Was ist Raum, was ist Zeit?
Wähle Beiträge von
# bis # FAQ
[/[Drucken]\]

Freigeisterhaus -> Wissenschaft und Technik

#1: Was ist Raum, was ist Zeit? Autor: stepWohnort: Germering BeitragVerfasst am: 09.04.2016, 00:16
    —
Ganz guter Artikel im April-SdW:

http://www.spektrum.de/magazin/sind-raum-und-zeit-das-produkt-quantenmechanischer-verschraenkungsprozesse/1400768

Es ist natürlich sehr schwer, so etwas anschaulich darzustellen. Aber ich halte das, basierend auf der sog. Maldacena-Dualität, für den besten Ansatz der vergangenen Jahrzehnte. Sozusagen das erste Mal, daß man sagen könnte "wir beginnen zu verstehen, was Raum und Zeit wirklich sind". Nebenbei bietet dieser Ansatz auch eine mögliche Erklärung für die Expansion des Universums.

Falls jemand das PDF will, bitte PN mit @.

#2: Re: Was ist Raum, was ist Zeit? Autor: Tso Wang BeitragVerfasst am: 09.04.2016, 15:10
    —
.

http://www.nature.com/news/the-quantum-source-of-space-time-1.18797

()

#3:  Autor: smallie BeitragVerfasst am: 09.04.2016, 17:14
    —
Kurzfassung:

Was ist Verschränkung? Warum passt meine Vorstellung von Verschränkung nicht zu dem, was im Artikel steht? Am Kopf kratzen





DE-SITTER-RAUM

Der Artikel erwähnt ihn überhaupt nicht, sondern steigt gleich beim Anti-De-Sitter-Raum ein. Das finde ich schade.

Cowen hat folgendes geschrieben:
Maldacena postulierte eine Beziehung zwische zwei scheinbar ganz unterschiedlichen Modelluniversen. Das eine heißt Anti-de-Sitter-Raum (AdS). Dieses Modell ähnelt unserem Kosmos - allerdings ohne Expansion oder Kontraktion -, ist dreidimensional, enthält Quantenteilchen und gehorcht den einsteinschen Feldgleichungen.

Anti-de-Sitter ist eine mathemagische Transformation des normalen de-Sitter-Raums. Er unterscheidet sich vom "normalen" Raum durch die an M.C. Escher erinnernde Projektion, die es salopp gesagt erlaubt, auf endlichem Raum unendlich viel unterzubringen.

Die Motivation dahinter: untersucht man den "normalen Raum", dann fällt es schwer, einen Grund anzugeben, warum man nun ein kleines oder ein großes Raumstück untersuchen sollte. Abgrenzungen - im Sinne von Ereignishorizonten - wären erstmal willkürlich, genau die will man ja berechnen, da wäre es schlecht, wenn man sie bereits in den Ansatz hineinsteckte.

Bei AdS hingegen braucht man nicht vorher abgrenzen, da passt alles auf wenige Pixel. zwinkern





Obendrein: in einer Sache ähnelt unser Kosmos weder de-Sitter- noch Anti-de-Sitter, denn in unserem Kosmos gibt es neben dem Raum, der kosmologischen Konstante beziehungsweise der Nullpunktsenergie auch noch gewöhnliche, meinetwegen auch dunkle, Materie. Mein Bauchgefühl sagt, es ist keine gute Idee, die Materie einfach wegzulassen.



VERSCHRÄNKUNG UND INFORMATIONSPARADOX

Was am Ereignishorizont genau passiert, ist seit den 1970er Jahren umstritten. Siehe die Debatte darüber, ob Schwarze Löcher Haare haben oder nicht. Wenn verschränkte Teilchenpaare nahe an einem Ereignishorizont entstehen, dann kann eins davon ins Schwarze Loch fallen, während das andere entkommt. (Hat Hawking so gesagt, und hinzugefügt, daß es in Wirklichkeit komplizierter ist.)

Der Gedanke ist problematisch, weil dabei Information über das Hineingefallene verlorengeht.

    - Hawking-Strahlung kann nicht gleichzeitig thermisch-zufällig sein und obendrein Information über einen Raumfahrer, ein Raumschiff oder einen ganzen Stern enthalten, der hineingefallen ist.

    - Oder Hawkings Vorstellung ist falsch, dann handelt man sich neue Probleme in Form einer Feuerwand ein. Die, soweit ich es verstanden habe, von der Wahl der Koordinaten abhängig ist.

    - Oder es geht Quanteninformation verloren.

So oder so bleibt nur Mit den Augen rollen


Es wundert mich, daß der Artikel dazu nichts sagt.


Im Gegensatz zu unserem Kosmos ist ein AdS-Raum frei von Materie. Insbesondere auch frei von starken Feldern, die ein spontan entstehendes und vergehendes Teilchenpaar überhaupt erst trennen könnten. Der Artikel spricht viel von Verschränkung. Aber mir ist nicht klar, was da verschränkt sein soll? Außer der Nullpunktsenergie ist ja nichts da.


Ich könnte wüst spekulieren, daß es bei der Verschränkung um hypothetische Partikel wie das Inflaton geht.

Oder auch, daß nicht nur ein Partikelpaar spontan entsteht, sondern mehrere gleichzeitig, die dann so miteinander Wechselwirken, ohne sich zu annihilieren. Klingt aber nicht überzeugend, denn nach einer Wechselwirkung ist es doch vorbei mit der Verschränkung?




WELCHE SKALA HAT DIESER GRAPH?

Verschränkung ist unter "normalen" Bedingungen kurzlebig und kleinräumig. Es braucht ein Menge Apparatur, um eine Verschränkung aufrecht zu erhalten. Wie ist dann dieser Graph zu lesen? Geht es da um subatomare Größenordnungen oder um kosmologische?


Wenn's um atomare Größenordnungen geht, könnte ich mir einen Reim darauf machen. Aber es ist von bulk universe die Rede.

In meiner Lesart geht das Argument so:

Bei uns entsteht ein Paar verschränkter Teilchen. Sagen wir zwei Photonen. Das eine saust in Richtung Andromeda-Nebel davon und stellt so den Kausalzusammenhang zwischen der Milchstraße und dem Rest des beobachtbaren Universums her?

Ist es das, was der Artikel sagen will? Diese Auslegung finde ich nicht überzeugend. Warum muß das Photon verschränkt sein? Damit es nach Hause telefonieren kann? Reicht ein darüberstreichen des Lichtkegels nicht, um einen kausalen Zusammenhang herzustellen?



Und überhaupt:

- wie weit kommt ein verschränktes Photon, bevor es mit irgend etwas wechselwirkt? Da muß ich passen, tippe aber auf nicht so weit. Stimmt das?

- mag sein, daß die zugrundeliegenden Arbeiten das anders darstellen, in der Graphik jedenfalls sind die Partikel promisk verschränkt. So ist es in Wirklichkeit auch nicht, oder?




WURMLÖCHER

Cowen hat folgendes geschrieben:
Nach der ER = EPR-Hypothese ist die durch ein Wurmloch hergestellte Verbindung dasselbe wie eine Quantenverschränkung, nur in viel größerem Maßstab.


Eine kühne These. Das gefällt mir. Glauben tu' ich sie nicht.



Wurmlöcher - gibt es irgend ein Argument, das auch nur annähernd auf die Existenz solcher Dinger hindeutet? Die Wurmloch-Analogie bringt nichts Neues, das über Begriff Nicht-Lokalität hinausgeht. Oder ist mir da etwas entgangen?




PS:

Besonders bei der de-Sitter-Sache ohne Gewähr.

#4:  Autor: stepWohnort: Germering BeitragVerfasst am: 09.04.2016, 18:23
    —
smallie hat folgendes geschrieben:
in einer Sache ähnelt unser Kosmos weder de-Sitter- noch Anti-de-Sitter, denn in unserem Kosmos gibt es neben dem Raum, der kosmologischen Konstante beziehungsweise der Nullpunktsenergie auch noch gewöhnliche, meinetwegen auch dunkle, Materie. Mein Bauchgefühl sagt, es ist keine gute Idee, die Materie einfach wegzulassen.

Ja, wie im Artikel erwähnt, ist das modellierte Universum statisch. Ich denke aber, das ist bei Fragen nach der Natur der Raumzeit erstmal legitim und sogar eine gute Idee, auf diese Weise einen fundamentalen Zusammenhang zu suchen.

smallie hat folgendes geschrieben:
Im Gegensatz zu unserem Kosmos ist ein AdS-Raum frei von Materie. Insbesondere auch frei von starken Feldern, die ein spontan entstehendes und vergehendes Teilchenpaar überhaupt erst trennen könnten. Der Artikel spricht viel von Verschränkung. Aber mir ist nicht klar, was da verschränkt sein soll? Außer der Nullpunktsenergie ist ja nichts da.

Nee, das ist ganz anders gemeint. In dem (leeren) AdS-Universum gibt es das Phänomen "Raum", während die Quantenfelder und ihre Verschränkung in einem anderen Raum (der Hülle mit den Tensoren) leben. Es sind also nicht die uns bekannten Teilchen, sondern sie werden in einem mathematischen, erstmal hypothetischen Raum angenommen, und zwar gerade so, daß ihre Beschreibung mathematisch äquivalent zum Raum in "unserem" AdS-Universum ist. Das dies überhaupt geht, ist nicht selbstverständlich. Der Witz dabei ist, daß die Tensorfelder einer QFT genügen, mit der man einigermaßen gut umgehen kann.
Wenn sich das besser bestätigt, kann man u.a. zurückschließen und aus den Eigenschaften der Tensorfelder Voraussagen über Raumphänomene machen, und letztlich eine fundamentalere Erklärung für Raum und Zeit angeben.

Ich habe nach einem einfach verständlichen Analogon gesucht, mir ist aber leider noch keins eingefallen.

smallie hat folgendes geschrieben:
Verschränkung ist unter "normalen" Bedingungen kurzlebig und kleinräumig. Es braucht ein Menge Apparatur, um eine Verschränkung aufrecht zu erhalten.

Ja, in unserem Universum. Die dort geltenden QFTs haben ja auch ganz andere Parameter.

smallie hat folgendes geschrieben:
Wie ist dann dieser Graph zu lesen? Geht es da um subatomare Größenordnungen oder um kosmologische?

Ich würde sagen, es gilt für jede Größenordnung oberhalb der Plancklängen.

smallie hat folgendes geschrieben:
Bei uns entsteht ein Paar verschränkter Teilchen. Sagen wir zwei Photonen. Das eine saust in Richtung Andromeda-Nebel davon und stellt so den Kausalzusammenhang zwischen der Milchstraße und dem Rest des beobachtbaren Universums her? Ist es das, was der Artikel sagen will?

Nein, wie gesagt, die Verschränkung bezieht sich gar nicht auf die Teilchen in unserem Universum, sondern auf die der generierenden QFT in der hypothetischen Hülle.

smallie hat folgendes geschrieben:
- wie weit kommt ein verschränktes Photon, bevor es mit irgend etwas wechselwirkt? Da muß ich passen, tippe aber auf nicht so weit. Stimmt das?

Auf der Erde stimmt das jedenfalls. Aber Photonen gehorschen ja einer ganz anderen QFT.

smallie hat folgendes geschrieben:
- mag sein, daß die zugrundeliegenden Arbeiten das anders darstellen, in der Graphik jedenfalls sind die Partikel promisk verschränkt. So ist es in Wirklichkeit auch nicht, oder?

In dem CFT Raum ist es aber so, nehme ich an. Und wenn es da Dekohärenz gibt, gibts bei uns äquivalent einen neuen Raum, salopp gesagt. Wie "selten" das nun vorkommt, weiß ich auch nicht.

smallie hat folgendes geschrieben:
Wurmlöcher - gibt es irgend ein Argument, das auch nur annähernd auf die Existenz solcher Dinger hindeutet? Die Wurmloch-Analogie bringt nichts Neues, das über Begriff Nicht-Lokalität hinausgeht.

Wurmlöcher sind ein Raumzeitphänomen (jedenfalls wenn die ART stimmt). Daher ist es interessant, ob der Maldacena-Ansatz diese äquivalent voraussagt und welches Quantenphänomen im CFT Raum ihnen entspricht.

#5: Gravity and Entanglement Autor: Tso Wang BeitragVerfasst am: 10.04.2016, 10:15
    —
.

https://www.youtube.com/watch?v=WQU9yOtWrQk

()

#6:  Autor: stepWohnort: Germering BeitragVerfasst am: 10.04.2016, 13:03
    —
Danke, in dem Video ist noch einiges Weitere erklärt, so daß man einen besseren Eindruck von den Zusammenhängen bekommt.

#7:  Autor: zelig BeitragVerfasst am: 10.04.2016, 14:21
    —
step hat folgendes geschrieben:
Ich habe nach einem einfach verständlichen Analogon gesucht, mir ist aber leider noch keins eingefallen.


Für einen interessierten Laien wäre es sehr hilfrauch, wenn Dir das gelänge.
Vorläufig stelle ich mir eine riesige Menge von Seifenblasen vor, die sich vielfach in ihren Oberflächen überschneiden (oder Übergänge bilden?). Wir sind die zweidimensionalen Wesen, die diesmal auf der Innenseite einer dieser Seifenblase existieren. Wir haben bisher eine Theorie über die Innenregion ("Raum") unserer Blase, die wir beobachten können. Und wir haben eine Theorie über die Region auf der wir leben (Grenze des "Raums").
Insgesamt könnte es sein, daß die eine Theorie die gesamte Metaregion beschreibt, und die andere eine lokale Subregion. Bezogen auf das aktuelle Theme sind wir gerade von der Entdeckung fasziniert, daß Gravitation als lokale Beschreibung äquivalent (?) zu einer Untermenge der Metatheorie ist. Ferner könnte die Verschränkung konstitutiv für die (physikalische) Zeit sein.

Ich wünschte, ich hätte mich früher stärker für diese Themen interessiert. Wahrscheinlich zeige ich gerade vor allem meine eigenen Grenzen. : )

#8:  Autor: stepWohnort: Germering BeitragVerfasst am: 10.04.2016, 16:01
    —
Dasselbe nochmal anders erklärt, und mit einer Analogie (soup can label). Die Analogie gefällt mir persönlich nicht so, aber trotzdem:
https://www.sciencenews.org/article/entanglement-gravitys-long-distance-connection

Ein kurzer (leicht) und ein längerer (schwierig) Foliensatz:
http://people.brandeis.edu/~headrick/talks/EntanglementGeometry.pdf
http://gr20-amaldi10.edu.pl/userfiles/10-03_Tadashi%20Takayanagi%20-%20Entanglement%20Entropy%20and%20the%20Gravity___.pdf

#9:  Autor: smallie BeitragVerfasst am: 10.04.2016, 21:14
    —
step hat folgendes geschrieben:
smallie hat folgendes geschrieben:
in einer Sache ähnelt unser Kosmos weder de-Sitter- noch Anti-de-Sitter, denn in unserem Kosmos gibt es neben dem Raum, der kosmologischen Konstante beziehungsweise der Nullpunktsenergie auch noch gewöhnliche, meinetwegen auch dunkle, Materie. Mein Bauchgefühl sagt, es ist keine gute Idee, die Materie einfach wegzulassen.

Ja, wie im Artikel erwähnt, ist das modellierte Universum statisch. Ich denke aber, das ist bei Fragen nach der Natur der Raumzeit erstmal legitim und sogar eine gute Idee, auf diese Weise einen fundamentalen Zusammenhang zu suchen.

Da fehlt mir noch ein Baustein.

In einem statischen Universum sollte es nicht vorkommen, daß sich der gelbe und der rote Raumbereich voneinander trennen. Der Vorgang sieht mir recht dynamisch aus.



step hat folgendes geschrieben:
smallie hat folgendes geschrieben:
Im Gegensatz zu unserem Kosmos ist ein AdS-Raum frei von Materie. Insbesondere auch frei von starken Feldern, die ein spontan entstehendes und vergehendes Teilchenpaar überhaupt erst trennen könnten. Der Artikel spricht viel von Verschränkung. Aber mir ist nicht klar, was da verschränkt sein soll? Außer der Nullpunktsenergie ist ja nichts da.

Nee, das ist ganz anders gemeint. In dem (leeren) AdS-Universum gibt es das Phänomen "Raum", während die Quantenfelder und ihre Verschränkung in einem anderen Raum (der Hülle mit den Tensoren) leben. Es sind also nicht die uns bekannten Teilchen, sondern sie werden in einem mathematischen, erstmal hypothetischen Raum angenommen, und zwar gerade so, daß ihre Beschreibung mathematisch äquivalent zum Raum in "unserem" AdS-Universum ist.

"Verschränkt" in CFT, nicht aber in AdS - das erklärt, warum ich mit der Darstellung der Verschränkung nichts anfangen konnte. Danke, das war mir nicht klar.



step hat folgendes geschrieben:
smallie hat folgendes geschrieben:
Wie ist dann dieser Graph zu lesen? Geht es da um subatomare Größenordnungen oder um kosmologische?

Ich würde sagen, es gilt für jede Größenordnung oberhalb der Plancklängen.

Da gehe ich mit. Solange der betreffende Raumbereich nahe an der Schwarzschilddichte liegt. Nur dann gilt, daß Entropie proportional zu einer Oberfläche ist. Im normalen Raum ist sie grob gesagt eher proportional zum Volumen.


Wenn die Beschreibung der Welt im AdS-Raum äquivalent ist zur Beschreibung in CFT, dann sollten sich Zusammenhänge in einem System in die Sprache das anderen übertragen lassen.

Welche Ursachen gibt es - in herkömmlicher Darstellung -, daß sich Raumbereiche abspalten, die hinterher kausal getrennt sind? Zwei Gründe fallen mir ein:

    1) kosmische Expansion. Sei es Inflation, kosmologische Konstante oder eine, ermm, "variable" kosmologische Konstante.
    2) gravitativer Ereignishorizont, also Schwarzes Loch.



Zu 1) Denk an zwei Beobachter, von denen einer am "Nordpol", einer am "Südpol" des beobachtbaren Universums steht. Beide sind "heute" kausal voneinander getrennt, obwohl beide in unserem Lichtkegel liegen. Das obige Diagramm stellt das als disjunkte Raumbereiche, ja sogar als getrennte Universen, dar - was sie im Sinne einer gelösten Verschränkung in CFT auch sind. Daß es aber kontinuierlichen Raum dazwischen gibt, ist in der graphischen Darstellung verloren gegangen. Ich bin mir ziemlich sicher, daß die mathematische Darstellung des Sachverhaltes diesen Fehler nicht macht. Meine Kritik ist eher, daß die neuartige Darstellung den beiden Argumenten 1) und 2) nichts hinzufügt.

Zu 2) Ein Schwarzes Loch könnte ich mir als Raumbereich verstehen, der ebenfalls von unserem kausal getrennt ist. Ob nur vorübergehend oder für immer - eine Antwort darauf wäre die Lösung des Informationsparadoxons.


Wenn die Ads/CFT-Dualität zu diesen Fragen etwas beitragen könnte, dann wäre sie mehr als nur Spekulation. Interessanterweise stammen die Beispiele für die Nützlichkeit der Dualität, wie sie der SdW-Artikel nennt, aus ganz anderen Bereichen der Physik.



step hat folgendes geschrieben:
smallie hat folgendes geschrieben:
Wurmlöcher - gibt es irgend ein Argument, das auch nur annähernd auf die Existenz solcher Dinger hindeutet? Die Wurmloch-Analogie bringt nichts Neues, das über Begriff Nicht-Lokalität hinausgeht.

Wurmlöcher sind ein Raumzeitphänomen (jedenfalls wenn die ART stimmt). Daher ist es interessant, ob der Maldacena-Ansatz diese äquivalent voraussagt und welches Quantenphänomen im CFT Raum ihnen entspricht.

Meine Vorstellung geht so:

wäre ich Universumsbauer und könnte darum Masse beliebig im Raum platzieren, dann könnte ich auch Wurmlöcher erzeugen. Wie bei einer natürlichen kosmischen Entwicklung ein Wurmloch entstehen sollte, überfordert meine Phantasie.

Nichtsdestotrotz:

Wenn es gelingt, Gravitation mit den anderen Kräften zu vereinigen, dann ist es zumindest denkbar, daß sich das Phänomen "Wurmloch" analog auch bei den anderen Kräften zeigt. Insofern ziehe ich meinen Einwand zurück. Auch wenn es kein praktisches Beispiel für ein Wurmloch in der Welt der Schwerkraft gibt, so könnte es doch eins in der Welt der CFT-GUT-Verschränkung geben.

Das muß zwangsläufig so sein, wenn sich die Erscheinungen unseres Raumes tatsächlich um eine Dimension verringern lassen. Dann gibt es Korrelationen, die wir nicht sehen.



Noch ein ganz großes caveat hinterhergeschickt.

Die AdS/CFT-Dualität kann Korrelationen beschreiben kann, die wir nicht sehen. Aber warum sollten diese Korrelationen EPR-artig sein? Warum gilt ER = EPR, obwohl niemand EPR vertritt?

EPR ist die Behauptung, es gäbe verborgene Variablen. Bisher gibt es aber keinen Fall, der die Bellsche Ungleichung verletzt. Wenn AdS/CFT tatsächlich EPR bestätigen sollte und verborgene Variablen wieder salonfähig macht, wäre das eine Sensation.

#10:  Autor: smallie BeitragVerfasst am: 10.04.2016, 21:38
    —
zelig hat folgendes geschrieben:
step hat folgendes geschrieben:
Ich habe nach einem einfach verständlichen Analogon gesucht, mir ist aber leider noch keins eingefallen.


Für einen interessierten Laien wäre es sehr hilfrauch, wenn Dir das gelänge.

So vielleicht.

Hier wird der 3D-Raum mit verblüffendem Effekt auf 2D reduziert.



Bei AdS/CFT ist es umgekehrt. Da gibt es derartige Stützeffekte oder Korrelationen, obwohl wir sie nicht sehen.

#11:  Autor: stepWohnort: Germering BeitragVerfasst am: 11.04.2016, 15:32
    —
smallie hat folgendes geschrieben:
Die AdS/CFT-Dualität kann Korrelationen beschreiben kann, die wir nicht sehen. Aber warum sollten diese Korrelationen EPR-artig sein? Warum gilt ER = EPR, obwohl niemand EPR vertritt? EPR ist die Behauptung, es gäbe verborgene Variablen. Bisher gibt es aber keinen Fall, der die Bellsche Ungleichung verletzt. Wenn AdS/CFT tatsächlich EPR bestätigen sollte und verborgene Variablen wieder salonfähig macht, wäre das eine Sensation.

EPR steht hier einfach für Verschränkung, nämlich für die drei Forscher, die damals das erste Paper über Verschränkung schrieben:

Einstein, A.; Podolsky, B.; Rosen, N. (15 May 1935). "Can Quantum-Mechanical Description of Physical Reality Be Considered Complete?" - Physical Review 47 (10): 777–780.

Zu den anderen Fragen gibt es teilweise was in der verlinkten Präsentation, z.B. Fläche vs. Volumen und auch zur möglichen Auflösung des Informationsparadoxes und kosmischer Expansion durch diesen Ansatz. Ansonsten sobald ich wieder Zeit finde ...

#12:  Autor: smallie BeitragVerfasst am: 11.04.2016, 19:04
    —
Vorrausgeschickt: es fehlt ein "nicht" im Satz über die Verletzung der Bellschen Ungleichung.

step hat folgendes geschrieben:
smallie hat folgendes geschrieben:
Die AdS/CFT-Dualität kann Korrelationen beschreiben kann, die wir nicht sehen. Aber warum sollten diese Korrelationen EPR-artig sein? Warum gilt ER = EPR, obwohl niemand EPR vertritt? EPR ist die Behauptung, es gäbe verborgene Variablen. Bisher gibt es aber keinen Fall, der die Bellsche Ungleichung verletzt. Wenn AdS/CFT tatsächlich EPR bestätigen sollte und verborgene Variablen wieder salonfähig macht, wäre das eine Sensation.

EPR steht hier einfach für Verschränkung, nämlich für die drei Forscher, die damals das erste Paper über Verschränkung schrieben:

Einstein, A.; Podolsky, B.; Rosen, N. (15 May 1935). "Can Quantum-Mechanical Description of Physical Reality Be Considered Complete?" - Physical Review 47 (10): 777–780.

Ist mir bekannt.

Vergleiche Wikipedia:

Wikipedia hat folgendes geschrieben:
Bell's theorem - Historical background

In the early 1930s, the philosophical implications of the current interpretations of quantum theory troubled many prominent physicists of the day, including Albert Einstein. In a well-known 1935 paper, Boris Podolsky and co-authors Einstein and Nathan Rosen (collectively "EPR") sought to demonstrate by the EPR paradox that QM was incomplete. This provided hope that a more-complete (and less-troubling) theory might one day be discovered. But that conclusion rested on the seemingly reasonable assumptions of locality and realism (together called "local realism" or "local hidden variables", often interchangeably). In the vernacular of Einstein: locality meant no instantaneous ("spooky") action at a distance; realism meant the moon is there even when not being observed. These assumptions were hotly debated in the physics community, notably between Nobel laureates Einstein and Niels Bohr.

In his groundbreaking 1964 paper, "On the Einstein Podolsky Rosen paradox", physicist John Stewart Bell presented an analogy (based on spin measurements on pairs of entangled electrons) to EPR's hypothetical paradox. Using their reasoning, he said, a choice of measurement setting here should not affect the outcome of a measurement there (and vice versa). After providing a mathematical formulation of locality and realism based on this, he showed specific cases where this would be inconsistent with the predictions of QM theory.

https://en.wikipedia.org/wiki/Bell's_theorem


Ich halte mein Argument aufrecht - wenn AdS/CFT eine Aussage macht in Sachen EPR, - und das scheint mir der Fall zu sein, auch wenn es nicht ausdrücklich gesagt wurde -, dann wäre das eine Sensation.


step hat folgendes geschrieben:
Zu den anderen Fragen gibt es teilweise was in der verlinkten Präsentation, z.B. Fläche vs. Volumen und auch zur möglichen Auflösung des Informationsparadoxes und kosmischer Expansion durch diesen Ansatz. Ansonsten sobald ich wieder Zeit finde ...

Beim ersten drüberschauen gestern habe ich nach einem Drittel aufgegeben. Für mein Hausmacher-Niveau ist das dann doch etwas hoch. Weinen Mal sehen, ob nach gründlichem Lesen etwas 'rumkommt.

#13:  Autor: stepWohnort: Germering BeitragVerfasst am: 11.04.2016, 19:49
    —
@smallie - ich verstehe nicht, wie Du darauf kommst, es deute hier etwas auf verborgene Variablen hin. Wenn der AdS/CFT Ansatz stimmt, ist die quantisierte Raumzeit äquivalent zu einer QFT ohne Gravitation in einem um 1 niederdimensionalen Raum (typischerweise 4 statt 5). Wo sollen da die HV herkommen, die es in einer QFT ja eben nicht gibt?

Zum BH Information Paradox:

https://en.wikipedia.org/wiki/AdS/CFT_correspondence#Black_hole_information_paradox

Zitat:
The AdS/CFT correspondence resolves the black hole information paradox, at least to some extent [...] Any ... black hole corresponds to a configuration of particles on the boundary of anti-de Sitter space. These particles obey the usual rules of quantum mechanics and in particular evolve in a unitary fashion, so the black hole must also evolve in a unitary fashion, respecting the principles of quantum mechanics. In 2005, Hawking announced that the paradox had been settled in favor of information conservation by the AdS/CFT correspondence, and he suggested a concrete mechanism by which black holes might preserve information.

#14:  Autor: smallie BeitragVerfasst am: 12.04.2016, 19:58
    —
step hat folgendes geschrieben:
@smallie - ich verstehe nicht, wie Du darauf kommst, es deute hier etwas auf verborgene Variablen hin. Wenn der AdS/CFT Ansatz stimmt, ist die quantisierte Raumzeit äquivalent zu einer QFT ohne Gravitation in einem um 1 niederdimensionalen Raum (typischerweise 4 statt 5). Wo sollen da die HV herkommen, die es in einer QFT ja eben nicht gibt?

Das mußt du die Leute fragen, die EPR ins Spiel gebracht haben. Und bei EPR denke ich an verborgene Variable, Nicht-Lokalität oder Nicht-Realismus. Zu mindestens einem dieser Punkte sollte eine Aussage kommen, wenn EPR angerufen wird.


step hat folgendes geschrieben:
Zum BH Information Paradox:

https://en.wikipedia.org/wiki/AdS/CFT_correspondence#Black_hole_information_paradox

Zitat:
The AdS/CFT correspondence resolves the black hole information paradox, at least to some extent [...] Any ... black hole corresponds to a configuration of particles on the boundary of anti-de Sitter space. These particles obey the usual rules of quantum mechanics and in particular evolve in a unitary fashion, so the black hole must also evolve in a unitary fashion, respecting the principles of quantum mechanics. In 2005, Hawking announced that the paradox had been settled in favor of information conservation by the AdS/CFT correspondence, and he suggested a concrete mechanism by which black holes might preserve information.

Nun ja, Hawking schreibt weiter mit an Artikel, in denen die Sache aus anderem Winkel betrachtet.

Das neuste Ding sind supertranslations - frag mich nicht, was das ist, da muß ich passen. Siehe hier, ich zitiere nur den Teil zu Hawkings Mitwirkung:


Zitat:
Stephen Hawking's New Black-Hole Paper, Translated
January 8, 2016


SF: Stephen Hawking is an author on this paper, so I take it he agrees that his original argument was flawed in this way.


AS: Right. I think that’s why he got excited. People have made all kinds of crazy criticisms of his argument, and to the best of my impressions, he’s correctly objected to all of them. But this one, he heard it and he seemed to immediately agree that this was the key. In fact, as you’ve learned from what happened at Stockholm, he’s more certain than I am that this is the missing link in understanding black hole information. I’ve been surprised so many times in my career about how things turn out that I'm not making any predictions.

http://blogs.scientificamerican.com/dark-star-diaries/stephen-hawking-s-new-black-hole-paper-translated-an-interview-with-co-author-andrew-strominger/

#15:  Autor: stepWohnort: Germering BeitragVerfasst am: 12.04.2016, 20:18
    —
smallie hat folgendes geschrieben:
Und bei EPR denke ich an verborgene Variable, Nicht-Lokalität oder Nicht-Realismus. Zu mindestens einem dieser Punkte sollte eine Aussage kommen, wenn EPR angerufen wird.

Das stimmt sogar: Nichtlokalität in der CFT.

#16:  Autor: smallie BeitragVerfasst am: 12.04.2016, 20:36
    —
Ich hab' EPR als Kritik an dem Konzept abgespeichert.

Wenn die EPR-Arbeit geschrieben wurde, um zu sagen: seht her! Quantenmechanische Verschränkung ist Nicht-Lokal! dann verstehe ich, was gemeint ist, und mein Gedanke, da sei eine Sensation versteckt, war Unfug. Die populäre Darstellung von EPR geht eher so, daß sie sagten: Seht! Da ist eine Lücke im Gedankengebäude.

Allerdings wäre ich nicht überrascht, wenn die populäre Darstellung etwas anders geht, als das, was wirklich in der Arbeit steht.

#17:  Autor: smallie BeitragVerfasst am: 25.04.2016, 21:58
    —
Das EPR-Papier habe ich inzwischen gelesen und auch schon ein bisschen was dazu und zu Bell aufgeschrieben.

Nun ist mir gerade aufgefallen, daß ich doof bin.

Wenn ich die Sache richtig verstehe, dann sollte folgendes möglich sein:

    - geh' ins 3D-Kino.
    - platziere zwei Brillen hintereinander, und zwar zwei gegenseitig polarisierende Scheiben. Ergebnis: es geht kein Licht durch.
    - platziere drei Brillen hintereinander, so daß die mittlere um 45° zwischen den zwei entgegengesetzten steht. Ergebnis: es geht ein bisschen Licht durch.

Kommt das hin?

Frage

#18:  Autor: stepWohnort: Germering BeitragVerfasst am: 27.04.2016, 13:50
    —
smallie hat folgendes geschrieben:
Wenn ich die Sache richtig verstehe, dann sollte folgendes möglich sein:

    - geh' ins 3D-Kino.
    - platziere zwei Brillen hintereinander, und zwar zwei gegenseitig polarisierende Scheiben. Ergebnis: es geht kein Licht durch.
    - platziere drei Brillen hintereinander, so daß die mittlere um 45° zwischen den zwei entgegengesetzten steht. Ergebnis: es geht ein bisschen Licht durch.

Kommt das hin?

Ja, das ist korrekt.

Ohne Quantenphysik kann man sich das so veranschaulichen: Der Linear-Polfilter läßt nur eine Ebene durch. Er "betrachtet" dazu jede Lichtwelle als zusammengesetzt aus einer Komponente parallel und senkrecht zu seiner P-Richtung - was aufgrund der vektoriellen Natur des EM-Feldes möglich ist.

Wenn der erste Filter jetzt z.B. 0° und der zweite ß-Richtung durchläßt, dann "zerlegt" der zweite Filter das 0°-Licht in Linearkomponenten ß und ß-π/2. Es kommt also die relative Amplitude cos(ß) in ß-Richtung polarisiert durch.

Der dritte Filter macht es genauso: Wenn er α durchläßt, dann "zerlegt" der dritte Filter das ß-Licht in Linearkomponenten α und α-π/2. Es kommt also am Ende Amplitude cos(ß)*cos(ß-α) in α-Richtung polarisiert durch.

Ist der Hintegrund Deiner Frage etwa das Gedankenexperiment mit dem Zufallsgenerator, z.B. hier beschrieben?
http://scienceblogs.de/arte-fakten/2010/07/02/die-eprgeschichte-teil-3-ein-zufallsgenerator-fur-bell/

#19:  Autor: smallie BeitragVerfasst am: 28.04.2016, 23:16
    —
step hat folgendes geschrieben:
smallie hat folgendes geschrieben:
Wenn ich die Sache richtig verstehe, dann sollte folgendes möglich sein:

    - geh' ins 3D-Kino.
    - platziere zwei Brillen hintereinander, und zwar zwei gegenseitig polarisierende Scheiben. Ergebnis: es geht kein Licht durch.
    - platziere drei Brillen hintereinander, so daß die mittlere um 45° zwischen den zwei entgegengesetzten steht. Ergebnis: es geht ein bisschen Licht durch.

Kommt das hin?

Ja, das ist korrekt.

Jetzt bin ich gespannt, das mal in Echt auszuprobieren.


step hat folgendes geschrieben:
Ohne Quantenphysik kann man sich das so veranschaulichen: Der Linear-Polfilter läßt nur eine Ebene durch. Er "betrachtet" dazu jede Lichtwelle als zusammengesetzt aus einer Komponente parallel und senkrecht zu seiner P-Richtung - was aufgrund der vektoriellen Natur des EM-Feldes möglich ist.

Wenn der erste Filter jetzt z.B. 0° und der zweite ß-Richtung durchläßt, dann "zerlegt" der zweite Filter das 0°-Licht in Linearkomponenten ß und ß-π/2. Es kommt also die relative Amplitude cos(ß) in ß-Richtung polarisiert durch.

Der dritte Filter macht es genauso: Wenn er α durchläßt, dann "zerlegt" der dritte Filter das ß-Licht in Linearkomponenten α und α-π/2. Es kommt also am Ende Amplitude cos(ß)*cos(ß-α) in α-Richtung polarisiert durch.

So genau hätte ich das nicht sagen können.

Auf meinem Niveau - und rückwärts gedacht - lautet das: beim Durchgang durch einen Spalt - hier Polfilter - ist der Ort eines Teilchens festgelegt. Würde das Teilchen dann noch durch einen zweiten, um 90 Grad gedrehten Spalt in Abstand x gehen, wäre sowohl Ort und Geschwindigkeit des Teilchens bekannt.


step hat folgendes geschrieben:
Ist der Hintegrund Deiner Frage etwa das Gedankenexperiment mit dem Zufallsgenerator, z.B. hier beschrieben?
http://scienceblogs.de/arte-fakten/2010/07/02/die-eprgeschichte-teil-3-ein-zufallsgenerator-fur-bell/

Ja, genau.

Hab' inzwischen halbwegs fertig, was ich aufschreiben wollte. Ein kurzer, geschichtlicher Überblick zum Thema Verschränkung.





1926 - de Broglie behauptet die Wellennatur von Partikeln.
1926 - Schrödinger stellt seine Gleichung auf, mit dem Elektron als eine Partikelwelle.
1926 - Born sieht das Amplitudenquadrat der Wellenfunktion als Aufenthaltswahrscheinlichkeit des Teilchens

1927 - Heisenbergs Unschärferelation
1927 - Kopenhagener Interpretation

1932 - Nachweis des Neutrons

1935 - EPR.

EPR argumentiert in zwei Stufen.

    1) Kopenhaagener Kollaps

    Mit einer Messung wird der Zustand der Wellenfunktion festgestellt. Oder auf deutsch: man schaut, wo man das Partikel tatsächlich findet. Der Messvorgang führt dazu, daß der weitere Zustand des neuen Systems unbekannt ist.


    2) Was passiert, wenn zwei dieser Kollapse hintereinander passieren? Beziehungsweise an zwei verschränkten Teilchen? Der erste Kollaps hinterläßt das System in einem undefinierten Zustand, deshalb führen weitere Messungen zu logischen Widersprüchen.


1957 - Bohm und Aharonov kommen auf die übliche Darstellung mit dem Spin. Discussion of Experimental Proof for the Paradox of Einstein, Rosen, and Podolsk

1965 - Bell stellt seine Ungleichung auf.




Hier ein Beispiel, wie es aussähe, wenn in der Quantenwelt die gleichen Regeln herrschten wie in der makroskopischen. Statt verschränkter Photonen oder Elektronen nehme ich Spielkarten. Die sollen diese Eigenschaften haben:

    Farbe - schwarz oder weiß
    Schlag - rund oder eckig
    Zahl - gerade oder ungerade


Verschränkte Spielkarten haben dabei jeweils entgegengesetzte Eigenschaften. Wenn die eine schwarz ist, dann ist die andere weiß, und so weiter. Dies sind die möglichen Kombinationen, die dabei entstehen können. Die Spalten sind F, S, Z - Farbe, Schlag, Zahl.


Zitat:
F S Z

0 0 0
0 0 1
0 1 0
0 1 1
1 0 0
1 0 1
1 1 0
1 1 1



Nun erhalten Alice und Bob verschränkte Spielkarten, die sie jeweils auf eine beliebige Eigenschaft testen. Also entweder Farbe, Schlag. Oder Farbe, Zahl. Oder Schlag, Zahl. Sie notieren, ob sie gleich oder ungleich gemessen haben, G und U.


Zitat:
F S Z FS FZ SZ

0 0 0 G G G
0 0 1 G U U
0 1 0 U G U
0 1 1 U U G
1 0 0 U U G
1 0 1 U G U
1 1 0 G U U
1 1 1 G G G


Sie würden in 33% aller Fälle gleich messen. Warum die erste und letzte Zeile dabei keine Rolle spielen, kann ich leider nicht erklären.



Das Problem ist: im Quanten-Experiment ergeben sich andere Wahrscheinlichkeiten als diese Überlegung nahelegt. Es ist, als ob das Teilchen bei Bob wüßte, daß Alice sein verschränktes Gegenpart gemessen hat, sonst könnte es nicht zu einem anderen Meßergebnis kommen, als wenn Alice das Experiment nicht gemacht hätte.

Etwas fachlicher und ohne "Kartentricks" gesagt. Üblicherweise geht es um den Spin von Teilchen. Oder um Polarisationseben von Photonen. Photonen können Nord-Süd polarisiert sein oder Ost-West oder Südwest nach Nordost. Also so:

    | - /




Tja, und an der Stelle kam ein Kinobesuch Cool dazwischen, daraus entstand dann obige Frage.

#20:  Autor: smallie BeitragVerfasst am: 29.04.2016, 17:57
    —
Warum fällt es mir manchmal so schwer, meinen eigenen Unfug in statu nascendi zu erkennen? Nein

smallie hat folgendes geschrieben:
Auf meinem Niveau - und rückwärts gedacht - lautet das: beim Durchgang durch einen Spalt - hier Polfilter - ist der Ort eines Teilchens festgelegt. Würde das Teilchen dann noch durch einen zweiten, um 90 Grad gedrehten Spalt in Abstand x gehen, wäre sowohl Ort und Geschwindigkeit des Teilchens bekannt.

Das muß natürlich Ort und Impuls heißen.


Ok, kann passieren. Aber der hier ist grob:

smallie hat folgendes geschrieben:
Zitat:
F S Z FS FZ SZ

0 0 0 G G G
0 0 1 G U U
0 1 0 U G U
0 1 1 U U G
1 0 0 U U G
1 0 1 U G U
1 1 0 G U U
1 1 1 G G G


Sie würden in 33% aller Fälle gleich messen. Warum die erste und letzte Zeile dabei keine Rolle spielen, kann ich leider nicht erklären.

Genau deshalb heißt es ja Bellsche Ungleichung, weil von den 2 hoch n Zuständen des Systems immer zwei dabei sind, in denen Alice und Bob gleich messen.

#21: Re: Was ist Raum, was ist Zeit? Autor: uwebus BeitragVerfasst am: 15.05.2016, 17:54
    —
step hat folgendes geschrieben:
Ganz guter Artikel im April-SdW:

http://www.spektrum.de/magazin/sind-raum-und-zeit-das-produkt-quantenmechanischer-verschraenkungsprozesse/1400768


Auszug:
"Viele Physiker halten die Verschränkung für das grundlegende Quantenphänomen überhaupt. Einige mutmaßen jetzt sogar, Raum und Zeit seien Folgen quantenmechanischer Verschränkungsprozesse.

https://de.wikipedia.org/wiki/Prozess_(Begriffskl%C3%A4rung)
Prozess (Begriffsklärung)

Prozess, ein gerichteter Ablauf eines Geschehens
Latein: procedo vorwärts gehen, schreiten, hervortreten, -gehen, -kommen

Leute,

ihr beherrscht doch alle die deutsche Sprache. Wie kann denn der Ablauf eines Geschehens die Ursache dieses Geschehens sein? Zeit ist ein Maßstab für Veränderung, Veränderung ist ein Geschehen, welches mithilfe des Zeitmaßstabes beschrieben werden kann.

Was sind denn die Voraussetzungen dafür, daß es eine Veränderung überhaupt geben kann?
1) Das sich Verändernde muß eine Ausdehnung besitzen, da sich ein mathematischer Punkt nicht verändern kann, also ist Raum = Volumen+Inhalt die eine Voraussetzung für Veränderung.
2) Das sich räumlich Verändernde muß über ein inhärentes Ungleichgewicht verfügen, was zu einer STÄNDIGEN Veränderung seiner selbst führt, also das erzeugt, was man ZEIT nennt, das ist die zweite Voraussetzung.

Wer also Raum und Zeit erklären will, muß die Voraussetzungen des sich Verändernden erklären können, nicht dessen Veränderungsprozesse. ERST kommt das sich Verändernde, die Ursache, danach die Wirkung, der Veränderungsprozeß.

Deshalb braucht man den genannten Artikel gar nicht erst zu lesen, da hier Ursache und Wirkung vertauscht werden.

Manchmal habe ich den Eindruck, daß Physiker sich verhalten wie früher die Scholastiker, die sich auch ernsthaft darüber Gedanken machten, wie viele Engel wohl auf einer Nadelspitze Platz hätten, ohne jedoch darauf einzugehen, was Engel denn seien.

#22: Re: Was ist Raum, was ist Zeit? Autor: Bliss BeitragVerfasst am: 17.05.2016, 14:48
    —
uwebus hat folgendes geschrieben:
Quark

Shhh, wenn Erwachsene reden unterbricht man sie nicht.

#23:  Autor: zelig BeitragVerfasst am: 31.12.2016, 00:46
    —
Ein Teil des Streits über die Zeit im uwebus-Thread erübrigt sich vielleicht, wenn man akzeptiert, daß es sowohl die erlebte Zeit gibt, als auch die Zeit als wissenschaftliches Modell. Ähnlich wie man einerseits etwas schmeckt, und andererseits Lebensmittelchemiker wissen, welche Moleküle in welchen Kombinationen welchen Geschmack hervorrufen. Der Vergleich hinkt, klar.

Ausgehend von der Dreiteilung Vergangenheit, Gegenwart und Zukunft, würde mich mal interessieren, welches intuitive Bild der erlebten Zeit ihr habt, und wie ihr euch darin positioniert.
Bei mir ist es beispielsweise so, daß ich die Zukunft wie einen Luftstrom auf mich zukommen sehe, mein Standpunkt fixiert die Gegenwart ist, und die Vergangenheit der Strom, der an mir vorbei hinter meinen Rücken in die ferne fließt.

Und wo wir schon dabei sind, ich wünsche jetzt schon allen Mitforisten und stillen Mitlesern einen guten Rutsch ins neue Jahr.
In diesem Bild scheint man sich selber durch eine statische Zeit zu bewegen. : )

#24:  Autor: Marcellinus BeitragVerfasst am: 31.12.2016, 14:04
    —
zelig hat folgendes geschrieben:
Ein Teil des Streits über die Zeit im uwebus-Thread erübrigt sich vielleicht, wenn man akzeptiert, daß es sowohl die erlebte Zeit gibt, als auch die Zeit als wissenschaftliches Modell.


Und genau da haben wir die Probleme, weil es offenbar Schwierigkeiten macht, zu akzeptieren, daß "Zeit" ein wissenschaftliches, mithin von Menschen gemachtes Modell ist, und Zeiterleben ein Produkt sozialer Prozesse ist.

#25:  Autor: fwoWohnort: im Speckgürtel BeitragVerfasst am: 31.12.2016, 14:40
    —
Marcellinus hat folgendes geschrieben:
zelig hat folgendes geschrieben:
Ein Teil des Streits über die Zeit im uwebus-Thread erübrigt sich vielleicht, wenn man akzeptiert, daß es sowohl die erlebte Zeit gibt, als auch die Zeit als wissenschaftliches Modell.


Und genau da haben wir die Probleme, weil es offenbar Schwierigkeiten macht, zu akzeptieren, daß "Zeit" ein wissenschaftliches, mithin von Menschen gemachtes Modell ist, und Zeiterleben ein Produkt sozialer Prozesse ist.

@ Marcellinus:
Bau doch da mal ein "auch" in Deine Formulierungen, damit ich den Griffel halten kann:
Ein mal abgesehen davon, dass das Zeiterleben bzw. Zeitempfinden auch Bereiche kennt, die kein Produkt sozialer, sondern eines evolutionärer Prozessen sind (die Zeitempfindung ist z.B. für das Abschätzen von Geschwindigkeiten unabdingbar - erkläre mal diese Treffsicherheit ohne Zeitempfindung: https://www.youtube.com/watch?v=9ieWrWLjii0 ) ist auch in deiner Aussage enthalten, dass diese sozialen Prozesse auch zu unterschiedlichen Zeiterlebnissen führen können, und das sogar innerhalb einer Gesellschaft, situationsabhängig sogar innerhalb einer Person. Und bei dem in der Sprache abstrakt handelnden Menschen hat das sogar zu einem Zeitbegriff geführt, der gar nicht mehr direkt vom Zeiterleben abhängt, sondern Teil unserer Vorstellung von dieser Welt ist. Und das schon lange vor Newton.

Vielleicht sollten wir noch eine Ebne höher gehen:
Die natürliche Sprache - und damit auch das Handeln in und mit diesem Vehikel - ist nur möglich aufgrund einer nichtvorhandenen Eineindeutigkeit zwischen Symbol und Bedeutung. Das gilt auch für die Soziologie.

#26:  Autor: zelig BeitragVerfasst am: 31.12.2016, 14:53
    —
Marcellinus hat folgendes geschrieben:
zelig hat folgendes geschrieben:
Ein Teil des Streits über die Zeit im uwebus-Thread erübrigt sich vielleicht, wenn man akzeptiert, daß es sowohl die erlebte Zeit gibt, als auch die Zeit als wissenschaftliches Modell.


Und genau da haben wir die Probleme, weil es offenbar Schwierigkeiten macht, zu akzeptieren, daß "Zeit" ein wissenschaftliches, mithin von Menschen gemachtes Modell ist, und Zeiterleben ein Produkt sozialer Prozesse ist.


Wissenschaftliche Modelle sind ja immer von Menschen gemacht. Das sagt erst mal nichts über den ontologischen Status dessen aus, was modelliert wurde. Man könnte natürlich trotzdem der Auffassung sein, daß Zeit nicht existiert. Nur wird diese Auffassung nicht durch die obige Aussage begründet.

Zeit ist nicht direkt sinnlich erfahrbar. Die Dauer ist im Gegensatz zur räumlichen Ausdehnung flüchtig. Sie lässt sich nicht fassen. Dagegen hat das Urmeter eine scharfe Grenze zum Raum, der es umgibt. Der Raum ist Exklusiv. Wo das Urmeter ist, kann unser Körper nicht sein. Wir können nur mit den Fingern über seine Oberfläche streichen. Wir schließen die Augen, und wenn wir sie wieder öffnen, dann liegt es immer noch da. Das ist alles Intuition. Und die Intuition für die Zeit ist, daß da eigentlich nichts ist - weil wir kein Analogon zur räumlich ausgedehnten Substanz haben, die der Zeit zugrunde liegen könnte.
Nur, gerade die moderne Naturwissenschaft stellt die obige Intuition in Frage. Die Substanz, die das Urmeter ausmacht, verflüchtigt sich ebenso wie die Zeit, je genauer wir hinschauen.

edit: umformulierung

#27:  Autor: Marcellinus BeitragVerfasst am: 31.12.2016, 15:19
    —
zelig hat folgendes geschrieben:
Marcellinus hat folgendes geschrieben:
zelig hat folgendes geschrieben:
Ein Teil des Streits über die Zeit im uwebus-Thread erübrigt sich vielleicht, wenn man akzeptiert, daß es sowohl die erlebte Zeit gibt, als auch die Zeit als wissenschaftliches Modell.

Und genau da haben wir die Probleme, weil es offenbar Schwierigkeiten macht, zu akzeptieren, daß "Zeit" ein wissenschaftliches, mithin von Menschen gemachtes Modell ist, und Zeiterleben ein Produkt sozialer Prozesse ist.

Wissenschaftliche Modelle sind ja immer von Menschen gemacht. Das sagt erst mal nichts über den ontologischen Status dessen aus, was modelliert wurde. Man könnte natürlich trotzdem der Auffassung sein, daß Zeit nicht existiert. Nur wird diese Auffassung nicht durch die obige Aussage begründet.


Nun, wissenschaftliche Modelle sind Modelle, letztlich gedankliche Vorstellungen. Sie dienen der Orientierung in dieser Welt, der leider keine fertige Gebrauchsanweisung beiliegt. Die Frage ist aus meiner Sicht nicht, ob "Zeit" existiert (das ist eine philosophische Frage, keine wissenschaftliche), sondern wie realistisch die Vorstellungen sind, die wir damit verbinden. Einsteins Relativitätstheorie hat da unsere Vorstellungen erweitert, allerdings in den Bereich jenseits unserer Intuition. Für GPS ist das gut, für unser Verständnis nicht. zwinkern

#28:  Autor: zelig BeitragVerfasst am: 31.12.2016, 15:26
    —
Marcellinus hat folgendes geschrieben:
Die Frage ist aus meiner Sicht nicht, ob "Zeit" existiert (das ist eine philosophische Frage, keine wissenschaftliche), sondern wie realistisch die Vorstellungen sind, die wir damit verbinden.


Ah, ok. Dann habe ich dich nicht richtig verstanden. Natürlich ist die soziale Konstruktion Zeit faszinierend, wenn man sich vergegenwärtigt, daß sie sich vollends ausgebreitet hat aufgrund der Notwendigkeit, Fahrpläne für Züge zu erstellen.

#29:  Autor: Zumsel BeitragVerfasst am: 31.12.2016, 15:55
    —
zelig hat folgendes geschrieben:
Zeit ist nicht direkt sinnlich erfahrbar...


Nein, aber jede sinnliche Erfahrung ist zeitlich. Wie schon im anderen Thread gesagt, Kant lohnt sich auch heute noch:

http://www.zeno.org/Philosophie/M/Kant,+Immanuel/Kritik+der+reinen+Vernunft/I.+Transzendentale+Elementarlehre/Erster+Teil%3A+Die+transzendentale+%C3%84sthetik/2.+Abschnitt%3A+Von+der+Zeit

#30:  Autor: Marcellinus BeitragVerfasst am: 31.12.2016, 16:01
    —
fwo hat folgendes geschrieben:

Ein mal abgesehen davon, dass das Zeiterleben bzw. Zeitempfinden auch Bereiche kennt, die kein Produkt sozialer, sondern eines evolutionärer Prozessen sind (die Zeitempfindung ist z.B. für das Abschätzen von Geschwindigkeiten unabdingbar - erkläre mal diese Treffsicherheit ohne Zeitempfindung: https://www.youtube.com/watch?v=9ieWrWLjii0 ) ist auch in deiner Aussage enthalten, dass diese sozialen Prozesse auch zu unterschiedlichen Zeiterlebnissen führen können, und das sogar innerhalb einer Gesellschaft, situationsabhängig sogar innerhalb einer Person. Und bei dem in der Sprache abstrakt handelnden Menschen hat das sogar zu einem Zeitbegriff geführt, der gar nicht mehr direkt vom Zeiterleben abhängt, sondern Teil unserer Vorstellung von dieser Welt ist. Und das schon lange vor Newton.


Nein, das mit der Zwille ist kein "Zeiterleben". Für ein bewußtest Erleben geht das viel zu schnell. Das schafft nur unser Unterbewußtsein, mit viel, viel Übung. Genauso, wie dein Fuß auf der Bremse steht, bevor du noch begriffen hast, was da unerwartet aus der Seitenstraße kommt. Auch da, viel, viel Übung. Bei einem Fahranfänger rumst es da mit Sicherheit. Von angeboren also keine Spur. OK, evolutionär entstanden ist sicherlich unsere Fähigkeit, aus einer Fülle fast gleichartiger Abläufe ein Abschätzung vorzunehmen. Aber frage mal einen Bogenschützen, wie lange es dauert, bis das zuverlässig klappt.

Unsere heutige Vorstellung von dieser Welt, untrennbar gebunden an einen Minuten-, bei manchen sogar an einen Sekundentakt, ist jedenfalls für die Menschen in den Industrienationen allumfassend, so allumfassend, daß es sogar ihre Freizeit strukturiert, sehr zu unser aller Schaden übrigens, als auch historisch noch ziemlich neu.

Deine Einzelbeispiele von Zeitmessungsinstrumenten sprechen da nicht dagegen, sowenig wie das Auffinden einer antiken Batterie ein Beleg für die Elektrifizierung Babylons ist.

Galilei zB verwendete bei seinen Beschleunigungsexperimenten eine Einer, aus dem in einen dünnen Strahl Wasser auslief, das er auffing und wog, gelegentlich auch einfach nur seinen Pulsschlag, aber keine Uhr, die es ja zu seiner Zeit schon gab. In dem Augenblick ist es erst einmal nur der Vergleich von unterschiedlichen physikalischen Abläufen. Zu einer Vorstellung eines, gewissermaßen von den Objekten unabhängigen Zeitflusses, wie bei Newton, ist es da noch ein Weg. Und seit Einstein wissen wir, daß dieser "Zeitfluß" eben nicht unabhängig ist von den Objekten, deren Veränderung ihn bilden. Hier kann man, denke ich, reicht schön beobachten, wie sich das physikalische Modell der Zeit entwickelt und weiterentwickelt hat.

#31:  Autor: zelig BeitragVerfasst am: 31.12.2016, 16:20
    —
Zumsel hat folgendes geschrieben:
Wie schon im anderen Thread gesagt, Kant lohnt sich auch heute noch


Unbedingt!

#32:  Autor: Marcellinus BeitragVerfasst am: 31.12.2016, 17:01
    —
zelig hat folgendes geschrieben:
Zumsel hat folgendes geschrieben:
Wie schon im anderen Thread gesagt, Kant lohnt sich auch heute noch


Unbedingt!


Aber mehr wegen der Fragen, die er gestellt hat, weniger der Antworten wegen. zwinkern

#33:  Autor: Zumsel BeitragVerfasst am: 31.12.2016, 17:17
    —
Marcellinus hat folgendes geschrieben:
zelig hat folgendes geschrieben:
Zumsel hat folgendes geschrieben:
Wie schon im anderen Thread gesagt, Kant lohnt sich auch heute noch


Unbedingt!


Aber mehr wegen der Fragen, die er gestellt hat, weniger der Antworten wegen. zwinkern


Na ja, wenn ich zu wählen hätte zwischen seinen Antworten und dem, was du zu diesem Thema hier auftischst...

#34:  Autor: Marcellinus BeitragVerfasst am: 31.12.2016, 17:36
    —
Zumsel hat folgendes geschrieben:

Na ja, wenn ich zu wählen hätte ...


Mußt du aber nicht! Sehr glücklich

#35:  Autor: fwoWohnort: im Speckgürtel BeitragVerfasst am: 31.12.2016, 23:14
    —
Marcellinus hat folgendes geschrieben:
fwo hat folgendes geschrieben:

Ein mal abgesehen davon, dass das Zeiterleben bzw. Zeitempfinden auch Bereiche kennt, die kein Produkt sozialer, sondern eines evolutionärer Prozessen sind (die Zeitempfindung ist z.B. für das Abschätzen von Geschwindigkeiten unabdingbar - erkläre mal diese Treffsicherheit ohne Zeitempfindung: https://www.youtube.com/watch?v=9ieWrWLjii0 ) ist auch in deiner Aussage enthalten, dass diese sozialen Prozesse auch zu unterschiedlichen Zeiterlebnissen führen können, und das sogar innerhalb einer Gesellschaft, situationsabhängig sogar innerhalb einer Person. Und bei dem in der Sprache abstrakt handelnden Menschen hat das sogar zu einem Zeitbegriff geführt, der gar nicht mehr direkt vom Zeiterleben abhängt, sondern Teil unserer Vorstellung von dieser Welt ist. Und das schon lange vor Newton.


Nein, das mit der Zwille ist kein "Zeiterleben". Für ein bewußtest Erleben geht das viel zu schnell. Das schafft nur unser Unterbewußtsein, mit viel, viel Übung. Genauso, wie dein Fuß auf der Bremse steht, bevor du noch begriffen hast, was da unerwartet aus der Seitenstraße kommt. Auch da, viel, viel Übung. Bei einem Fahranfänger rumst es da mit Sicherheit. Von angeboren also keine Spur. OK, evolutionär entstanden ist sicherlich unsere Fähigkeit, aus einer Fülle fast gleichartiger Abläufe ein Abschätzung vorzunehmen. Aber frage mal einen Bogenschützen, wie lange es dauert, bis das zuverlässig klappt.

Das war jetzt in verschiedener Hinsicht etwas schnell geschossen: Das Schießen mit der Zwille findet komplett bewusst statt. Natürlich braucht es Training, aber dieses Training führt zu Automatisierungen und nicht zu Verschiebungen ins Unterbewusste, was Du schon daran erkennen kannst, dass Du auch automatisiertes Verhalten bewusst verändern kannst, wobei allerdings der Vorteil der Automatisierung, nämlich das höhere Tempo flöten geht. Der zweite Denkfehler besteht darin, dass Du nichts trainieren kannst, was nicht schon da ist: Ich erkenne unterschiedliche Geschwindigkeiten an den unterschiedlichen zeitlichen Abläufen - woran sonst? Das Erkennen von Geschwindigkeiten setzt also die Empfindung von Zeit voraus. (Dieses besondere Beispiel habe ich wegen seiner besonderen Komplexität genommen, die sich aus dem bewegten Objekt in Verbindung mit dem ballistischen Projektil ergibt. Hier erfolgt tatsächlich eine (im Detail natürlich unbewusste) Verrechnung verschiedener Geschwindigkeiten.

Zeit findet ja nicht nur im Stunden-, Tages- oder Jahresmaßstab statt, sondern auch in viel kleineren Zeiträumen. Dass wir sie da wahrnehmen, habe ich damit gezeigt, dass wir Geschwindigkeit wahrnehmen und unterscheiden können - auch relativ hohe Geschwindigkeiten. Das muss ich nicht bei jedem Tier voraussetzen, da viele der Reaktionen, die wir auf Geschwindigkeit beziehen würden, direkt analog leichter steuerbar sind als auf das Abstraktum Geschwindigkeit - das war auch der Grund für mein sehr komplexes Beispiel.

Zeit wird grundsätzlich auch durch den Bezug von vorher zu nachher wahrgenommen. Un zu zeigen, dass auch das keiner sozialen Konstruktion bedarf, sei an das Beispiel der Orangs erinnert, die ein individuelles Wissen erwerben, in welcher Jahreszeit innerhalb eines relativ großen Wandergebietes welche Früchte wo reif werden. Was den Tag angeht, verfügen wir über eine eingebaute "Uhr", das Stichwort dazu ist die circadiane Rhythmik. Wer regelmäig mit Tieren zu tun hat, weiß, dass auch das keine menschliche "Erfindung" ist.

Marcellinus hat folgendes geschrieben:
Unsere heutige Vorstellung von dieser Welt, untrennbar gebunden an einen Minuten-, bei manchen sogar an einen Sekundentakt, ist jedenfalls für die Menschen in den Industrienationen allumfassend, so allumfassend, daß es sogar ihre Freizeit strukturiert, sehr zu unser aller Schaden übrigens, als auch historisch noch ziemlich neu.

Deine Einzelbeispiele von Zeitmessungsinstrumenten sprechen da nicht dagegen, sowenig wie das Auffinden einer antiken Batterie ein Beleg für die Elektrifizierung Babylons ist. ....

Hier ist Deine Argumentation nicht redlich: In dem Beispiel, das Du anführst, ist die Batterie nur eine mögliche Interpretation, die aber selbst, wenn sie richtig ist, nicht auf eine Elekrifizierung Babylons schließen lässt, weil sie selbst auch noch keinen Beleg für ein Verständnis der Elektrizität mit ihren Möglichkeiten darstellt. Wenn diese Interpretation richtig wäre, würde sie allenfalls belegen, dass man in der Lage war, mit dem Phänomen Spannung ein bisschen Schabernack zu treiben.

Das Beispiel, das ich mit den römischen Soldaten gebracht habe, zeigt aber ein Zeitempfinden ähnlich dem unseren nicht als obskuren Einzelfund sondern als gesellschaftliche Realität. Auch die griechischen Diskussionen um ein heliozentrisches Weltbild mit ihrer eigenen Erklärung der jahreszeitlich wechselnden zeigen Sternbilder zeigen - nicht in der Gesellschaft, sondern unter den Diskutierenden*** - einen intellektuellen Zeitbegriff, der von unseren heutigen nicht weit entfernt ist. Gleichzeitig haben wir gesellschaftlich in vielen Bereichen natürlich den Zeitbegriff, den Du als den einzigen betonst - aber nicht immer und überall, wie die römischen Soldaten zeigen.

Es ist wahrscheinlich viel zu stark vereinfacht, davon auszugehen, dass der Begriff Zeit zu einer Zeit und in einer Gesellschaft nur eine Bedeutung hat.

*** mein Beispiel mit dem Umfang der Erde gehört übrigens nicht in diesen Zusammenhang - der wurde über eine Winkeldifferenz des höchsten Sonnenstandes über einen Nord-Süd-Abstand berechnet. Das war also dummes Zeuch. Sorry. Aber der Gesamtzusammenhang ist zu abstrakt, als dass es uns besonders leicht fiele, ihn aus dem Stegreif fehlerfrei zu denken. siehe http://freigeisterhaus.de/viewtopic.php?p=2079267#2079267

#36:  Autor: Marcellinus BeitragVerfasst am: 01.01.2017, 19:24
    —
@fwo
Wir kommen jetzt von "Höcksken auf Stöcksken", wie man bei uns sagt. "Automatisiertes Verhalten" ist aus meiner Sicht unterbewußtes Verhalten, aber ich habe keine Lust, mit dir darüber zu streiten. Ich denke, wir sollten uns in diesem wie in anderen Punkten darauf einigen, daß wir uns nicht einig sind.

#37:  Autor: fwoWohnort: im Speckgürtel BeitragVerfasst am: 01.01.2017, 21:44
    —
Marcellinus hat folgendes geschrieben:
@fwo
Wir kommen jetzt von "Höcksken auf Stöcksken", wie man bei uns sagt. "Automatisiertes Verhalten" ist aus meiner Sicht unterbewußtes Verhalten, aber ich habe keine Lust, mit dir darüber zu streiten. Ich denke, wir sollten uns in diesem wie in anderen Punkten darauf einigen, daß wir uns nicht einig sind.

Sehr glücklich Nichts dagegen - wenn Du diesem Angebot nicht zustimmen kannst:
fwo hat folgendes geschrieben:
....
Es ist wahrscheinlich viel zu stark vereinfacht, davon auszugehen, dass der Begriff Zeit zu einer Zeit und in einer Gesellschaft nur eine Bedeutung hat.....

#38:  Autor: der kleine FritzWohnort: Planet Erde BeitragVerfasst am: 01.01.2017, 22:10
    —
fwo hat folgendes geschrieben:

Zeit wird grundsätzlich auch durch den Bezug von vorher zu nachher wahrgenommen.


Und mit welchem unserer Sinne wird die Zeit wahrgenommen, nach welchem Grundsatz? Mit den Augen rollen
Was wird denn da wahrgenommen?

#39:  Autor: fwoWohnort: im Speckgürtel BeitragVerfasst am: 01.01.2017, 23:01
    —
der kleine Fritz hat folgendes geschrieben:
fwo hat folgendes geschrieben:

Zeit wird grundsätzlich auch durch den Bezug von vorher zu nachher wahrgenommen.


Und mit welchem unserer Sinne wird die Zeit wahrgenommen, nach welchem Grundsatz? Mit den Augen rollen
Was wird denn da wahrgenommen?

Vielleicht möchtest Du mir diese Sache ja etwas erleichtern, indem Du mir erkläfrst, wie genau wir das machen, dass wir Raum wahrnehmen. Das ist ja anscheinend ein unstrittiges Thema.

Nachdem ich das verstanden habe, werde ich dann versuchen, das Analoge zu Zeit zu formulieren.

#40:  Autor: der kleine FritzWohnort: Planet Erde BeitragVerfasst am: 02.01.2017, 10:12
    —
fwo hat folgendes geschrieben:
der kleine Fritz hat folgendes geschrieben:
fwo hat folgendes geschrieben:

Zeit wird grundsätzlich auch durch den Bezug von vorher zu nachher wahrgenommen.


Und mit welchem unserer Sinne wird die Zeit wahrgenommen, nach welchem Grundsatz? Mit den Augen rollen
Was wird denn da wahrgenommen?


Vielleicht möchtest Du mir diese Sache ja etwas erleichtern, indem Du mir erkläfrst, wie genau wir das machen, dass wir Raum wahrnehmen. Das ist ja anscheinend ein unstrittiges Thema.



Nein, wenn du zum erleichtern Raum brauchst, such dir eine kleine Hütte! und versuche dort meine Frage zur Zeit zu beantworten!! Lachen

Zu deiner Information: "Raum ist in der kleinsten Hütte!"

#41:  Autor: fwoWohnort: im Speckgürtel BeitragVerfasst am: 02.01.2017, 13:39
    —
der kleine Fritz hat folgendes geschrieben:
fwo hat folgendes geschrieben:
der kleine Fritz hat folgendes geschrieben:
fwo hat folgendes geschrieben:

Zeit wird grundsätzlich auch durch den Bezug von vorher zu nachher wahrgenommen.


Und mit welchem unserer Sinne wird die Zeit wahrgenommen, nach welchem Grundsatz? Mit den Augen rollen
Was wird denn da wahrgenommen?


Vielleicht möchtest Du mir diese Sache ja etwas erleichtern, indem Du mir erkläfrst, wie genau wir das machen, dass wir Raum wahrnehmen. Das ist ja anscheinend ein unstrittiges Thema.



Nein, wenn du zum erleichtern Raum brauchst, such dir eine kleine Hütte! und versuche dort meine Frage zur Zeit zu beantworten!! Lachen

Zu deiner Information: "Raum ist in der kleinsten Hütte!"

zwinkern
Ein Teil dessen, was ich Dir auf diese Frage im anderen Thread geantwortet habe, ist mir tatsächlich beim Scheißen durch den Kopf gegangen. Ich hoffe, die Qualität ist eine andere als die des Materials, das mir in dieser Zeit durch den Anus ging. Aber derartige geistige Vorarbeit in zeitlich eigentlich anders gewidmeten Momenten erlauben es dann, auch ziemlich komplexe Zusammenhänge einmal relativ flüssig zwischendurch zu formulieren.

#42:  Autor: fwoWohnort: im Speckgürtel BeitragVerfasst am: 02.01.2017, 16:35
    —
Marcellinus hat folgendes geschrieben:
zelig hat folgendes geschrieben:
Ein Teil des Streits über die Zeit im uwebus-Thread erübrigt sich vielleicht, wenn man akzeptiert, daß es sowohl die erlebte Zeit gibt, als auch die Zeit als wissenschaftliches Modell.


Und genau da haben wir die Probleme, weil es offenbar Schwierigkeiten macht, zu akzeptieren, daß "Zeit" ein wissenschaftliches, mithin von Menschen gemachtes Modell ist, und Zeiterleben ein Produkt sozialer Prozesse ist.

@ Marcellinus:

Nachdem ich das Ganze bei den unterschiedlichsten Tätigkeiten doch noch ein paar Mal durch meinen Kopf bewegt habe (siehe die Dialoge mit dem kleinen Fritz), muss ich dann doch noch mal nachfragen:

Es ist klar, dass die Termine früher von anderen Göttern als dem Umsatz und dem Profit gesetzt wurden, das waren da die Sonne mit den Sonnenwenden, Zeus, der Regengott oder die Wanderung der Lachse. Aber wie stellst Du Dir ohne einen Zeitbegriff vor, dass man diese Termine, deren Begehungen ja z.T. mit erheblichem zu planenden Aufwand verbunden war, trotz z.T. langwieriger Vorbereitung einhalten konnte?

Wie stellst Du Dir vor, dass Alexander der Große seine Schlachten ohne Zeitplan planen konnte, oder wie stellst Du Dir nur die bereits hochgradig arbeitsteilige Gesellschaft des alten Griechenland mit Wochenmarkt und allem drum und dran ohne ein zeitlich bestimmtes Management vor, von termingebundenen Auftragsarbeiten der Handwerker (z.B. zu den religiösen Festen) ganz zu schweigen. Das alles bedurfte nicht nur einer Denkarbeit zum Thema Zeit, sondern die musste auch kommuniziert werden.

Ich kann mir das alles ohne einen Zeitbegriff, der unserem ziemlich ähnlich war, nicht vorstellen, Du anscheinend schon. Wie hat das funktioniert?

#43:  Autor: smallie BeitragVerfasst am: 02.01.2017, 18:58
    —
zelig hat folgendes geschrieben:
Zeit ist nicht direkt sinnlich erfahrbar. Die Dauer ist im Gegensatz zur räumlichen Ausdehnung flüchtig. Sie lässt sich nicht fassen.

Meine Empfindungen, Bilder und Gedanken dazu sagen das Gegenteil.

An erster Stelle natürlich: atmen. Ich sitze hier und atme ein, aus, Pause, ein, aus, ... Das ist für mich als sinnliche Erfahrung unmittelbar mit dem Erleben von Zeit verbunden.


Zur Dauer:

Zitat:
Li Bai (701-762)

Wir sitzen zusammen, der Berg und ich,
bis nur der Berg übrigbleibt.



https://en.wikiquote.org/wiki/Li_Bai


(Das Zitat kenne ich aus Sid Meier's AlphaCentauri, nicht daß der Eindruck entsteht, ich sei in altchinesischer Poesie bewandert.)



zelig hat folgendes geschrieben:
Und die Intuition für die Zeit ist, daß da eigentlich nichts ist - weil wir kein Analogon zur räumlich ausgedehnten Substanz haben, die der Zeit zugrunde liegen könnte.

Wirklich nicht? Drei Beispiele sind so augenfällig, daß sie sogar zur Basis von Zeiteinheiten wurden.

Die Dauer der Erddrehung, die Umlaufzeit des Mondes um die Erde und die Umlaufzeit der Erde um die Sonne.

Die historischen oder prähistorischen Bauten zur Bestimmung von Tag-und-Nacht-Gleiche und Sonnwende zeugen davon, daß Menschen schon lange einen Sinn für Zeit hatten und obendrein in der Lage waren, Zeit mit Hilfe von Apparaten "dingfest" zu machen.

#44:  Autor: Marcellinus BeitragVerfasst am: 02.01.2017, 21:34
    —
fwo hat folgendes geschrieben:
Marcellinus hat folgendes geschrieben:
zelig hat folgendes geschrieben:
Ein Teil des Streits über die Zeit im uwebus-Thread erübrigt sich vielleicht, wenn man akzeptiert, daß es sowohl die erlebte Zeit gibt, als auch die Zeit als wissenschaftliches Modell.


Und genau da haben wir die Probleme, weil es offenbar Schwierigkeiten macht, zu akzeptieren, daß "Zeit" ein wissenschaftliches, mithin von Menschen gemachtes Modell ist, und Zeiterleben ein Produkt sozialer Prozesse ist.

@ Marcellinus:

Nachdem ich das Ganze bei den unterschiedlichsten Tätigkeiten doch noch ein paar Mal durch meinen Kopf bewegt habe (siehe die Dialoge mit dem kleinen Fritz), muss ich dann doch noch mal nachfragen:

Es ist klar, dass die Termine früher von anderen Göttern als dem Umsatz und dem Profit gesetzt wurden, das waren da die Sonne mit den Sonnenwenden, Zeus, der Regengott oder die Wanderung der Lachse. Aber wie stellst Du Dir ohne einen Zeitbegriff vor, dass man diese Termine, deren Begehungen ja z.T. mit erheblichem zu planenden Aufwand verbunden war, trotz z.T. langwieriger Vorbereitung einhalten konnte?

Wie stellst Du Dir vor, dass Alexander der Große seine Schlachten ohne Zeitplan planen konnte, oder wie stellst Du Dir nur die bereits hochgradig arbeitsteilige Gesellschaft des alten Griechenland mit Wochenmarkt und allem drum und dran ohne ein zeitlich bestimmtes Management vor, von termingebundenen Auftragsarbeiten der Handwerker (z.B. zu den religiösen Festen) ganz zu schweigen. Das alles bedurfte nicht nur einer Denkarbeit zum Thema Zeit, sondern die musste auch kommuniziert werden.

Ich kann mir das alles ohne einen Zeitbegriff, der unserem ziemlich ähnlich war, nicht vorstellen, Du anscheinend schon. Wie hat das funktioniert?


Was fragst du mich das, wo du doch die Antwort schon hast? Du hast doch schon entschieden, daß unser Zeitbegriff ihr Zeitbegriff war (oder, wie du so schön schreibst, „ziemlich ähnlich“ war, was immer das heißen mag). Daher habe ich geschrieben:

Marcellinus hat folgendes geschrieben:
@fwo
Ich denke, wir sollten uns in diesem wie in anderen Punkten darauf einigen, daß wir uns nicht einig sind.


Was erwartest du also von mir? Genauso gut könnte ich jemandem zu erklären suchen, wie die Pyramiden ohne Kran und Bagger gebaut worden sind, der das nicht für möglich hält. Wir könnten es nicht. Richtig! Wir haben für alles einen genauen Zeitplan - um ihn dann nicht einzuhalten. Wann wird der BER noch mal fertig?

Was würde es nützen, wenn ich dir sage, daß im Altertum Kriege maximal auf Jahreszeiten genau geplant wurden (und Generale auch heute wissen, oder wissen sollten, daß kein Kriegsplan den ersten Schuss übersteht), was, wenn ich dir sage, daß die attischen Bauern keine genaueren Zeitpläne kannten als eben diese Jahreszeiten, und eben, wie gesagt, die Termine besonderer Feiertage? Würdest du mir glauben, daß man schon Tage vorher anreiste, mitnahm, was man hatte, so wie es die Eltern und Großeltern schon getan hatten (was hätte man sonst tun sollten?).

Du gehst davon aus, daß der Zeiterleben der Menschen sich seit der Antike nicht geändert hat, und „praktisch dem Zeitbegriff der heutigen Physik“ entsprach und entspricht.

Ich bin dagegen der Ansicht, daß der Zeitbegriff der heutigen Physik etwas ist, was sich unter wenigen Spezialisten über die letzten Jahrhunderte entwickelt hat (mit unzweifelhaften Vorläufern in der Antike), mit dem Ziel einer Modellvorstellung, die es erlaubt unterschiedliche physikalische Abläufe zeitlich vergleichen zu können. Das Beispiel von Galilei hatte ich erwähnt. Der letzte Höhepunkt dieser Entwicklung ist die Relativitätstheorie Einsteins. Sie zeigt in ihrer Unanschaulichkeit übrigens auch die Distanz zu unserem Zeiterleben, eine Distanz die es meiner Ansicht immer gegeben hat, ziemlich sicher übrigens auch im persönlichen Leben eben dieser Physiker.

Unser heutiges Zeiterleben ist etwas, das wir unsere Kindern von Kindesbeinen mühsam beibringen, und das nicht denkbar ist ohne die Allgegenwart von Uhren, in jeder Wohnung, an fast jedem Handgelenk, und nie mehr als einen Augenblick entfernt. Es ist in unserem Habitus derartig fest verankert, daß die meisten von uns es für die natürliche Art halten, diese Welt zu erleben, ist aber in dieser Totalität noch keine 150 Jahre alt. Die Stichworte sind Eisenbahnen, Industrialisierung und Fließbandarbeit (du kennst den Film „Modern Times“?).

Aber was hat das alles für einen Sinn? Du bist fest davon überzeugt, daß unser physikalischer Zeitbegriff unveränderlich seit den ersten Hochkulturen existiert, und unser Zeiterleben (wie auch das meiste unseres sonstigen Habitus, wie wir beide aus anderen Diskussionen wissen) quasi angeboren ist. Dabei ist es ziemlich unerheblich, ob es sich dabei um ein Kant’sches a-priori handelt, oder ein Produkt der Evolution. Der Homo Sapiens schaut nach dieser Vorstellung seit den ersten Tagen in Afrika vor 70.000 Jahren mit den gleichen (oder „ziemlich ähnlichen“) Augen auf diese Welt wie wir heute. Alles, was man über diesen Sapiens herausfinden könne, finde man daher auch an jedem Einzelnen von uns heute, abgesehen vielleicht von einem Mehr an Wissen.

Ich bin dagegen der Ansicht, daß sich seitdem nicht nur unser Wissen vermehrt hat, unsere Modellvorstellungen von dieser Welt, sondern auch unsere Einstellungen, Wertungen, Gefühle, unser ganzes Erleben, unser sozialer Habitus. Sozialer Habitus, weil man meiner Ansicht nach Menschen nicht verstehen kann, wenn man sie nicht im Plural sieht, nicht in den Gesellschaften, die sie miteinander bilden. Und Gesellschaften sind Geflechte wechselseitiger Abhängigkeiten, die sich teilweise über Jahrhunderte entwickelt haben, ungeplant, aber strukturiert und in die jeder Mensch irgendwann hineingewachsen ist. Nicht nur diese Gesellschaften haben sich entwickelt, sondern damit auch die einzelnen Menschen, die sie bilden. Das wird meistens übersehen.

Ich möchte diese Entwicklungen etwas besser verstehen, Entwicklungen, die, wenn ich das richtig sehe, aus deiner Sicht überhaupt nicht stattgefunden haben. Du suchst hinter den Veränderung das Unveränderliche, ich versuche, die Art und die Mechanismen der Veränderungen zu verstehen. Da sind Missverständnisse unvermeidlich, um es vorsichtig zu formulieren.

#45:  Autor: Zumsel BeitragVerfasst am: 03.01.2017, 11:06
    —
Marcellinus hat folgendes geschrieben:
Aber was hat das alles für einen Sinn? Du bist fest davon überzeugt, daß unser physikalischer Zeitbegriff unveränderlich seit den ersten Hochkulturen existiert, und unser Zeiterleben (wie auch das meiste unseres sonstigen Habitus, wie wir beide aus anderen Diskussionen wissen) quasi angeboren ist. Dabei ist es ziemlich unerheblich, ob es sich dabei um ein Kant’sches a-priori handelt, oder ein Produkt der Evolution. Der Homo Sapiens schaut nach dieser Vorstellung seit den ersten Tagen in Afrika vor 70.000 Jahren mit den gleichen (oder „ziemlich ähnlichen“) Augen auf diese Welt wie wir heute. Alles, was man über diesen Sapiens herausfinden könne, finde man daher auch an jedem Einzelnen von uns heute, abgesehen vielleicht von einem Mehr an Wissen.


Stell doch mal ein Modell unserer alltäglichen Welt vor, das ohne linearen Zeitbegriff auskommt, das keine Annahmen über Nacheinander, Gleichzeitigkeit etc. beinhaltet und trotzdem praxistauglich ist. Denn wenn eine solche "Sicht in die Welt" möglich ist, muss sie sich ja zumindest theoretisch beschreiben lassen. Ansonsten ist das alles nur Blabla.

#46:  Autor: der kleine FritzWohnort: Planet Erde BeitragVerfasst am: 03.01.2017, 11:40
    —
smallie hat folgendes geschrieben:
zelig hat folgendes geschrieben:
Zeit ist nicht direkt sinnlich erfahrbar. Die Dauer ist im Gegensatz zur räumlichen Ausdehnung flüchtig. Sie lässt sich nicht fassen.



zelig hat folgendes geschrieben:
Und die Intuition für die Zeit ist, daß da eigentlich nichts ist - weil wir kein Analogon zur räumlich ausgedehnten Substanz haben, die der Zeit zugrunde liegen könnte.


Wirklich nicht? Drei Beispiele sind so augenfällig, daß sie sogar zur Basis von Zeiteinheiten wurden.

Die Dauer der Erddrehung, die Umlaufzeit des Mondes um die Erde und die Umlaufzeit der Erde um die Sonne.



Diese Beispiele stehen nicht für eine Wahrnehmung bezw, Existenz von (unsichtbarer!) Zeit, sondern was du tatsächlich wahrnehmen kannst sind unterschiedliche Bewegungen von materiellen Objekten!

Zur Basis von willkürlichen Zeiteinheiten wurden diese Bewegungen aber erst durch uns genormt, um die Dauer von Bewegungen anschaulich und mathematisch darstellen zu können.

Newton hat es ja ähnlich formuliert;

„Und die Zeit ist keine Sache sondern ein "Maß der Dauer", das selbst nicht meßbar ist sondern definiert werden muß. Maße existieren nicht sondern sie gelten, sind also etwas vom Geist gesetztes…..“
Das wäre zu widerlegen

#47:  Autor: Marcellinus BeitragVerfasst am: 03.01.2017, 11:57
    —
Zumsel hat folgendes geschrieben:

Stell doch mal ein Modell unserer alltäglichen Welt vor, das ohne linearen Zeitbegriff auskommt, ... Ansonsten ist das alles nur Blabla.


Hat jemand, außer dir, von einem "nichtlinearen" Zeitbegriff gesprochen? Also blabla ...

#48:  Autor: Zumsel BeitragVerfasst am: 03.01.2017, 12:07
    —
Marcellinus hat folgendes geschrieben:
Zumsel hat folgendes geschrieben:

Stell doch mal ein Modell unserer alltäglichen Welt vor, das ohne linearen Zeitbegriff auskommt, ... Ansonsten ist das alles nur Blabla.


Hat jemand, außer dir, von einem "nichtlinearen" Zeitbegriff gesprochen? Also blabla ...


Was unterschied denn hinsichtlich des Zeitverständnisses den Blick des Steinzeitmenschen in die Welt so fundamental von dem unsrigen- außer, dass er keine Fahrpläne im Kopf haben musste?

#49:  Autor: fwoWohnort: im Speckgürtel BeitragVerfasst am: 03.01.2017, 12:19
    —
Marcellinus hat folgendes geschrieben:
....
Du gehst davon aus, daß der Zeiterleben der Menschen sich seit der Antike nicht geändert hat, und „praktisch dem Zeitbegriff der heutigen Physik“ entsprach und entspricht.
....

Nein, das tue ich nicht, und das habe ich auch nicht geschrieben.
Ich gehe allerdings auch nicht davon aus, dass die Mehrheit der heutigen Menschen etwas mit dem Zeitbegriff der heutigen Physik anzufangen weiß, nur weil sie sich nach technisch höchst aufwändigen Zeitgebern richten.
Was ich geschrieben habe war eine Antwort hierauf:
Marcellinus hat folgendes geschrieben:
....
Und genau da haben wir die Probleme, weil es offenbar Schwierigkeiten macht, zu akzeptieren, daß "Zeit" ein wissenschaftliches, mithin von Menschen gemachtes Modell ist ....

Was ich dann gemacht habe, war zu zeigen, dass nicht nur wir, sondern sogar ein Orang sein Verhalten über das Jahr zeitlich zu steuern in der Lage ist, allerdings kann er das nur aus eigenen Erfahrungen, und insgesamt, dass es auch Bereiche der zeitlichen Wahrnehmung gibt, die wenig mit sozialer Konstruktion zu tun haben.

Das besondere am Menschen ist aber, dass er sich zeitlich verabreden kann und das auch getan haben muss, seit er die sprachlichen Fähigkeiten dazu hat - ich habe auch dazu Beispiel gebracht, in denen das sehr früh geklappt haben muss - sonst wären wir nicht, wo wir heute sind.

Dann habe ich festgestellt, dass die arbeitsteilige Gesellschaft damit gleichzeitig eine zeitliche Organisation besessen haben muss, die weit über die der Jäger und Sammler hinausgegangen ist.

Das hat alles sehr wenig mit Wissenschaft und dem Zeitbegriff der heutigen Physik zu tun, sondern zeigt nur, dass in den Gesellschaften ein Zeitbegriff existiert haben muss, der das alles möglich gemacht hat, ein Zeitbegriff, der nicht nur eine zeitliche Reihung erlaubte, sondern auch in der Lage war, nähere (nach heutigem Maßstab innerhalb von Minuten oder Stunden) und entferntere (innerhalb von Monaten) zukünftige Zeitpunkte in der Kommunikation zu adressieren.

Es würde allerdings tatsächlich wenig nützen, mir zu sagen daß im Altertum Kriege maximal auf Jahreszeiten genau geplant wurden - es wäre mir nämlich nicht neu, und wenn Du genau liest, habe ich von der Planung einzelner Schlachten geschrieben, und nicht von der ganzer Kriegszüge - also nichts, was dieser Aussage von Dir widersprochen hätte. Bei meiner Formulierung geht es um Stunden, maximal um Tage.

Wo ich Dir dann wieder widersprochen habe, war in der Wissenschaft, die sich nachweislich schon seit den Ägyptern um die Präzisierung der Zeitmessung bemüht hat. Und was wir hierzu gefunden haben, sind eben keine Objekte, in die man etwas hineininterpretieren muss; wir vermuten die Funktion dieser Geräte nicht nur, wir kennen auch die wissenschaftlichen Dialoge dazu, genauso wie wir die Dialoge um die praktische Zeitmessung bei den römischen Soldaten, also mitten in der Gesellschaft kennen.

Meine Neugierde bezüglich der Veränderung ist wahrscheinlich kaum kleiner als Deine, allerdings vermute ich ihre Ursache woanders: Weniger im Begriff der Zeit, als im geänderten Stellenwert, den die Zeit für uns hat. Die größere Abhängigkeit von der äußeren Taktung liegt für mich aber eher an der abnehmenden Kompetenz den Einzelnen für den Gesamtprozess, der Hebel, der den Menschen zum Gut macht, das nach den selben Regeln bewirtschaftet wird wie Maschinen, ist die kurzfristige Ersetzbarkeit. Denn im Prinzip gab es auch die äußere Taktung des Einzelnen wie am heutigen Fließband schon früher: In Ketten an den Rudern wurde den Menschen der Schlag schon so hart vorgegeben wie uns heute. Aber versuch mal einen Schmied unter Druck zu setzen, der x Jahre gelernt hat, um so gut zu sein, wie er heute ist, und von dem bekannt ist, dass es im Umkreis von 10 Tagesreisen niemanden gibt, der ihn ersetzen kann. Auch der Weizen wächst nicht schneller, wenn ich die Uhr schneller laufen lasse. Diese Leute konntest Du zwar brutal unterdrücken, aber jeder Versuch einer Beschleunigung wäre kontraproduktiv gewesen.

Deshalb: Wenn ich etwas lernen soll - dagegen habe ich ja nichts, im Gegenteil, nur deshalb diskutiere ich überhaupt mit - beschwere Dich bitte nicht über meine Uneinsichtigkeit, sondern zeige mir einfach in der Sache, wo ich falsch liege. Aber zeigen bedeutet nicht, nur einfach etwas anderes zu behaupten.

#50:  Autor: fwoWohnort: im Speckgürtel BeitragVerfasst am: 03.01.2017, 12:36
    —
der kleine Fritz hat folgendes geschrieben:
smallie hat folgendes geschrieben:
zelig hat folgendes geschrieben:
Zeit ist nicht direkt sinnlich erfahrbar. Die Dauer ist im Gegensatz zur räumlichen Ausdehnung flüchtig. Sie lässt sich nicht fassen.



zelig hat folgendes geschrieben:
Und die Intuition für die Zeit ist, daß da eigentlich nichts ist - weil wir kein Analogon zur räumlich ausgedehnten Substanz haben, die der Zeit zugrunde liegen könnte.


Wirklich nicht? Drei Beispiele sind so augenfällig, daß sie sogar zur Basis von Zeiteinheiten wurden.

Die Dauer der Erddrehung, die Umlaufzeit des Mondes um die Erde und die Umlaufzeit der Erde um die Sonne.



Diese Beispiele stehen nicht für eine Wahrnehmung bezw, Existenz von (unsichtbarer!) Zeit, sondern was du tatsächlich wahrnehmen kannst sind unterschiedliche Bewegungen von materiellen Objekten!

Zur Basis von willkürlichen Zeiteinheiten wurden diese Bewegungen aber erst durch uns genormt, um die Dauer von Bewegungen anschaulich und mathematisch darstellen zu können.

Newton hat es ja ähnlich formuliert;

„Und die Zeit ist keine Sache sondern ein "Maß der Dauer", das selbst nicht meßbar ist sondern definiert werden muß. Maße existieren nicht sondern sie gelten, sind also etwas vom Geist gesetztes…..“
Das wäre zu widerlegen

Nein, das wäre nicht zu widerlegen, sondern dass Einheiten willkürlich gesetzt werden müssen, ist in der Wissenschaft heute ein Allgemeinplatz. Nicht nur für die Zeit, sondern auch für alle anderen Größen. Aber das hatten wir bereits.

#51:  Autor: smallie BeitragVerfasst am: 04.01.2017, 20:03
    —
Zumsel erwähnte, daß bereits die alten Griechen über Zeit nachgedacht haben. Kurzer Exkurs zu den Zeitvorstellungen von Zenon und Newton:


Zenon und der Pfeil

Betrachte einen fliegenden Pfeil zur Zeit t. In diesem einen Moment läßt sich der Pfeil nicht von einem ruhenden unterscheiden. Also kann der Pfeil gar nicht fliegen.

Etwas moderner gesagt: in 0 Sekunden legt der Pfeil 0 Meter zurück, seine Geschwindigkeit ist 0/0, also unbestimmt.



Newton und die Infinitesimalen

Netwon hat die Flexionenrechnung entwickelt, um veränderliche Größen besser beschreiben zu können. Modern gesagt betrachtet er den Grenzwert von delta x / delta t für t gegen 0. Der Wert ist dann nicht mehr unbestimmt, wie bei Zenon.

Eng damit verbunden ist die Impulserhaltung, die in den Newtonschen Gesetzen steckt. actio = reactio und Ohne Krafteinwirkung keine Geschwindigkeitsänderung: F = m * a. Sehr salopp gesagt passt auch Zenon wieder, wenn ich mir den in der Luft stehenden Pfeil mit einem Impuls versehen denke. Die Vorstellung einer Erhaltungsgröße fehlte den Griechen.

#52:  Autor: smallie BeitragVerfasst am: 04.01.2017, 20:47
    —
der kleine Fritz hat folgendes geschrieben:
smallie hat folgendes geschrieben:
zelig hat folgendes geschrieben:
Zeit ist nicht direkt sinnlich erfahrbar. Die Dauer ist im Gegensatz zur räumlichen Ausdehnung flüchtig. Sie lässt sich nicht fassen.



zelig hat folgendes geschrieben:
Und die Intuition für die Zeit ist, daß da eigentlich nichts ist - weil wir kein Analogon zur räumlich ausgedehnten Substanz haben, die der Zeit zugrunde liegen könnte.


Wirklich nicht? Drei Beispiele sind so augenfällig, daß sie sogar zur Basis von Zeiteinheiten wurden.

Die Dauer der Erddrehung, die Umlaufzeit des Mondes um die Erde und die Umlaufzeit der Erde um die Sonne.



Diese Beispiele stehen nicht für eine Wahrnehmung bezw, Existenz von (unsichtbarer!) Zeit, sondern was du tatsächlich wahrnehmen kannst sind unterschiedliche Bewegungen von materiellen Objekten!

Jetzt habe ich den Faden verloren. Wofür oder wogegen ist das ein Argument?

Natürlich steht das für die Existenz von Zeit, wenn sich etwas verändert. Ohne Zeit keine Veränderung. Ohne Zeit keine Wahrnehmung.

Würdest du so weit gehen und sagen, die Schwerkraft ist unsichtbar, denn tatsächlich sehen wir nur wie Gegenstände zu Boden fallen? Das ist zwar auf eine gewisse Weise richtig, auf eine andere Weise aber sehr spitzfindig.

Besser so: daß es Zeit gibt ist offensichtlich. Daß sich Zeit nicht so absolut und linear verhält, wie es sich unser Primatenhirn vorstellt, ist eine neue Erkenntnis.



der kleine Fritz hat folgendes geschrieben:
Zur Basis von willkürlichen Zeiteinheiten wurden diese Bewegungen aber erst durch uns genormt, um die Dauer von Bewegungen anschaulich und mathematisch darstellen zu können.

Willkürlich?

Dann hätte man den Tag auch zu 2/5 oder 8/6 Erddrehungen festlegen können. Oder das Jahr zu 100 oder 1000 Erddrehungen. Das wäre willkürlich. Stattdessen wurden immer ausgefeiltere Kalender entwickelt, mit Schaltjahren und neuerdings Schaltsekunden, um den Kalender synchron mit der astronomischen Zeit zu halten. Das ist alles andere als willkürlich.

Willkürlich ist allerdings die Einteilung in Stunden, Minuten und Sekunden. Das war ein historischer Zufall, weil die alten Sumerer und Babylonier im 12er-System zählten. Darum die Uhr mit 12 Stunden, 60 Minuten und 60 Sekunden.



der kleine Fritz hat folgendes geschrieben:
Newton hat es ja ähnlich formuliert;

„Und die Zeit ist keine Sache sondern ein "Maß der Dauer", das selbst nicht meßbar ist sondern definiert werden muß. Maße existieren nicht sondern sie gelten, sind also etwas vom Geist gesetztes…..“
Das wäre zu widerlegen

Daß Zeit keine Sache ist, möchte ich gar nicht widerlegen.

Ansonsten lassen sich natürlich viele Newton-Zitate anführen.

Zitat:
Es gibt eine objektiv existierende, unabhängig von allen Geschehnissen verlaufende Zeit. Alle Körper bewegen sich in einem objektiv existierenden Raum, der unabhängig is von allen darin befindlichen Körpern. Alle veränderlichen Größen sind physikalische Größen, die von der objektiv ablaufenden Zeit abhängen.

http://www.studentshelp.de/p/referate/02/822.htm




PS:

Mit Einstein gedacht, ist das mit der Länge auch nicht so einfach. Ein bewegter Beobachter stellt für das Urmeter nämlich eine andere Länge als 1 m fest. Zustimmung

#53:  Autor: der kleine FritzWohnort: Planet Erde BeitragVerfasst am: 05.01.2017, 20:04
    —
smallie hat folgendes geschrieben:
der kleine Fritz hat folgendes geschrieben:
smallie hat folgendes geschrieben:
zelig hat folgendes geschrieben:
Zeit ist nicht direkt sinnlich erfahrbar. Die Dauer ist im Gegensatz zur räumlichen Ausdehnung flüchtig. Sie lässt sich nicht fassen.



zelig hat folgendes geschrieben:
Und die Intuition für die Zeit ist, daß da eigentlich nichts ist - weil wir kein Analogon zur räumlich ausgedehnten Substanz haben, die der Zeit zugrunde liegen könnte.


Wirklich nicht? Drei Beispiele sind so augenfällig, daß sie sogar zur Basis von Zeiteinheiten wurden.

Die Dauer der Erddrehung, die Umlaufzeit des Mondes um die Erde und die Umlaufzeit der Erde um die Sonne.



Diese Beispiele stehen nicht für eine Wahrnehmung bezw, Existenz von (unsichtbarer!) Zeit, sondern was du tatsächlich wahrnehmen kannst sind unterschiedliche Bewegungen von materiellen Objekten!

Jetzt habe ich den Faden verloren. Wofür oder wogegen ist das ein Argument?

Natürlich steht das für die Existenz von Zeit, wenn sich etwas verändert. Ohne Zeit keine Veränderung. Ohne Zeit keine Wahrnehmung.

Würdest du so weit gehen und sagen, die Schwerkraft ist unsichtbar, denn tatsächlich sehen wir nur wie Gegenstände zu Boden fallen? Das ist zwar auf eine gewisse Weise richtig, auf eine andere Weise aber sehr spitzfindig.

Besser so: daß es Zeit gibt ist offensichtlich. Daß sich Zeit nicht so absolut und linear verhält, wie es sich unser Primatenhirn vorstellt, ist eine neue Erkenntnis.



Das sind zunächst nur Behauptungen, die des Beweises und der Erklärung bedürfen.

Also welche Eigenschaft der Zeit vollbringt wie den physikalischren Vorgang, Einfluss auf den Aufenthaltsort von Materie im Raum zu verändern?
(Ohne Zeit keine Veränderung.)

Und in welcher Form nimmt Zeit Einfluss darauf, dass du etwas sehen, hören, schmecken, fühlen oder riechen kannst – also eine Wahrnehmung empfindest?
(Ohne Zeit keine Wahrnehmung.)

Was deinen Vergleich mit der unsichtbaren Schwerkraft anbelangt, so siehst du ja beim fallenden Apfel eine „offensichtliche“ Wirkung. Welche Wirkung ist aber nach deiner Behauptung bei der Zeit offensichtlich?

#54:  Autor: Kramer BeitragVerfasst am: 06.01.2017, 01:07
    —
Hallo kleiner Fritz,

ich denke, das passt besser in diesen Thread:

der kleine Fritz hat folgendes geschrieben:

Aber sag mir bitte noch,
wie lange dauert denn das "Jetzt"?

mfG


Kannst Du mir sagen, wie gross "hier" ist?

SCNR

Cool

#55:  Autor: Kramer BeitragVerfasst am: 06.01.2017, 01:09
    —
zelig hat folgendes geschrieben:
Zeit ist nicht direkt sinnlich erfahrbar.


Findest Du? Ich habe irgendwo mal gelesen: "Malerei ist Kunst in der Fläche. Bildhauerei ist Kunst im Raum. Musik ist Kunst in der Zeit."

#56:  Autor: fwoWohnort: im Speckgürtel BeitragVerfasst am: 06.01.2017, 03:35
    —
der kleine Fritz hat folgendes geschrieben:
....
Was deinen Vergleich mit der unsichtbaren Schwerkraft anbelangt, so siehst du ja beim fallenden Apfel eine „offensichtliche“ Wirkung. Welche Wirkung ist aber nach deiner Behauptung bei der Zeit offensichtlich?

Welch kühne Behauptung.
Erklär doch bitte mal genau welche Eigenschaft der Schwerkraft es macht, dass der Apfel fällt,
und wie die das macht.

btw: Wie kommst Du nur auf das schräge Brett, dass die Zeit an der Materie zerbeln müsste? Das braucht sie doch gar nicht, weil der Raum schon an der zerrt . denn ohne Raum ist schließlich auch keine Bewegung möglich.

Aber da Du die Frage nicht gestellt hast, wie der Raum das macht, kannst D uns das wohl erklären.

#57:  Autor: der kleine FritzWohnort: Planet Erde BeitragVerfasst am: 06.01.2017, 08:59
    —
fwo hat folgendes geschrieben:
der kleine Fritz hat folgendes geschrieben:
....
Was deinen Vergleich mit der unsichtbaren Schwerkraft anbelangt, so siehst du ja beim fallenden Apfel eine „offensichtliche“ Wirkung. Welche Wirkung ist aber nach deiner Behauptung bei der Zeit offensichtlich?

Welch kühne Behauptung.
Erklär doch bitte mal genau welche Eigenschaft der Schwerkraft es macht, dass der Apfel fällt,
und wie die das macht.

btw: Wie kommst Du nur auf das schräge Brett, dass die Zeit an der Materie zerbeln müsste? Das braucht sie doch gar nicht, weil der Raum schon an der zerrt . denn ohne Raum ist schließlich auch keine Bewegung möglich.

.


Jetzt hast du du mir ein überzeugendes Argument geliefert, dass der Raum an der Zeit zerrt, wer hötte das gedacht.
Und ich hatte mir eingebildet die Zeit zerrt am Raum! Errare humanum est.
So eine Zerrerei aber auch im Universum. Auf den Arm nehmen Auf den Arm nehmen Auf den Arm nehmen

#58:  Autor: der kleine FritzWohnort: Planet Erde BeitragVerfasst am: 06.01.2017, 09:06
    —
Kramer hat folgendes geschrieben:
Hallo kleiner Fritz,

ich denke, das passt besser in diesen Thread:

der kleine Fritz hat folgendes geschrieben:

Aber sag mir bitte noch,
wie lange dauert denn das "Jetzt"?

mfG


Kannst Du mir sagen, wie gross "hier" ist?

SCNR

Cool


Nein, warum sollte ich?

#59:  Autor: fwoWohnort: im Speckgürtel BeitragVerfasst am: 06.01.2017, 12:28
    —
der kleine Fritz hat folgendes geschrieben:
fwo hat folgendes geschrieben:
der kleine Fritz hat folgendes geschrieben:
....
Was deinen Vergleich mit der unsichtbaren Schwerkraft anbelangt, so siehst du ja beim fallenden Apfel eine „offensichtliche“ Wirkung. Welche Wirkung ist aber nach deiner Behauptung bei der Zeit offensichtlich?

Welch kühne Behauptung.
Erklär doch bitte mal genau welche Eigenschaft der Schwerkraft es macht, dass der Apfel fällt,
und wie die das macht.

btw: Wie kommst Du nur auf das schräge Brett, dass die Zeit an der Materie zerbeln müsste? Das braucht sie doch gar nicht, weil der Raum schon an der zerrt . denn ohne Raum ist schließlich auch keine Bewegung möglich.

.


Jetzt hast du du mir ein überzeugendes Argument geliefert, dass der Raum an der Zeit zerrt, wer hötte das gedacht.
Und ich hatte mir eingebildet die Zeit zerrt am Raum! Errare humanum est.
So eine Zerrerei aber auch im Universum. Auf den Arm nehmen Auf den Arm nehmen Auf den Arm nehmen

Das beruhigt mich. Ich hatte schon befürchtet, Du würdest dieses Argument nicht sehen.

#60:  Autor: zelig BeitragVerfasst am: 06.01.2017, 13:24
    —
Mal ganz unabhängig vom Diskussionsverlauf ein Aufsatz aus 2014:

Zitat:
Nach einer inneren Uhr, die unser Zeitgefühl im Sekunden- bis Minutenbereich steuert, suchten Hirnforscher bislang vergeblich. Neue Erkenntnisse deuten nun darauf hin, dass hier unser Körpergefühl eine zentrale Rolle spielt: Selbstwahrnehmung und Zeiterleben sind augenscheinlich untrennbar miteinander verbunden.


http://www.spektrum.de/news/wie-unser-gefuehl-fuer-die-zeit-entsteht/1309744


Das Statement über die fehlende direkte sinnliche Wahrnehmung von Zeit, würde ich insofern verteidigen, als daß wir über kein zeitliches Korrelat zur räumlich ausgedehnten Substanz verfügen. Aber ich zweifle damit nicht den ontologischen Status der Zeit an. Andersrum zweifle ich eher den ontologischen Status der Substanz an, die uns aufgrund unserer menschlichen Beschaffenheit fast zwingend eine falsche Intuition vermittelt.
Allerdings spielt das im Mesokomos keine Rolle. Unsere Sinne sind das Ergebnis eine langen Prozesses. Wir sind, was unsere Bedürfnise betrifft, bestens an diese Realität angepasst. Nur haben wir halt seit einigen Jahrzehnten aufgrund des naturwissenschaftlichen Fortschritts die Grenzen des Mesokosmos' in Bereiche erweitert, die uns (beispielsweise über die Nutzung des GPS) zeigen, wie sehr uns die Intuition täuschen kann.

#61:  Autor: SkeptikerWohnort: 129 Goosebumpsville BeitragVerfasst am: 06.01.2017, 13:44
    —
Am ziemlich präzisen Funktionieren der GPS-Technologie sieht man aber, dass die Wissenschaft die Struktur der Zeit im Zusammenwirken mit anderen physikalischen Größen sehr gut verstanden hat.

#62:  Autor: zelig BeitragVerfasst am: 06.01.2017, 13:46
    —
Skeptiker hat folgendes geschrieben:
Am ziemlich präzisen Funktionieren der GPS-Technologie sieht man aber, dass die Wissenschaft die Struktur der Zeit im Zusammenwirken mit anderen physikalischen Größen sehr gut verstanden hat.


Ja, im Rahmen unsere momentanen Erkenntnisfähigkeit auf jeden Fall.

#63:  Autor: fwoWohnort: im Speckgürtel BeitragVerfasst am: 06.01.2017, 14:21
    —
zelig hat folgendes geschrieben:
....
Das Statement über die fehlende direkte sinnliche Wahrnehmung von Zeit, würde ich insofern verteidigen, als daß wir über kein zeitliches Korrelat zur räumlich ausgedehnten Substanz verfügen. ...

Gibt es denn wirklich eine sinnliche Wahrnehmung von Raum? Doch nur insofern, als wir - solange wir gesund sind - eine Empfindung davon haben, wo sich unsere eigenen Körperteile relativ zum Kopf befinden, das aber auch nur, solange wir sie, und wenn es im Mikrobereich ist, bewegen, und wo wir andere, Raum einnehmende Materie berühren. Ansonsten gilt auch hier: Unsere Sinne sind nicht geeignet, statische Information zu erfassen - die Kontinuität der subjektiven Welt entsteht im Gehirn selbst.

Aber genau genommen ist die räumlichen Wahrnehmung ein Konstrukt aus der Auswertung von Reizen, die selbst nichts mit der Weltqualität Raum zu tun haben: Licht, Druck, Schwerkraft, Schall.

Wir sind uns des Räumlichen nur so sicher, weil es in dieser Konstruktion schon taktil erfahrbar ist, bevor wir die Welt in Worte fassen, und weil wir schon als Säugling üben, unsere Richtung darin zu bestimmen: Eine echte Raumerfahrung machen wir nur, wenn wir die Zeit hinzunehmen, um uns selbst im Raum zu bewegen.

p.s. zur Raumwahrnehmungg speziell zu dieser Diskussion: Ich sollte nicht unerwähnt lassen, dass wir selbst die Zeit nutzen (können), um den Raum wahrzunehmen: Blinde können - ähnlich Fledermäusen - lernen sich am Echo von Klicklauten zu orientieren: Unsere Richtungswahrnehmung bei Tönen ist möglich durch die zeitliche Differenz im Ankommen der Laute an beiden Ohren.

@ zelig, weil Dir meine Zitierweise regelmäßig aufstößt: Dem nicht zitierten stimme ich zu.
EDIT: ps.


Zuletzt bearbeitet von fwo am 06.01.2017, 17:47, insgesamt einmal bearbeitet

#64:  Autor: SkeptikerWohnort: 129 Goosebumpsville BeitragVerfasst am: 06.01.2017, 15:59
    —
zelig hat folgendes geschrieben:
Skeptiker hat folgendes geschrieben:
Am ziemlich präzisen Funktionieren der GPS-Technologie sieht man aber, dass die Wissenschaft die Struktur der Zeit im Zusammenwirken mit anderen physikalischen Größen sehr gut verstanden hat.


Ja, im Rahmen unsere momentanen Erkenntnisfähigkeit auf jeden Fall.


Wobei nicht alles, was technisch funktioniert, nur deshalb funktioniert, weil die entsprechenden Gesetzmäßigkeiten alle korrekt verstanden wurden.

Wenn man an die Vergangenheit der Menschheit denkt, etwa an das technische Erzeugen des ersten Feuers, da hat der Mensch damals praktisch nichts von irgendwelchen physikalischen und mathematischen Gesetzen verstanden. Trotzdem hat es funktioniert!

Aber je bewusster eine Technologie entwickelt wird, desto größer wird die Notwendigkeit des Verstehens relevanter Gesetzmäßigkeiten und Zusammenhänge.

So sind im Laufe der menschlichen Geschichte Erkenntnis und Technik eine immer größere Einheit miteinander eingegangen, während diese beiden Elemente in der menschlichen Frühzeit praktisch nahezu völlig getrennt voneinander waren.

Insofern ist z.B. die Zeit zum einen eine subjektive Wahrnehmung von Mensch & Tier, aber wir sehen heute, dass sich dahinter auch etwas Obektives verbirgt, welches im Bewusstsein keineswegs erst entsteht, erzeugt wird, sondern sich darin - in mehr oder weniger verzerrter Form - abbildet ...-

#65:  Autor: Hebart BeitragVerfasst am: 07.01.2017, 01:15
    —
Skeptiker hat folgendes geschrieben:
zelig hat folgendes geschrieben:
Skeptiker hat folgendes geschrieben:
Am ziemlich präzisen Funktionieren der GPS-Technologie sieht man aber, dass die Wissenschaft die Struktur der Zeit im Zusammenwirken mit anderen physikalischen Größen sehr gut verstanden hat.


Ja, im Rahmen unsere momentanen Erkenntnisfähigkeit auf jeden Fall.


Wobei nicht alles, was technisch funktioniert, nur deshalb funktioniert, weil die entsprechenden Gesetzmäßigkeiten alle korrekt verstanden wurden.

Wenn man an die Vergangenheit der Menschheit denkt, etwa an das technische Erzeugen des ersten Feuers, da hat der Mensch damals praktisch nichts von irgendwelchen physikalischen und mathematischen Gesetzen verstanden. Trotzdem hat es funktioniert!

Aber je bewusster eine Technologie entwickelt wird, desto größer wird die Notwendigkeit des Verstehens relevanter Gesetzmäßigkeiten und Zusammenhänge.

So sind im Laufe der menschlichen Geschichte Erkenntnis und Technik eine immer größere Einheit miteinander eingegangen, während diese beiden Elemente in der menschlichen Frühzeit praktisch nahezu völlig getrennt voneinander waren.

Insofern ist z.B. die Zeit zum einen eine subjektive Wahrnehmung von Mensch & Tier, aber wir sehen heute, dass sich dahinter auch etwas Obektives verbirgt, welches im Bewusstsein keineswegs erst entsteht, erzeugt wird, sondern sich darin - in mehr oder weniger verzerrter Form - abbildet ...-


Du bist bei Deinen Gedankengängen nicht mehr/noch nicht (such es Dir aus, hehe..) ZEITGEMÄß... Jedenfalls stehst Du mitten im Geschehen zwinkern

#66:  Autor: der kleine FritzWohnort: Planet Erde BeitragVerfasst am: 07.01.2017, 09:19
    —
Skeptiker hat folgendes geschrieben:
Am ziemlich präzisen Funktionieren der GPS-Technologie sieht man aber, dass die Wissenschaft die Struktur der Zeit im Zusammenwirken mit anderen physikalischen Größen sehr gut verstanden hat.


Hallo Skeptiker.
schalte mal deine Skepsis ein.

Um die von dir genannte Struktur der Zeit zu verstehen, ist ja deren struktureller Aufbau Voraussetzug. Strukturell bedeutet aber auch zwingend materiell, und es ist eine absurde Vorstellung, dass und wie massebehaftete Zeit vom Raum verzerrt werden kann. Eine imaginäre Zeit dürfte kaum zu Wechselwirkungen fähig sein.

Kannst du nähere Angaben dazu machen, welcher Art der Struktur der Zeit die Wissenschaft im Zusammenhang mit anderen (welchen) physikalischen Größen erkannt hat?

#67:  Autor: fwoWohnort: im Speckgürtel BeitragVerfasst am: 07.01.2017, 11:16
    —
der kleine Fritz hat folgendes geschrieben:
Skeptiker hat folgendes geschrieben:
Am ziemlich präzisen Funktionieren der GPS-Technologie sieht man aber, dass die Wissenschaft die Struktur der Zeit im Zusammenwirken mit anderen physikalischen Größen sehr gut verstanden hat.


Hallo Skeptiker.
schalte mal deine Skepsis ein.

Um die von dir genannte Struktur der Zeit zu verstehen, ist ja deren struktureller Aufbau Voraussetzug. Strukturell bedeutet aber auch zwingend materiell, und es ist eine absurde Vorstellung, dass und wie massebehaftete Zeit vom Raum verzerrt werden kann. Eine imaginäre Zeit dürfte kaum zu Wechselwirkungen fähig sein.

Kannst du nähere Angaben dazu machen, welcher Art der Struktur der Zeit die Wissenschaft im Zusammenhang mit anderen (welchen) physikalischen Größen erkannt hat?

Ich würde den Skeptiker erstmal fragen wollen, was er denn mit dem Ausdruck "Struktur der Zeit" überhaupt meint.

@der kleine Fritz: Wie kommst Du eigentlich darauf, dass Struktur Materie voraussetzt?

#68:  Autor: Er_Win BeitragVerfasst am: 07.01.2017, 12:35
    —
fwo hat folgendes geschrieben:

Ich würde den Skeptiker erstmal fragen wollen, was er denn mit dem Ausdruck "Struktur der Zeit" überhaupt meint.


Der Frage schließe ich mich an.

btw. zumindest mein "Struktur"-Begriff impliziert keine Materie.

Ein strukturelles Element "der Zeit" ist für mich die Frage: "gequantelt oder kontinuierlich"

#69:  Autor: SkeptikerWohnort: 129 Goosebumpsville BeitragVerfasst am: 07.01.2017, 19:48
    —
der kleine Fritz hat folgendes geschrieben:
Skeptiker hat folgendes geschrieben:
Am ziemlich präzisen Funktionieren der GPS-Technologie sieht man aber, dass die Wissenschaft die Struktur der Zeit im Zusammenwirken mit anderen physikalischen Größen sehr gut verstanden hat.


Hallo Skeptiker.
schalte mal deine Skepsis ein.


Skepsis ist wie immer auf "on". zwinkern

der kleine Fritz hat folgendes geschrieben:
Um die von dir genannte Struktur der Zeit zu verstehen, ist ja deren struktureller Aufbau Voraussetzug. Strukturell bedeutet aber auch zwingend materiell, und es ist eine absurde Vorstellung, dass und wie massebehaftete Zeit vom Raum verzerrt werden kann. Eine imaginäre Zeit dürfte kaum zu Wechselwirkungen fähig sein.

Kannst du nähere Angaben dazu machen, welcher Art der Struktur der Zeit die Wissenschaft im Zusammenhang mit anderen (welchen) physikalischen Größen erkannt hat?


Die Zeit eines Körpers hängt ab von seiner Geschwindigkeit und der Gravitation, der er ausgesetzt ist. Der Körper verändert seine Koordinaten in der Raumzeit in einem bestimmten Bezugssystem; er bewegt sich messbar im Raum und in der Zeit in einem bestimmten Bezugssystem.

Seine relativ zu anderen Körpern in anderen Bezugssystemen stehende, eigene Zeit bezogen auf seine inneren Ereignisse wird von den genannten physikalischen Größen beeinflusst, welche Zeit und Raum jeweils für sich genommen *krümmen*. So verändern also die sich bewegenden Körper die umgebende Raumzeit und diese beeinflusst durch ihre veränderte Struktur auch die Bewegung dieser Körper selber.

Man könnte sagen, es ist eine wechselseitige Dialektik.

Die Zeit bildet eine Einheit mit dem 3-dimensionalen Raum. Der Zeitpfeil ist anders als die Raumkoordinaten allein auf zukünftige Zustände/Konstellationen ausgerichtet, so wie eine Strahlung nur in Richtung weg von ihrer Strahlungsquelle eine Richtung hat, jedoch keine Umkehrrichtung. Dies hängt auch zusammen mit dem Gesetz der Entropie.

Wie bei der Lichtgeschwindigkeit muss man also auch bei der Zeit dessen jeweiliges Bezugssystem (Gravitationskräfte, Vakuum ja/nein) angeben, um eine Synchronisation zu erreichen. Die Lichtgeschwindigkeit ist einzig im Vakuum konstant, die inneren Ereignisse eine Körpers sind in ihrer Zeitspanne auch nur bei gleichbleibenden Bedingungen konstant.

Zeit und Raum werden von materiellen Bedingungen beeinflusst und beeinflussen diese Bedingungen wiederum auch.

Beim GPS-System müssen gemäß der Geschwindigkeit und dem Gravitationseinfluss eines Satelliten dessen Eigenzeit sowie die zeitliche Differenz zwischen Senden und Empfangen korrigiert werden, um genau die Koordinaten zu bekommen, die man auf der Erde zu einem bestimmten *Jetzt* besitzt.

Dass dies tatsächlich gelingt, beweist den wechselseitigen und quantifizierbaren Einfluss von Zeit & Raum auf sich bewegende Körper.

@Physik-Spezialisten: Bitte ggf. um Korrekturen und Ergänzungen.

#70:  Autor: der kleine FritzWohnort: Planet Erde BeitragVerfasst am: 09.01.2017, 11:09
    —
Skeptiker hat folgendes geschrieben:
der kleine Fritz hat folgendes geschrieben:
Skeptiker hat folgendes geschrieben:
Am ziemlich präzisen Funktionieren der GPS-Technologie sieht man aber, dass die Wissenschaft die Struktur der Zeit im Zusammenwirken mit anderen physikalischen Größen sehr gut verstanden hat.


Hallo Skeptiker.
schalte mal deine Skepsis ein.


Skepsis ist wie immer auf "on". zwinkern

der kleine Fritz hat folgendes geschrieben:
Um die von dir genannte Struktur der Zeit zu verstehen, ist ja deren struktureller Aufbau Voraussetzug. Strukturell bedeutet aber auch zwingend materiell, und es ist eine absurde Vorstellung, dass und wie massebehaftete Zeit vom Raum verzerrt werden kann. Eine imaginäre Zeit dürfte kaum zu Wechselwirkungen fähig sein.

Kannst du nähere Angaben dazu machen, welcher Art der Struktur der Zeit die Wissenschaft im Zusammenhang mit anderen (welchen) physikalischen Größen erkannt hat?


Die Zeit eines Körpers hängt ab von seiner Geschwindigkeit und der Gravitation, der er ausgesetzt ist. Der Körper verändert seine Koordinaten in der Raumzeit in einem bestimmten Bezugssystem; er bewegt sich messbar im Raum und in der Zeit in einem bestimmten Bezugssystem.



Halihalo Skeptiker

Frei nach Goethes „Faust“:
„Hier steh ich nun, ich armer Tor und bin so klug als wie zuvor“

1. Wo erklärst du denn nun die Erkenntnisse der Wissenschaft über die Struktur der Zeit?
2 Wo war auch nur ein einziges Wort deiner Skepsis zu lesen?
3. Was verstehst du denn unter „die Zeit“ eines Körpers? WO hat er die denn??

Danke für deine Erklärung im voraus!

#71:  Autor: SkeptikerWohnort: 129 Goosebumpsville BeitragVerfasst am: 10.01.2017, 08:12
    —
der kleine Fritz hat folgendes geschrieben:
Skeptiker hat folgendes geschrieben:
der kleine Fritz hat folgendes geschrieben:
Skeptiker hat folgendes geschrieben:
Am ziemlich präzisen Funktionieren der GPS-Technologie sieht man aber, dass die Wissenschaft die Struktur der Zeit im Zusammenwirken mit anderen physikalischen Größen sehr gut verstanden hat.


Hallo Skeptiker.
schalte mal deine Skepsis ein.


Skepsis ist wie immer auf "on". zwinkern

der kleine Fritz hat folgendes geschrieben:
Um die von dir genannte Struktur der Zeit zu verstehen, ist ja deren struktureller Aufbau Voraussetzug. Strukturell bedeutet aber auch zwingend materiell, und es ist eine absurde Vorstellung, dass und wie massebehaftete Zeit vom Raum verzerrt werden kann. Eine imaginäre Zeit dürfte kaum zu Wechselwirkungen fähig sein.

Kannst du nähere Angaben dazu machen, welcher Art der Struktur der Zeit die Wissenschaft im Zusammenhang mit anderen (welchen) physikalischen Größen erkannt hat?


Die Zeit eines Körpers hängt ab von seiner Geschwindigkeit und der Gravitation, der er ausgesetzt ist. Der Körper verändert seine Koordinaten in der Raumzeit in einem bestimmten Bezugssystem; er bewegt sich messbar im Raum und in der Zeit in einem bestimmten Bezugssystem.



Halihalo Skeptiker

Frei nach Goethes „Faust“:
„Hier steh ich nun, ich armer Tor und bin so klug als wie zuvor“

1. Wo erklärst du denn nun die Erkenntnisse der Wissenschaft über die Struktur der Zeit?
2 Wo war auch nur ein einziges Wort deiner Skepsis zu lesen?
3. Was verstehst du denn unter „die Zeit“ eines Körpers? WO hat er die denn??

Danke für deine Erklärung im voraus!


Hi klein fritzchen! Smilie

Gib mir ein bisschen Zeit, dann nehme ich ausführlicher und konkreter Stellung. Raum habe ich genug.

In einem nahen Zeitraum will ich zur Materialität von Zeit Stellung nehmen. Meine Skepsis ist immer da, wo Aussagen schlecht begründet sind, dann wacht sie auf. Ansonsten schläft sie.

Der Begriff der Eigenzeit eines Körpers bzw eines Teilchens spielt beim GPS jedenfalls auch technisch eine wichtige Rolle.

Bis dahin ...-! zwinkern

#72:  Autor: der kleine FritzWohnort: Planet Erde BeitragVerfasst am: 10.01.2017, 10:04
    —
Skeptiker hat folgendes geschrieben:



Zitat:
Hi klein fritzchen! Smilie

Gib mir ein bisschen Zeit, dann nehme ich ausführlicher und konkreter Stellung. Raum habe ich genug.


Hallo Skeptiker,
wieviel Zeit benötigst du denn?

Ich könnte dir das Kilo Zeit für 99 Cent anbieten!
Musst sie aber selber abholen und den Raum dafür mitbringen.
Am besten eignet sich ein Schuhkarton, da passt gerade 1 Kilo rein!
Dann tscüß.
Lachen

#73:  Autor: smallie BeitragVerfasst am: 10.01.2017, 23:37
    —
zelig hat folgendes geschrieben:
Das Statement über die fehlende direkte sinnliche Wahrnehmung von Zeit, würde ich insofern verteidigen, als daß wir über kein zeitliches Korrelat zur räumlich ausgedehnten Substanz verfügen.

Du verteidigst das hartnäckig, ich kann's per Selbsterfahrung ebenso hartnäckig nicht nachvollziehen. Könnte es sich hier am Ende um eine mentale Typsache handeln? So wie introvertiert vs. extrovertiert? Dann wärst du ein Raum-Typ und ich ein Zeit-Typ.

Zum Beispiel sind es von mir bis in die Stadtmitte 15 Gehminuten. München ist eine Stunde und eine Viertelte weit weg. Man spricht von Tagesreisen und meint heute damit etwas anderes als zu Zeiten Bachs. Als Flächenmaß gibt es das altertümliche Tagwerk. Die Astronomen messen in Lichtjahren. Arbeit verrechnet man in Manntagen, -monaten oder -jahren. Und so weiter.

Jetzt kannst du einwenden, diese Beispiele seien immer noch keine sinnliche Wahrnehmung von Zeit. Witziger Weise liefert dein Link ein Beispiel für diese sinnliche Wahrnehmung.




zelig hat folgendes geschrieben:
Mal ganz unabhängig vom Diskussionsverlauf ein Aufsatz aus 2014:

Zitat:
Nach einer inneren Uhr, die unser Zeitgefühl im Sekunden- bis Minutenbereich steuert, suchten Hirnforscher bislang vergeblich. Neue Erkenntnisse deuten nun darauf hin, dass hier unser Körpergefühl eine zentrale Rolle spielt: Selbstwahrnehmung und Zeiterleben sind augenscheinlich untrennbar miteinander verbunden.


http://www.spektrum.de/news/wie-unser-gefuehl-fuer-die-zeit-entsteht/1309744


Im Kern passt der Ansatz schon. Zeiterleben ist das Erleben von sich ändernden Körperzuständen.

smallie hat folgendes geschrieben:
An erster Stelle natürlich: atmen. Ich sitze hier und atme ein, aus, Pause, ein, aus, ... Das ist für mich als sinnliche Erfahrung unmittelbar mit dem Erleben von Zeit verbunden.

Wenn den Hirnforschern das erst jetzt auffällt, dann waren sie zuvor ziemlich verkopft.


Eine Sache fehlt mir im Artikel: in "Bunkerversuchen" stellte sich heraus, das Menschen ohne Kontakt zum Tag/Nacht-Rhythmus zu 25-Stunden-Tagen neigen. Falls das stimmt, gäbe es eine Art innere Uhr.


Hier kann sich der Autor nicht entscheiden, was stimmt:

Spektrum hat folgendes geschrieben:
Nach einer inneren Uhr, die unser Zeitgefühl im Sekunden- bis Minutenbereich steuert, suchten Hirnforscher bislang vergeblich.


Dann aber heißt es:

Spektrum hat folgendes geschrieben:
Zeitwahrnehmung im Experiment

In einem Experiment zur Zeitwahrnehmung hörten die Versuchspersonen einen Ton von festgelegter Dauer. Währenddessen nahm die Aktivität in der hinteren Inselrinde (gelb im Hirnschnitt) kontinuierlich bis zum Ende des durch den Ton markierten Zeitintervalls zu. Da diese Hirnregion fortlaufend körperliche Zustände repräsentiert, könnte die Aktivitätszunahme die Aufnahme von Körpersignalen widerspiegeln und so als Zeitmesser dienen.


Das widerspricht sich doch, oder?

#74:  Autor: zelig BeitragVerfasst am: 11.01.2017, 09:06
    —
smallie hat folgendes geschrieben:
zelig hat folgendes geschrieben:
Das Statement über die fehlende direkte sinnliche Wahrnehmung von Zeit, würde ich insofern verteidigen, als daß wir über kein zeitliches Korrelat zur räumlich ausgedehnten Substanz verfügen.

Du verteidigst das hartnäckig, ich kann's per Selbsterfahrung ebenso hartnäckig nicht nachvollziehen. Könnte es sich hier am Ende um eine mentale Typsache handeln? So wie introvertiert vs. extrovertiert? Dann wärst du ein Raum-Typ und ich ein Zeit-Typ.


Interessante Typologie. Obwohl Menschen sicherlich unterschiedliche räumliche und zeitliche Wahrnehmungen haben, glaube ich nicht, daß man sie in diese 2 Kategorien einteilen könnte. ; )


smallie hat folgendes geschrieben:

Eine Sache fehlt mir im Artikel: in "Bunkerversuchen" stellte sich heraus, das Menschen ohne Kontakt zum Tag/Nacht-Rhythmus zu 25-Stunden-Tagen neigen. Falls das stimmt, gäbe es eine Art innere Uhr.


Aus meiner Sicht wäre die schwankend-unzuverlässige Wahrnehmung zeitlicher Abläufe eher ein Argument pro meiner Ausgangsthese: Ohne die Möglichkeit, genau definierte Zyklen mittels gegenständlicher Wahrnehmung abzählen zu können, verliert sich die zeitliche Wahrnehmung ins Ungefähre. Die verstrichene Zeit in der Isolation kann nur noch geschätzt werden. Und wenn ich mich richtig erinnere, gibt es Berichte über Langzeitisolationen in Höhlen, mit dem Verlust der Fähigkeit, die verstrichene Zeit auch nur annähernd richtig einzuschätzen.

Ob du in der Isolation einen Zollstock dabei hast, der dir ermöglicht, auch nach langer Zeit räumliche Abstände ziemlich genau zu vermessen. Oder ob Du zufällig gerade eine Sanduhr im Gepäck dabei hattest, die dir ermöglicht hat, die verstrichene Dauer ziemlich genau zu vermessen, als du für ein halbes Jahr in einer Höhle verschütt gegangen bist - in beiden Fällen kannst Du die Messung nur vornehmen, weil etwas Gegenständliches -die Substanz- vorhanden war. Ein temporales, sinnlich erfahrbares Gegenstück zur Substanz existiert nicht. Danke daß Du mich gezwungen hast darüber nachzudenken.

#75:  Autor: der kleine FritzWohnort: Planet Erde BeitragVerfasst am: 11.01.2017, 11:47
    —
zelig hat folgendes geschrieben:



Ob du in der Isolation einen Zollstock dabei hast, der dir ermöglicht, auch nach langer Zeit räumliche Abstände ziemlich genau zu vermessen. Oder ob Du zufällig gerade eine Sanduhr im Gepäck dabei hattest, die dir ermöglicht hat, die verstrichene Dauer ziemlich genau zu vermessen, als du für ein halbes Jahr in einer Höhle verschütt gegangen bist - in beiden Fällen kannst Du die Messung nur vornehmen, weil etwas Gegenständliches -die Substanz- vorhanden war. Ein temporales, sinnlich erfahrbares Gegenstück zur Substanz existiert nicht. Danke daß Du mich gezwungen hast darüber nachzudenken.


Ja was denn nun – macht man eine Messung der langen Zeit oder der verstrichener Dauer?

Zeit als abstrakten Begriff kann man nicht messen, Zeit ist ja immer erst ein Ergebnis der Messung der Dauer eines Vorgangs in der jeweils angegebenen Maßeinheit. Für diese wiederum wurde auch eine bestimmte Dauer von physikalischen Bewegungen als Norm gesetzt.

Hab ich dich recht verstanden, dass Raum und Zeit etwas substanziell Gegenständliches sind (dann wäre die Substanz zu beschreiben) oder meinst du damit Zollstock und Uhr? Und mit der Uhr misst man ja keine Zeit, sondern vergleicht nur die Dauer von Bewegungen miteinander.


quote gerichtet. vrolijke

#76:  Autor: zelig BeitragVerfasst am: 11.01.2017, 14:32
    —
der kleine Fritz hat folgendes geschrieben:

Hab ich dich recht verstanden, dass Raum und Zeit etwas substanziell Gegenständliches sind (dann wäre die Substanz zu beschreiben) oder meinst du damit Zollstock und Uhr? Und mit der Uhr misst man ja keine Zeit, sondern vergleicht nur die Dauer von Bewegungen miteinander.


In diesem Zweig der Unterhaltung geht es um die Überlegung, daß der Grund für die Verwirrungen über die Anschauung der Zeit darin liegen könnte, daß kein (zeitlich ausgedehntes) Analogon zur (räumlich ausgedehnten) Substanz existiert, das uns über unsere Sinne erfahrbar wäre.
Ich sage damit nichts über den ontologischen Status von Raum und Zeit aus. Aber es scheint doch so zu sein, daß Konzepte von Raum intuitiv eingänglicher sind, als Konzepte von Zeit.

#77:  Autor: fwoWohnort: im Speckgürtel BeitragVerfasst am: 11.01.2017, 14:43
    —
@ zelig

Dass unsere Wahrnehmung des Raumes letztlich auch sekundär ist, hatte ich hier schon mal beschrieben. Auch da sollten wir davon ausgehen, dass die Wissenschaft es schwer haben wird, ein eingebautes Metermaß für die Entfernungen jenseits der Armeslänge zu finden. Warum steht die Raumenpfindung trotzdem für Dich außer Frage?

Dass die circadiane Rhythmik irgendwann einmal versagt, wenn sie nicht regelmäßig durch Tageswechsel neu angestoßen wird, sagt nicht, dass dass sie nicht existiert. Das wäre ungefähr so, als würdest Du sagen, dass eine mechanische Uhr keine Unruh haben könne, weil ihr Federwerk irgendwann abgelaufen ist.

Dass bei uns eine "Uhr" im Ultrakurzzeitbereich existieren muss, darauf hatte ich im selben Post wie zum Raum auch schon hingewiesen.

Dass ein hinreichend präziser Taktgeber für den Kurzzeitbereich bisher nicht gefunden wurde, sagt nicht, dass es ihn nicht gibt. Ich würde ihn weiterhin als Hypothese stehen lassen.

Der Hintergrund ist folgender:

Wie stellst Du Dir die Organsiation auch nur einer steinzeitlichen Gesellschaft mit den nötigen Verabredungen (habe ich schon erwähnt: z.B. bei einer Jagd, die großräumig genug ist, einen direkten Kontakt nicht mehr zu erlauben) ohne einen irgendwie gearteten Zeitbegriff überhaupt vor?
In Anbetracht der Tatsache, dass wir in Sprache denken und kommunizieren, ist der Anfang dieser zeitlichen Verabredungen selbstverständlich nur in einer zeitlichen Reihung vorstellbar: Erst das, dann das, dann das.

Ich würde den Gedanken sogar umdrehen:
Innerhalb der Entwicklung der Sprache ist die Entdeckung der Zeit als Denk- und Kommunikationsgegenstand mit eine der Voraussetzungen für den Erfolg des Evolutionsprojektes Mensch, weil erst das den hohen Organisationsgrad durch die Vereinbarungsfähigkeit der zeitlichen Organisation möglich gemacht hat. Angeboren und durch äußere Faktoren getaktet kannte "die Natur" das ja schon lange.

Die Verbindung der Zeit mit der Religion, die Marcellinus hier als Gegenmodell aufstellen möchte, ist nicht wirklich ein Gegenmodell, sondern zeigt mir nur, wie wichtig denen diese Zeit war: Um zu zeigen, was ich damit meine, möchte ich Marcellinus' soziologische Argumentation mal auf einem anderen Gebiet persiflieren: Der Regen war denen völlig egal, den haben sie gar nicht wahrgenommen, die haben nur an den Regengott geglaubt.

Der religiöse Ritus ist evolutionär nur eine gesicherte Form der Tradition, kein Zweck an sich.

Wenn wir heute sehen, dass unsere Sprache nicht einfach irgendwann einmal da war, sondern zugleich auch Ausdruck der Objekte war, mit denen wir uns beschäftigt haben (ich gebe zu bedenken, dass diese einfache Formulierung für einen rekursiven Prozess steht) muss man doch fragen, welche Notwendigkeit überhaupt bestand, den Tag als solchen zu benamsen, oder den Monat oder die Jahreszeit oder die Tageszeit. Warum sollte das bei der Zeit anders sein als etwa bei den Pflanzen, bei denen vor der Einrichtung der Wissenschaft nur Namen für die Pflanzen existierten, die man auch benutzte?

Und wir wissen aus der Archäologie, dass man bereits sehr früh sehr hohen Aufwand betrieb, um diese Zeit genauer zu kalibrieren. Zur Befriedigung der Götter? Siehe oben.

#78:  Autor: zelig BeitragVerfasst am: 11.01.2017, 15:14
    —
fwo hat folgendes geschrieben:
Warum steht die Raumenpfindung trotzdem für Dich außer Frage?


Weil es im Mesokosmos keine menschliche Verwirrung über sie gibt. Dagegen wird die Zeit, und wie der Mensch in ihr steht, als rätselhaft empfunden.


Auf die anderen Überlegungen gehe ich jetzt nicht ein, weil ich dazu nichts gesagt habe.

#79:  Autor: Marcellinus BeitragVerfasst am: 11.01.2017, 17:51
    —
fwo hat folgendes geschrieben:

Die Verbindung der Zeit mit der Religion, die Marcellinus hier als Gegenmodell aufstellen möchte, ist nicht wirklich ein Gegenmodell, sondern zeigt mir nur, wie wichtig denen diese Zeit war…


Marcellinus hat nur versucht, dir klar zu machen, daß es den Menschen der frühen Hochkulturen vor allem darum ging, Zeitpunkte von besonderer sozialer Relevanz zu bestimmen, und die orientierten sich vor allem an regelmäßig wiederkehrenden Himmelserscheinungen. Es ging also nicht eigentlich um Zeitmessung, sondern um Himmelsbeobachtung, wie ihr Zeitverständnis auch nicht in erster Linie linear wie unser heutiges war, sondern zyklisch, orientiert an der regelmäßigen Wiederkehr gesellschaftlich signifikanter Ereignisse.

fwo hat folgendes geschrieben:

Dass ein hinreichend präziser Taktgeber für den Kurzzeitbereich bisher nicht gefunden wurde, sagt nicht, dass es ihn nicht gibt. Ich würde ihn weiterhin als Hypothese stehen lassen.
[…]
Wie stellst Du Dir die Organsiation auch nur einer steinzeitlichen Gesellschaft mit den nötigen Verabredungen (habe ich schon erwähnt: z.B. bei einer Jagd, die großräumig genug ist, einen direkten Kontakt nicht mehr zu erlauben) ohne einen irgendwie gearteten Zeitbegriff überhaupt vor?


Das ist doch nun ganz einfach! Jeder Steinzeitjäger hatte eine Uhr in der Tasche, um die tägliche Jagd um 17:45 Uhr nicht zu verpassen. Deshalb nannte man die Zeit ja auch U(h)rzeit! Sehr glücklich

#80:  Autor: Kramer BeitragVerfasst am: 11.01.2017, 18:06
    —
zelig hat folgendes geschrieben:
Aber es scheint doch so zu sein, daß Konzepte von Raum intuitiv eingänglicher sind, als Konzepte von Zeit.


Vielleicht liegt das daran, dass die Räume/Entfernungen, in denen wir uns bewegen, klein genug sind, um sie zu begreifen. Das wirkliche Ausmass des Raumes (also des Universums) ist für uns ebenso unbegreiflich, wie die Ewigkeit. Ich sehe da durchaus eine Analogie zur Zeit: Wir begreifen den Raum, weil wir ihn verstehbare Einheiten einteilen. Mein Zimmer, mein Haus, mein Hof, mein Dorf, die Strecke zum nächsten Dorf usw. Bei der Zeit machen wir etwas Ähnliches, wir teilen sie in begreifbare Abschnitte ein, wir haben z.B. einen Tagesrhythmus, bestimmte Zeiten, zu denen wir Essen usw. Wird uns dieser Rhythmus genommen, verlieren wir die Orientierung in der Zeit. So wie wir die Orientierung im Raum verlieren würden, wenn man uns in eine total fremde Umgebung beamen würde.

#81:  Autor: Er_Win BeitragVerfasst am: 11.01.2017, 18:52
    —
Marcellinus hat folgendes geschrieben:

Das ist doch nun ganz einfach! Jeder Steinzeitjäger hatte eine Uhr in der Tasche, um die tägliche Jagd um 17:45 Uhr nicht zu verpassen. Deshalb nannte man die Zeit ja auch U(h)rzeit! Sehr glücklich


Auf diese Steinzeitjäger gehen dann wohl auch die Steine mechanischer Uhren zurÜck ... Auf den Arm nehmen

Liege ich ansonsten richtig, dass du im Gegensatz zu den meisten anderen Postern eine Objektivierbarkeit der Zeit bezweifelst. Ev. ähnlich wie das Zeilinger schon 2003 in einem Interview ausgedrückt hat:

Ja, aus Gründen, die ich nicht verstehe, gibt es bei den Philosophen eine Scheu vor radikalen Positionen. Und die Bohrsche Position ist radikal. Warum die Philosophen diese Scheu haben, weiß ich nicht. Offenbar versuchen sie, möglichst viel von der Vorstellung einer Wirklichkeit zu retten, die unabhängig von den Experimenten ist, die wir mit ihr anstellen.

#82:  Autor: Marcellinus BeitragVerfasst am: 11.01.2017, 21:10
    —
Er_Win hat folgendes geschrieben:

Liege ich ansonsten richtig, dass du im Gegensatz zu den meisten anderen Postern eine Objektivierbarkeit der Zeit bezweifelst.


Ich denke, man muß ein paar Dinge auseinanderhalten. „Die Zeit“ ist ein menschengemachter Begriff, der für Modellvorstellungen steht, die sich im Laufe unserer Geschichte entwickelt haben. Davon unterscheiden sollte man unser Zeitgefühl, das ein Produkt unserer sozialen Entwicklung ist. Und schließlich Methoden zur Zeitmessung.

Beginnen wir mit den Vorstellungen, die wir heute, will heißen seit Einstein, von unserer Welt haben. Wir nennen es Raum-Zeit-Kontinuum, wobei Raum wie Zeit nicht einfach so da sind, sondern von den Objekten dieses Kontinuums gebildet werden, durch ihre Entstehung, ihre Ausdehnung wie ihre Bewegung, wobei leicht zu sehen ist, daß eine Bewegung im Raum auch immer eine in der Zeit ist. Das eine ist also vom anderen nicht zu trennen.

Was es also ganz offensichtlich gibt, sind Zeitabläufe, die Dauer von Prozessen, manche voneinander abhängig, andere nicht, manche wiederholen sich, andere nicht. „Die Zeit“ als etwas, was von der Dauer solcher Prozesse unabhängig ist, ist ein menschengemachter Begriff, und zwar meiner Ansicht nach einer, der überholt ist.

Wie alle Begriffe ist auch der der Zeit im Prozeß unserer gesellschaftlichen Entwicklung entstanden. War er ursprünglich einer zur Regelung sozialer Beziehungen, Bestimmung der Termine immer wiederkehrender Feste, Bestimmung der besten Zeit für die Aussaat, Ermittlung des „Willens der Götter“ durch Bestimmung von wiederkehrenden Himmelsereignissen, habe wir heute einen Zeitbegriff, der an den Vorstellungen der physikalischen Wissenschaften orientiert ist.

Zeitmessung passierte dabei immer und passiert bis heute durch den Vergleich von nicht oder nur schlecht wiederholbaren Vorgängen mit Ablaufmustern, die sich regelmäßig und mit einer großen Genauigkeit wiederholen lassen. So verwendete Galilei bei seinen Beschleunigungsversuchen zuerst nur seinen Pulsschlag als regelmäßiges Ablaufmuster, danach einen gleichmäßigen Wasserfluß; interessanterweise keine Uhren, obwohl es die damals schon gab. Sie dienten damals einem ganz anderen Zweck, nämlich der sozialen Zeitmessung, und waren ihm auch wohl nicht genau genug.

Heute wächst schon jedes Kind in den Industriestaaten mit der Uhr auf. Unsere Gesellschaften wären nicht funktionsfähig ohne eine sekundengenaue Synchronisation unserer sozialen Abläufe. Schon geringe Verspätungen unserer Verkehrs- und Transportmittel führen zu einem Chaos. Selbst die Drehung unserer Erde um sich selbst und ihr Umlauf um die Sonne ist uns scheinbar nicht mehr genau genug. Das alles ist uns so sehr „in Fleisch und Blut übergegangen“, daß wir es für die natürliche Form der Zeitwahrnehmung halten, obwohl unsere Kinder Jahre brauchen, um diese Vorstellungen zu erlernen. Daß sie das nicht ist, kann man selbst heute noch in vielen Ländern dieser Erde beobachten, und galt umso mehr in vergangenen Zeiten. Nichts desto trotz scheint dieser Umstand, daß Zeit eine menschengemachte Vorstellung ist, einigen hier sehr schwer zu fallen.

Ein Grund dafür mag sein, daß unsere Zeitmessung, und damit kommen wir zu ihrem objektiven Gehalt, mittlerweile einen so hohen Grad an Genauigkeit erreicht hat, daß wir sogar den Einfluß von räumlichen Veränderung, von wechselnden Geschwindigkeiten auf Geschehensabläufe berechnen können. Einsteins Relativitätstheorie hat dafür die gedanklichen Grundlage gebildet. Sie mag, wie jede wissenschaftliche Modellvorstellung, auch immer noch Fantasievorstellungen enthalten, also zukünftigen Verbesserungen zugänglich sein, aber sie ist doch zumindest so realistisch, daß eine großer Teil der Navigation auf unserer Erde darauf beruht.

Allerdings zeigt das zweierlei. Einerseits hat die Relativitätstheorie die Vorstellung einer unveränderlichen Zeit, gewissermaßen unabhängig von den sich bewegenden Objekten, ins Reich der Fantasie verwiesen. Das ist also keine Frage der Philosophie, sondern ein durch Tatsachenbeobachtungen recht gut belegtes wissenschaftliches Modell.

Und andererseits hat unserer Zeitgefühl damit nicht Schritt gehalten. Wir sind von Kindesbeinen an so an den gleichmäßigen, ja unerbittlichen Takt unserer Uhren gewöhnt, daß wir „Zeit“ selbst für eine Art von Objekt halten, eine Art von „Strom“. Wir denken uns die Objekte, uns selbst wohl eingeschlossen, wie etwas Zeitloses, was in diesem Strom schwimmt. Daher wohl auch die beliebte Vorstellung der SF, den Strom der Zeit umkehren zu können, was schon deshalb nicht geht, weil es diesen Strom nicht gibt.

#83:  Autor: smallie BeitragVerfasst am: 11.01.2017, 23:01
    —
zelig hat folgendes geschrieben:
smallie hat folgendes geschrieben:
zelig hat folgendes geschrieben:
Das Statement über die fehlende direkte sinnliche Wahrnehmung von Zeit, würde ich insofern verteidigen, als daß wir über kein zeitliches Korrelat zur räumlich ausgedehnten Substanz verfügen.

Du verteidigst das hartnäckig, ich kann's per Selbsterfahrung ebenso hartnäckig nicht nachvollziehen. Könnte es sich hier am Ende um eine mentale Typsache handeln? So wie introvertiert vs. extrovertiert? Dann wärst du ein Raum-Typ und ich ein Zeit-Typ.


Interessante Typologie. Obwohl Menschen sicherlich unterschiedliche räumliche und zeitliche Wahrnehmungen haben, glaube ich nicht, daß man sie in diese 2 Kategorien einteilen könnte. ; )

*Geh zurück auf Los und ziehe keine 4000 ein*

Dann bleibt die Frage offen, warum unsere Vorstellungswelt inkompatibel ist. Ich könnte noch einen weiteren Absatz an Beispielen zeitlich ausgedehnter Analogien bringen - vom 3-Minuten-Ei bis zu der knappen Stunde, die brauner Reis braucht. Das ist für mich so offensichtlich, daß ich nicht verstehe, wie du das anders empfinden kannst.

Ich denke bei der Sache auch an die östliche Vorstellung von der Welt als ewigem Wandel oder mit Heraklit an "alles fließt".


zelig hat folgendes geschrieben:
smallie hat folgendes geschrieben:

Eine Sache fehlt mir im Artikel: in "Bunkerversuchen" stellte sich heraus, das Menschen ohne Kontakt zum Tag/Nacht-Rhythmus zu 25-Stunden-Tagen neigen. Falls das stimmt, gäbe es eine Art innere Uhr.


Aus meiner Sicht wäre die schwankend-unzuverlässige Wahrnehmung zeitlicher Abläufe eher ein Argument pro meiner Ausgangsthese: Ohne die Möglichkeit, genau definierte Zyklen mittels gegenständlicher Wahrnehmung abzählen zu können, verliert sich die zeitliche Wahrnehmung ins Ungefähre. Die verstrichene Zeit in der Isolation kann nur noch geschätzt werden. Und wenn ich mich richtig erinnere, gibt es Berichte über Langzeitisolationen in Höhlen, mit dem Verlust der Fähigkeit, die verstrichene Zeit auch nur annähernd richtig einzuschätzen.

Das kenn ich anders.


Zitat:
Cave Dwellers, 1938

Renowned sleep researcher Nathaniel Kleitman and a colleague spent a month underground to test the body’s natural rhythms.

The Mammoth Cave experiment demonstrated that human bodies maintain a roughly 24-hour temperature cycle even in the absence of external cues, and that our sleepiness ebbs and flows in synch with this temperature cycle. “This study was important, [a] step towards identifying the [human circadian rhythm],” says Siegel.

http://www.the-scientist.com/?articles.view/articleNo/45359/title/Cave-Dwellers--1938/



zelig hat folgendes geschrieben:
Ob du in der Isolation einen Zollstock dabei hast, der dir ermöglicht, auch nach langer Zeit räumliche Abstände ziemlich genau zu vermessen. Oder ob Du zufällig gerade eine Sanduhr im Gepäck dabei hattest, die dir ermöglicht hat, die verstrichene Dauer ziemlich genau zu vermessen, als du für ein halbes Jahr in einer Höhle verschütt gegangen bist - in beiden Fällen kannst Du die Messung nur vornehmen, weil etwas Gegenständliches -die Substanz- vorhanden war. Ein temporales, sinnlich erfahrbares Gegenstück zur Substanz existiert nicht. Danke daß Du mich gezwungen hast darüber nachzudenken.

Hmm. Substanz ist für dich also etwas statisches und unveränderliches?

Mist, jetzt fällt mir Drafi Deutscher ein: "Marmor, Stein und Eisen bricht ..." Sorry. Wenn mir jetzt darüber die Milch sauer wird, ist das ein gültiges Beispiel für die zeitabhängige Qualität von Substanzen? Läßt du radioaktive Substanzen gelten, die mit einer charakteristischen Halbwertzeit zerfallen?



Machen wir ein Experiment, das ohne Substanzen auskommt.

Ich denk' an drei meiner Lieblingsmusikstücke und du auch. Wir klopfen das Tempo und vergleichen dann mit dem Original. Das ganze mit 1000 Versuchspersonen. Dann hätten wir etwas in der Hand zur Genauigkeit des Kurzzeit-Zeitgefühls.

#84:  Autor: smallie BeitragVerfasst am: 11.01.2017, 23:18
    —
fwo hat folgendes geschrieben:
@ zelig

Dass unsere Wahrnehmung des Raumes letztlich auch sekundär ist, hatte ich hier schon mal beschrieben. Auch da sollten wir davon ausgehen, dass die Wissenschaft es schwer haben wird, ein eingebautes Metermaß für die Entfernungen jenseits der Armeslänge zu finden. Warum steht die Raumenpfindung trotzdem für Dich außer Frage?

Hab' in meine Introduction to Psychology geschaut. Dort ist eine Arbeit zur Größenwahrnehmung aus dem Jahr 1957 erwähnt. Untersucht wurde die Größe eines Bildes auf der Netzhaut und die zugehörige Entfernungsschätzung. Achtjährige schneiden dabei schlechter ab als Erwachsene.


Zitat:
Apparent visual size as a function of Distance for Children and Adults
Zeigler/Leibowitz - 1957




Größen- und Abstandswahrnehmung ist als mindestens teilweise erlernt.


fwo hat folgendes geschrieben:
Dass die circadiane Rhythmik irgendwann einmal versagt, wenn sie nicht regelmäßig durch Tageswechsel neu angestoßen wird, sagt nicht, dass dass sie nicht existiert. Das wäre ungefähr so, als würdest Du sagen, dass eine mechanische Uhr keine Unruh haben könne, weil ihr Federwerk irgendwann abgelaufen ist.

Wenn die Aussage von Kleitman richtig ist, dann versagt die circadiane Rhythmik gar nicht. Wo der Zyklus der Frau herkommt läßt sich an dieser Stelle ebenfalls hinterfragen.


fwo hat folgendes geschrieben:
Dass bei uns eine "Uhr" im Ultrakurzzeitbereich existieren muss, darauf hatte ich im selben Post wie zum Raum auch schon hingewiesen.

Dass ein hinreichend präziser Taktgeber für den Kurzzeitbereich bisher nicht gefunden wurde, sagt nicht, dass es ihn nicht gibt. Ich würde ihn weiterhin als Hypothese stehen lassen.

Wie sonst könnten Musiker synchron bleiben?

Meine Spekulation wäre, daß es keine Uhr im eigentlichen Sinn gibt, sondern das die Zahl der "Zeitscheiben" herangezogen wird. Die zeitliche Auflösung unserer Wahrnehmung ist begrennzt. Am feinsten dürfte Auge und Gehör auflösen. Irgendwo zwischen 1/20tel oder 1/15tel Sekunde. Musikalisch entspricht das etwa einer 32tel Note bei 120 bpm.

#85:  Autor: Kramer BeitragVerfasst am: 11.01.2017, 23:24
    —
smallie hat folgendes geschrieben:

Meine Spekulation wäre, daß es keine Uhr im eigentlichen Sinn gibt, sondern das die Zahl der "Zeitscheiben" herangezogen wird. Die zeitliche Auflösung unserer Wahrnehmung ist begrennzt. Am feinsten dürfte Auge und Gehör auflösen. Irgendwo zwischen 1/20tel oder 1/15tel Sekunde. Musikalisch entspricht das etwa einer 32tel Note bei 120 bpm.


Dazu habe ich vor einigen Tagen schon mal was geschrieben:

Kramer hat folgendes geschrieben:
der kleine Fritz hat folgendes geschrieben:
Woran erkennt man denn beim Meßsubjekt wann dessen „Jetzt“ beginnt und wann es endet, um seine Dauer überhaupt messen zu können, falls eine solche Messung auch möglich ist!


So eine Messung ist möglich. Die kannst Du selber vornehmen. Die Grundlage für diese Messung ist ein reales Problem: Die Latenz bei digitalen Aufnahmesystemen.

Situation ohne Latenz: Du sitzt als Musiker mit Deinem Musikinstrument im Studio und spielst eine Taste, zupfst eine Seite, singst einen Ton und hörst das Resultat direkt über Deinen Monitor/Kopfhörer - quasi in Lichtgeschwindigkeit - ohne Verzögerung.

Situation mit Latenz: Du sitzt in einem Studio mit digitaler Aufnahmetechnik, das Signal muss erst von analog zu digital gewandelt werden, dann digital verarbeitet werden und anschliessend wieder von digital zu analog gewandelt werden. Dabei vergeht Zeit, bis Du z.B. den Tastendruck auf einen Keyboard hörst. Diese Zeit nennt man Latenz. Man könnte sie ja auch "Jetzt-Effekt" nennen.

Bei Latenzen um 5-7 Millisekunden haben die meisten Menschen das Gefühl, dass ihre Aktion (Tastendruck, das Zupfen einer Seite) gleichzeitig mit der akustischen Reaktion erfolgt. Ab Latenzen über 10 Millisekunden fangen viele Musiker an, sich unwohl zu fühlen und nicht mehr korrekt im Rhythmus spielen zu können. Das heisst: Das was ich "jetzt" mache, ist nicht mehr vereinbar mit dem, was ich als Reaktion auf diese "jetzige" Handlung erwarte. Diese Schwelle ist bei allen Menschen unterschiedlich und auch abhängig von der Aktion, aber ab 10 Millisekunden steigt die Wahrscheinlichkeit, dass jemand erkennt, dass "Jetzt" und "Jetzt" nicht mehr identisch sind. Das hängt halt auch davon ab, wie tempogenau Du agieren musst. Schlagzeuger müssen da mehr Feeling für das Mikrotiming haben als die Typen aus der Streicher-Sektion.

Du kannst das gerne mal selber ausprobieren. Nimm Dir ein Mikro und versuche einen vernünftigen Satz zu sagen, während Du einen Kopfhörer auf hast, über den Du Deine Worte um 20 oder 30 Millisekunden verzögert hörst. Das wird Dich total irritieren. Sobald die Verzögerungszeit aber in Richtung 10 Millisekunden geht, wird das Problem verschwinden. Und unter 7 Millisekunden wirst Du kaum einen Unterschied zum normalen Sprechen feststellen. Das ist keine Esoterik, sondern Praxiserfahrung.

#86:  Autor: fwoWohnort: im Speckgürtel BeitragVerfasst am: 12.01.2017, 03:12
    —
Kramer hat folgendes geschrieben:
....
Kramer hat folgendes geschrieben:
....
Du kannst das gerne mal selber ausprobieren. Nimm Dir ein Mikro und versuche einen vernünftigen Satz zu sagen, während Du einen Kopfhörer auf hast, über den Du Deine Worte um 20 oder 30 Millisekunden verzögert hörst. Das wird Dich total irritieren. Sobald die Verzögerungszeit aber in Richtung 10 Millisekunden geht, wird das Problem verschwinden. Und unter 7 Millisekunden wirst Du kaum einen Unterschied zum normalen Sprechen feststellen. Das ist keine Esoterik, sondern Praxiserfahrung.

Das war jetzt ein Beispiel für Zeitwahrnehmung durch die Wahrnehmung einer künstlichen Asynchronität zwischen eigener Lauterzeugung und Lautwahrnehmung.

Ich hatte die Klickortung schon einmal erwähnt. Bei der Klickortung, wie sie heute zur Unterstützung für Blinde gelehrt wird, wird im Bereich 0,5 bis 12m eine Tiefenauflösung von 10cm angegeben. Um das leisten zu können, ist eine zeitliche Auflösung von unter einer Millisekunde nötig.

#87:  Autor: fwoWohnort: im Speckgürtel BeitragVerfasst am: 12.01.2017, 03:24
    —
smallie hat folgendes geschrieben:
....
Meine Spekulation wäre, daß es keine Uhr im eigentlichen Sinn gibt, sondern das die Zahl der "Zeitscheiben" herangezogen wird. Die zeitliche Auflösung unserer Wahrnehmung ist begrennzt. Am feinsten dürfte Auge und Gehör auflösen. Irgendwo zwischen 1/20tel oder 1/15tel Sekunde. Musikalisch entspricht das etwa einer 32tel Note bei 120 bpm.

Ich habe jetzt keinen Link dafür greifbar, aber die zeitliche Auflösung des Auges ist kontrastabhängig. Bei starkem Kontrast zur Umgebung (deshalb ist das Kino abgedunkelt und beim klassischen Kino hat das Bild auf der Leinwand keine Verweildauer) sind wir in der Lage etwa 15 bis 18 Bilder pro Sekunde noch als zerhackt wahrzunehmen. Wenn die Bilder nicht aus dem Dunkeln leuchten, beginnt das Bild erst bei 60 Bildern ruhig zu stehen.

#88:  Autor: Kramer BeitragVerfasst am: 12.01.2017, 03:44
    —
fwo hat folgendes geschrieben:
Um das leisten zu können, ist eine zeitliche Auflösung von unter einer Millisekunde nötig.


Das halte ich für durchaus möglich. Aber was man dabei hört, ist kein zeitlicher Unterschied zwischen dem Originalsignal und dem Echo, sondern eine Verfärbung des Signals durch die Phasenverschiebung beider Signale. Da löschen sich bestimmte Frequenzen aus, während sich andere verstärken. Für Sehende gibt es keinen Grund, ihr Gehör für solche Unterschiede zu trainieren, aber auch sie hören den Unterschied - er wird sogar in der Musikproduktion als Effekt genutzt: https://de.wikipedia.org/wiki/Phaser_(Musik)

#89:  Autor: Kramer BeitragVerfasst am: 12.01.2017, 03:57
    —
fwo hat folgendes geschrieben:

Ich habe jetzt keinen Link dafür greifbar, aber die zeitliche Auflösung des Auges ist kontrastabhängig. Bei starkem Kontrast zur Umgebung (deshalb ist das Kino abgedunkelt und beim klassischen Kino hat das Bild auf der Leinwand keine Verweildauer) sind wir in der Lage etwa 15 bis 18 Bilder pro Sekunde noch als zerhackt wahrzunehmen. Wenn die Bilder nicht aus dem Dunkeln leuchten, beginnt das Bild erst bei 60 Bildern ruhig zu stehen.


Das Auge ist auf jeden Fall träger als das Gehör. 60 Bilder pro Sekunde wären im Audiobereich eine Auflösung von 60 Hz. Damit könnte man nicht mal eine Tuba entsprechend aufzeichnen. Eine CD hat eine Auflösung/Samplerate von 44.100 HZ. Was mich zu einem anderen Punkt der Wahrnehmung von Zeit bringt: Die Tonhöhe. Vogelgezwitscher könnte man mit einen Gehör, das nur eine 60-Hz-Auflösung hat, gar nicht wahrnehmen. Wir nehmen Zeit nicht nur durch zeitliche Differenzen von verschiedenen Signalen war, sondern Veränderungen in der Zeit als unterschiedliche Tonhöhen. Und bevor wir überhaupt ein Konzept davon haben, was Raum ist, nehmen wir vor der Geburt die Stimme unserer Mutter wahr - und nach der Geburt die Stimme unserer Mutter und unseres Vaters. Wir hören subjektiv eine höhere und eine tiefere Stimme, mit denen wir eine bestimmte "Funktion" und "Vertrautheit" verbinden, aber was wir faktisch hören, sind nur zeitlich unterschiedlich schnelle Schwingungen.

#90:  Autor: SkeptikerWohnort: 129 Goosebumpsville BeitragVerfasst am: 12.01.2017, 08:28
    —
Marcellinus hat folgendes geschrieben:
Er_Win hat folgendes geschrieben:
Liege ich ansonsten richtig, dass du im Gegensatz zu den meisten anderen Postern eine Objektivierbarkeit der Zeit bezweifelst.


Ich denke, man muß ein paar Dinge auseinanderhalten. „Die Zeit“ ist ein menschengemachter Begriff, der für Modellvorstellungen steht, die sich im Laufe unserer Geschichte entwickelt haben.


@Marcellinus: Wäre es möglich, auf Er_Wins Frage zu antworten?

Bezweifelst du die Objektivierbarkeit der Zeit?

#91:  Autor: der kleine FritzWohnort: Planet Erde BeitragVerfasst am: 12.01.2017, 11:20
    —
Skeptiker hat folgendes geschrieben:
Marcellinus hat folgendes geschrieben:
Er_Win hat folgendes geschrieben:
Liege ich ansonsten richtig, dass du im Gegensatz zu den meisten anderen Postern eine Objektivierbarkeit der Zeit bezweifelst.


Ich denke, man muß ein paar Dinge auseinanderhalten. „Die Zeit“ ist ein menschengemachter Begriff, der für Modellvorstellungen steht, die sich im Laufe unserer Geschichte entwickelt haben.


@Marcellinus: Wäre es möglich, auf Er_Wins Frage zu antworten?

Bezweifelst du die Objektivierbarkeit der Zeit?


Hallo Skeptiker

Was verstehst du unter Objektivierbarkeit der Zeit?

Das heißt doch nach meinem Sprachverständnis, das Mysterium „Zeit“ ist nicht, kann aber zu einem Objekt gemacht werden und damit Masse erhalten.
Wie die Verwandlung von Nichts in materielle „Zeit“ vor sich geht bedarf natürlich der wissenschaftlichen Erklärung, die du mir ja versprochen hattest!

Bitte erlöse mich von meinen Zweifeln!

@marcellinus:

Da kann ich dir natürlich beiplichten, denn ohne den Beobachter/Beurteiler Mensch käme niemand auf die aburde Idee im Universum zerren Raum und Zeit aneinender herum! Mit den Augen rollen

#92:  Autor: fwoWohnort: im Speckgürtel BeitragVerfasst am: 12.01.2017, 11:26
    —
Kramer hat folgendes geschrieben:
fwo hat folgendes geschrieben:

Ich habe jetzt keinen Link dafür greifbar, aber die zeitliche Auflösung des Auges ist kontrastabhängig. Bei starkem Kontrast zur Umgebung (deshalb ist das Kino abgedunkelt und beim klassischen Kino hat das Bild auf der Leinwand keine Verweildauer) sind wir in der Lage etwa 15 bis 18 Bilder pro Sekunde noch als zerhackt wahrzunehmen. Wenn die Bilder nicht aus dem Dunkeln leuchten, beginnt das Bild erst bei 60 Bildern ruhig zu stehen.


Das Auge ist auf jeden Fall träger als das Gehör. 60 Bilder pro Sekunde wären im Audiobereich eine Auflösung von 60 Hz. Damit könnte man nicht mal eine Tuba entsprechend aufzeichnen. Eine CD hat eine Auflösung/Samplerate von 44.100 HZ. Was mich zu einem anderen Punkt der Wahrnehmung von Zeit bringt: Die Tonhöhe. Vogelgezwitscher könnte man mit einen Gehör, das nur eine 60-Hz-Auflösung hat, gar nicht wahrnehmen. Wir nehmen Zeit nicht nur durch zeitliche Differenzen von verschiedenen Signalen war, sondern Veränderungen in der Zeit als unterschiedliche Tonhöhen. Und bevor wir überhaupt ein Konzept davon haben, was Raum ist, nehmen wir vor der Geburt die Stimme unserer Mutter wahr - und nach der Geburt die Stimme unserer Mutter und unseres Vaters. Wir hören subjektiv eine höhere und eine tiefere Stimme, mit denen wir eine bestimmte "Funktion" und "Vertrautheit" verbinden, aber was wir faktisch hören, sind nur zeitlich unterschiedlich schnelle Schwingungen.

Da bringst Du jetzt was durcheinander: Die Tonhöhe wird nicht über eine zeitliche Auflösung differenziert, sondern über eine örtliche anhand von örtlich bestimmbaren Resonanzen in der Cochlea. Hier geht es also um die Wellenlänge. (Ich habe zur Sicherheit gerade selber nachgesehen, ob mein inzwischen Jahrzehnte altes Wissen zur Sinnesphysiologie noch up to date ist - ist es zu meiner Erleichterung)

#93:  Autor: Marcellinus BeitragVerfasst am: 12.01.2017, 11:37
    —
Skeptiker hat folgendes geschrieben:
Marcellinus hat folgendes geschrieben:
Er_Win hat folgendes geschrieben:
Liege ich ansonsten richtig, dass du im Gegensatz zu den meisten anderen Postern eine Objektivierbarkeit der Zeit bezweifelst.


Ich denke, man muß ein paar Dinge auseinanderhalten. „Die Zeit“ ist ein menschengemachter Begriff, der für Modellvorstellungen steht, die sich im Laufe unserer Geschichte entwickelt haben.


@Marcellinus: Wäre es möglich, auf Er_Wins Frage zu antworten?

Bezweifelst du die Objektivierbarkeit der Zeit?


Lies einfach nach, was ich geschrieben habe. "Zeit" ist eine Modellvorstellung. Entsprechend der Einstein'schen Relativitätstheorie begreifen Physiker sie heute als Aspekt des vierdimensionalen Raum-Zeit-Kontinuums, und dieser Aspekt ist ziemlich gut durch Tatsachenbeobachtungen bestätigt. Wenn du das unter Objektivierbarkeit verstehst, dann ist es das. Aber es bleibt ein menschengemachtes Modell, wie realistisch auch immer, und morgen mögen wir Dinge entdecken, die in diesem Modell bisher nicht berücksichtig wurden, die falsch sind oder unvollständig. "Das Ding an sich" ist eine Vorstellung der Philosophie, nicht der Wissenschaft.

#94:  Autor: fwoWohnort: im Speckgürtel BeitragVerfasst am: 12.01.2017, 11:46
    —
Kramer hat folgendes geschrieben:
fwo hat folgendes geschrieben:
Um das leisten zu können, ist eine zeitliche Auflösung von unter einer Millisekunde nötig.


Das halte ich für durchaus möglich. Aber was man dabei hört, ist kein zeitlicher Unterschied zwischen dem Originalsignal und dem Echo, sondern eine Verfärbung des Signals durch die Phasenverschiebung beider Signale. Da löschen sich bestimmte Frequenzen aus, während sich andere verstärken. Für Sehende gibt es keinen Grund, ihr Gehör für solche Unterschiede zu trainieren, aber auch sie hören den Unterschied - er wird sogar in der Musikproduktion als Effekt genutzt: https://de.wikipedia.org/wiki/Phaser_(Musik)

Das kommt in Nahbereich sicher dazu, aber auf der von mir verlinkten Website sprechen sie selbst von Verzögerung des Echos, also Laufzeitlänge:
Zitat:
Echos können ab einer Entfernung von 30cm bis etwa 200m interpretiert werden. Objekte, die kleiner als etwa 2cm sind können aufgrund der minimal erzeugbaren Wellenlänge nicht wahrgenommen werden. Für die Nähe ist der schwache hohe Knall, der vorn am Gaumen mit E-Lippen erzeugt wird besser, da er wegen der kleinen Wellenlänge eine hohe Auflösung ermöglicht. Der weiter hinten am Gaumen erzeugte Power-Klick mit offenen O-Lippen reicht weiter, bei geringerer Auflösung. Der Schall wird von unterschiedlichen Materialien relativ eindeutig unterschiedlich reflektiert, dadurch kann eine Vielzahl von Oberflächen und Materialdichten unterschieden werden. Die Entfernung des Objektes zum „Betrachter“ ist, besser sogar als von Sehenden, durch die Verzögerung des Echos sehr genau hörbar (im Bereich 50cm bis 12m können Unterschiede von 10cm gehört werden). Und nicht zuletzt sogar die Form eines Objektes ist durch mehrmaliges Zungenklicken gut erkennbar.

Für Überlagerungen, wie Du sie ansprichst, brächtest Du zu einer guten Interpretierbarkeit auch einen sauberen Dauerton, die arbeiten aber mit "Klicks", also sehr kurzen Explosionslauten, die bei größeren Entfernungen schon wegen ihrer eigenen zeitlichen Ausdehnung am Ort der Erzeugung und Wahrnehmung keine Überlagerung mehr herstellen können.

Um noch auf ein etwas eindeutiger zu interpretierendes Phänomen hinzuweisen, das Du als Musiker wahrscheinlich aus der Theorie der Stereophonie besser kennst als ich, ist die Ortung externer Schallquellen, die im unteren Frequenzbereich nur über Laufzeitdifferenzen wahrgenommen wird. Da kommen wir zu einer vergleichbaren zeitlichen Auflösung.

#95:  Autor: der kleine FritzWohnort: Planet Erde BeitragVerfasst am: 12.01.2017, 11:50
    —
Kramer hat folgendes geschrieben:


Wir nehmen Zeit nicht nur durch zeitliche Differenzen von verschiedenen Signalen war, sondern Veränderungen in der Zeit als unterschiedliche Tonhöhen.


Hallo Kramer

Und wer das Pech hat taub zu sein? Traurig

Ich bin zwar nicht taub, kann aber trotzdem auch keine Zeit wahrnehmen, ich hör da einfach nichts. Mit welchem Sinn nimmst du denn „Zeit“ wahr?

#96:  Autor: fwoWohnort: im Speckgürtel BeitragVerfasst am: 12.01.2017, 12:03
    —
der kleine Fritz hat folgendes geschrieben:
....ohne den Beobachter/Beurteiler Mensch käme niemand auf die aburde Idee im Universum zerren Raum und Zeit aneinender herum! Mit den Augen rollen

Welch bahnbrechende Erkenntnis. Aber ich gebe zu bedenken:
Ohne den Beobachter/Beurteiler Mensch käme auch niemand auf die absurde Idee, zu behaupten, die Erde drehte sich um die Sonne. Aber wir gehen davon aus, dass sie das auch täte, wenn die Evolution bei den Einzellern steckengeblieben wäre und niemand in unserer Weise zusähe, wir wir auch davon ausgehen, dass sie es vor etwa 4 Milliarden Jahren schon tat, als die ersten Einzeller auf der Erde begannen, sich zu entwickeln.

#97:  Autor: zelig BeitragVerfasst am: 12.01.2017, 12:25
    —
smallie hat folgendes geschrieben:
zelig hat folgendes geschrieben:
smallie hat folgendes geschrieben:
zelig hat folgendes geschrieben:
Das Statement über die fehlende direkte sinnliche Wahrnehmung von Zeit, würde ich insofern verteidigen, als daß wir über kein zeitliches Korrelat zur räumlich ausgedehnten Substanz verfügen.

Du verteidigst das hartnäckig, ich kann's per Selbsterfahrung ebenso hartnäckig nicht nachvollziehen. Könnte es sich hier am Ende um eine mentale Typsache handeln? So wie introvertiert vs. extrovertiert? Dann wärst du ein Raum-Typ und ich ein Zeit-Typ.


Interessante Typologie. Obwohl Menschen sicherlich unterschiedliche räumliche und zeitliche Wahrnehmungen haben, glaube ich nicht, daß man sie in diese 2 Kategorien einteilen könnte. ; )

*Geh zurück auf Los und ziehe keine 4000 ein*

Dann bleibt die Frage offen, warum unsere Vorstellungswelt inkompatibel ist. Ich könnte noch einen weiteren Absatz an Beispielen zeitlich ausgedehnter Analogien bringen - vom 3-Minuten-Ei bis zu der knappen Stunde, die brauner Reis braucht. Das ist für mich so offensichtlich, daß ich nicht verstehe, wie du das anders empfinden kannst.

Ich denke bei der Sache auch an die östliche Vorstellung von der Welt als ewigem Wandel oder mit Heraklit an "alles fließt".

Ich habe den Eindruck, in das Statement wird zuviel hineininterpretiert. Was auch immer „inkompatibel“ sein soll, bedenke, ich spekuliere damit nur über den Grund, warum Zeit und Raum von Menschen kategorial unterschiedlich wahrgenommen werden.

smallie hat folgendes geschrieben:
zelig hat folgendes geschrieben:
Ob du in der Isolation einen Zollstock dabei hast, der dir ermöglicht, auch nach langer Zeit räumliche Abstände ziemlich genau zu vermessen. Oder ob Du zufällig gerade eine Sanduhr im Gepäck dabei hattest, die dir ermöglicht hat, die verstrichene Dauer ziemlich genau zu vermessen, als du für ein halbes Jahr in einer Höhle verschütt gegangen bist - in beiden Fällen kannst Du die Messung nur vornehmen, weil etwas Gegenständliches -die Substanz- vorhanden war. Ein temporales, sinnlich erfahrbares Gegenstück zur Substanz existiert nicht. Danke daß Du mich gezwungen hast darüber nachzudenken.

Hmm. Substanz ist für dich also etwas statisches und unveränderliches?

Nein. Ich verwende den Begriff um einen philosophischen Kontext zu erzeugen, damit die Spekulation nicht als Aussage über Materie, also die physische Welt missverstanden werden kann. Das ist der einzige, vielleicht übertriebene Grund


smallie hat folgendes geschrieben:

Mist, jetzt fällt mir Drafi Deutscher ein: "Marmor, Stein und Eisen bricht ..." Sorry. Wenn mir jetzt darüber die Milch sauer wird, ist das ein gültiges Beispiel für die zeitabhängige Qualität von Substanzen? Läßt du radioaktive Substanzen gelten, die mit einer charakteristischen Halbwertzeit zerfallen?
Machen wir ein Experiment, das ohne Substanzen auskommt.
Ich denk' an drei meiner Lieblingsmusikstücke und du auch. Wir klopfen das Tempo und vergleichen dann mit dem Original. Das ganze mit 1000 Versuchspersonen. Dann hätten wir etwas in der Hand zur Genauigkeit des Kurzzeit-Zeitgefühls.

Das verstehe ich nicht, glaube aber, das hat auch nichts mit dem Gegenstand unserer Diskussion zu tun.

#98:  Autor: fwoWohnort: im Speckgürtel BeitragVerfasst am: 12.01.2017, 12:36
    —
der kleine Fritz hat folgendes geschrieben:
...Mit welchem Sinn nimmst du denn „Zeit“ wahr?

Nicht mit einem bestimmten Sinn nehmen wir die Zeit wahr, sondern aus der Interpretation der Signale, die über unterschiedliche Sinne bei uns einlaufen.

Es ist wie beim Raum: Dafür haben wir auch keinen eingebauten Zollstock, sondern wir nehmen den dreidimensionalen Raum über die Auswertung von Druck-, Licht- oder Tonsignalen wahr.

Gegenfrage an Dich, um mal von der Kalibrierung und Vermessung wegzukommen und das etwas basaler anzugehen: Woher weißt Du von der zeitlichen Reihenfolge von Ereignissen, wenn Du Zeit nicht wahrnimmst? Warum schaffst Du es nicht, alle Threads dieses Forums gleichzeitig zu bedienen und auch Posts zu beantworten, die erst morgen geschrieben werden? Warum meinst Du überhaupt, so alt zu sein wie Du bist und nicht noch gar nicht geboren?

Und wie erklärst Du Dir, dass etwa wir beide, die über die Zeit anscheinend völlig uneins sind, keine Differenzen über die Reihenfolge unserer Posts haben und uns wahrscheinlich auch über die Tage Vorgestern und Gestern und unsere Pläne für Morgen unterhalten können?

#99:  Autor: zelig BeitragVerfasst am: 12.01.2017, 12:54
    —
Kramer hat folgendes geschrieben:
zelig hat folgendes geschrieben:
Aber es scheint doch so zu sein, daß Konzepte von Raum intuitiv eingänglicher sind, als Konzepte von Zeit.


Vielleicht liegt das daran, dass die Räume/Entfernungen, in denen wir uns bewegen, klein genug sind, um sie zu begreifen. Das wirkliche Ausmass des Raumes (also des Universums) ist für uns ebenso unbegreiflich, wie die Ewigkeit. Ich sehe da durchaus eine Analogie zur Zeit: Wir begreifen den Raum, weil wir ihn verstehbare Einheiten einteilen. Mein Zimmer, mein Haus, mein Hof, mein Dorf, die Strecke zum nächsten Dorf usw. Bei der Zeit machen wir etwas Ähnliches, wir teilen sie in begreifbare Abschnitte ein, wir haben z.B. einen Tagesrhythmus, bestimmte Zeiten, zu denen wir Essen usw. Wird uns dieser Rhythmus genommen, verlieren wir die Orientierung in der Zeit. So wie wir die Orientierung im Raum verlieren würden, wenn man uns in eine total fremde Umgebung beamen würde.


Das kann ich nachvollziehen, und sehe es ähnlich. Wir existieren in einem bestimmten Skalenbereich, den Mesokosmos, der die Randbedingungen der Erfahrbarkeit setzt. Wir erfassen die Wirklichkeit in kleinen, mehr oder weniger willkürlich gesetzten Einheiten. Wobei die Begriffe Ewigkeit und Unendlichkeit nach meiner Vorstellung eigentlich die Aufhebung von Raum und Zeit bedeuten. Was ist jenseits vom Big Bang? Jedenfalls etwas anderes als die Raumzeit, in der wir existieren.

edit: umformuliert

#100:  Autor: Kramer BeitragVerfasst am: 12.01.2017, 16:00
    —
fwo hat folgendes geschrieben:

Da bringst Du jetzt was durcheinander: Die Tonhöhe wird nicht über eine zeitliche Auflösung differenziert, sondern über eine örtliche anhand von örtlich bestimmbaren Resonanzen in der Cochlea. Hier geht es also um die Wellenlänge.


Es handelt sich um eine zeitliche Auflösung, die räumlich gemessen wird. Eigentlich man das bei einer Welle kaum trennen.

#101:  Autor: fwoWohnort: im Speckgürtel BeitragVerfasst am: 12.01.2017, 16:23
    —
Kramer hat folgendes geschrieben:
fwo hat folgendes geschrieben:

Da bringst Du jetzt was durcheinander: Die Tonhöhe wird nicht über eine zeitliche Auflösung differenziert, sondern über eine örtliche anhand von örtlich bestimmbaren Resonanzen in der Cochlea. Hier geht es also um die Wellenlänge.


Es handelt sich um eine zeitliche Auflösung, die räumlich gemessen wird. Eigentlich man das bei einer Welle kaum trennen.

Ich trenne es deshalb, weil wir (d.h. der Organismus) hier die Tonhöhe, also die Frequenz selbst als Qualität verarbeiten, wir sind an dieser Stelle nicht an der Information Zeit interessiert wie bei der Geschwindigkeits- oder der Raumempfindung. Wenn Du diese Trennung nicht machst, kannst Du bei der Wahrnehmung der Farben, die auf den unterschiedlichen Energiefrachten beruht, auf noch viel höhere Frequenzen, also kürzere Zeiten kommen. Aber auch da ist die weiterverarbeitete Information die Frequenz als eigene Qualität und nicht irgendeine Zeit.

Ich bin also dafür, den Begriff der zeitlichen Auflösung innerhalb unserer Sinne auf den Bereich zu beschränken, in dem es die Information der Dauer eines Vorgangs die ist, die der Organismus weiterverarbeitet.

#102:  Autor: Kramer BeitragVerfasst am: 12.01.2017, 16:28
    —
fwo hat folgendes geschrieben:

Das kommt in Nahbereich sicher dazu, aber auf der von mir verlinkten Website sprechen sie selbst von Verzögerung des Echos, also Laufzeitlänge:
Zitat:
Echos können ab einer Entfernung von 30cm bis etwa 200m interpretiert werden. Objekte, die kleiner als etwa 2cm sind können aufgrund der minimal erzeugbaren Wellenlänge nicht wahrgenommen werden. Für die Nähe ist der schwache hohe Knall, der vorn am Gaumen mit E-Lippen erzeugt wird besser, da er wegen der kleinen Wellenlänge eine hohe Auflösung ermöglicht. Der weiter hinten am Gaumen erzeugte Power-Klick mit offenen O-Lippen reicht weiter, bei geringerer Auflösung. Der Schall wird von unterschiedlichen Materialien relativ eindeutig unterschiedlich reflektiert, dadurch kann eine Vielzahl von Oberflächen und Materialdichten unterschieden werden. Die Entfernung des Objektes zum „Betrachter“ ist, besser sogar als von Sehenden, durch die Verzögerung des Echos sehr genau hörbar (im Bereich 50cm bis 12m können Unterschiede von 10cm gehört werden). Und nicht zuletzt sogar die Form eines Objektes ist durch mehrmaliges Zungenklicken gut erkennbar.


Dort steht nur, dass die Entfernung durch die Verzögerung des Echos sehr genau hörbar ist. Ob dass durch die tatsächliche Wahrnehmung zweier unterschiedlicher Signale (Originalsignal und Echo) oder durch die Klangunterschiede durch die Überlagerung beider Signale geschieht, steht dort nicht. Ich halte es jedenfalls für sehr unwahrscheinlich, dass man Laufzeitunterschiede von unter 1 ms als Echo wahrnehmen kann.

Zitat:
Für Überlagerungen, wie Du sie ansprichst, brächtest Du zu einer guten Interpretierbarkeit auch einen sauberen Dauerton, die arbeiten aber mit "Klicks", also sehr kurzen Explosionslauten, die bei größeren Entfernungen schon wegen ihrer eigenen zeitlichen Ausdehnung am Ort der Erzeugung und Wahrnehmung keine Überlagerung mehr herstellen können.


Ein Dauerton wäre für grössere Entfernungen ungeeignet, also dort, wo tatsächlich anhand des Echos die Entfernung gemessen wird. Man hätte zwar auch hier eine Phasenauslöschung/verstärkung, aber die würde nichts mehr über die Entfernung aussagen. Je nach Tonhöhe/Wellenlänge wiederholt sich das Muster der Auslöschung und Verstärkung entlang der Schallwelle. Ausserdem würde das Gehör durch so einen Dauerton ermüden.

Auch im Nahbereich ist hier ein kurzer Klickton anwendbar, im Text ist ja die Rede davon, dass sogar die Form eines Objektes durch mehrmaliges Zungenklicken ermitteln lässt. Es steht zwar nicht im Text, aber ich gehe davon aus, dass die Anwender dabei in mehrere Richtungen klicken und anhand der Klangunterschiede Aussagen über Entfernung Form eines Objekts machen können.

#103:  Autor: Kramer BeitragVerfasst am: 12.01.2017, 18:30
    —
fwo hat folgendes geschrieben:

Ich trenne es deshalb, weil wir (d.h. der Organismus) hier die Tonhöhe, also die Frequenz selbst als Qualität verarbeiten, wir sind an dieser Stelle nicht an der Information Zeit interessiert wie bei der Geschwindigkeits- oder der Raumempfindung.


Ich trenne hier, weil es technisch erforderlich ist. Du kannst die Welt photographisch einfrieren, ohne dass sie an Farbenpracht verliert. Akustisch geht das nicht.

BTW: Ich habe gerade mal einen Test gemacht und diverse Klick-Laute durch ein Delay geschickt, das die Verzögerung in 0,1ms-Schritten auflöst. Je nach Beschaffenheit des Clicks konnte ich ab ca. 5,5ms eine zeitliche Verzögerung wahrnehmen, die sich allerdings eher wie ein kurzes Kratzen anhörte. Es klang immer noch wie ein Signal, mit einem etwas breiteren und rauerem Einschwingvorgang. Ab ca. 7 ms konnte ich zwei verschiedene Signale erahnen. Im Bereich unter 5ms habe ich nur Verfärbungen gehört, die umso intensiver wurden, je kürzer das Echo war. Man hört den Unterschied der Verfärbung z.B. bei dem Wechsel zwischen 0,1ms zu 0,2ms deutlicher, als zwischen 4,1ms und 4,2ms.

#104:  Autor: der kleine FritzWohnort: Planet Erde BeitragVerfasst am: 12.01.2017, 19:56
    —
fwo hat folgendes geschrieben:
der kleine Fritz hat folgendes geschrieben:
...Mit welchem Sinn nimmst du denn „Zeit“ wahr?



Gegenfrage an Dich, um mal von der Kalibrierung und Vermessung wegzukommen und das etwas basaler anzugehen: Woher weißt Du von der zeitlichen Reihenfolge von Ereignissen, wenn Du Zeit nicht wahrnimmst?


Hallo fwo

Ein Ereignis ist doch immer das Ergebnis der Bewegung eines Objektes im Raum.
Bewegungen sind determiniert, haben also immer eine physikalische Urasche die auch deren Reihenfolge bestimmt.

Wie soll das „wahrnehmen von Zeit“ als ein durchaus unterschiedlicher subjektiv/
individueller Vorgang, Einfluss auf die Reihenfolge von Ereignissen nehmen?

Das wäre zu erklären, wenn möglich dazu nicht die Ereignisse von 5 Milliarden Erdgeschichte Jahren erklären!

Wenn dich ein solches Ereignis näher interessiert, z, B, der von dir mal genannte 100 m Lauf, so kannst du dessen Dauer ja mit der genormten Dauer von Bewegungen vergleichen, damit du endlich mal zu deiner Zeit kommst! Lachen

#105:  Autor: fwoWohnort: im Speckgürtel BeitragVerfasst am: 12.01.2017, 20:19
    —
der kleine Fritz hat folgendes geschrieben:
fwo hat folgendes geschrieben:
der kleine Fritz hat folgendes geschrieben:
...Mit welchem Sinn nimmst du denn „Zeit“ wahr?



Gegenfrage an Dich, um mal von der Kalibrierung und Vermessung wegzukommen und das etwas basaler anzugehen: Woher weißt Du von der zeitlichen Reihenfolge von Ereignissen, wenn Du Zeit nicht wahrnimmst?


Hallo fwo

Ein Ereignis ist doch immer das Ergebnis der Bewegung eines Objektes im Raum.
Bewegungen sind determiniert, haben also immer eine physikalische Urasche die auch deren Reihenfolge bestimmt.

Wie soll das „wahrnehmen von Zeit“ als ein durchaus unterschiedlicher subjektiv/
individueller Vorgang, Einfluss auf die Reihenfolge von Ereignissen nehmen?
....

Das ist doch gar nicht die Frage. Du fragst jemanden anders, mit welchen Sinnen er denn die Zeit wahrnnähme, als könnte er das gar nicht.
Ich mache Dich nur darauf aufmerksam, dass Du die zeitliche Reihenfolge von Ereignissen wahrnimmst. Das kannst Du nicht, ohne auch Zeit wahrzunehmen. Und jetzt frage ich Dich, mit welchen Sinnen Du das denn machst. Beantworte doch einfach erstmal diese Frage.

der kleine Fritz hat folgendes geschrieben:
...
Wie soll das „wahrnehmen von Zeit“ als ein durchaus unterschiedlicher subjektiv/
individueller Vorgang, Einfluss auf die Reihenfolge von Ereignissen nehmen?
....

Woher soll ich das wissen? Die eigentliche Frage ist aber eine ganz andere: Woher weißt Du, dass es nicht geht, wenn Du keine Sensorik für dieses Objekt Zeit hast, mithin überhaupt keine Ahnung hast, worüber wir uns gerade unterhalten?

#106:  Autor: fwoWohnort: im Speckgürtel BeitragVerfasst am: 12.01.2017, 21:37
    —
Kramer hat folgendes geschrieben:
fwo hat folgendes geschrieben:

Ich trenne es deshalb, weil wir (d.h. der Organismus) hier die Tonhöhe, also die Frequenz selbst als Qualität verarbeiten, wir sind an dieser Stelle nicht an der Information Zeit interessiert wie bei der Geschwindigkeits- oder der Raumempfindung.


Ich trenne hier, weil es technisch erforderlich ist. Du kannst die Welt photographisch einfrieren, ohne dass sie an Farbenpracht verliert. Akustisch geht das nicht.
....

Jetzt wird es interessant.
Natürlich kannst Du den analogen Vorgang zu einer Photographie auch in der Akustik veranstalten: Du setzt einfach das Frequenzgemisch eines Augenblicks als konstantes Dauergeräusch fort.

Genau das mache ich nämlich mit dem Bild. Ich erzeuge einen konstanten Dauerreiz.

Der Unterschied liegt in unserer Auswertung, d.h. der Weise, in der wir die Information aus den unterschiedlichen Reizen entnehmen: Für uns wahrnehmbare Information ist immer Information über die Differenz verschiedener "Punkte" (wir werden noch sehen, warum ich die "" gesetzt habe).

Der Unterschied zwischen dem optischen und dem akustischen Dauerreiz liegt in der Struktur der unterschiedlichen Kanäle: Die Information eines Bildes liegt in den Differenzen zwischen den unterschiedlichen Bildpunkten, die (ich nehme jetzt nur das Bild eines Auges und verzichte auf Rauminformation) auf einer Kugelfläche angeordnet sind: Jeder Punkt ist definiert über die Winkel im Raum. Das Bild auf meiner Netzhaut zeigt also viele unterschiedliche Punkte mit unterschiedlichen Ausprägungen der Eigenschaft Farbe und enthält damit für mich Information, obwohl es theoretisch ein konstanter Dauerreiz ist.

Der akustische Dauerreiz ist für uns sinnlos, obwohl wir sein Freqenzspektrum grundsätzlich ähnlich aufzulösen imstande sind wie bei dem Bild. Aber die Wahrnehmung eines konstanten Geräusches stellt für uns keine Information dar, weil die Punkte, deren Differenz zueinander für uns Information bedeuten, Zeitpunkte sind und Zeitpunkte in einem konstanten Dauergeräusch keine Differenz besitzen - die alleinige Feststellung der Frequenzen in unserem Dauergeräusch ist also ohne Informationswert, oder andersherum: die physikalische Zeit einer Schwingung im Bereich hörbaren Schalls ist für uns ähnlich informativ wie die physikalische Zeit im Bereich der Schwingung sichtbaren Lichts - nämlich gar nicht. Noch einmal anders: Die Tonhöhe allein hat, genauso wie die Farbfrequenz, für uns keine zeitliche Bedeutung, obwohl beide physikalisch analog mit der Zeit verknüpft sind. Es besteht allerdings der Unterschied, dass die Zeit in unserem optischen Kanal eine mögliche Dimension ist, in unserem akustischen eine unerlässliche. Aber eben Zeit in einem anderen Maßstab als dem, der im Modulationsumfang der der Träger eine Rolle spielt.

#107:  Autor: Kramer BeitragVerfasst am: 12.01.2017, 23:21
    —
fwo hat folgendes geschrieben:

Natürlich kannst Du den analogen Vorgang zu einer Photographie auch in der Akustik veranstalten: Du setzt einfach das Frequenzgemisch eines Augenblicks als konstantes Dauergeräusch fort.


Das kannst Du gerne mal versuchen. Eine typische Belichtungszeit für ein Foto ist 1/125 sek, das sind 8 ms. Schneide aus einer beliebigen Aufnahme 8 ms raus und loope die Aufnahme zu einem Dauergeräusch. Das wird nicht im Entferntesten an die Ursprungsaufnahme erinnern und eher wie ein Störgeräusch klingen.

#108:  Autor: Kramer BeitragVerfasst am: 12.01.2017, 23:41
    —
fwo hat folgendes geschrieben:

Der Unterschied zwischen dem optischen und dem akustischen Dauerreiz liegt in der Struktur der unterschiedlichen Kanäle:


Um zu sehen, ob wir das gleiche meinen: Wir waren ja bei dem Thema Farbwahrnehung vs. Tonhöhenwahrnehmung. Beides basiert auf Schwingungen, aber ein Foto verhält sich da nicht analog zu einer Audioaufnahme. Ein Foto ist nur ein Filter, der z.B. aus dem Sonnenlicht bestimmte Spektralanteile herausfiltert/nicht reflektiert und so nur die Anteile des Lichts zurückwirft, die nicht gefiltert wurden. Das sehen wir dann als Farben. (Das war jetzt Holzfällerphysik, ich weiss - aber es sollte klar sein, was ich meine.) Bei der Aufzeichnung von Schall funktioniert das nicht, die Schwingungen müssen gespeichert und beim Abspielen neu erzeugt werden - und das braucht Zeit. Eine 1/125 Sekunden lange Aufnahme von "was auch immer", würde kein Mensch als "was auch immer" erkennen. Bei der Farbwahrnehmung erzeugen nicht die Farben die Schwingung, sondern sie Filtern die vorhandene Schwingung des Lichts. Bei der akustischen Wahrnehmung muss der Tonerzeuger selber eine Schwingung erzeugen und dazu braucht er Zeit.

#109:  Autor: Kramer BeitragVerfasst am: 12.01.2017, 23:59
    —
Noch mal kurz hier zu:

fwo hat folgendes geschrieben:

Da bringst Du jetzt was durcheinander: Die Tonhöhe wird nicht über eine zeitliche Auflösung differenziert, sondern über eine örtliche anhand von örtlich bestimmbaren Resonanzen in der Cochlea. Hier geht es also um die Wellenlänge. (Ich habe zur Sicherheit gerade selber nachgesehen, ob mein inzwischen Jahrzehnte altes Wissen zur Sinnesphysiologie noch up to date ist - ist es zu meiner Erleichterung)


Das Ohr funktioniert vergleichbar zu einem Klavier mit geöffnetem Deckel und gehaltenem Sustain-Pedal - so, dass alle Saiten frei Schwingen können. Wenn Du in das Klavier hinein singst, schwingen die Saiten mit, die zu den Frequenzen in Deiner Stimme passen und es entsteht eine Art Nachhall. Wenn Du dann schaust, welche Saiten schwingen, kannst Du zwar den Ort des Nachhalls bestimmen, aber nicht dessen Entstehung. Wenn Du den Kammerton A singst, wird die Saite mit dem Ton A mitschwingen (und die Saiten mit den Dazugehörigen Obertönen, aber das lass ich jetzt mal aussen vor). Diese Schwingung entsteht dadurch, dass der Schall Deiner Stimme mit 440 hZ schwingt, die Saite wird 440 mal in der Sekunde durch die Schwingung in der Luft hin und her bewegt. Und weil das ihrer Eigenschwingung entspricht, schwingt sie auch danach noch etwas weiter.

Die Bereiche in der Hörschnecke, die mit dem Schall resonieren, sind zwar örtlich angeordnet, aber damit sie Schwingen, braucht es einen bestimmten Impuls mit einer ganz bestimmten zeitlichen Auflösung.

#110:  Autor: fwoWohnort: im Speckgürtel BeitragVerfasst am: 13.01.2017, 10:53
    —
Kramer hat folgendes geschrieben:

.... Das wird nicht im Entferntesten an die Ursprungsaufnahme erinnern und eher wie ein Störgeräusch klingen.

Ja. Ich habe auch nichts Anderes behauptet. Es ist ein aus technischer Sicht in bestimmter Weise vergleichbares Gegenstück zur Photographie, hergestellt, um die Unterschiede zwischen den beiden Kanälen Auge und Ohr zu demonstrieren.
Es geht dabei immer noch um die Frage, wie sinnvoll es ist, die akustische Schwingungsdauer als Grenzwert der zeitlichen Wahrnehmung zu bezeichnen. (Genaugenommen müsste man dann sogar nicht die Schwingungsdauer nehmen, sondern die Differenz zwischen zwei für uns unterscheidbaren Schwingungsdauern.)
Kramer hat folgendes geschrieben:
....Bei der akustischen Wahrnehmung muss der Tonerzeuger selber eine Schwingung erzeugen und dazu braucht er Zeit.

Ja. Und die gebe ich ihm mit dem Dauergeräusch.
Kramer hat folgendes geschrieben:
Noch mal kurz hier zu:

fwo hat folgendes geschrieben:

Da bringst Du jetzt was durcheinander: Die Tonhöhe wird nicht über eine zeitliche Auflösung differenziert, sondern über eine örtliche anhand von örtlich bestimmbaren Resonanzen in der Cochlea. Hier geht es also um die Wellenlänge. (Ich habe zur Sicherheit gerade selber nachgesehen, ob mein inzwischen Jahrzehnte altes Wissen zur Sinnesphysiologie noch up to date ist - ist es zu meiner Erleichterung)


Das Ohr funktioniert vergleichbar zu einem Klavier mit geöffnetem Deckel und gehaltenem Sustain-Pedal - so, dass alle Saiten frei Schwingen können. Wenn Du in das Klavier hinein singst, schwingen die Saiten mit, die zu den Frequenzen in Deiner Stimme passen und es entsteht eine Art Nachhall. Wenn Du dann schaust, welche Saiten schwingen, kannst Du zwar den Ort des Nachhalls bestimmen, aber nicht dessen Entstehung. Wenn Du den Kammerton A singst, wird die Saite mit dem Ton A mitschwingen (und die Saiten mit den Dazugehörigen Obertönen, aber das lass ich jetzt mal aussen vor). Diese Schwingung entsteht dadurch, dass der Schall Deiner Stimme mit 440 hZ schwingt, die Saite wird 440 mal in der Sekunde durch die Schwingung in der Luft hin und her bewegt. Und weil das ihrer Eigenschwingung entspricht, schwingt sie auch danach noch etwas weiter.

Die Bereiche in der Hörschnecke, die mit dem Schall resonieren, sind zwar örtlich angeordnet, aber damit sie Schwingen, braucht es einen bestimmten Impuls mit einer ganz bestimmten zeitlichen Auflösung.

Ja, so etwa hatte ich das auch im Kopf, als ich zur Sicherheit nochmal unter Cochlea nachsah - und trotzdem nein. Du machst hier den selben Fehler, den ich auch in dem hier noch einmal beantworteten Post mache: Ich spreche von örtlicher Auflösung, obwohl der Organismus keine Ortsinformation verarbeitet, sondern die Frequenzinformation, wobei ihn die Frequenz nicht in den quantitativen physikalischen Basisgrößen interessiert, sondern als quasi qualitative Eigenschaft. Deshalb war diese Formulierung von der örtlichen Auflösung Blödsinn. Sie entstammt der Betrachtung der technischen (oder besser gesagt physiologischen) Umsetzung des physikalischen Reizes in die verarbeitete Information.
Die Information selbst in diesem Kanal liegt in Tonbündeln unterschiedlicher Höhen in einer definierten zeitlichen Abfolge, dem Rhythmus. Dieser Rhythmus ist das, was es für den Organismus zeitlich aufzulösen gilt, das ist die Zeitinformation, mit der der Empfänger etwas anfängt, sei es als Kramer um ein Solo von Clapton oder Worte der Freundin zu erkennen, oder als Vogel den Reviergesang des Rivalen. Die Frequenz wird als allgemeine Toneigenschaft und nicht als Zeitinformation genutzt, auch wenn sie technisch eine erhält. Deshalb würde ich bei der Umsetzung der Frequenz in die benutzte Information nicht von zeitlicher Auflösung reden - (an dieser Stelle ist die relevante Auflösung keine zeitliche sondern die Trennschärfe innerhalb der Frequenzen).

Im Prinzip hatte ich das alles in dem letzten Post aber schon gesagt. Wenn Du den jetzt mit diesen zusätzliche Erläuterungen nochmal durchliest immer noch nicht überzeugend findest, gebe ich es auf. Interessant war die Diskussion trotzdem.

#111:  Autor: der kleine FritzWohnort: Planet Erde BeitragVerfasst am: 13.01.2017, 10:55
    —
fwo hat folgendes geschrieben:


der kleine Fritz hat folgendes geschrieben:
...
Wie soll das „wahrnehmen von Zeit“ als ein durchaus unterschiedlicher subjektiv/
individueller Vorgang, Einfluss auf die Reihenfolge von Ereignissen nehmen?
....


Woher soll ich das wissen? Die eigentliche Frage ist aber eine ganz andere: Woher weißt Du, dass es nicht geht, wenn Du keine Sensorik für dieses Objekt Zeit hast, mithin überhaupt keine Ahnung hast, worüber wir uns gerade unterhalten?

(Fett von mir)

Vielen Dank für deinen Hinweis---- netter Diskussionsbeitrag-
Wichtiger ist jedoch Deine Ahnung über das von dir genannte "Objekt Zeit"

#112:  Autor: fwoWohnort: im Speckgürtel BeitragVerfasst am: 13.01.2017, 11:26
    —
der kleine Fritz hat folgendes geschrieben:
fwo hat folgendes geschrieben:


der kleine Fritz hat folgendes geschrieben:
...
Wie soll das „wahrnehmen von Zeit“ als ein durchaus unterschiedlicher subjektiv/
individueller Vorgang, Einfluss auf die Reihenfolge von Ereignissen nehmen?
....


Woher soll ich das wissen? Die eigentliche Frage ist aber eine ganz andere: Woher weißt Du, dass es nicht geht, wenn Du keine Sensorik für dieses Objekt Zeit hast, mithin überhaupt keine Ahnung hast, worüber wir uns gerade unterhalten?

(Fett von mir)

Vielen Dank für deinen Hinweis---- netter Diskussionsbeitrag-
Wichtiger ist jedoch Deine Ahnung über das von dir genannte "Objekt Zeit"

Sehr glücklich
Nette Zusammenstellung.

Aber Du hast freundlicherweise über die Zitatfunktion die Verlinkung zu den kompletten Originalen erhalten, so dass sich jeder den Zusammenhang ansehen kann, in dem das da steht.

p.s. Ich gehe auch davon aus, dass Du den Satz, den Du da zitiert hast, nicht verstanden hast: Dieses "keine Ahnung", das Du da gefettet hast, steht in einem Konditionalsatz, der auf eine gewisse Widersprüchlichkeit hinweist.

#113:  Autor: Kramer BeitragVerfasst am: 13.01.2017, 17:06
    —
fwo hat folgendes geschrieben:
Kramer hat folgendes geschrieben:

.... Das wird nicht im Entferntesten an die Ursprungsaufnahme erinnern und eher wie ein Störgeräusch klingen.

Ja. Ich habe auch nichts Anderes behauptet. Es ist ein aus technischer Sicht in bestimmter Weise vergleichbares Gegenstück zur Photographie, hergestellt, um die Unterschiede zwischen den beiden Kanälen Auge und Ohr zu demonstrieren.


Nein. Es ist kein Gegenstück zur Fotografie. Was Du mit dieser "Technik" herstellst, ist kein Abbild des Originalsignals, also keine kurze Momentaufnahme, sondern etwas völlig Neues. Die Länge des Schnippsels, den Du loopst, wird zu einer eigenen komplexen Wellenform, die dann entsprechend Deiner Belichtungszeit (z.B. 8 ms - 1/125 Sek) schwingt, also hier mit 125 Hz - und das liegt bereits im hörbaren Bereich.Da Du bei einem beliebigen Schnitt nie den Nulldurchgang aller Wellenformen erwischst, erhältst Du ausserdem eine Wellenform, die im Original gar nicht vorkommt, dass heisst zusätzlich zur hörbaren Oszillation entstehen künstliche Obertöne, die im Original gar nicht vorhanden sind. Darum klingen 8 ms geloopt von AC/DC nicht sehr viel anders als 8 ms von Mozart.

#114:  Autor: smallie BeitragVerfasst am: 13.01.2017, 18:38
    —
Kramer hat folgendes geschrieben:
BTW: Ich habe gerade mal einen Test gemacht und diverse Klick-Laute durch ein Delay geschickt, das die Verzögerung in 0,1ms-Schritten auflöst. Je nach Beschaffenheit des Clicks konnte ich ab ca. 5,5ms eine zeitliche Verzögerung wahrnehmen, die sich allerdings eher wie ein kurzes Kratzen anhörte. Es klang immer noch wie ein Signal, mit einem etwas breiteren und rauerem Einschwingvorgang. Ab ca. 7 ms konnte ich zwei verschiedene Signale erahnen. Im Bereich unter 5ms habe ich nur Verfärbungen gehört, die umso intensiver wurden, je kürzer das Echo war. Man hört den Unterschied der Verfärbung z.B. bei dem Wechsel zwischen 0,1ms zu 0,2ms deutlicher, als zwischen 4,1ms und 4,2ms.

Lachen Lachen Lachen

Laus mich doch der Affe. Da schreiben wir sechs Jahre im gleichen Forum und ich weiß nicht, daß du Musiker bist. Geschockt Laß doch mal was hören. Cool

Deinen Test kann ich übrigens bestätigen. Selber vor ein paar Jahren ausprobiert, allerdings nicht mit einem Click, sondern nur mit kürzest möglichem Attack/Decay bei einem Synthesizer. Da kam ein bisschen mehr heraus, zwischen 10 und 20 ms, wenn ich mich recht erinnere. Das passt ganz gut zu deinen Werten.

#115:  Autor: smallie BeitragVerfasst am: 13.01.2017, 18:52
    —
zelig hat folgendes geschrieben:
smallie hat folgendes geschrieben:
zelig hat folgendes geschrieben:
smallie hat folgendes geschrieben:
Du verteidigst das hartnäckig, ich kann's per Selbsterfahrung ebenso hartnäckig nicht nachvollziehen. Könnte es sich hier am Ende um eine mentale Typsache handeln? So wie introvertiert vs. extrovertiert? Dann wärst du ein Raum-Typ und ich ein Zeit-Typ.


Interessante Typologie. Obwohl Menschen sicherlich unterschiedliche räumliche und zeitliche Wahrnehmungen haben, glaube ich nicht, daß man sie in diese 2 Kategorien einteilen könnte. ; )

*Geh zurück auf Los und ziehe keine 4000 ein*

Dann bleibt die Frage offen, warum unsere Vorstellungswelt inkompatibel ist. Ich könnte noch einen weiteren Absatz an Beispielen zeitlich ausgedehnter Analogien bringen - vom 3-Minuten-Ei bis zu der knappen Stunde, die brauner Reis braucht. Das ist für mich so offensichtlich, daß ich nicht verstehe, wie du das anders empfinden kannst.

Ich denke bei der Sache auch an die östliche Vorstellung von der Welt als ewigem Wandel oder mit Heraklit an "alles fließt".

Ich habe den Eindruck, in das Statement wird zuviel hineininterpretiert. Was auch immer „inkompatibel“ sein soll, bedenke, ich spekuliere damit nur über den Grund, warum Zeit und Raum von Menschen kategorial unterschiedlich wahrgenommen werden.

Im Raum kann ich mich einigermaßen frei bewegen. Naja, zumindest auf der Erdoberfläche.

Bei der Zeit ist es vorwärts immer, rückwärts nimmer.


zelig hat folgendes geschrieben:
smallie hat folgendes geschrieben:

Mist, jetzt fällt mir Drafi Deutscher ein: "Marmor, Stein und Eisen bricht ..." Sorry. Wenn mir jetzt darüber die Milch sauer wird, ist das ein gültiges Beispiel für die zeitabhängige Qualität von Substanzen? Läßt du radioaktive Substanzen gelten, die mit einer charakteristischen Halbwertzeit zerfallen?
Machen wir ein Experiment, das ohne Substanzen auskommt.
Ich denk' an drei meiner Lieblingsmusikstücke und du auch. Wir klopfen das Tempo und vergleichen dann mit dem Original. Das ganze mit 1000 Versuchspersonen. Dann hätten wir etwas in der Hand zur Genauigkeit des Kurzzeit-Zeitgefühls.

Das verstehe ich nicht, glaube aber, das hat auch nichts mit dem Gegenstand unserer Diskussion zu tun.

War ja auch assoziativ ziemlich, ahhm, überbordend. zwinkern

Den letzten Punkt nochmal anders: daß Musiker das Tempo halten können, zeugt doch eigentlich von der Existenz einer Zeitwahrnehmung?

#116:  Autor: smallie BeitragVerfasst am: 13.01.2017, 19:00
    —
fwo hat folgendes geschrieben:
Warum schaffst Du es nicht, ... auch Posts zu beantworten, die erst morgen geschrieben werden?

Das ist jetzt einfach. Mr. Green

Meine Antwort lautet schon heute: "Nein, es ist nicht der Islam." Ich sag's dir dann, wenn du den zugehörigen, auslösenden Text geschrieben hast.

Bitte nicht!

#117:  Autor: Kramer BeitragVerfasst am: 14.01.2017, 01:44
    —
fwo hat folgendes geschrieben:

Im Prinzip hatte ich das alles in dem letzten Post aber schon gesagt. Wenn Du den jetzt mit diesen zusätzliche Erläuterungen nochmal durchliest immer noch nicht überzeugend findest, gebe ich es auf.


Ich weiss nicht, ob ich es überzeugend finde, ich verstehe es nicht ganz. Vielleicht reden wir auch aneinander vorbei und meinen eigentlich das Gleiche, nur diskutieren wir jeweils von einer anderen Warte aus?

#118:  Autor: Kramer BeitragVerfasst am: 14.01.2017, 01:59
    —
smallie hat folgendes geschrieben:
Laus mich doch der Affe. Da schreiben wir sechs Jahre im gleichen Forum und ich weiß nicht, daß du Musiker bist. Geschockt Laß doch mal was hören. Cool


Aprilscherz 2010 - der Soundtrack war von mir.

Zitat:
Deinen Test kann ich übrigens bestätigen. Selber vor ein paar Jahren ausprobiert, allerdings nicht mit einem Click, sondern nur mit kürzest möglichem Attack/Decay bei einem Synthesizer. Da kam ein bisschen mehr heraus, zwischen 10 und 20 ms, wenn ich mich recht erinnere. Das passt ganz gut zu deinen Werten.


Ich habe ein Signal genommen, dass Klicklauten, die man man mit dem Mund machen kann, möglichst nahe kommt. Mein Signal war 0,4 ms lang - noch kürzer klang einfach nicht mehr nach etwas, das auch in der Natur vorkommen könnte. Ausserdem braucht das Signal ja etwas "Body" um das Phasing bei den kurzen Zeiten hörbar zu machen. Mit ganz kurzen Signalen erhält man nur Reissverschluss-ähnliche Ergebnisse ohne charakteristischen Frequenzgang. Ich vermute mal, dass das Decay Deines Synths sich für das Experiment nicht kurz genug einstellen liess.

#119:  Autor: zelig BeitragVerfasst am: 14.01.2017, 12:54
    —
smallie hat folgendes geschrieben:
Den letzten Punkt nochmal anders: daß Musiker das Tempo halten können, zeugt doch eigentlich von der Existenz einer Zeitwahrnehmung?


Naja, die Prämisse kann man durchaus anzweifeln.







*scherz* :D






Spaß beiseite. Klar haben wir eine Zeitwahrnehmung. Wir haben eine ganze Reihe unterschiedlicher Zyklen der Natur, die uns Orientierung geben. Sonnenaufgang, Jahreszeiten, Ernte. Schlaf, Atem, Herzschlag. Schwangerschaft, Kindheit, Alter. Das sind Beispiele für Ereignisse und Phasen, die uns erlauben, den Verlauf des Lebens in zeitlich mehr oder weniger genaue Abschnitte einzuteilen. Der Stoffwechsel zwischen den Zellen ist fein abgestimmt und komplex, Herz und Lunge existieren als Organe, weil sie die Sauerstoffsättigung in einem Regelkreislauf auf einem konstanten Niveau halten. Damit das Gehirn arbeiten kann, muss Synchronizität bei der Signalverarbeitung und -erzeugung herrschen.
Die Existenz von Leben, wie wir es kennen, wäre ohne Zyklizität nicht denkbar. Wieso sollte also unsere eigene Natur davon ausgenommen sein?

#120:  Autor: SkeptikerWohnort: 129 Goosebumpsville BeitragVerfasst am: 14.01.2017, 17:45
    —
der kleine Fritz hat folgendes geschrieben:
Skeptiker hat folgendes geschrieben:


Hi klein fritzchen! Smilie

Gib mir ein bisschen Zeit, dann nehme ich ausführlicher und konkreter Stellung. Raum habe ich genug.


Hallo Skeptiker,
wieviel Zeit benötigst du denn?

Ich könnte dir das Kilo Zeit für 99 Cent anbieten!
Musst sie aber selber abholen und den Raum dafür mitbringen.
Am besten eignet sich ein Schuhkarton, da passt gerade 1 Kilo rein!
Dann tscüß.
Lachen


quote gerichtet. vrolijke

In den Karton lege ich das folgende Stück Musik:

Supertramp - Rudy

Deine Uhr wird währenddessen 485 s Zeit messen, sofern sie hinreichend genau geht.

der kleine Fritz hat folgendes geschrieben:
Skeptiker hat folgendes geschrieben:
Marcellinus hat folgendes geschrieben:
Er_Win hat folgendes geschrieben:
Liege ich ansonsten richtig, dass du im Gegensatz zu den meisten anderen Postern eine Objektivierbar8keit der Zeit bezweifelst.


Ich denke, man muß ein paar Dinge auseinanderhalten. „Die Zeit“ ist ein menschengemachter Begriff, der für Modellvorstellungen steht, die sich im Laufe unserer Geschichte entwickelt haben.


@Marcellinus: Wäre es möglich, auf Er_Wins Frage zu antworten?

Bezweifelst du die Objektivierbarkeit der Zeit?


Hallo Skeptiker

Was verstehst du unter Objektivierbarkeit der Zeit?

Das heißt doch nach meinem Sprachverständnis, das Mysterium „Zeit“ ist nicht, kann aber zu einem Objekt gemacht werden und damit Masse erhalten.
Wie die Verwandlung von Nichts in materielle „Zeit“ vor sich geht bedarf natürlich der wissenschaftlichen Erklärung, die du mir ja versprochen hattest!

Bitte erlöse mich von meinen Zweifeln!


Objektiv real ist alles, was sich messen lässt mit geeigneten Messinstrumenten, aber nicht alles lässt sich messen, auch wenn es objektive Realität besitzen mag.

Nun kann man messen: Zeit, Länge, Geschwindigkeit, Masse und Energie.

In idealistischen Philosophien sind allerdings Zeit und Raum sozusagen die *Kartons* für Materie und Energie, welche als leere Behälter weiterhin existieren würden, wenn man Materie und Energie aus ihnen heraus saugen würde mit einem eigens dafür konstruierten Strohmann ... äh ... Strohhalm.

Es gibt ja auch einen analogen Idealismus nach dem auch der Geist ohne Materie existieren könne, quasi Geist und Denken ohne Gehirn, ohne Körper.

Nach dem Verständnis neuester und best begründeter Theorien und Forschungen hängen aber wohl Zeit, Raum sowie sich bewegende Materie bzw. materielle Bewegungen untrennbar zusammen; es gibt nicht das eine ohne das andere - auch keine Zeit ohne Materie, ohne Bewegung derselben.

Man kann sagen, dass durch die Bewegung der Materie eine stets spezifische Geometrie (Form) der umgebenden Raumzeit geschaffen wird und jene spezielle Geometrie gibt quasi die Bewegundräume für die Materie vor - ein dialektischer, untrennbarer Zusammenhang.

Anders formuliert spricht man auch von einer spezifischen Krümmung der Raumzeit oder einer gekrümmten Geometrie, welche Gravitation als Kraft obselet werden lässt. Für große Geschwindigkeiten und Massen ist die Newton'sche Theorie zumindest unpräzise und eigentlich überholt. Das Universum ist - wie man heute weiß - nicht statisch - sondern in ständiger Dynamik und Entwicklung.

Die Berechnung der Raumgeometrie unter Berücksichtigung relativistischer Effekte bei Zeit und Raumdimensionen erfolgt bei Einstein mittels seiner so genannten Feldgleichungen. Durch eine hohe Geschwindigkeit verläuft die Eigenzeit eines Körpers langsamer als bei niedriger Geschwindigkeit und ebenso verläuft seine Eigenzeit langsamer unter dem Einfluss starker Gravitation im Vergleich zum Einfluss geringer Gravitation. Bei Satelliten und somit beim GPS-System muss man beide Effekte berücksichtigen, um die Eigenzeit der Satelliten mit der Zeit eines Standortes auf der Erde zu synchronisieren.

Diese Einflüsse der Geschwindigkeit und Masse auf die Eigenzeit (und auf die Länge) eines bewegten Objektes zeigen die Wechselwirkungen zwischen Materie, Raum und Zeit. So wie geistige Prozesse eigentlich materielle Prozesse und Bewegungen sind bzw. von diesen abhängen, so sind auch Raum und Zeit ohne sich bewegende Materie nicht existent. Oder anders formuliert: Die gesamte Welt ist eigentlich nichts anderes als sich bewegende Materie.

der kleine Fritz hat folgendes geschrieben:
@marcellinus:

Da kann ich dir natürlich beiplichten, denn ohne den Beobachter/Beurteiler Mensch käme niemand auf die aburde Idee im Universum zerren Raum und Zeit aneinender herum! Mit den Augen rollen


Das ist Schopenhauer: "Die Welt als Wille und Vorstellung". Nach dieser Vorstellung muss erst jemand hingucken, bevor die Welt da ist. Das ist natürlich nichts anders als die Ablehnung des wissenschaftlichen Objektivitätspostulates an sich. Materialistisch betrachtet ist es jedoch genau umgekehrt: Nicht das objektive Sein hängt vom Bewusstsein ab, sondern das Bewusstsein spiegelt eine objektive Realität wieder, so ungenau dies auch sein mag; das Subjekt ist Produkt der Objektivität anstatt vice versa.

Marcellinus hat folgendes geschrieben:
Skeptiker hat folgendes geschrieben:
Marcellinus hat folgendes geschrieben:
Er_Win hat folgendes geschrieben:
Liege ich ansonsten richtig, dass du im Gegensatz zu den meisten anderen Postern eine Objektivierbarkeit der Zeit bezweifelst.


Ich denke, man muß ein paar Dinge auseinanderhalten. „Die Zeit“ ist ein menschengemachter Begriff, der für Modellvorstellungen steht, die sich im Laufe unserer Geschichte entwickelt haben.


@Marcellinus: Wäre es möglich, auf Er_Wins Frage zu antworten?

Bezweifelst du die Objektivierbarkeit der Zeit?


Lies einfach nach, was ich geschrieben habe. "Zeit" ist eine Modellvorstellung. Entsprechend der Einstein'schen Relativitätstheorie begreifen Physiker sie heute als Aspekt des vierdimensionalen Raum-Zeit-Kontinuums, und dieser Aspekt ist ziemlich gut durch Tatsachenbeobachtungen bestätigt. Wenn du das unter Objektivierbarkeit verstehst, dann ist es das. Aber es bleibt ein menschengemachtes Modell, wie realistisch auch immer, und morgen mögen wir Dinge entdecken, die in diesem Modell bisher nicht berücksichtig wurden, die falsch sind oder unvollständig.


Bertolt Brecht hat folgendes geschrieben:
"Was tun Sie", wurde Herr K. gefragt, "wenn Sie einen Menschen lieben?" "Ich mache einen Entwurf von ihm", sagte Herr K., "und sorge, daß er ihm ähnlich wird." "Wer? Der Entwurf?" "Nein", sagte Herr K., "der Mensch."


Der Fortschritt der menschlichen Erkenntnis besteht nicht darin, dass der Mensch Entwürfe=Modelle macht und die Welt an diese Modelle anpasst, sondern umgekehrt. Die Wissenschaft passt ihre Vorstellungen an die Welt selber an und begründet sie auch mit ebendieser. Somit ist also dein Satz "Aber es bleibt ein menschengemachtes Modell" einfach daneben. Denn *Modelle* werden eben gerade nicht irgendwie beliebig *gemacht*, sondern wenn, dann werden sie erkannt. Deshalb sind sie auch nicht einfach zufällig bzw. ohne gute Gründe so, wie sie sind und es sind beileibe keine soziologischen Gründe. In deiner Philosophie, die ich für eine grottenschlechte halte, gibt es gar keinen Erkenntnisfortschritt, sondern im Grunde genommen ein mehr oder weniger buntes Pottpourie von *Modellen* und *Vorstellungen*, welche sich theoretisch auch wiederholen und im Kreise drehen können, ohne dass es eine Höherentwicklung und einen Fortschritt der Erkennntnis auch hinsichtliche ihrer Richtigkeit gibt. zwar fügst du mantramäßig ständig hinzu, dass sich alles *entwickelt* habe, aber in deine komische *Entwicklung* schließt du ja im Grunde genommen auch eine Rückentwicklung der Erkennntis ein, da ja eh alles nur *Modell* sei. Mit deinem Wort *Entwicklung* sagst du also eigentlich nur, dass es irgendwann nicht da war, und zu einem anderen Zeitpunkt war es dann eben da.

Marcellinus hat folgendes geschrieben:
"Das Ding an sich" ist eine Vorstellung der Philosophie, nicht der Wissenschaft.


Dass z.B. die Zeit auch eine subjektive Dimension hat, bedeutete für Einstein und andere moderne Wissenschaftler nicht, dass sie keine objektive Existenz hätte. Auch die relativistischen Effekte - die man übrigens unter Normalbedingungen nicht sinnlich wahrnehmen kann - ändern an der Objektivität der Zeit nichts.

Entscheidend ist hier, dass gerade der grundsätzliche Unterschied zwischen Subjektivität und Objektivität nicht dadurch einfach verwischt werden kann, indem man feststellt, dass Dinge bzw. Erscheinungen der Dinge erkennbar sind. Denn gerade durch die methodische Erkenntnis sind Ding und Subjekt einerseits miteinander verknüpft, andererseits jedoch voneinander erst unterscheidbar, sofern Erkenntnis sich z.B. in allgemeinen, objektiv nachprüfbaren Gesetzmäßigkeiten niederschlägt.

Die objektive Prüfung von Gesetzmäßigkeiten der Dinge ist gerade das Merkmal moderner Wissenschaft, und die Dinge enthüllen ihre Wesenheiten durch das Erkennen ihrer Gesetzmäßigkeiten und Beziehungen zu anderen Dingen. Dadurch hören sie aber auch auf, nur Dinge zu sein und verwandeln sich gleichzeitig in Verhältnisse (zu anderen Dingen), und zwar in objektive Verhältnisse ...-

#121:  Autor: fwoWohnort: im Speckgürtel BeitragVerfasst am: 14.01.2017, 19:05
    —
Kramer hat folgendes geschrieben:
fwo hat folgendes geschrieben:

Im Prinzip hatte ich das alles in dem letzten Post aber schon gesagt. Wenn Du den jetzt mit diesen zusätzliche Erläuterungen nochmal durchliest immer noch nicht überzeugend findest, gebe ich es auf.


Ich weiss nicht, ob ich es überzeugend finde, ich verstehe es nicht ganz. Vielleicht reden wir auch aneinander vorbei und meinen eigentlich das Gleiche, nur diskutieren wir jeweils von einer anderen Warte aus?

Unsere Diskussion mit Dir ging von mir aus nur um den einen Punkt, was man sinnvollerweise als sinnvolle untere Grenze der Erkennbarkeit einer Zeitspanne bezeichnen sollte, es ging um das zeitliche Auflösungsvermögen des menschlichen Gehörs, und ich habe dafür plädiert, nur da von zeitlichem Auflösungsvermögen zu sprechen, wo der entsprechende Organismus diese Dauer auch als Dauer benutzt.

Ansonsten sind wir uns wahrscheinlich einig.

#122:  Autor: Marcellinus BeitragVerfasst am: 14.01.2017, 20:13
    —
Skeptiker hat folgendes geschrieben:

… das Bewusstsein spiegelt eine objektive Realität wieder …


Das ist selbst für deine Verhältnisse naiv!

Skeptiker hat folgendes geschrieben:

… aber in deine komische *Entwicklung* schließt du ja im Grunde genommen auch eine Rückentwicklung der Erkennntis ein, da ja eh alles nur *Modell* sei. …


Oh, auch ein blindes Huhn findet mal ein Korn! Es ist nicht nur möglich, es ist schon vorgekommen, und leider auch nicht ausgeschlossen, daß es sich wiederholt.

Skeptiker hat folgendes geschrieben:

Marcellinus hat folgendes geschrieben:
"Das Ding an sich" ist eine Vorstellung der Philosophie, nicht der Wissenschaft.

Die objektive Prüfung von Gesetzmäßigkeiten der Dinge ist gerade das Merkmal moderner Wissenschaft, und die Dinge enthüllen ihre Wesenheiten durch das Erkennen ihrer Gesetzmäßigkeiten und Beziehungen zu anderen Dingen. Dadurch hören sie aber auch auf, nur Dinge zu sein und verwandeln sich gleichzeitig in Verhältnisse (zu anderen Dingen), und zwar in objektive Verhältnisse ...-


„Gesetzmäßigkeit“ …, „Wesenheit“ …, „objektive Verhältnisse“ …, heiliger Karl, da gräbst du aber ziemlich tief in der philosophischen Rumpelkammer!

Skeptiker hat folgendes geschrieben:

In deiner Philosophie, die ich für eine grottenschlechte halte, …


Als jemand, dessen Vorstellung im wesentlichen in der Philosophie des 19. Jh. stehengeblieben ist, schreibt genau der Richtige! Lachen

#123:  Autor: der kleine FritzWohnort: Planet Erde BeitragVerfasst am: 14.01.2017, 21:53
    —
Skeptiker hat folgendes geschrieben:
der kleine Fritz hat folgendes geschrieben:
Skeptiker hat folgendes geschrieben:


Hi klein fritzchen! Smilie
Gib mir ein bisschen Zeit, dann nehme ich ausführlicher und konkreter Stellung. Raum habe ich genug.


Hallo Skeptiker,
wieviel Zeit benötigst du denn?

Ich könnte dir das Kilo Zeit für 99 Cent anbieten!
Musst sie aber selber abholen und den Raum dafür mitbringen.
Am besten eignet sich ein Schuhkarton, da passt gerade 1 Kilo rein!
Dann tscüß.
Lachen


In den Karton lege ich das folgende Stück Musik:

Supertramp - Rudy

Deine Uhr wird währenddessen 485 s Zeit messen, sofern sie hinreichend genau geht.



Hallo Skeptiker,

erst mal vielen Dank für deine verbale Mühe. Das hat mir gefallen und ich habe alles aufmerksam gelesen. Ich bitte um Nachsicht wenn ich - aus Zeitmangel Verlegen - nicht auf alle Probleme eingehen kann.

Aber zur Musik im Karton Sehr glücklich will ich mal einen Versuch machen meine Ansicht darzustellen.
Mit meiner Uhr messe ich keine "Zeit", sondern die Dauer eines Vorganges, nämlich 485 s
Und Sekunden sind keine Zeit sondern eine genormte Maßeinheit der Dauer einer Bewegung,

Die sogenannte Zeimessung ist einfach nur ein Zählvogang der Anzahl von Sekunden auf dem Zifferblatt der Uhr während der Dauer deiner Musik!

Es soll aber Leute geben, die keine Uhr benötigen, die können die Zeit Wahrnehmen! Mit den Augen rollen

Zitatenmurks gerichtet. Bitte achte im Zukunft darauf! vrolijke

#124:  Autor: Kramer BeitragVerfasst am: 15.01.2017, 01:18
    —
fwo hat folgendes geschrieben:

Unsere Diskussion mit Dir ging von mir aus nur um den einen Punkt, was man sinnvollerweise als sinnvolle untere Grenze der Erkennbarkeit einer Zeitspanne bezeichnen sollte, es ging um das zeitliche Auflösungsvermögen des menschlichen Gehörs, und ich habe dafür plädiert, nur da von zeitlichem Auflösungsvermögen zu sprechen, wo der entsprechende Organismus diese Dauer auch als Dauer benutzt.


Da ist wohl einiges durcheinander geraten. Mein Beispiel mit den Tonhöhen bezog sich nicht auf die Diskussion über die Wahrnehmungsgrenze, sondern auf die Frage, ob wir Zeit ähnlich intuitiv wahrnehmen können wie Raum. Zwar nehmen wir unterschiedliche Tonhöhen nicht als zeitliche Unterschiede wahr, aber das, was wir da wahrnehmen, ist ein zeitliches Phänomen.

#125:  Autor: smallie BeitragVerfasst am: 15.01.2017, 14:22
    —
Kramer hat folgendes geschrieben:
fwo hat folgendes geschrieben:

Unsere Diskussion mit Dir ging von mir aus nur um den einen Punkt, was man sinnvollerweise als sinnvolle untere Grenze der Erkennbarkeit einer Zeitspanne bezeichnen sollte, es ging um das zeitliche Auflösungsvermögen des menschlichen Gehörs, und ich habe dafür plädiert, nur da von zeitlichem Auflösungsvermögen zu sprechen, wo der entsprechende Organismus diese Dauer auch als Dauer benutzt.


Da ist wohl einiges durcheinander geraten. Mein Beispiel mit den Tonhöhen bezog sich nicht auf die Diskussion über die Wahrnehmungsgrenze, sondern auf die Frage, ob wir Zeit ähnlich intuitiv wahrnehmen können wie Raum. Zwar nehmen wir unterschiedliche Tonhöhen nicht als zeitliche Unterschiede wahr, aber das, was wir da wahrnehmen, ist ein zeitliches Phänomen.

Jetzt habt ihr mich auf etwas gebracht. Stichwort akustische Illusion.

1000 oder 10000 Schwingungen pro Sekunde kann das Ohr nicht zeitlich auflösen. Es geht über den Umweg eines "Zungenfrequenzmessers" = den Härchen in der Gehörschnecke.

Nun hat aber dieser Frequenzmesser selbst eine bestimmte zeitliche Auflösung. Vermutlich irgendwas zwischen 5 und 20 ms. Was unter dieser Auflösung liegt, wird vom Gehirn ergänzt. Als Test: der Hörbereich geht über mehr als 8 Oktaven. Wenn ich ein Signal baue, daß diesen Frequenzbereich in kurzer Zeit durchläuft, sollte das Ohr nicht mehr in der Lage sein, der Frequenzänderung kontinuierlich zu folgen.

Zum Vergleich beim Auge. Wenn es rasch genug jeweils eine dieser Zeilen als Einzelbild sieht, entsteht daraus der Eindruck, das X würde sich von links nach rechts bewegen.

Code:

[X] [ ] [ ] [ ]
[ ] [X] [ ] [ ]
[ ] [ ] [X] [ ]
[ ] [ ] [ ] [X]


Bei geeignetem Material treten dabei optische Täuschungen auf. In Westernfilmen scheinen die Speichenräder von Kutschen rückwärts zu laufen.

Genau das müßte es analog beim Hören auch geben. Ich weiß nur noch nicht, was das geeignete Material sein könnte, um einen akustischen Rückwärts-Speichen-Effekt hervorzurufen.

#126:  Autor: smallie BeitragVerfasst am: 15.01.2017, 14:37
    —
smallie hat folgendes geschrieben:
Nun ist mir gerade aufgefallen, daß ich doof bin.

Guten Morgen, smallie. Auch schon wach? Obiges kennt man als Aliasing.

#127:  Autor: smallie BeitragVerfasst am: 15.01.2017, 14:44
    —
zelig hat folgendes geschrieben:
smallie hat folgendes geschrieben:
Den letzten Punkt nochmal anders: daß Musiker das Tempo halten können, zeugt doch eigentlich von der Existenz einer Zeitwahrnehmung?


Naja, die Prämisse kann man durchaus anzweifeln.

Mr. Green

Es klopft an der Tür.
Sagt der Bassist: "Das muß unser Drummer sein."
Sagt der Sänger: "Woher weißt du das?"
Bassist: "Das Klopfen wird schneller."



zelig hat folgendes geschrieben:
Spaß beiseite. Klar haben wir eine Zeitwahrnehmung. Wir haben eine ganze Reihe unterschiedlicher Zyklen der Natur, die uns Orientierung geben. Sonnenaufgang, Jahreszeiten, Ernte. Schlaf, Atem, Herzschlag. Schwangerschaft, Kindheit, Alter. Das sind Beispiele für Ereignisse und Phasen, die uns erlauben, den Verlauf des Lebens in zeitlich mehr oder weniger genaue Abschnitte einzuteilen. Der Stoffwechsel zwischen den Zellen ist fein abgestimmt und komplex, Herz und Lunge existieren als Organe, weil sie die Sauerstoffsättigung in einem Regelkreislauf auf einem konstanten Niveau halten. Damit das Gehirn arbeiten kann, muss Synchronizität bei der Signalverarbeitung und -erzeugung herrschen.

Dann sind zumindest unsere praktischen Ansschauungen doch nicht inkompatibel. zwinkern


zelig hat folgendes geschrieben:
Die Existenz von Leben, wie wir es kennen, wäre ohne Zyklizität nicht denkbar. Wieso sollte also unsere eigene Natur davon ausgenommen sein?

Wenn ich das gesagt hätte, dann würde da ein größerer gedanklicher Unterbau zum Thema Zyklizität dahinterstecken. Bei dir auch?

#128: Mehrerkenntnis Autor: SkeptikerWohnort: 129 Goosebumpsville BeitragVerfasst am: 15.01.2017, 17:01
    —
Marcellinus hat folgendes geschrieben:
Skeptiker hat folgendes geschrieben:

… das Bewusstsein spiegelt eine objektive Realität wieder …


Das ist selbst für deine Verhältnisse naiv!


Der wissenschaftlich-technische Fortschritt und das technische und organisatorische Funktionieren immer komplexerer Anwendungen zeigt doch, dass die objektive Realität so schlecht wohl nicht verstanden wird vom Menschen. Und zwar über das bloße Überleben hinaus. Die Menschheit produziert gewissermaßen nicht nur ökonomische Mehrprodukte und Mehrwerte, sondern auch über das bloße Überleben ein wachsendes Potenzial an Mehrerkenntnis.

die Evidenz dessen soll naiv sein? Naiv wäre es, die Augen davor zu verschließen, so wie ein Kind, welches denkt, die Welt sei dann verschwunden.

Marcellinus hat folgendes geschrieben:
Skeptiker hat folgendes geschrieben:
… aber in deine komische *Entwicklung* schließt du ja im Grunde genommen auch eine Rückentwicklung der Erkennntis ein, da ja eh alles nur *Modell* sei. …


Oh, auch ein blindes Huhn findet mal ein Korn! Es ist nicht nur möglich, es ist schon vorgekommen, und leider auch nicht ausgeschlossen, daß es sich wiederholt.


Das kann nur dann vorkommen, wenn bereits erreichte wissenschaftliche Erkenntnisse vergessen oder unter den Teppich gekehrt werden. Das sind aber dann politische Vorgänge, die mit dem wissenschaftlichen Fortschritt und der Richtigkeit ihrer Erkenntnisse überhaupt nichts zu tun hat.

Marcellinus hat folgendes geschrieben:
Skeptiker hat folgendes geschrieben:
Marcellinus hat folgendes geschrieben:
"Das Ding an sich" ist eine Vorstellung der Philosophie, nicht der Wissenschaft.

Die objektive Prüfung von Gesetzmäßigkeiten der Dinge ist gerade das Merkmal moderner Wissenschaft, und die Dinge enthüllen ihre Wesenheiten durch das Erkennen ihrer Gesetzmäßigkeiten und Beziehungen zu anderen Dingen. Dadurch hören sie aber auch auf, nur Dinge zu sein und verwandeln sich gleichzeitig in Verhältnisse (zu anderen Dingen), und zwar in objektive Verhältnisse ...-


„Gesetzmäßigkeit“ …, „Wesenheit“ …, „objektive Verhältnisse“ …, heiliger Karl, da gräbst du aber ziemlich tief in der philosophischen Rumpelkammer!


Ich sprach von der Erkenntnis als vermittelnder Instanz zwischen dem Subjekt und dem objektiven Ding und Verhältnis. Wenn das für dich nur Buzzwörter sind, dann sagt das etwas über dich aus, sonst nichts.

Marcellinus hat folgendes geschrieben:
Skeptiker hat folgendes geschrieben:
In deiner Philosophie, die ich für eine grottenschlechte halte, …


Als jemand, dessen Vorstellung im wesentlichen in der Philosophie des 19. Jh. stehengeblieben ist, schreibt genau der Richtige! Lachen


Das 19. Jh. ist groß. Es gab da viele verschiedene und gegensätzliche Philosophien. Es gab gute und schlechte, rückwärtsgewandte und zukunftsträchtige.

Das weißt du doch, oder ...-? Auf den Arm nehmen

#129:  Autor: Marcellinus BeitragVerfasst am: 15.01.2017, 20:46
    —
Skeptiker hat folgendes geschrieben:
Marcellinus hat folgendes geschrieben:
Skeptiker hat folgendes geschrieben:

… das Bewusstsein spiegelt eine objektive Realität wieder …


Das ist selbst für deine Verhältnisse naiv!


Der wissenschaftlich-technische Fortschritt und das technische und organisatorische Funktionieren immer komplexerer Anwendungen zeigt doch, dass die objektive Realität so schlecht wohl nicht verstanden wird vom Menschen. Und zwar über das bloße Überleben hinaus. Die Menschheit produziert gewissermaßen nicht nur ökonomische Mehrprodukte und Mehrwerte, sondern auch über das bloße Überleben ein wachsendes Potenzial an Mehrerkenntnis.

die Evidenz dessen soll naiv sein? Naiv wäre es, die Augen davor zu verschließen, so wie ein Kind, welches denkt, die Welt sei dann verschwunden.


Nein, naiv ist nur die Vorstellung, die „objektive Realität spiegele sich im menschlichen Bewusstseins“. Das ist simpler Induktionismus.

Skeptiker hat folgendes geschrieben:

Marcellinus hat folgendes geschrieben:
Skeptiker hat folgendes geschrieben:
… aber in deine komische *Entwicklung* schließt du ja im Grunde genommen auch eine Rückentwicklung der Erkennntis ein, da ja eh alles nur *Modell* sei. …


Oh, auch ein blindes Huhn findet mal ein Korn! Es ist nicht nur möglich, es ist schon vorgekommen, und leider auch nicht ausgeschlossen, daß es sich wiederholt.


Das kann nur dann vorkommen, wenn bereits erreichte wissenschaftliche Erkenntnisse vergessen oder unter den Teppich gekehrt werden. Das sind aber dann politische Vorgänge, die mit dem wissenschaftlichen Fortschritt und der Richtigkeit ihrer Erkenntnisse überhaupt nichts zu tun hat.


Ach, und „wissenschaftlicher Fortschritt“ hat nichts mit Politik zu tun? Wie war das noch mit der Atombombe? Ob Wissenschaften fortschreiten oder zurück, ist immer auch eine Frage der Politik. Das solltest du eigentlich wissen.

Skeptiker hat folgendes geschrieben:

Marcellinus hat folgendes geschrieben:
Skeptiker hat folgendes geschrieben:
In deiner Philosophie, die ich für eine grottenschlechte halte, …


Als jemand, dessen Vorstellung im wesentlichen in der Philosophie des 19. Jh. stehengeblieben ist, schreibt genau der Richtige! Lachen


Das 19. Jh. ist groß. Es gab da viele verschiedene und gegensätzliche Philosophien. Es gab gute und schlechte, rückwärtsgewandte und zukunftsträchtige.

Das weißt du doch, oder ...-? Auf den Arm nehmen


Nein, du sagst doch selbst, daß „meine Philosophie“ eine „grottenschlechte“ sei. Das 19. Jh. war auch nicht „groß“ (wie immer man das messen wollte), nicht einmal besonders lang. Das 20. Jh. war zB einen Tag länger. Nur woran du „gute“ von „schlechter“ (mal ganz abgesehen von meiner), „rückwärtsgewandter“ von „zukunftsträchtiger“ Philosophie unterscheiden willst, das wird wohl dein Geheimnis bleiben.

Bessere von schlechterer Wissenschaft zu unterscheiden, mag ja noch angehen, Sachkenntnis mal vorausgesetzt. Aber Philosophie? Da sich Philosophen auch auf sonst nichts einigen können, warum gerade auf ein verbindliches Qualitätskriterium? Bis zum Beweis des Gegenteils vermute ich daher mal, es dürfte sich um eine Geschmacksfrage handeln.

#130:  Autor: Kramer BeitragVerfasst am: 15.01.2017, 23:44
    —
smallie hat folgendes geschrieben:
smallie hat folgendes geschrieben:
Nun ist mir gerade aufgefallen, daß ich doof bin.

Guten Morgen, smallie. Auch schon wach? Obiges kennt man als Aliasing.


Richtig, aber mir wäre neu, dass es das auch in der Natur gibt. Das optische Aliasing (Stroboskopeffekt) entsteht ja auch nicht durch Beschränkungen unseres Sehvermögens, sondern durch die geringe Abtastrate von Filmaufnahmen.

#131:  Autor: smallie BeitragVerfasst am: 15.01.2017, 23:59
    —
Marcellinus hat folgendes geschrieben:
Skeptiker hat folgendes geschrieben:

… das Bewusstsein spiegelt eine objektive Realität wieder …


Das ist selbst für deine Verhältnisse naiv!

Eigentlich stand da "das Bewusstsein spiegelt eine objektive Realität wieder, so ungenau dies auch sein mag."


Marcellinus hat folgendes geschrieben:
„Gesetzmäßigkeit“ …, „Wesenheit“ …, „objektive Verhältnisse“ …, heiliger Karl, da gräbst du aber ziemlich tief in der philosophischen Rumpelkammer!

Zu dem Thema gibt es einen berühmt-berüchtigten Artikel:

Zitat:
Transgressing the Boundaries: Towards a Transformative Hermeneutics of Quantum Gravity

Viele Naturwissenschaftler, besonders Physiker, hängen einem alten Dogma an, das zusammengefasst lautet: es gibt eine äußere Welt, deren Eigenschaften unabhängig sind von menschlichen Wesen, daß diese Eigenschaften in "ewige" Naturgesetze aufgeschlüsselt werden können, und daß menschliche Wesen verläßliches Wissen darüber erlangen können durch die Anwendung der wissenschaftlichen Methode.

Tiefgreifende konzeptionelle Änderungen im zwanzigsten Jahrhundert haben diese Kartesisch-Newtonsche Metaphysik unterminiert. [...] Es wurde offensichtlich, daß die physikalische "Wirklichkeit" im Kern ein soziales und linguistisches Konstrukt ist. Wissenschaftliche Erkenntnis ist weit davon entfernt, objektiv zu sein, stattdessen werden Machtverhältnisse reflektiert und kodifiziert. Der Wahrheitsanspruch der Wissenschaft ist theorieüberladen und selbstbezüglich.

Westliche Physik bedient sich seit Galileo der Mathematik. Aber wessen Mathematik? Wie Aronowitz feststellte, kann weder Logik noch Mathematik der Kontaminierung durch das Soziale entkommen.




[many natural scientists, and especially physicist] cling to the dogma imposed by the long post-Enlightenment hegemony over the Western intellectual outlook, which can be summarized briefly as follows: that there exists an external world, whose properties are independent of any individual human being and indeed of humanity as a whole; that these properties are encoded in ``eternal'' physical laws; and that human beings can obtain reliable, albeit imperfect and tentative, knowledge of these laws by hewing to the ``objective'' procedures and epistemological strictures prescribed by the (so-called) scientific method.

But deep conceptual shifts within twentieth-century science have undermined this Cartesian-Newtonian metaphysics; revisionist studies in the history and philosophy of science have cast further doubt on its credibility; and, most recently, feminist and poststructuralist critiques have demystified the substantive content of mainstream Western scientific practice, revealing the ideology of domination concealed behind the façade of ``objectivity''. It has thus become increasingly apparent that physical ``reality'', no less than social ``reality'', is at bottom a social and linguistic construct; that scientific ``knowledge", far from being objective, reflects and encodes the dominant ideologies and power relations of the culture that produced it; that the truth claims of science are inherently theory-laden and self-referential; and consequently, that the discourse of the scientific community, for all its undeniable value, cannot assert a privileged epistemological status with respect to counter-hegemonic narratives emanating from dissident or marginalized communities.

[...] mainstream Western physical science has, since Galileo, been formulated in the language of mathematics. But whose mathematics? The question is a fundamental one, for, as Aronowitz has observed, ``neither logic nor mathematics escapes the `contamination' of the social.'

http://www.physics.nyu.edu/sokal/transgress_v2/transgress_v2_singlefile.html

Das fällt mir immer ein, wenn jemand die Subjektivitätskarte spielt.

Objektiv ist das natürlich sehr, sehr gemein von mir. zynisches Grinsen Verlegen

#132:  Autor: SkeptikerWohnort: 129 Goosebumpsville BeitragVerfasst am: 16.01.2017, 08:24
    —
Marcellinus hat folgendes geschrieben:
Skeptiker hat folgendes geschrieben:
… das Bewusstsein spiegelt eine objektive Realität wieder …


Das ist selbst für deine Verhältnisse naiv!


smallie hat folgendes geschrieben:
Eigentlich stand da "das Bewusstsein spiegelt eine objektive Realität wieder, so ungenau dies auch sein mag."


Beachte bitte die Vollständigkeit meiner Aussagen. Weder ein Zerpflücken von Sätzen in Busswörter noch ein Snippen wichtiger Halbsätze sind einer inhaltlichen Auseinandersetzung angemessen, sondern das ist Schlampigkeit in der Diskussion.

Marcellinus hat folgendes geschrieben:
Skeptiker hat folgendes geschrieben:
Der wissenschaftlich-technische Fortschritt und das technische und organisatorische Funktionieren immer komplexerer Anwendungen zeigt doch, dass die objektive Realität so schlecht wohl nicht verstanden wird vom Menschen. Und zwar über das bloße Überleben hinaus. Die Menschheit produziert gewissermaßen nicht nur ökonomische Mehrprodukte und Mehrwerte, sondern auch über das bloße Überleben ein wachsendes Potenzial an Mehrerkenntnis.

die Evidenz dessen soll naiv sein? Naiv wäre es, die Augen davor zu verschließen, so wie ein Kind, welches denkt, die Welt sei dann verschwunden.


Nein, naiv ist nur die Vorstellung, die „objektive Realität spiegele sich im menschlichen Bewusstseins“. Das ist simpler Induktionismus.


Wie gesagt, erst mal Erfassen, was der andere sagen will und zum zweiten Eingehen, auf das, was er ergänzt und ausführt. Wo bleibt denn deine Antwort auf meine Hinweise offenbar richtigen Verständnisses der Welt durch "das technische und organisatorische Funktionieren immer komplexerer Anwendungen", wie ich schrieb?

Marcellinus hat folgendes geschrieben:
Skeptiker hat folgendes geschrieben:
Das kann nur dann vorkommen, wenn bereits erreichte wissenschaftliche Erkenntnisse vergessen oder unter den Teppich gekehrt werden. Das sind aber dann politische Vorgänge, die mit dem wissenschaftlichen Fortschritt und der Richtigkeit ihrer Erkenntnisse überhaupt nichts zu tun hat.


Ach, und „wissenschaftlicher Fortschritt“ hat nichts mit Politik zu tun? Wie war das noch mit der Atombombe? Ob Wissenschaften fortschreiten oder zurück, ist immer auch eine Frage der Politik. Das solltest du eigentlich wissen.


Die Atombombe ist Technik. Die Wissenschaft ist nicht die Technik. Mit den Augen rollen

Was ich meinte war: es ist politisch möglich und es geschieht auch, wissenschaftliche Forschungen und Verständnisse unter den Teppich zu kehren. Das ändert doch aber nicht an der menschlichen Fähigkeit, richtige Erkenntnisse zu produzieren und darin weiter voran zu schreiten, sich der Realität immer ein Stück weiter anzunähern, im Zuge dieses fortlaufenden Prozesses auch ggf. neue Paradigmen zu entwickeln auf den Stufen bereits erreichter Erkenntnisse.

Also bitte keine Verwechslungen zwischen Wissenschaft, Politik und Philosophie! freakteach

Marcellinus hat folgendes geschrieben:
Skeptiker hat folgendes geschrieben:
Marcellinus hat folgendes geschrieben:
Skeptiker hat folgendes geschrieben:
In deiner Philosophie, die ich für eine grottenschlechte halte, …


Als jemand, dessen Vorstellung im wesentlichen in der Philosophie des 19. Jh. stehengeblieben ist, schreibt genau der Richtige! Lachen


Das 19. Jh. ist groß. Es gab da viele verschiedene und gegensätzliche Philosophien. Es gab gute und schlechte, rückwärtsgewandte und zukunftsträchtige.

Das weißt du doch, oder ...-? Auf den Arm nehmen


Nein, du sagst doch selbst, daß „meine Philosophie“ eine „grottenschlechte“ sei. Das 19. Jh. war auch nicht „groß“ (wie immer man das messen wollte), nicht einmal besonders lang. Das 20. Jh. war zB einen Tag länger. Nur woran du „gute“ von „schlechter“ (mal ganz abgesehen von meiner), „rückwärtsgewandter“ von „zukunftsträchtiger“ Philosophie unterscheiden willst, das wird wohl dein Geheimnis bleiben.

Bessere von schlechterer Wissenschaft zu unterscheiden, mag ja noch angehen, Sachkenntnis mal vorausgesetzt. Aber Philosophie? Da sich Philosophen auch auf sonst nichts einigen können, warum gerade auf ein verbindliches Qualitätskriterium? Bis zum Beweis des Gegenteils vermute ich daher mal, es dürfte sich um eine Geschmacksfrage handeln.


Da haben wir es wieder. So wie für dich wissenschaftliche Theorie einfach nur von Zeit zu Zeit wechselnde *Vostellungen* (*bloß menschengemacht Modelle*) seien, so siehst du auch ein bunten Supermarkt von Ideen, wenn du unterschiedliche philophische Ansätze siehst. Alles ist für dich beliebig und die Gründe interessieren dich nicht. Also muss der Philospoph des 21. Jahrhunderst seinen *Geschmack* sprechen lassen. Das ist ein Subjektivismus, wie er sich auch bei samsons Punkt 2 seiner *Angebote* findet. Wahr sei, was du für wahr befindest.

Und dann noch dein Hinweis auf das 19. Jahrhundert. Ist dir überhaupt klar, dass bereits Demokrit Theorien über die Widerspiegelung der Welt im Bewusstsein erarbeitet hat und dass dieser Faden in ganz neuer umfassenderer und elaborierter Form später wieder aufgegriffen wurde und weiterhin entwickelt wird?

Und ist für dich der Faschismus weil 20. Jahrundert progressiver als der der Marxismus weil 19. Jahrhundert?

Schau: Eine wissenschaftliche Theorie ist keine Ware mit einem eingebauten Verfallsdatum, welches du in deinem weltanschaulichen Bunke nur abwarten musst, um dann, wenn es scheinbar vorbei ist, wieder heraus zu kommen, um anzuknüpfen an vorphilosophische und allerlei irrationale Philosophieversatzsstücke.

Auch Philosophie ist kein Kaugummi, das du nach einiger Zeit, wenn es ausgelutscht ist, einfach ausspucken kannst ...-

#133:  Autor: Er_Win BeitragVerfasst am: 16.01.2017, 09:41
    —
Skeptiker hat folgendes geschrieben:


Da haben wir es wieder. So wie für dich wissenschaftliche Theorie einfach nur von Zeit zu Zeit wechselnde *Vostellungen* (*bloß menschengemacht Modelle*) seien, so siehst du auch ein bunten Supermarkt von Ideen, wenn du unterschiedliche philophische Ansätze siehst. Alles ist für dich beliebig und die Gründe interessieren dich nicht. Also muss der Philospoph des 21. Jahrhunderst seinen *Geschmack* sprechen lassen. Das ist ein Subjektivismus, wie er sich auch bei samsons Punkt 2 seiner *Angebote* findet. Wahr sei, was du für wahr befindest.

[...]

Schau: Eine wissenschaftliche Theorie ist keine Ware ...


Jetzt tust du @Marcellinus unrecht bzw. interpretierst etwas zu viel ...

Bei deinem Glauben an eine bereits (weit?) fortgeschrittene Erkenntnis einer "objekiven Realität" bin ich auch der Ansicht, dass das einem sozialen Verständnis "der Wissenschaft" als Art "objektiver Ersatzreligion" geschuldet ist.

Mir gefällt da gut eine begriffliche Unterscheidung von "Realität" und "Wirk_lichkeit". Die Wissenschaft hat eine enorm beschleunigte Erkenntnis über die reproduzierbaren, reduktionistischen Wirk-Zusammenhänge in vielen Einzeldisziplinen gezeitigt.

Aber schon ihre eigenen heutigen Basismodelle werfen in Bezug auf die Erkenntnis einer "objektiven Gesamtrealität" mehr Fragen auf, als beantwortet werden - erinnerlich liegen demnach wirk-strukturelle Erkenntnisse zu >95% noch fast völlig "im Dunkeln".

Und eine Theorie selbst ist keine Ware, aber das gesamte Forschungsumfeld unterliegt sozialen und markt-"gerechten" Prinzipien viel eher als rein wissenschaftstheoretischen.

In dem Sinne kann man dem Zitat von @smallie durchaus zustimmen:
Zitat:
Tiefgreifende konzeptionelle Änderungen im zwanzigsten Jahrhundert haben diese Kartesisch-Newtonsche Metaphysik unterminiert. [...] Es wurde offensichtlich, daß die physikalische "Wirklichkeit" im Kern ein soziales und linguistisches Konstrukt ist. Wissenschaftliche Erkenntnis ist weit davon entfernt, objektiv zu sein, stattdessen werden Machtverhältnisse reflektiert und kodifiziert. Der Wahrheitsanspruch der Wissenschaft ist theorieüberladen und selbstbezüglich.

#134:  Autor: SkeptikerWohnort: 129 Goosebumpsville BeitragVerfasst am: 16.01.2017, 11:38
    —
Er_Win hat folgendes geschrieben:
Skeptiker hat folgendes geschrieben:
Da haben wir es wieder. So wie für dich wissenschaftliche Theorie einfach nur von Zeit zu Zeit wechselnde *Vostellungen* (*bloß menschengemacht Modelle*) seien, so siehst du auch ein bunten Supermarkt von Ideen, wenn du unterschiedliche philophische Ansätze siehst. Alles ist für dich beliebig und die Gründe interessieren dich nicht. Also muss der Philospoph des 21. Jahrhunderst seinen *Geschmack* sprechen lassen. Das ist ein Subjektivismus, wie er sich auch bei samsons Punkt 2 seiner *Angebote* findet. Wahr sei, was du für wahr befindest.

[...]

Schau: Eine wissenschaftliche Theorie ist keine Ware ...


Jetzt tust du @Marcellinus unrecht bzw. interpretierst etwas zu viel ...

Bei deinem Glauben an eine bereits (weit?) fortgeschrittene Erkenntnis einer "objekiven Realität" bin ich auch der Ansicht, dass das einem sozialen Verständnis "der Wissenschaft" als Art "objektiver Ersatzreligion" geschuldet ist.


Du meinst *Wissenschaftsgläubigkeit*, gelle? Cool

Ich bin durchaus folgender Meinung:

Albert Einstein hat folgendes geschrieben:
Es ist absolut möglich, dass jenseits der Wahrnehmung unserer Sinne ungeahnte Welten verborgen sind.


Gleichwohl ist die menschliche Erkenntnis durchaus - im Vergleich zur Steinzeit - relativ deutlich fortgeschritten; dies ist nun wirklich keine Sache des (unbegründeten) Glaubens, sondern einer eindrucksvollen Evidenz.

Er_Win hat folgendes geschrieben:
Mir gefällt da gut eine begriffliche Unterscheidung von "Realität" und "Wirk_lichkeit". Die Wissenschaft hat eine enorm beschleunigte Erkenntnis über die reproduzierbaren, reduktionistischen Wirk-Zusammenhänge in vielen Einzeldisziplinen gezeitigt.

Aber schon ihre eigenen heutigen Basismodelle werfen in Bezug auf die Erkenntnis einer "objektiven Gesamtrealität" mehr Fragen auf, als beantwortet werden - erinnerlich liegen demnach wirk-strukturelle Erkenntnisse zu >95% noch fast völlig "im Dunkeln".


Das ist jetzt sehr allgemein formuliert von dir und insofern schwierig zu diskutieren. Aber natürlich sind bestimmte Strukturen und Zusammenhänge bisher noch weitgehend und komplett unverstanden, etwa das Bewusstsein, der Aufbau der Materie oder die Struktur des Universums und seine Geschichte. Auch um Zeit & Raum gibt es viele Rätsel. Zusammenhänge zwischen Zeit, Energie und Entropie sind aber immerhin formulierbar.

Aber vielleicht ist das auch gar kein so großes Problem, da das, was die Menschheit ungeachtet dessen weiß, schon sehr viel ist und man sollte dieses Wissen sinnvoll nutzen, finde ich.

Er_Win hat folgendes geschrieben:
Und eine Theorie selbst ist keine Ware, aber das gesamte Forschungsumfeld unterliegt sozialen und markt-"gerechten" Prinzipien viel eher als rein wissenschaftstheoretischen.


Ja, das ist bedauerlich. Wir würden uns sicher wünschen, dass internationale Kooperationen bei der Erforschung neuer Medikamente forciert würden statt nationale Forschungen nach immer mächtigeren und komplexeren Waffensystemen oder "marktgerechten" Autos etc.

Somit ist die Wissenschaft heute die Magd, die Dienerin des Profits. Und was liegt nicht schon alles in den Schubladen der Erfinder, was der Menschheit von großem Nutzen wäre.

Er_Win hat folgendes geschrieben:
In dem Sinne kann man dem Zitat von @smallie durchaus zustimmen:
Zitat:
Tiefgreifende konzeptionelle Änderungen im zwanzigsten Jahrhundert haben diese Kartesisch-Newtonsche Metaphysik unterminiert. [...] Es wurde offensichtlich, daß die physikalische "Wirklichkeit" im Kern ein soziales und linguistisches Konstrukt ist. Wissenschaftliche Erkenntnis ist weit davon entfernt, objektiv zu sein, stattdessen werden Machtverhältnisse reflektiert und kodifiziert. Der Wahrheitsanspruch der Wissenschaft ist theorieüberladen und selbstbezüglich.


Das betrifft die Interpretation von Erkenntnissen, die in der Tat vielfach ideologisch ist (siehe meine Signatur!).

Aber die Erkenntnisfähigkeit des Menschen an sich ist etwas anderes und darum geht es ja ...-!

#135:  Autor: Er_Win BeitragVerfasst am: 16.01.2017, 14:31
    —
Skeptiker hat folgendes geschrieben:


Er_Win hat folgendes geschrieben:
Mir gefällt da gut eine begriffliche Unterscheidung von "Realität" und "Wirk_lichkeit". Die Wissenschaft hat eine enorm beschleunigte Erkenntnis über die reproduzierbaren, reduktionistischen Wirk-Zusammenhänge in vielen Einzeldisziplinen gezeitigt.

Aber schon ihre eigenen heutigen Basismodelle werfen in Bezug auf die Erkenntnis einer "objektiven Gesamtrealität" mehr Fragen auf, als beantwortet werden - erinnerlich liegen demnach wirk-strukturelle Erkenntnisse zu >95% noch fast völlig "im Dunkeln".


Das ist jetzt sehr allgemein formuliert von dir und insofern schwierig zu diskutieren.


Wo liegt die Schwierigkeit ? Immerhin ist der Begriff "objektive Realität" um nichts weniger allgemein und vermutlich fehlt uns beiden die Genialität eines @uwebus, welcher das ganze technisch mit wenigen die gesamte QM "verdrängenden" Gleichungen "eindrücklich" erklären kann.

Btw. die von mir genannten >95% beziehen sich auf die Angaben über den ca. 4-5% Anteil baryonischer Materie

#136:  Autor: Marcellinus BeitragVerfasst am: 16.01.2017, 18:08
    —
Er_Win hat folgendes geschrieben:
Skeptiker hat folgendes geschrieben:

Da haben wir es wieder. So wie für dich wissenschaftliche Theorie einfach nur von Zeit zu Zeit wechselnde *Vostellungen* (*bloß menschengemacht Modelle*) seien, so siehst du auch ein bunten Supermarkt von Ideen, wenn du unterschiedliche philophische Ansätze siehst. Alles ist für dich beliebig und die Gründe interessieren dich nicht. Also muss der Philospoph des 21. Jahrhunderst seinen *Geschmack* sprechen lassen. Das ist ein Subjektivismus, wie er sich auch bei samsons Punkt 2 seiner *Angebote* findet. Wahr sei, was du für wahr befindest.

Jetzt tust du @Marcellinus unrecht bzw. interpretierst etwas zu viel ...


Ach, laß man! Was soll ich tun, um Skeptiker zufrieden zu stellen? Ich gebe ganz ehrlich zu, daß ich einen Augenblick überlegt habe, ob ihn versuchen soll, ihn zu überzeugen, daß ich kein „Subjektivist“ bin. Aber dann habe ich mich dagegen entschieden. Irgendwie habe ich keine Lust. Statt dessen lieber etwas Provokation, wenn’s gefällt. zwinkern

Ja, es gibt eine Realität, aber wir sind noch nie dagewesen, noch nicht mal auf Besuch! Wir sehen diese Welt nicht wie sie ist, wir sehen eine Konstruktion unseres Gehirns, und die ist nun einmal nicht an Erkenntnis orientiert, sondern nur am Überleben. Die Erdbeeren, die wir sehen, sind nicht rot, die Blätter nicht grün, die Bäume nicht kleiner, nur weil sie weiter weg sind und die Gesichter, die wir im dunklen Wald sehen, sind einfach nur eine Vorsichtsmaßnahme unseres Gehirns. Von „Widerspiegelung der Welt im Bewusstsein“ also keine Rede (da irrte der alte Grieche).

Dazu ein kleines Zitat: „Das Hirn ist viel mehr eine hypernervöse Alarmanlage, gebaut um uns Überleben zu lassen und nicht dafür, dass wir die Welt begreifen. Erkenntnisgewinn ist einfach nicht sein Zweck, sondern nur eine Nebenwirkung – die nur eintritt, wenn wir uns echt Mühe geben. Ansonsten funktioniert die Alarmanlage vollautomatisch, mit einfachen Mechanismen, den Denkzwängen.

Der wichtigste davon: Das Gehirn erkennt keine Zusammenhänge. Es konstruiert sie. Dauernd und in Hochgeschwindigkeit und vollautomatisch: Zwischen dem Mond und dem eigenen Verhalten, zwischen Medikamenten und der eigenen Gesundheit, zwischen allem was zeitlich knapp beieinander hängt. Das macht es, um ja keinen Zusammenhang zu übersehen. So wie eine Alarmanlage Fehlalarme in Kauf nimmt, baut das Hirn unzählige falsche Zusammenhänge, um ja keinen wesentlichen echten zu übersehen.“


Mit anderen Worten, unsere Denkfähigkeit beruht vor allem auf einer besonderen Fähigkeit: der Fantasie! Mit ihrer Hilfe konstruieren wir Zusammenhänge, und schauen dann, wie weit wir damit kommen, oder wie Popper zu Recht sagte: "Wir sind mit der Wirklichkeit immer genau in den Momenten in Kontakt, in denen unsere Theorien an ihr scheitern.“

Wir Menschen haben uns vom Anfang unserer Geschichte an Modelle von dieser Welt gemacht, Modelle von Zusammenhängen zwischen den Beobachtungen, die unser Gehirn uns liefert. Dabei haben wir offenbar eine gewisse Vorliebe für Erklärungen, die auf persönlichen Verursachern beruhen, vermutlich weil es sicherer war, hinter jedem Geräusch einen Säbelzahntiger zu vermuten, als ihn auch nur ein einziges Mal zu übersehen. So vermutete man hinter dem Regen eben Regengeister. Wenn der Schamane seine Regentanz machte, und es einige Zeit später regnete, hatte sich die Erklärung bestätigt. So funktionieren unsere Gehirne bis heute.

Lange Zeit haben die Menschen beim Nachdenken über die unbelebte wie die belebte Natur absolute, endgültige und dogmatische Antworten auf ihre Fragen, vor allem auf solche von gefühlsmäßiger Bedeutung in den Handlungen und Absichten von persönlichen Verursachern, Göttern und Geistern, gesucht (und auch gefunden), und manche tun das bis in die Gegenwart.

Erst mühsam, Schritt für Schritt, sehr spät und nach zahlreichen (Ent-)Täuschungen und keineswegs auf allen Gebieten gleichermaßen haben die Menschen angefangen, nach unpersönlichen und dafür empirisch nachprüfbaren Modellen von Zusammenhänge zu suchen. Dabei gehört beides untrennbar zusammen, Tatsachenbeobachtung und Modellbildung, Empirie und Theorie. Ohne Tatsachen keine realistischen Modelle, ohne Modelle keine realistischen Beobachtungen. Es ist ein wechselseitiger Prozeß, ohne bestimmbaren Anfang oder Ende, und auch selbst nur als Prozeßmodell, als vierdimensionales Modell zu verstehen.

Ja, die Menschen haben Erkenntnisfortschritte gemacht, Fortschritte an nachprüfbarem Wissen, aber man muß nicht nur in die Medizingeschichte schauen, um zu bemerken, daß viele dieser Erkenntnisfortschritte mit Irrtümern behaftet waren. Ja, Fortschritt ist nicht nur eine subjektive Überzeugung, sondern eine beobachtbare Tatsache, aber der Fortschritt ist weder gradlinig noch zwangsläufig und er ist auch nicht auf allen Gebieten menschlicher Geistestätigkeiten gleich groß.

#137:  Autor: vrolijkeWohnort: Stuttgart BeitragVerfasst am: 16.01.2017, 18:56
    —
Marcellinus hat folgendes geschrieben:
Schöner Text


Daumen hoch!

#138:  Autor: smallie BeitragVerfasst am: 16.01.2017, 23:13
    —
Er_Win hat folgendes geschrieben:
Und eine Theorie selbst ist keine Ware, aber das gesamte Forschungsumfeld unterliegt sozialen und markt-"gerechten" Prinzipien viel eher als rein wissenschaftstheoretischen.

Das stimmt. Mal sehen wie viele Leute noch wie lange an Supersymmetrie festhalten, obwohl die Luft dafür schon ziemlich dünn geworden ist.


Er_Win hat folgendes geschrieben:
In dem Sinne kann man dem Zitat von @smallie durchaus zustimmen:
Zitat:
Tiefgreifende konzeptionelle Änderungen im zwanzigsten Jahrhundert haben diese Kartesisch-Newtonsche Metaphysik unterminiert. [...] Es wurde offensichtlich, daß die physikalische "Wirklichkeit" im Kern ein soziales und linguistisches Konstrukt ist. Wissenschaftliche Erkenntnis ist weit davon entfernt, objektiv zu sein, stattdessen werden Machtverhältnisse reflektiert und kodifiziert. Der Wahrheitsanspruch der Wissenschaft ist theorieüberladen und selbstbezüglich.

Ich war hundsgemein.

Der Text stammt von Sokal aus seinem Hoax. Du darfst mich mit einem Schimpfwort belegen für dieses fiese Manöver.

An Marcellinus: du vertrittst ein postmodernes Progamm zur Vernichtung jeglicher Wahrheit. Hochtrabend gesagt gibt es keinen Grund für einen ontologischen Nihilismus. fwo sagt ganz schlicht: "Ich glaube an die Existenz der Welt in der ich lebe."

Es stimmt ja, daß alles nur ein Modell ist. Trotzdem enthalten neue Modelle üblicherweise die alten. Der Mondflug läßt sich auch mit Newton verwirklichen. Die Boulevard-Schlagzeile müßte lauten: "Einsteins neue Theorie bestätigt Newtons Gesetze in 95% aller praktischen Fälle."

#139:  Autor: smallie BeitragVerfasst am: 16.01.2017, 23:20
    —
Marcellinus hat folgendes geschrieben:
Ja, die Menschen haben Erkenntnisfortschritte gemacht, Fortschritte an nachprüfbarem Wissen, aber man muß nicht nur in die Medizingeschichte schauen, um zu bemerken, daß viele dieser Erkenntnisfortschritte mit Irrtümern behaftet waren. Ja, Fortschritt ist nicht nur eine subjektive Überzeugung, sondern eine beobachtbare Tatsache, aber der Fortschritt ist weder gradlinig noch zwangsläufig und er ist auch nicht auf allen Gebieten menschlicher Geistestätigkeiten gleich groß.

Sag sowas nicht. Am Ende stellt sich noch heraus, daß du dir mit Skeptiker eigentlich einig bist, obwohl du seine Sicht naiv nanntest.

#140:  Autor: Er_Win BeitragVerfasst am: 17.01.2017, 09:40
    —
smallie hat folgendes geschrieben:
Er_Win hat folgendes geschrieben:
Und eine Theorie selbst ist keine Ware, aber das gesamte Forschungsumfeld unterliegt sozialen und markt-"gerechten" Prinzipien viel eher als rein wissenschaftstheoretischen.

Das stimmt. Mal sehen wie viele Leute noch wie lange an Supersymmetrie festhalten, obwohl die Luft dafür schon ziemlich dünn geworden ist.


Dabei gefällt mir der grundsätzliche theoretische Ansatz von Susy ...

smallie hat folgendes geschrieben:

Er_Win hat folgendes geschrieben:
In dem Sinne kann man dem Zitat von @smallie durchaus zustimmen:
Zitat:
Tiefgreifende konzeptionelle Änderungen im zwanzigsten Jahrhundert haben diese Kartesisch-Newtonsche Metaphysik unterminiert. [...] Es wurde offensichtlich, daß die physikalische "Wirklichkeit" im Kern ein soziales und linguistisches Konstrukt ist. Wissenschaftliche Erkenntnis ist weit davon entfernt, objektiv zu sein, stattdessen werden Machtverhältnisse reflektiert und kodifiziert. Der Wahrheitsanspruch der Wissenschaft ist theorieüberladen und selbstbezüglich.

Ich war hundsgemein.

Der Text stammt von Sokal aus seinem Hoax. Du darfst mich mit einem Schimpfwort belegen für dieses fiese Manöver.


schon ok - daher die übertrieben polemisch/pathetisch angehauchte Formulierung.

Nur sagen "hoaxfrei" zB. Zeilinger oder auch schon Heisenberg dasselbe, wenn es um die "Objektivierung" des Seins bzw. unserer Erkenntnis darüber geht:

"Die Naturwissenschaft beschreibt und erklärt die Natur nicht einfach, so wie sie "an sich" ist. Sie ist vielmehr ein Teil des Wechselspiels zwischen der Natur und uns selbst."

"Die Ontologie des Materialismus beruhte auf der Illusion, daß man die Art der Existenz, das unmittelbar Faktische der uns umgebenden Welt, auf die Verhältnisse im atomaren Bereich extrapolieren könne." (Heisenberg)


"Schlimmer" noch der Heisenberg Schüler Dürr, der für seine Aussagen von manchen Kollegen als "alterssenil" statt als "altersweise" bezeichet wird.

Zum Thema, bis hin zur heutigen Einbettung/Genese der Wissenschaft in ein/em (neoliberales/n) Wirtschaftssystem ganz lesenswert:
Wissen ist Macht. Erkenntnistheoretische Spielarten eines Prinzips (Karin Ulrich)

#141:  Autor: smallie BeitragVerfasst am: 19.01.2017, 00:18
    —
Er_Win hat folgendes geschrieben:
smallie hat folgendes geschrieben:
Er_Win hat folgendes geschrieben:
Und eine Theorie selbst ist keine Ware, aber das gesamte Forschungsumfeld unterliegt sozialen und markt-"gerechten" Prinzipien viel eher als rein wissenschaftstheoretischen.

Das stimmt. Mal sehen wie viele Leute noch wie lange an Supersymmetrie festhalten, obwohl die Luft dafür schon ziemlich dünn geworden ist.


Dabei gefällt mir der grundsätzliche theoretische Ansatz von Susy ...

Allen hat er gefallen. Vielen gefällt er immer noch.

Aber genaugenommen ist SUSY keine Theorie, sondern nur eine These. Um Theorie im strengen Sinn zu sein, müßte sie Ergebnisse vorweisen können. Mein Formulierungs-Vorschlag: Thesen sind wie Waren. Sie werden heiß gehandelt oder links liegen gelassen. Die Wahrheit hingegen ist nicht käuflich. (Errm. Oh. Grummel. Bitte den letzten Satz streichen.)




Er_Win hat folgendes geschrieben:
smallie hat folgendes geschrieben:
Ich war hundsgemein.

schon ok

Uff.



Er_Win hat folgendes geschrieben:
Nur sagen "hoaxfrei" zB. Zeilinger oder auch schon Heisenberg dasselbe, wenn es um die "Objektivierung" des Seins bzw. unserer Erkenntnis darüber geht:

"Die Naturwissenschaft beschreibt und erklärt die Natur nicht einfach, so wie sie "an sich" ist. Sie ist vielmehr ein Teil des Wechselspiels zwischen der Natur und uns selbst."

Das ist mir zu blumig. Ein Wechselspiel zu behaupten ist trivial. Ohne konkret zu benennen, wie das Wechselspiel aussieht, kann das alles Mögliche bedeuten. Jeder kann den Satz auf seine Weise lesen und zustimmen, auch wenn man unausgesprochen ganz unterschiedliches hineinliest.

Mit anderen Worten: der Satz macht eigentlich gar keine Aussage. nee


Er_Win hat folgendes geschrieben:
"Die Ontologie des Materialismus beruhte auf der Illusion, daß man die Art der Existenz, das unmittelbar Faktische der uns umgebenden Welt, auf die Verhältnisse im atomaren Bereich extrapolieren könne." (Heisenberg)

Erstens:

Im Heisenberg-Zitat kommt ein gewisser Schock zum Ausdruck über die Andersartigkeit der Verhältnisse im atomaren Bereich. Als Spekulation: könnte es sein, daß die Verwunderung irgendwann nachläßt, so in hundert oder fünfhundert Jahren, weil man dann lange genug praktisch mit der Sache gearbeitet und ein Gefühl dafür entwickelt hat? Differential- und Integralrechnung war früher etwas für Gelehrte, heute ist es Schulstoff.


Zweitens: *kicher*

Weißt du was ich darauf antworten wollte - genau das was im Amazon-Anreißer deines Links zu Wissen ist Macht. Erkenntnistheoretische Spielarten eines Prinzips steht:


Feynman hat folgendes geschrieben:
Was ich nicht nachbauen kann, habe ich nicht verstanden.
What I cannot create, I do not understand.

Zeilinger kann Dinge aus der Quantenwelt recht gut nachbauen. So gesehen hat er einiges verstanden, auch wenn er selber es nicht wahrhaben will.

Daß es im atomaren Bereich anders läuft ist für "das unmittelbar Faktische der uns umgebenden Welt" völlig belanglos. Quantenphysik spielt weder eine Rolle für die Umlaufbahn des Mondes noch für die nicht-Newtonsche Periheldrehung des Merkurs.


Feynman hat folgendes geschrieben:
Stell dir nicht die Frage, soweit du es vermeiden kannst, "Aber wie kann es genau so sein? Denn die Frage zieht dich in einen Strudel, in eine Sackgasse, aus der noch niemand herausgekommen ist. Niemand weiß, warum es genau so ist."

Do not keep saying to yourself, if you can possibly avoid it, "But how can it be like that?" because you will get "down the drain", into a blind alley from which nobody has yet escaped. Nobody knows how it can be like that.

Da ist was dran.

Kritisch angemerkt gilt aber auch, daß Wissenschaft gerade durch die Frage "wie kann es so sein" fortschreitet.



Das hier finde ich ziemlich gut:

Marcellinus hat folgendes geschrieben:
... wie Popper zu Recht sagte: "Wir sind mit der Wirklichkeit immer genau in den Momenten in Kontakt, in denen unsere Theorien an ihr scheitern.“

Nur daß dieser Kontakt mit Wirklichkeit natürlich nicht aufhört, sobald der Moment vorbei ist. Auch heute ist noch bekannt, daß die Periheldrehung des Merkur eben nicht mit Newton erklärbar ist. Zu wissen, wie es nicht ist, das ist auch eine Form des Wissens - und keine schlechte. Gibt es dafür ein Wort? Ausschluß-Ontologie? Inverse Objektivität?

Weitergesponnen ist der Kontakt mit der Wirklichkeit eine Folge immer genaueren Hinschauens. Früher mit bloßem Auge, später mit Fernrohr, Teleskop oder Teilchenbeschleuniger. Der Kontakt mit der Wirklichkeit läßt sich erzwingen. Zumindest solange, bis die technischen Möglichkeiten ausgereizt sind.

#142:  Autor: Marcellinus BeitragVerfasst am: 19.01.2017, 00:28
    —
Kleine Frage am Rande: Warum kommen eure Beispiele für das, was eurer Ansicht Wissenschaft ist oder zu sein hat, eigentlich immer aus dem Bereich der Physik?

#143:  Autor: smallie BeitragVerfasst am: 19.01.2017, 00:33
    —
Kramer hat folgendes geschrieben:
smallie hat folgendes geschrieben:
smallie hat folgendes geschrieben:
Nun ist mir gerade aufgefallen, daß ich doof bin.

Guten Morgen, smallie. Auch schon wach? Obiges kennt man als Aliasing.


Richtig, aber mir wäre neu, dass es das auch in der Natur gibt. Das optische Aliasing (Stroboskopeffekt) entsteht ja auch nicht durch Beschränkungen unseres Sehvermögens, sondern durch die geringe Abtastrate von Filmaufnahmen.

Mir will auch kein Beispiel einfallen.

Wenn es wirklich keins gibt, dann muß ich nochmal über mein Verständnis der Trägheit des Auges nachdenken. Ich dachte, das daß letztlich ebenso auf eine bestimmte Abtastrate hinausläuft. fwo hat dazu schon geschrieben, daß es auf die Umstände ankommt. Helligkeit, usw. Hmm. Es gibt da noch die Sakkaden beim Auge, die spielen da sicher auch mit 'rein.

#144:  Autor: Kramer BeitragVerfasst am: 19.01.2017, 01:11
    —
smallie hat folgendes geschrieben:

Wenn es wirklich keins gibt, dann muß ich nochmal über mein Verständnis der Trägheit des Auges nachdenken. Ich dachte, das daß letztlich ebenso auf eine bestimmte Abtastrate hinausläuft. fwo hat dazu schon geschrieben, daß es auf die Umstände ankommt. Helligkeit, usw. Hmm. Es gibt da noch die Sakkaden beim Auge, die spielen da sicher auch mit 'rein.


Dazu findet man einiges bei Wiki:
https://de.wikipedia.org/wiki/Zeitliches_Aufl%C3%B6sungsverm%C3%B6gen
https://de.wikipedia.org/wiki/Nachbildwirkung
https://de.wikipedia.org/wiki/Stroboskopeffekt

#145:  Autor: smallie BeitragVerfasst am: 19.01.2017, 01:35
    —
Marcellinus hat folgendes geschrieben:
Kleine Frage am Rande: Warum kommen eure Beispiele für das, was eurer Ansicht Wissenschaft ist oder zu sein hat, eigentlich immer aus dem Bereich der Physik?

Das ist eine sehr gute Frage. Zwei widersprüchliche Antworten darauf.


Wenn sich Ontologie per Goethe-Wort damit beschäftigt, was die Welt im Innersten zusammenhält und ausmacht, ist Physik die relevante Wissenschaft. Und nur die Physik, denn welcher Philosoph hat chemische Vorgänge wie die Oxydation von Eisen je als ontologisch bedeutsam eingestuft? Oder geologische Vorgänge wie die Plattentektonik? Und so weiter...

Insofern ist der Fokus auf die Physik nachvollziehbar.



Andererseits bestreite ich deine Behauptung. Alle naselang berufe ich mich auf Darwin oder rede von kultureller Evolution. Nicht mal Bibel-Rezeption bleibt verschont. zwinkern

Wie siehst du das? Ist Evolution auch "nur ein Modell"? Oder ist es eine Erfahrungstatsache? So wie Raum, Zeit, Gravitation und Quantenverschränkung Erfahrungstatsachen sind. (Mist, jetzt habe ich bereits die Antwort vorgegeben.)

#146:  Autor: Kramer BeitragVerfasst am: 19.01.2017, 02:29
    —
Marcellinus hat folgendes geschrieben:
Kleine Frage am Rande: Warum kommen eure Beispiele für das, was eurer Ansicht Wissenschaft ist oder zu sein hat, eigentlich immer aus dem Bereich der Physik?


Meine Beispiele hier kommen eigentlich alle aus der Musik. Mehr oder weniger. Das Experimentieren mit Delay-Zeiten unter einer Millisekunde ist mehr der Neugier geschuldet, musikalisch sinnvoll kann man das kaum nutzen. Aber wenn es um das Thema Zeit geht, finde ich, dass Musik ein gutes Beispiel dafür ist, dass wir Zeit ähnlich intuitiv und spontan empfinden können, wie die Grösse eines Raumes. Wenn ein Stück groovt, dann hat das etwas mit Zeit zu tun. Und wenn nicht, dann auch. Wenn wir den Klang eines Instrumentes als schön und interessant empfinden, dann hat das auch etwas mit Zeit zu tun, nämlich damit, wie sich der Klang über einen Zeitraum hinweg entwickelt. Nimm irgendeinen einzelnen Ton irgendeines Instrumentes, der Dir persönlich gefällt. Und dann reduziere den Faktor Zeit auf ein Mindestmass - was dabei heraus kommt, klingt nicht mehr wie das Instrument, sondern wie ein billiges Heimkeyboard aus den 80er Jahren.

#147:  Autor: Marcellinus BeitragVerfasst am: 19.01.2017, 12:14
    —
smallie hat folgendes geschrieben:
Marcellinus hat folgendes geschrieben:
Kleine Frage am Rande: Warum kommen eure Beispiele für das, was eurer Ansicht Wissenschaft ist oder zu sein hat, eigentlich immer aus dem Bereich der Physik?

Das ist eine sehr gute Frage. Zwei widersprüchliche Antworten darauf.

Wenn sich Ontologie per Goethe-Wort damit beschäftigt, was die Welt im Innersten zusammenhält und ausmacht, ist Physik die relevante Wissenschaft. Und nur die Physik, denn welcher Philosoph hat chemische Vorgänge wie die Oxydation von Eisen je als ontologisch bedeutsam eingestuft? Oder geologische Vorgänge wie die Plattentektonik? Und so weiter...

Insofern ist der Fokus auf die Physik nachvollziehbar.


Ontologie ist ein Thema der Philosophie, Physik eine Wissenschaft. Philosophie beschäftigt sich mit dem „Absoluten“, sucht nach der „Wahrheit“; theoretisch-empirische Wissenschaften dagegen (sehr verkürzt) nach Modellen beobachtbarer Zusammenhänge. Ich denke, das sollte man nicht durcheinanderschmeißen.


smallie hat folgendes geschrieben:

Wie siehst du das? Ist Evolution auch "nur ein Modell"? Oder ist es eine Erfahrungstatsache? So wie Raum, Zeit, Gravitation und Quantenverschränkung Erfahrungstatsachen sind. (Mist, jetzt habe ich bereits die Antwort vorgegeben.)


Evolution ist eine beobachtbare Tatsache, Evolutionstheorie ein Modell dafür. Das eine ist Empirie, das andere Theorie. Wobei die Tücke darin besteht, daß man das Modell der Evolutionstheorie braucht, um Evolution wahrzunehmen. Hier merkt man vor allem die Beschränkungen meiner (unserer?) Sprache. Evolution ist ein Prozeß, unsere Wahrnehmung auch. Unsere Sprache neigt aber dazu, Veränderungen auf Unveränderliches zu reduzieren. Was in diesem Falle (wie vermutlich in vielen anderen Fällen auch) nicht funktioniert.

#148:  Autor: Marcellinus BeitragVerfasst am: 19.01.2017, 12:27
    —
Kramer hat folgendes geschrieben:

Meine Beispiele hier kommen eigentlich alle aus der Musik. Mehr oder weniger. Das Experimentieren mit Delay-Zeiten unter einer Millisekunde ist mehr der Neugier geschuldet, musikalisch sinnvoll kann man das kaum nutzen. Aber wenn es um das Thema Zeit geht, finde ich, dass Musik ein gutes Beispiel dafür ist, dass wir Zeit ähnlich intuitiv und spontan empfinden können, wie die Grösse eines Raumes. Wenn ein Stück groovt, dann hat das etwas mit Zeit zu tun. Und wenn nicht, dann auch. Wenn wir den Klang eines Instrumentes als schön und interessant empfinden, dann hat das auch etwas mit Zeit zu tun, nämlich damit, wie sich der Klang über einen Zeitraum hinweg entwickelt. Nimm irgendeinen einzelnen Ton irgendeines Instrumentes, der Dir persönlich gefällt. Und dann reduziere den Faktor Zeit auf ein Mindestmass - was dabei heraus kommt, klingt nicht mehr wie das Instrument, sondern wie ein billiges Heimkeyboard aus den 80er Jahren.


Ich denke, daß wir auch hier wieder an die Beschränkungen unserer Sprache kommen. Begriffe sind gedankliche Werkzeuge zur Orientierung in dieser Welt, und wenn sie mit der beobachtbaren Wirklichkeit nicht Schritt halten, führen sie uns in die Irre.

Was du mit Zeitgefühl meinst, ist etwas anderes als das, was ich darunter verstehe. Zeitgefühl als die Wahrnehmung kurzer aufeinander folgender Ereignisse, wie zB Töne, scheint mir etwas anderes zu sein als das Zeitgefühl, in dem sich „Lebenswendepunkte“ Geburten, Hochzeiten, Todesfälle, Kirege, Hungersnöte, Wirtschaftskrisen neiderschlagen, aber auch der Wechsel der Jahreszeiten.

Ersteres ist vielleicht angeboren, obwohl ich auch da nicht sicher bin. Dafür habe ich einfach schon zu viele Menschen erlebt, deren Musikempfinden vollkommen unterschiedlich ist, ja sich sogar im Laufe eines Lebens verändern kann. Bei letzterem bin ich dagegen sicher (und dafür gibt es auch empirische Belege), daß ein solches Zeitgefühl, daß sich in Kalendern und heute in der Allgegenwart der Uhren ausdrückt, Ergebnis von sozialen Prozessen ist, deren Zwänge wie Möglichkeiten sich im individuellen Habitus der einzelnen Menschen „individualisieren“, so sehr, daß sie ihnen als eine „natürliche“ Eigenschaft aller Menschen erscheinen, obwohl sie in dieser Form noch keine 150 Jahre alt sind.

#149:  Autor: fwoWohnort: im Speckgürtel BeitragVerfasst am: 19.01.2017, 14:10
    —
Marcellinus hat folgendes geschrieben:
Kramer hat folgendes geschrieben:

Meine Beispiele hier kommen eigentlich alle aus der Musik. Mehr oder weniger. Das Experimentieren mit Delay-Zeiten unter einer Millisekunde ist mehr der Neugier geschuldet, musikalisch sinnvoll kann man das kaum nutzen. Aber wenn es um das Thema Zeit geht, finde ich, dass Musik ein gutes Beispiel dafür ist, dass wir Zeit ähnlich intuitiv und spontan empfinden können, wie die Grösse eines Raumes. Wenn ein Stück groovt, dann hat das etwas mit Zeit zu tun. Und wenn nicht, dann auch. Wenn wir den Klang eines Instrumentes als schön und interessant empfinden, dann hat das auch etwas mit Zeit zu tun, nämlich damit, wie sich der Klang über einen Zeitraum hinweg entwickelt. Nimm irgendeinen einzelnen Ton irgendeines Instrumentes, der Dir persönlich gefällt. Und dann reduziere den Faktor Zeit auf ein Mindestmass - was dabei heraus kommt, klingt nicht mehr wie das Instrument, sondern wie ein billiges Heimkeyboard aus den 80er Jahren.


Ich denke, daß wir auch hier wieder an die Beschränkungen unserer Sprache kommen. Begriffe sind gedankliche Werkzeuge zur Orientierung in dieser Welt, und wenn sie mit der beobachtbaren Wirklichkeit nicht Schritt halten, führen sie uns in die Irre.

Was du mit Zeitgefühl meinst, ist etwas anderes als das, was ich darunter verstehe. Zeitgefühl als die Wahrnehmung kurzer aufeinander folgender Ereignisse, wie zB Töne, scheint mir etwas anderes zu sein als das Zeitgefühl, in dem sich „Lebenswendepunkte“ Geburten, Hochzeiten, Todesfälle, Kirege, Hungersnöte, Wirtschaftskrisen neiderschlagen, aber auch der Wechsel der Jahreszeiten.

Ersteres ist vielleicht angeboren, obwohl ich auch da nicht sicher bin. Dafür habe ich einfach schon zu viele Menschen erlebt, deren Musikempfinden vollkommen unterschiedlich ist, ja sich sogar im Laufe eines Lebens verändern kann. Bei letzterem bin ich dagegen sicher (und dafür gibt es auch empirische Belege), daß ein solches Zeitgefühl, daß sich in Kalendern und heute in der Allgegenwart der Uhren ausdrückt, Ergebnis von sozialen Prozessen ist, deren Zwänge wie Möglichkeiten sich im individuellen Habitus der einzelnen Menschen „individualisieren“, so sehr, daß sie ihnen als eine „natürliche“ Eigenschaft aller Menschen erscheinen, obwohl sie in dieser Form noch keine 150 Jahre alt sind.

@Marcellinus

Diese Zeitgefühl ist das wo wir auseinander sind. Und zwar nicht, in dem, wie die Leute früher waren, sondern in dem, wie sie heute sind. Ich bezweifel, dass wir uns da wirklich im Gefühl verändert haben. Was sich verändert hat ist die Präzision in der Kalibrierung der äußeren Taktgeber und deren Wichtigkeit im Alltag. Dass es sich da nicht um ein Zeitgefühl handelt, kannst Du bei der Jugend sehen, die eben nicht mehr durch Minuten und Sekunden getaktet wird, sondern durch das Eintreffen der nächsten Whatsapp-Nachricht auf dem Handy. Wenn die nicht kommt, können die den ganzen Tag verschlafen.

Die präzise Zeit, auf die Du Dich beziehst, die in der Gesellschaft existiert, existiert auch nur dort, bzw. in der Technik, die die Individuen zur Gesellschaft verbindet. Wie der Mensch als Individuum die Zeit sieht, ist in der technisierten Welt allerdings nur zu sehen, solange er nicht primär als Mitglied funktioniert, also in der Jugend (s.o.), im Urlaub und als Arbeitsloser oder Rentner.

Und da wird er in seinen Verabredungen ähnlich "unzuverlässig" wie unsere Vorgänger seit Beginn der verabredeten Kalibrierung der Zeit.

#150:  Autor: Marcellinus BeitragVerfasst am: 19.01.2017, 22:25
    —
fwo hat folgendes geschrieben:

Diese Zeitgefühl ist das wo wir auseinander sind. Und zwar nicht, in dem, wie die Leute früher waren, sondern in dem, wie sie heute sind. Ich bezweifel, dass wir uns da wirklich im Gefühl verändert haben.


Ich könnte jetzt auf die Anthropologie verweisen, auf den Vergleich von Stämmen, die ohne Kontakt zu modernen Zivilisationen waren, und komplett andere Formen der Zeitwahrnehmung hatten, zB nicht einmal eine Vorstellung von ihrem eigenen Alter hatten. Ich könnte auf die Phoenix-Sendereihe „Der taumelnde Kontinent“ verweisen, in der gerade Umwälzung der Selbstwahrnehmung großer Bevölkerungsgruppen durch die Industrialisierung am Beginn des 20. Jh. thematisiert wurde.

Aber ich vermute, es würde dich nicht überzeugen. Unsere Grundannahmen sind zu unterschiedlich. Du gehst davon aus, daß sich unsere Wahrnehmung dieser Welt evolutionär entwickelt und seitdem nicht wesentlich verändert hat. Ich gehe davon aus, daß unserer sozialer Habitus, die Art und Weise, wie wir uns selbst in dieser Welt wahrnehmen, Produkt der sozialen Prozesse ist, die unsere Gesellschaftsentwicklung bestimmt haben.

Ich halte mein Modell für besser als deines (nein, ein Mangel an Selbstbewusstsein ist nicht mein Problem zwinkern), da es nicht nur besser die Sozio- und Psychogenese unserer Gesellschaften erklärt (etwas, was die Soziologie von der Historie unterscheidet, bzw unterscheiden sollte), sondern auch viele der Probleme und Mißverständnisse, die Menschen und Menschengruppen in der aktuellen globalisierten Welt im Verkehr miteinander haben.

Aber am Ende ist es ein Modell, und welches man für besser oder schlechter hält, ist immer auch eine Frage der Entscheidung, solange sich in der eigenen Kultur nicht eines von beiden als bestimmendes Erklärungsmodell durchgesetzt hat. Bis das passiert ist, sollten wir bei meinem Vorschlag bleiben: uns darauf zu einigen, daß wir uns nicht einig sind. Smilie

#151:  Autor: fwoWohnort: im Speckgürtel BeitragVerfasst am: 19.01.2017, 23:16
    —
Marcellinus hat folgendes geschrieben:
fwo hat folgendes geschrieben:

Diese Zeitgefühl ist das wo wir auseinander sind. Und zwar nicht, in dem, wie die Leute früher waren, sondern in dem, wie sie heute sind. Ich bezweifel, dass wir uns da wirklich im Gefühl verändert haben.


Ich könnte jetzt auf die Anthropologie verweisen, auf den Vergleich von Stämmen, die ohne Kontakt zu modernen Zivilisationen waren, und komplett andere Formen der Zeitwahrnehmung hatten, zB nicht einmal eine Vorstellung von ihrem eigenen Alter hatten.


Das finde ich jetzt interessant. Weißt Du etwas über den Zahlenraum dieser Kulturen? Ich rate mal, dass die geografisch rund um den Äquator anzusiedeln sind.
Was mich da auch interessieren würde, das wären linguistische Arbeiten über diese Kulturen (s.u.).

Marcellinus hat folgendes geschrieben:
Ich könnte auf die Phoenix-Sendereihe „Der taumelnde Kontinent“ verweisen, in der gerade Umwälzung der Selbstwahrnehmung großer Bevölkerungsgruppen durch die Industrialisierung am Beginn des 20. Jh. thematisiert wurde.

Aber ich vermute, es würde dich nicht überzeugen. Unsere Grundannahmen sind zu unterschiedlich. Du gehst davon aus, daß sich unsere Wahrnehmung dieser Welt evolutionär entwickelt und seitdem nicht wesentlich verändert hat.


Wenn Du evolutionär nur auf die genetische Evolution beziehst, irrst Du. Mir ist klar, dass speziell das abstrakte Konzept Zeit nicht älter sein kann als unsere Sprache. Unsere Sprache ist gleichzeitig auch die Welt, in der wir denken, das gemeinsame Wissen sammeln und unsere persönliche Welt konstruieren. Menschliche Sprache in der üblichen Bedeutung ist nicht älter als ca 100 000 Jahre.
Was aber alle Sprachen gemeinsam haben, ist, dass sie als Kommunikationskanal einer hochsozialen Art optimiert wurden, wenn sie auch vielleicht von verschiedenen Konzepten ausgehen. Das macht Übersetzungen ähnlich schwierig wie die Migration von Datenbeständen, die zwar im Prinzip das selbe externe Objekt beschreiben, sich aber in der inneren Objektstruktur gewaltig unterscheiden können. Das ist auch der Hintergrund, vor dem ich den Anthropologen misstraue, weil ich regelmäßig vorgeführt bekomme, dass ihnen nicht einmal klar ist, welche Folgen es für unser Bild des frühen Menschen hat, dass die Sprache so jung ist.

Marcellinus hat folgendes geschrieben:
Ich gehe davon aus, daß unserer sozialer Habitus, die Art und Weise, wie wir uns selbst in dieser Welt wahrnehmen, Produkt der sozialen Prozesse ist, die unsere Gesellschaftsentwicklung bestimmt haben.

Ich halte mein Modell für besser als deines (nein, ein Mangel an Selbstbewusstsein ist nicht mein Problem zwinkern), da es nicht nur besser die Sozio- und Psychogenese unserer Gesellschaften erklärt (etwas, was die Soziologie von der Historie unterscheidet, bzw unterscheiden sollte), sondern auch viele der Probleme und Mißverständnisse, die Menschen und Menschengruppen in der aktuellen globalisierten Welt im Verkehr miteinander haben.

Aber am Ende ist es ein Modell, und welches man für besser oder schlechter hält, ist immer auch eine Frage der Entscheidung, solange sich in der eigenen Kultur nicht eines von beiden als bestimmendes Erklärungsmodell durchgesetzt hat. Bis das passiert ist, sollten wir bei meinem Vorschlag bleiben: uns darauf zu einigen, daß wir uns nicht einig sind. Smilie

Gegen diese Einigung habe ich gar nichts.

Aber sollte sie uns davon abhalten, uns in der Sache zu streiten? Bei allem Respekt?
Hier sind so wenige, mit denen ich streiten kann, um etwas dabei zu lernen.

#152:  Autor: smallie BeitragVerfasst am: 20.01.2017, 23:42
    —
Kramer hat folgendes geschrieben:
smallie hat folgendes geschrieben:

Wenn es wirklich keins gibt, dann muß ich nochmal über mein Verständnis der Trägheit des Auges nachdenken. Ich dachte, das daß letztlich ebenso auf eine bestimmte Abtastrate hinausläuft. fwo hat dazu schon geschrieben, daß es auf die Umstände ankommt. Helligkeit, usw. Hmm. Es gibt da noch die Sakkaden beim Auge, die spielen da sicher auch mit 'rein.


Dazu findet man einiges bei Wiki:
https://de.wikipedia.org/wiki/Zeitliches_Aufl%C3%B6sungsverm%C3%B6gen
https://de.wikipedia.org/wiki/Nachbildwirkung
https://de.wikipedia.org/wiki/Stroboskopeffekt

Jetzt ist mir anhand der Links doch noch ein Beispiel eingefallen. Es könnte aber falsch sein.

Kürzlich im Zirkus gekauft: ein Plastikstab, die obere Hälfte ist batteriegetrieben drehbar. Oben sind Schnüre befestigt, an deren Enden Leuchtkugeln sitzen. In schummriger Beleuchtung in Drehung versetzt sehe ich ein Nachbild in Form von Kreisen. Ich sehe aber die Kugeln selbst stroboskopartig an bestimmten Stellen stehend, dann springen sie zur nächsten Stelle. Und das noch nicht mal in regelmäßigen zeitlichen Abständen, sondern abhängig davon, wie ich meine Augen bewege.

Soweit wäre das das gesuchte Beispiel. Allerdings erinnere ich mich dunkel an Fälle, an Arbeitsunfälle, in denen Arbeiter in sich drehende Maschinen gegriffen haben, weil sie stillzustehen schienen. Die Netzfrequenz des Stromes führte zu einem Stroboskopeffekt in der Beleuchtung. Das könnte auch bei meinem Beispiel der Fall sein. Müßte ich mal im Mondlicht ausprobieren, fernab jeglicher künstlichen Beleuchtung.

#153:  Autor: zelig BeitragVerfasst am: 21.01.2017, 13:38
    —
Zitat:
Mit Quantenbits zur Raumzeit
Astrophysiker und Quantenforscher untersuchen gemeinsam, wie kosmische Phänomene mit subatomaren Gesetzen zusammenhängen. Sie versprechen sich neue Einsichten in die grundlegende Struktur von Raum und Zeit.


http://www.spektrum.de/news/die-struktur-von-raum-und-zeit-wird-von-astrophysikern-und-quantenforschern-untersucht/1435484

#154:  Autor: smallie BeitragVerfasst am: 21.01.2017, 21:43
    —
Marcellinus hat folgendes geschrieben:
smallie hat folgendes geschrieben:
Marcellinus hat folgendes geschrieben:
Kleine Frage am Rande: Warum kommen eure Beispiele für das, was eurer Ansicht Wissenschaft ist oder zu sein hat, eigentlich immer aus dem Bereich der Physik?

Das ist eine sehr gute Frage. Zwei widersprüchliche Antworten darauf.

Wenn sich Ontologie per Goethe-Wort damit beschäftigt, was die Welt im Innersten zusammenhält und ausmacht, ist Physik die relevante Wissenschaft. Und nur die Physik, denn welcher Philosoph hat chemische Vorgänge wie die Oxydation von Eisen je als ontologisch bedeutsam eingestuft? Oder geologische Vorgänge wie die Plattentektonik? Und so weiter...

Insofern ist der Fokus auf die Physik nachvollziehbar.


Ontologie ist ein Thema der Philosophie, Physik eine Wissenschaft. Philosophie beschäftigt sich mit dem „Absoluten“, sucht nach der „Wahrheit“; theoretisch-empirische Wissenschaften dagegen (sehr verkürzt) nach Modellen beobachtbarer Zusammenhänge. Ich denke, das sollte man nicht durcheinanderschmeißen.

Das ist die übliche Lesart. Ganz persönlich kann ich nur sagen, daß ich Begriffe wie Ontologie nur im Notfall verwende. Ich denke in Geschichten und Beispielen.

Kurzer Exkurs: der Fleck von Arago.

    Bei Newton hat das Licht Teilchennatur. Später wurde das in Zweifel gezogen, also hat die Französiche Akademie der Wissenschaften einen Wettbewerb zu ausgeschrieben. Fresnel hat eine Wellentheorie des Lichtes eingereicht. Poisson, einer aus der Jury, hat Fresnels Theorie durchdacht und kam zum Ergebnis: "So ein Blödsinn, dann würde ein kreisförmiges Objekt einen Schatten werfen, mit einem Lichtpunkt in der Mitte." Der Leiter der Akademie, Arago, hat den Versuch gemacht: den Lichtpunkt im Schatten gibt es tatsächlich.


Das ist eine nackte Tatsache. Ganz egal in welches ontologische Gewand ich diese Tatsache kleide, die Geschichte und die Tatsache bleibt. Hier ist die Wahrheit zu Hause, der Rest ist strenggenommen Auslegungssache, Interpretation. Das kann fürchterlich in die Hose gehen, ist in der Wissenschaftsgeschichte auch oft genug in die Hose gegangen. Muß aber nicht immer in die Hose gehen.

Gute Modelle haben Vorhersagekraft. Eine Sternstunde der Newtonschen Gravitation war die Entdeckung des Neptuns. Aus den Bahnunregelmäßigkeiten des Uranus konnte die Position des Neptuns vorherberechnet werden. Er wurde dann auch prompt dort gefunden. Der oben erwähnte Arago schrieb damals, Le Verrier habe einen Planeten mit der Spitze seiner Feder entdeckt.

Das ist letztlich der alte Disput zwischen den Falsifikationisten und den Bayesianern. (Dazu habe ich noch was zu berichten, das auf der ersten Seite dieses Threads vorkam.)

Kurzfassung: wie läßt sich etwas über Ontologie sagen, ohne auf die Begriffe der jeweiligen Fachdisziplinen zurückzugreifen?


Marcellinus hat folgendes geschrieben:
smallie hat folgendes geschrieben:

Wie siehst du das? Ist Evolution auch "nur ein Modell"? Oder ist es eine Erfahrungstatsache? So wie Raum, Zeit, Gravitation und Quantenverschränkung Erfahrungstatsachen sind. (Mist, jetzt habe ich bereits die Antwort vorgegeben.)


Evolution ist eine beobachtbare Tatsache, Evolutionstheorie ein Modell dafür. Das eine ist Empirie, das andere Theorie.

Ja und nein aus vielerlei Gründen. (Ich schaff's gerade nicht, die passabel aufzuschreiben.)

Der wichtigste Grund: Kritiker der ET sagen, sie sei nur eine Tautologie. Das teile ich, bis auf das Wörtchen "nur". ET ist logisch zwingend, sobald es Abstammung mit Veränderung und Selektion gibt.

Damit das nicht zu Off-Topic wird: Zeit ist ganz grundlegend für Evolution. Ohne Zeit keine Evolution, und zwar Zeit in geologischen Dimensionen. Stichwort Zeitwahrnehmung als soziales Phänomen: was machen folgende Vorstellungen mit einer Gesellschaft? Beziehungsweise: unter welchen Voraussetzungen kommen Gesellschaften auf diese Vorstellungen?

    - die Welt ist ewig und ewig im Fluß. (LaoTse)
    - die Welt ist ewig und war schon immer so. (Alte Griechen. Ahhm. Hoffentlich.)
    - die Welt ist 6000 Jahre alt. (Bischof von Usher)
    - die Welt ist hunderte von Millionen Jahren alt. (Lyell)
    - die Welt ist fast 4 Milliarden Jahre alt? (Hubble ff.)


fwo hat es bereits angesprochen: die Entwicklung sozialer Zeitwahrnehmungen unterliegt selbst bestimmten Regeln der kulturellen Evolution.


Marcellinus hat folgendes geschrieben:
Wobei die Tücke darin besteht, daß man das Modell der Evolutionstheorie braucht, um Evolution wahrzunehmen.

Das halte ich für eine logische Unmöglichkeit.

Es stimmt zwar, daß bereits vor Darwin über Evolution nachgedacht wurde, von Lamarck, Erasmus Darwin, etc. Ohne solche Vorläufertheorien wäre Darwin wohl tatsächlich nicht auf die ET gekommen.

Andererseits, welche Beobachtungen brauchte Darwin, um auf seine Ideen zu kommen?

Der wichtigste Erkenntnis war vermutlich die Erkenntnis der geologischen Zeiträume. Kurz vor Darwin hatte man erkannt, daß die Welt viel älter als 6000 Jahre sein mußte. Darwin selbst hat versteinerte Muscheln auf Andengipfeln gefunden und beobachtet, wie sie die Küsteenlinie nach einem Erdbeben um eine Meter gehoben hatte.

Hinzu kamen Fossileinfunde ausgestorbener Tierarten und die Artendynamik auf frisch besiedelten Inseln. Schließlich ist er noch über Malthus gestolpert.



Zur Sprache:

Marcellinus hat folgendes geschrieben:
Hier merkt man vor allem die Beschränkungen meiner (unserer?) Sprache. Evolution ist ein Prozeß, unsere Wahrnehmung auch. Unsere Sprache neigt aber dazu, Veränderungen auf Unveränderliches zu reduzieren. Was in diesem Falle (wie vermutlich in vielen anderen Fällen auch) nicht funktioniert.


Und auch das, soweit es sich nicht auf Kramer bezieht:

Marcellinus hat folgendes geschrieben:
Ich denke, daß wir auch hier wieder an die Beschränkungen unserer Sprache kommen. Begriffe sind gedankliche Werkzeuge zur Orientierung in dieser Welt, und wenn sie mit der beobachtbaren Wirklichkeit nicht Schritt halten, führen sie uns in die Irre.

Im Kern gebe ich dir Recht. Mit Begriffen geht oft eine Verabsolutierung der Begriffe einher. Dabei wird vergessen, daß Begriffe auf einer beschränkten Datenbasis basieren.

#155:  Autor: Kramer BeitragVerfasst am: 22.01.2017, 01:49
    —
Marcellinus hat folgendes geschrieben:
Was du mit Zeitgefühl meinst, ist etwas anderes als das, was ich darunter verstehe. Zeitgefühl als die Wahrnehmung kurzer aufeinander folgender Ereignisse, wie zB Töne, scheint mir etwas anderes zu sein als das Zeitgefühl, in dem sich „Lebenswendepunkte“ Geburten, Hochzeiten, Todesfälle, Kirege, Hungersnöte, Wirtschaftskrisen neiderschlagen, aber auch der Wechsel der Jahreszeiten.


Ich verstehe was Du meinst, aber ich bin mir nicht sicher, ob wir für die von Dir genannten Zeiträume wirklich ein "Gefühl" haben. Den Wechsel der Jahreszeiten erfahren wir schon als Kinder, da sehe ich am ehesten eine Chance, dass man dafür im Laufe des Lebens ein Gefühl empfinden kann. Aber Wirtschaftskrisen oder Todesfälle? Ich denke, dass Lebenswendepunkte oft die Eigenschaft haben, dass sie völlig unverhofft eintreten und keinem erkennbaren Rhythmus folgen. Sie bilden ein Muster, das man erst im Nachhinein erkennt, quasi als Bilanz des eigenen Lebens, aber sie ergeben kein erkennbares, wiederkehrendes Muster, wie z.B. der Wechsel der Jahreszeiten.

Zitat:
Ersteres ist vielleicht angeboren, obwohl ich auch da nicht sicher bin. Dafür habe ich einfach schon zu viele Menschen erlebt, deren Musikempfinden vollkommen unterschiedlich ist, ja sich sogar im Laufe eines Lebens verändern kann.


Hier muss man m.E. unterscheiden zwischen dem Musikempfinden/persönlichen Geschmack und dem damit verbundenen Zeitempfinden. Das Zeitempfinden ist objektiver. Wenn der Produzent bei der Aufnahme sagt "Ihr spielt gut, aber der Song gefällt mir nicht" ist das ein subjektives Urteil. Sagt er "Ich weiss nicht, ob der Song sich verkaufen wird, aber euer Timing ist Mist" dann ist das eine nachprüfbare Aussage. Und wenn eine Band, eine Kapelle, ein Tanzorchester die Sache mit der Zeit, das Timing, den Groove nicht drauf hat, dann wird sie auch bei vielen Menschen nicht gut ankommen, selbst wenn diese persönlich auf den Stil der Band stehen.


Zitat:
Bei letzterem bin ich dagegen sicher (und dafür gibt es auch empirische Belege), daß ein solches Zeitgefühl, daß sich in Kalendern und heute in der Allgegenwart der Uhren ausdrückt, Ergebnis von sozialen Prozessen ist, deren Zwänge wie Möglichkeiten sich im individuellen Habitus der einzelnen Menschen „individualisieren“, so sehr, daß sie ihnen als eine „natürliche“ Eigenschaft aller Menschen erscheinen, obwohl sie in dieser Form noch keine 150 Jahre alt sind.


Inwiefern sollten Todesfälle, Geburten, Wirtschaftskrisen, Naturkatastrophen das individuelle Zeitgefühl der Betroffenen so sehr prägen, dass es ihnen als natürliche Eigenschaft aller Menschen erscheint? Es gibt Menschen, die verlieren ihre Eltern während ihrer Kindheit, andere im frühen Erwachsenenalter und wieder andere sterben vor ihren Eltern. Und das erleben Menschen nicht erst seit 150 Jahren.

#156:  Autor: schtonk BeitragVerfasst am: 22.01.2017, 02:29
    —
Kramer hat folgendes geschrieben:
Marcellinus hat folgendes geschrieben:
Was du mit Zeitgefühl meinst, ist etwas anderes als das, was ich darunter verstehe. Zeitgefühl als die Wahrnehmung kurzer aufeinander folgender Ereignisse, wie zB Töne, scheint mir etwas anderes zu sein als das Zeitgefühl, in dem sich „Lebenswendepunkte“ Geburten, Hochzeiten, Todesfälle, Kirege, Hungersnöte, Wirtschaftskrisen neiderschlagen, aber auch der Wechsel der Jahreszeiten.


Ich verstehe was Du meinst, aber ich bin mir nicht sicher, ob wir für die von Dir genannten Zeiträume wirklich ein "Gefühl" haben. Den Wechsel der Jahreszeiten erfahren wir schon als Kinder, da sehe ich am ehesten eine Chance, dass man dafür im Laufe des Lebens ein Gefühl empfinden kann. Aber Wirtschaftskrisen oder Todesfälle? Ich denke, dass Lebenswendepunkte oft die Eigenschaft haben, dass sie völlig unverhofft eintreten und keinem erkennbaren Rhythmus folgen. Sie bilden ein Muster, das man erst im Nachhinein erkennt, quasi als Bilanz des eigenen Lebens, aber sie ergeben kein erkennbares, wiederkehrendes Muster, wie z.B. der Wechsel der Jahreszeiten.


Im Terminus Zeitgefühl steckt alles subjektive Empfinden der Welt. Zeit wird manchmal empfunden, weil man sie nicht nach Einheiten einteilen will oder kann, weil ein Abschnitt von ihr eine spezielle Besonderheit aufweist. Und dies beeinflusst unsere Gefühle. Dieser Sachverhalt wird doch häufig in belletristischen Romanen beschrieben:

Nach dem Tode seiner Frau kamen ihm die Tage endlos lang vor, und die Nächte zogen sich end- und schlaflos quälend dahin.
Im Urlaub schienen die Tage kürzer zu sein als sonst, sie vergingen wie im Fluge.

Na ja, so ungefähr jedenfalls. Die von Marcellinus angesprochenen Gegebenheiten sind besondere Situationen, die uns die Zeit u.U. anders erleben lassen - nach Gefühl. So jedenfalls habe ich ihn verstanden.
Und außerdem kommt es mir momentan so vor, als wollten die kalten Tage gar kein Ende mehr nehmen...


Kramer hat folgendes geschrieben:
Und wenn eine Band, eine Kapelle, ein Tanzorchester die Sache mit der Zeit, das Timing, den Groove nicht drauf hat, dann wird sie auch bei vielen Menschen nicht gut ankommen, selbst wenn diese persönlich auf den Stil der Band stehen.

Ja. It must schwing Smilie

#157:  Autor: sponorWohnort: München BeitragVerfasst am: 23.01.2017, 10:45
    —
zelig hat folgendes geschrieben:
Zitat:
Mit Quantenbits zur Raumzeit
Astrophysiker und Quantenforscher untersuchen gemeinsam, wie kosmische Phänomene mit subatomaren Gesetzen zusammenhängen. Sie versprechen sich neue Einsichten in die grundlegende Struktur von Raum und Zeit.


http://www.spektrum.de/news/die-struktur-von-raum-und-zeit-wird-von-astrophysikern-und-quantenforschern-untersucht/1435484

Hab's noch nicht lesen können, aber der aktuelle Titel vom SdW klingt schon sehr geil:
"Quantenverschränkte Schwarze Löcher"... Let's Rock

Ich würde sagen: Du kannst deine Zivilisation "fortgeschritten" nennen, wenn sie quantenverschränkte Schwarze Löcher im Labor herstellen kann.

#158:  Autor: smallie BeitragVerfasst am: 25.01.2017, 00:07
    —
In eigener Sache:

meine quasi-legasthenische Folter der deutschen Schriftsprache ist euch sicher schon aufgefallen, ich hoffe nicht allzu sinnverstellend.

Jetzt geht das auch mit Zahlen los. Geschockt

smallie hat folgendes geschrieben:
- die Welt ist fast 4 Milliarden Jahre alt? (Hubble ff.)

Das ist das Alter der Erde.

Richtig wäre gewesen: "fast 14 Milliarden Jahre". Das war kein Tippfehler, sondern ein blackout. Seufz.

#159:  Autor: smallie BeitragVerfasst am: 25.01.2017, 00:20
    —
zelig hat folgendes geschrieben:
Zitat:
Mit Quantenbits zur Raumzeit
Astrophysiker und Quantenforscher untersuchen gemeinsam, wie kosmische Phänomene mit subatomaren Gesetzen zusammenhängen. Sie versprechen sich neue Einsichten in die grundlegende Struktur von Raum und Zeit.


http://www.spektrum.de/news/die-struktur-von-raum-und-zeit-wird-von-astrophysikern-und-quantenforschern-untersucht/1435484

Jetzt sind wir wieder beim Ausgangsthema des Threads. AdS-CFT-Dualität usw. Ich war schon damals etwas skeptisch, soweit ich mir das als Laie erlauben kann.

Ganz grundlegend sehe ich zwei oder drei Ungereimtheiten bei der gegenwärtigen Vorstellung von der physikalischen Zeit.


Reversibilität

Viele physikalische Gesetze sind zeitlich umkehrbar, wenn auch die Richtungen umgekehrt werden. Klassisches Beispiel: eine Schachtel mit einer Trennwand, auf der einen Seite heiße Luft, auf der anderen kalte. Entferne ich die Trennwand, dann gleichen sich die Temperaturen aus. Jetzt könnte ich nach dem Temperaturausgleich die Flugrichtungen der Teilchen umkehren, so daß sie in die Richtung fliegen, aus der sie kamen. Hokus Pokus ... Dann entwickelt sich das System zurück in den Zustand, in dem heiß und kalt getrennt war. Das widerspricht aber dem Gedanken, daß die Entropie immer zunimmt.


Nicht-lokale QM und lokale AR

Quantenverschränkung ist nicht-lokal. Eine Überlagerung von Quantenwellen kann aus örtlich getrennten Einzelwellen bestehen. Zumindest sieht es für uns so aus. Allgemeine Relativität ist lokal, die lokale Form der Raumzeit hängt von der Energie in diesem Raum ab. Wie passt das zusammen?


Big Bang und Entropie

Entropie nimmt nie ab. Nach heutigem Verständnis wird der größte Teile der in Masse gebundenen Energie unseres Universums irgendwann zerstrahlen. Sei es dadurch, daß Schwarze Löcher zerstrahlen, sei es aufgrund von Protonenzerfall - falls es ihn gibt. Am Ende steht maximale Entropie. Am Anfang war die Entropie sehr gering, was die Frage aufwirft, aus welchem Zustand noch niedrigerer Entropie unser Universum entstand.


Wenn die neue Sache aus deinem Link hier hilft, dann wären wir einen Schritt weiter. Ich seh' aber nicht, wie sie genau weiterhelfen sollte.

#160:  Autor: smallie BeitragVerfasst am: 25.01.2017, 00:29
    —
sponor hat folgendes geschrieben:
Hab's noch nicht lesen können, aber der aktuelle Titel vom SdW klingt schon sehr geil:
"Quantenverschränkte Schwarze Löcher"... Let's Rock

Eine Aussage fehlt im Artikel - falls mich mein Laienverständnis der Sache nicht trügt: verschränkt bedeutet verschränkt mit einer fernen Zukunft oder gar erst im Unendlichen. Wie der Artikel weiterhelfen sollte, etwas über die nächste Sekunde, über Morgen oder über in tausend Jahren auszusagen, verstehe ich nicht.

Was dort steht ist reine Spekulation. Soll nicht heißen, das sie falsch sein muß, aber eine gesunde Skepsis ist angebracht. Das Thema "Thesen als Waren" wurde bereits angesprochen. Ich persönlich denke, daß es eine neue Beobachtung braucht, um in Sachen Quantengravitation weiterzukommen.

Was Maldacena beschreibt, ist eine These, die sich nicht an einer konkreten Beobachtung aufhängt, die auch keine konkrete Beobachtung erklären kann, weil sie, Zitat Maldacena, "nicht mathematisch formuliert" ist.

Maldacena, SDW hat folgendes geschrieben:
Und obwohl wir noch keine keine mathematischen Werkzeuge dafür haben, gibt es bereits Überlegungen, solche Gebilde könnten Ursache der Raumzeit selbst sein.


Auf Deutsch gesagt: Maldacena und ER = EPR ist eine intuitive Bauchgefühl-These. Damit sie mich überzeugt, müßt sie eine konkrete Geschichte erzählen können - und das geht nicht, wenn es weder einen experimentellen Beleg gibt, noch eine Mathematik, die einen Beleg vorhersagen könnte.

#161:  Autor: smallie BeitragVerfasst am: 28.01.2017, 22:11
    —
smallie hat folgendes geschrieben:
Das ist letztlich der alte Disput zwischen den Falsifikationisten und den Bayesianern. (Dazu habe ich noch was zu berichten, das auf der ersten Seite dieses Threads vorkam.)

Damals ist mir ein Experiment eingefallen. Das Ergebnis:

smallie 0 : Wirklichkeit 1



smallie hat folgendes geschrieben:
Das EPR-Papier habe ich inzwischen gelesen und auch schon ein bisschen was dazu und zu Bell aufgeschrieben.

Nun ist mir gerade aufgefallen, daß ich doof bin.

Wenn ich die Sache richtig verstehe, dann sollte folgendes möglich sein:

    - geh' ins 3D-Kino.
    - platziere zwei Brillen hintereinander, und zwar zwei gegenseitig polarisierende Scheiben. Ergebnis: es geht kein Licht durch.
    - platziere drei Brillen hintereinander, so daß die mittlere um 45° zwischen den zwei entgegengesetzten steht. Ergebnis: es geht ein bisschen Licht durch.

Kommt das hin?


Worauf step meinte:

step hat folgendes geschrieben:
Ja, das ist korrekt.

Ohne Quantenphysik kann man sich das so veranschaulichen: Der Linear-Polfilter läßt nur eine Ebene durch. Er "betrachtet" dazu jede Lichtwelle als zusammengesetzt aus einer Komponente parallel und senkrecht zu seiner P-Richtung - was aufgrund der vektoriellen Natur des EM-Feldes möglich ist.

Mittlerweile habe ich das ausprobiert. Es geht nicht. Es geht noch nicht mal, daß bei zwei entgegengesetzten Filtern kein Licht mehr durchkommt.

Oh weh.

Nach diesem Negativresultat müßte ich zwingend zum Schluß kommen: Quantenphysik ist Mumpitz. Wissenschaftliche Methode, Falsifizierung, und so weiter. Tu' ich aber nicht. Stattdessen erfinde ich dreist eine Ausrede und behaupte: die Güte der Polfilter von 3D-Kino-Brillen reicht nicht aus, um den Effekt hervorzubringen. Dummerweise reicht die Güte der Polfilter, um einen 3D-Effekt zu erzeugen. Dazu fällt mir keine Ausrede mehr ein. Euch?

#162:  Autor: Kramer BeitragVerfasst am: 29.01.2017, 01:58
    —
smallie hat folgendes geschrieben:
Dummerweise reicht die Güte der Polfilter, um einen 3D-Effekt zu erzeugen. Dazu fällt mir keine Ausrede mehr ein. Euch?


Ich gehe öfter mal mit einem Freund ins Kino, der ein wahrer Film- und Film-Technik-Freak ist. Wenn ich mit ihm 3D-Filme schaue, leiht er mir eine seiner teuren 3D-Brillen von Panasonic (oder so). Bei anderen Gelegenheiten benutze ich halt die handelsüblichen Pappbrillen. Und auch wenn der 3D-Effekt mit beiden Brillen funktioniert, sehe ich mit den Pappbrillen mehr Artefakte und Doppelbilder, manche 3D-Effekte wirken auch weniger plastisch, als mit den teureren Brillen.

#163:  Autor: sponorWohnort: München BeitragVerfasst am: 30.01.2017, 13:30
    —
Kramer hat folgendes geschrieben:
smallie hat folgendes geschrieben:
Dummerweise reicht die Güte der Polfilter, um einen 3D-Effekt zu erzeugen. Dazu fällt mir keine Ausrede mehr ein. Euch?


Ich gehe öfter mal mit einem Freund ins Kino, der ein wahrer Film- und Film-Technik-Freak ist. Wenn ich mit ihm 3D-Filme schaue, leiht er mir eine seiner teuren 3D-Brillen von Panasonic (oder so). Bei anderen Gelegenheiten benutze ich halt die handelsüblichen Pappbrillen. Und auch wenn der 3D-Effekt mit beiden Brillen funktioniert, sehe ich mit den Pappbrillen mehr Artefakte und Doppelbilder, manche 3D-Effekte wirken auch weniger plastisch, als mit den teureren Brillen.

Das sind oft (meist?) keine einfachen, linear-polarisierenden Filter mehr, sondern es wird zirkular-polarisiertes Licht benutzt, oder gleich ganz was anderes mit Farben.

#164:  Autor: zelig BeitragVerfasst am: 17.10.2017, 17:55
    —
In einem Beitrag über das Quantenuniversum in der Spektrum habe ich gelesen, daß "alle möglichen Ausgänge eines Experiments in verschiedenen Zweigen der Realität verwirklicht werden." Und es heißt, "In dieser Vorstellung existieren zahlreiche Universen, in denen jeweils eigene zahlreiche Gesetze gelten."

Ich glaube das hilft mir eine Frage zu einem alten Unverständnis zu formulieren. Wenn einerseits nicht postuliert wird, daß wahllos einfach alles existiert, und andererseits viele Physiken existieren. Wie kann man - auch nur theoretisch - das Mögliche vom Unmöglichen unterscheiden? Wie definieren Theoretiker die Trennlinie?

#165:  Autor: stepWohnort: Germering BeitragVerfasst am: 17.10.2017, 18:37
    —
zelig hat folgendes geschrieben:
In einem Beitrag über das Quantenuniversum in der Spektrum habe ich gelesen, daß "alle möglichen Ausgänge eines Experiments in verschiedenen Zweigen der Realität verwirklicht werden." Und es heißt, "In dieser Vorstellung existieren zahlreiche Universen, in denen jeweils eigene zahlreiche Gesetze gelten."

Ich glaube das hilft mir eine Frage zu einem alten Unverständnis zu formulieren. Wenn einerseits nicht postuliert wird, daß wahllos einfach alles existiert, und andererseits viele Physiken existieren. Wie kann man - auch nur theoretisch - das Mögliche vom Unmöglichen unterscheiden? Wie definieren Theoretiker die Trennlinie?

Das geht grob gesagt so:

Man verallgemeinert seine Theorien (z.B. QFT) so weit, daß sie alle bekannten Theorien als Grenzfälle enthalten und möglichst einfach aussehen. Z.B. kommt man dann auf eine Gruppe von Stringtheorien, Lie-Gruppen usw. Die enthalten aber immer noch Konstanten, die sich nicht zwangsweise ergeben oder sogar ganz spezielle, "unwahrscheinliche" Werte annehmen müssen, um die heimische "Realität" abzubilden - z.B. Kopplungskonstanten.

Jetzt kommt der Trick: Man definiert ein weiteres noch allgemeineres Quantenfeld (oder Gruppe), das diese Konstante im Rahmen einer spontanen Symmetriebrechung produziert. Das ist keineswegs ein Taschenspielertrick, sondern in der QFT fundamental angelegt und in anderen QFT-Kontexten auch schon beobachtet worden. Aus Sicht dieser übergeordneten Theorie ist der Wert der Konstanten jetzt nur noch eine Art Stichprobe oder einer von vielen/allen möglichen Erwartungswerten.

Diese Schlüsse sind übrigens nicht so willkürlich, wie sie auf den ersten Blick scheinen. Man könnte sagen, sie werden durch die Quantenphysik erzwungen. Wenn man das nicht immer weiter treibt, hat man u.a. das Problem, daß die Quantenphysik nicht stimmen kann, daß man die Feinabstimmung nicht erklären kann (also nicht mangels Wissen, sondern prinzipiell nicht), usw.

Obiges ist extrem vereinfacht und daher auch nicht ganz korrekt, aber es erklärt das Prinzip.

Jetzt zu Deiner Frage: Diese Trennlinie gibt es in der modernen Physik eigentlich gar nicht mehr. In der Praxis liegt sie immer bei der gerade konkretesten TOE, die keine oder möglichst wenige hidden variables besitzt. Es gibt aber schon Physiker, die vermuten, daß tatsächlich alles "realisiert" (*) ist, was mathematisch widerspruchsfrei ist.

Liest sich jetzt alles sehr metaphysisch, zugegeben. Für mich zählt aber nur, ob man damit weiterreichende und bessere Vorraussagen bekommt als mit spezifischeren Teiltheorien, die z.B. in der Nähe des Urknalls oder im CERN versagen.


(*) "realisiert" oder "real" im Sinne des Realismus macht hier natürlich kaum noch Sinn.

#166:  Autor: zelig BeitragVerfasst am: 17.10.2017, 19:24
    —
step hat folgendes geschrieben:
zelig hat folgendes geschrieben:
In einem Beitrag über das Quantenuniversum in der Spektrum habe ich gelesen, daß "alle möglichen Ausgänge eines Experiments in verschiedenen Zweigen der Realität verwirklicht werden." Und es heißt, "In dieser Vorstellung existieren zahlreiche Universen, in denen jeweils eigene zahlreiche Gesetze gelten."

Ich glaube das hilft mir eine Frage zu einem alten Unverständnis zu formulieren. Wenn einerseits nicht postuliert wird, daß wahllos einfach alles existiert, und andererseits viele Physiken existieren. Wie kann man - auch nur theoretisch - das Mögliche vom Unmöglichen unterscheiden? Wie definieren Theoretiker die Trennlinie?

Das geht grob gesagt so:

Man verallgemeinert seine Theorien (z.B. QFT) so weit, daß sie alle bekannten Theorien als Grenzfälle enthalten und möglichst einfach aussehen. Z.B. kommt man dann auf eine Gruppe von Stringtheorien, Lie-Gruppen usw. Die enthalten aber immer noch Konstanten, die sich nicht zwangsweise ergeben oder sogar ganz spezielle, "unwahrscheinliche" Werte annehmen müssen, um die heimische "Realität" abzubilden - z.B. Kopplungskonstanten.

Jetzt kommt der Trick: Man definiert ein weiteres noch allgemeineres Quantenfeld (oder Gruppe), das diese Konstante im Rahmen einer spontanen Symmetriebrechung produziert. Das ist keineswegs ein Taschenspielertrick, sondern in der QFT fundamental angelegt und in anderen QFT-Kontexten auch schon beobachtet worden. Aus Sicht dieser übergeordneten Theorie ist der Wert der Konstanten jetzt nur noch eine Art Stichprobe oder einer von vielen/allen möglichen Erwartungswerten.

Diese Schlüsse sind übrigens nicht so willkürlich, wie sie auf den ersten Blick scheinen. Man könnte sagen, sie werden durch die Quantenphysik erzwungen. Wenn man das nicht immer weiter treibt, hat man u.a. das Problem, daß die Quantenphysik nicht stimmen kann, daß man die Feinabstimmung nicht erklären kann (also nicht mangels Wissen, sondern prinzipiell nicht), usw.

Obiges ist extrem vereinfacht und daher auch nicht ganz korrekt, aber es erklärt das Prinzip.

Jetzt zu Deiner Frage: Diese Trennlinie gibt es in der modernen Physik eigentlich gar nicht mehr. In der Praxis liegt sie immer bei der gerade konkretesten TOE, die keine oder möglichst wenige hidden variables besitzt. Es gibt aber schon Physiker, die vermuten, daß tatsächlich alles "realisiert" (*) ist, was mathematisch widerspruchsfrei ist.

Liest sich jetzt alles sehr metaphysisch, zugegeben. Für mich zählt aber nur, ob man damit weiterreichende und bessere Vorraussagen bekommt als mit spezifischeren Teiltheorien, die z.B. in der Nähe des Urknalls oder im CERN versagen.


(*) "realisiert" oder "real" im Sinne des Realismus macht hier natürlich kaum noch Sinn.


Vielen Dank! Sehr interessant.

#167:  Autor: Marcellinus BeitragVerfasst am: 17.10.2017, 19:32
    —
step hat folgendes geschrieben:

(*) "realisiert" oder "real" im Sinne des Realismus macht hier natürlich kaum noch Sinn.


Und mit Naturwissenschaft hat das auch nichts mehr zu tun.

#168:  Autor: stepWohnort: Germering BeitragVerfasst am: 17.10.2017, 20:40
    —
Marcellinus hat folgendes geschrieben:
Und mit Naturwissenschaft hat das auch nichts mehr zu tun.

Ja und nein, würde ich sagen. Zumindest die Deutung "alles Mathematische ..." würde auch ich als im strengeren Sinne unwissenschaftlich ansehen. U.a. weil es schwer vorstellbar ist, woher da die Falsifizierbarkeit kommen soll.

Den Teil mit der Symmetriebrechung und den Gruppen dagegen würde ich als Wissenschaft durchgehen lassen, denn so etwas kann man - zumindest prinzipiell - nachprüfen, etwa indem man Zustände herstellt und beobachtet, in denen die Symmetrie noch nicht gebrochen ist.

Ich denke zudem, daß der klassische Realitätsbegriff schon viel früher aufgegeben werden muß, spätestens mit der Quantenphysik.

#169:  Autor: Marcellinus BeitragVerfasst am: 17.10.2017, 21:12
    —
step hat folgendes geschrieben:
Marcellinus hat folgendes geschrieben:
Und mit Naturwissenschaft hat das auch nichts mehr zu tun.

Ja und nein, würde ich sagen. Zumindest die Deutung "alles Mathematische ..." würde auch ich als im strengeren Sinne unwissenschaftlich ansehen. U.a. weil es schwer vorstellbar ist, woher da die Falsifizierbarkeit kommen soll.

Den Teil mit der Symmetriebrechung und den Gruppen dagegen würde ich als Wissenschaft durchgehen lassen, denn so etwas kann man - zumindest prinzipiell - nachprüfen, etwa indem man Zustände herstellt und beobachtet, in denen die Symmetrie noch nicht gebrochen ist.

Ich denke zudem, daß der klassische Realitätsbegriff schon viel früher aufgegeben werden muß, spätestens mit der Quantenphysik.


Der "klassische Realitätsbegriff" ist letztlich ein philosophischer. Theoretisch-empirische Wissenschaften sind der organisierte Versuch, nachprüfbare Modelle zu erstellen, die sich durch Tatsachenbeobachtungen belegen lassen. Wo die Modelle sich so weit von den beobachtbaren Tatsachen entfernen, daß sie sich nach unserem heutigen Kenntnisstand prinzipiell nicht mehr überprüfen lassen, hat man sich wohl von den Wissenschaften entfernt, und betreibt eher mathematisch-physikalische Spekulation. Aber wenn sich jemand findet, der das finanziert, warum nicht? zwinkern

#170:  Autor: smallie BeitragVerfasst am: 17.10.2017, 22:44
    —
zelig hat folgendes geschrieben:
In einem Beitrag über das Quantenuniversum in der Spektrum habe ich gelesen, daß "alle möglichen Ausgänge eines Experiments in verschiedenen Zweigen der Realität verwirklicht werden." Und es heißt, "In dieser Vorstellung existieren zahlreiche Universen, in denen jeweils eigene zahlreiche Gesetze gelten."

Ich glaube das hilft mir eine Frage zu einem alten Unverständnis zu formulieren. Wenn einerseits nicht postuliert wird, daß wahllos einfach alles existiert, und andererseits viele Physiken existieren. Wie kann man - auch nur theoretisch - das Mögliche vom Unmöglichen unterscheiden? Wie definieren Theoretiker die Trennlinie?

Laß' mich eine Analogie aus der Welt der Musik bauen. Kennst du schon meine vereinheitlichte String-, Blas- und Schlagtheorie? Mr. Green

Die Elementarpartikel der Musik sind Schwingungen. Alle Musik läßt sich als Summe von Frequenzen f darstellen, die sich mit Zeit t ändern. Damit kann ich alle Musik beschreiben, die je existiert hat und je existieren wird. Beethoven, Chuck Berry und Chick Corea sind nur individuelle Ausprägungen meiner Theorie. Cool, nicht?

Nur sind wir jetzt kein bisschen klüger, warum Beethoven nicht wie Chuck Berry klingt. skeptisch



Falls das zu spinnert war, habe ich noch ein historisches Beispiel.

Die Titus-Bode-Regel ist eine Formel aus der Astronomie, mit der sich die Planetenabstände im Sonnensystem vorhersagen lassen. Sie stimmt bis auf etwa 5%. Wenn man nur unser Sonnensystem kennt, liegt es nahe, die Regel als allgemeines Gesetz zu behaupten, das in anderen Systemen (oder Universen) ebenso gilt, eventuell mit anderen Koeffizienten.

Ohne andere Systeme beobachtet zu haben, bleibt die Regel Spekulation. Sie ist übrigens falsch.

#171:  Autor: zelig BeitragVerfasst am: 18.10.2017, 07:49
    —
smallie hat folgendes geschrieben:
zelig hat folgendes geschrieben:
In einem Beitrag über das Quantenuniversum in der Spektrum habe ich gelesen, daß "alle möglichen Ausgänge eines Experiments in verschiedenen Zweigen der Realität verwirklicht werden." Und es heißt, "In dieser Vorstellung existieren zahlreiche Universen, in denen jeweils eigene zahlreiche Gesetze gelten."

Ich glaube das hilft mir eine Frage zu einem alten Unverständnis zu formulieren. Wenn einerseits nicht postuliert wird, daß wahllos einfach alles existiert, und andererseits viele Physiken existieren. Wie kann man - auch nur theoretisch - das Mögliche vom Unmöglichen unterscheiden? Wie definieren Theoretiker die Trennlinie?

Laß' mich eine Analogie aus der Welt der Musik bauen. Kennst du schon meine vereinheitlichte String-, Blas- und Schlagtheorie? :mrgreen:

Die Elementarpartikel der Musik sind Schwingungen. Alle Musik läßt sich als Summe von Frequenzen f darstellen, die sich mit Zeit t ändern. Damit kann ich alle Musik beschreiben, die je existiert hat und je existieren wird. Beethoven, Chuck Berry und Chick Corea sind nur individuelle Ausprägungen meiner Theorie. Cool, nicht?

Nur sind wir jetzt kein bisschen klüger, warum Beethoven nicht wie Chuck Berry klingt. :?



Falls das zu spinnert war, habe ich noch ein historisches Beispiel.


Das ist aus meiner laienhaften Sicht überhaupt nicht spinnert, weil man daran auch gleich einen skeptischen Einwand verdeutlichen kann. Im geschilderten Multiversum müsste man die meisten Universen wohl als Kakophonien bezeichnen. Das Beethoven-Universum und das von Chick Corea wären seltene Ausnahmen. Deine Beispiele wären Schaum auf dem Meer unfassbar bizarrer musikalischer Ausformungen, die die Intuition für natürliche Entitäten sprengen würde. Und zu denen, um deine Metapher zu verlassen, gehören Lebewesen, also auch Menschen.
Ist das nicht trotz der Faszination ein gewichtiger Einwand?

#172:  Autor: stepWohnort: Germering BeitragVerfasst am: 18.10.2017, 13:25
    —
zelig an smallie hat folgendes geschrieben:
Deine Beispiele wären Schaum auf dem Meer unfassbar bizarrer musikalischer Ausformungen, die die Intuition für natürliche Entitäten sprengen würde. Und zu denen, um deine Metapher zu verlassen, gehören Lebewesen, also auch Menschen.
Ist das nicht trotz der Faszination ein gewichtiger Einwand?

Wieso bzw. wogegen genau soll das ein Einwand sein? Außer evtl. gegen smallies Analogie zwinkern

smallie hat folgendes geschrieben:
Nur sind wir jetzt kein bisschen klüger, warum Beethoven nicht wie Chuck Berry klingt.

Wieso nicht? Wir können sogar viel besser sagen, ob ein Unterschied and den akustischen Parametern liegt oder z.B. an der kulturellen Vorprägung des Rezipienten oder dem dazu gezeigten Bauchtanz. Bei den letzten 2 Effekten hinkt natürlich die Analogie, weil wesentlich ein weiteres komplexes System ins Spiel kommt (der Rezipient), was meiner Ansicht nach bei der QFT nicht der Fall ist - klar wird die auch von Menschen gemacht, erhält aber durch Experiment und Mathematik eine hohe Operationalisierbarkeit und übersubjektive Unabhängigkeit.

Auch ein intelligenter Alien mit völlig anderer Biologie und Kultur würde die QFT und Lie-Gruppe verstehen und auf das Universum abbilden können. Bei Beethoven und Chuck Berry wäre eine Verständigung schwieriger, evtl. aber bei deren rezeptionsunabhängigen Anteilen (Frequenzverhältnisse, Rhythmus u.ä.) möglich.

#173:  Autor: zelig BeitragVerfasst am: 18.10.2017, 13:55
    —
step hat folgendes geschrieben:
zelig an smallie hat folgendes geschrieben:
Deine Beispiele wären Schaum auf dem Meer unfassbar bizarrer musikalischer Ausformungen, die die Intuition für natürliche Entitäten sprengen würde. Und zu denen, um deine Metapher zu verlassen, gehören Lebewesen, also auch Menschen.
Ist das nicht trotz der Faszination ein gewichtiger Einwand?

Wieso bzw. wogegen genau soll das ein Einwand sein? Außer evtl. gegen smallies Analogie ;-)


Der Einwand gilt für alle Relationen menschlicher Natur. Um zwei zu nennen: Wir gerieten in massive Erklärungsnot, wenn Duplikate von uns ohne Gehirn oder Bewußtsein das gleiche Verhalten aufwiesen - vorausgesetzt, die Existenz des entsprechenden Universums wäre mathematisch erlaubt. Das betrifft das Selbstverständnis. Ein zweiter wäre die Natur menschlicher Erkenntnis selber. Der Einwand ist zugegebenermaßen eher irrelevant für das theoretische Konstrukt. Nur darf man das sich das Multiversum auch nicht als eine simple Vervielfachung der Realität darstellen. Letzteres ist jetzt weniger an Dich gerichtet.

#174:  Autor: stepWohnort: Germering BeitragVerfasst am: 18.10.2017, 15:42
    —
zelig hat folgendes geschrieben:
Wir gerieten in massive Erklärungsnot, wenn Duplikate von uns ohne Gehirn oder Bewußtsein das gleiche Verhalten aufwiesen - vorausgesetzt, die Existenz des entsprechenden Universums wäre mathematisch erlaubt. Das betrifft das Selbstverständnis.

Was uns daran auf den ersten Blick schockiert, wäre ja wohl die Kombination aus "komplexes Verhalten" und "kein Gehirn oder Vergleichbares" - Komplexität hat aber viel mit Logik zu tun, und es könnte gut sein, daß eine solche Kombi in keinem Universum möglich ist. Falls aber doch - wieso sollte es (prinzipiell) unerklärbar sein?

Und ja, mit den Identitäten haben wir natürlich ein Problem, aber das ist mE auch kein Einwand, sondern ein intuitives Akzeptanzproblem, ähnlich wie die Entdeckung der Evolution Akzeptanzprobleme bei vielen Gläubigen zur Folge hatte.

zelig hat folgendes geschrieben:
Ein zweiter wäre die Natur menschlicher Erkenntnis selber. Der Einwand ist zugegebenermaßen eher irrelevant für das theoretische Konstrukt. Nur darf man das sich das Multiversum auch nicht als eine simple Vervielfachung der Realität darstellen. Letzteres ist jetzt weniger an Dich gerichtet.

Da stimme ich zu, eher würde passen, daß die klassische Realität eine Art zugänglicher Teil der multiversalen Realität ist.

#175:  Autor: zelig BeitragVerfasst am: 18.10.2017, 15:47
    —
step hat folgendes geschrieben:
zelig hat folgendes geschrieben:
Wir gerieten in massive Erklärungsnot, wenn Duplikate von uns ohne Gehirn oder Bewußtsein das gleiche Verhalten aufwiesen - vorausgesetzt, die Existenz des entsprechenden Universums wäre mathematisch erlaubt. Das betrifft das Selbstverständnis.

Was uns daran auf den ersten Blick schockiert, wäre ja wohl die Kombination aus "komplexes Verhalten" und "kein Gehirn oder Vergleichbares" - Komplexität hat aber viel mit Logik zu tun, und es könnte gut sein, daß eine solche Kombi in keinem Universum möglich ist. Falls aber doch - wieso sollte es (prinzipiell) unerklärbar sein?

Und ja, mit den Identitäten haben wir natürlich ein Problem, aber das ist mE auch kein Einwand, sondern ein intuitives Akzeptanzproblem, ähnlich wie die Entdeckung der Evolution Akzeptanzprobleme bei vielen Gläubigen zur Folge hatte.

zelig hat folgendes geschrieben:
Ein zweiter wäre die Natur menschlicher Erkenntnis selber. Der Einwand ist zugegebenermaßen eher irrelevant für das theoretische Konstrukt. Nur darf man das sich das Multiversum auch nicht als eine simple Vervielfachung der Realität darstellen. Letzteres ist jetzt weniger an Dich gerichtet.

Da stimme ich zu, eher würde passen, daß die klassische Realität eine Art zugänglicher Teil der multiversalen Realität ist.


Das da sollte sich auf meine beiden Einwände beziehen. Habe ich schlecht formuliert.
zelig hat folgendes geschrieben:
Der Einwand ist zugegebenermaßen eher irrelevant für das theoretische Konstrukt.

#176:  Autor: johWohnort: Kevelaer BeitragVerfasst am: 18.10.2017, 18:07
    —
Es gibt Wissenschaftler, die der Meinung sind,



The multiverse isn't science any more. It's mathematical fantasy. And it's utterly fruitless "research".
Übersetzt aus dem Englisch von Bing

Das Multiversum ist keine Wissenschaft mehr. Es ist mathematische Fantasie. Und es ist völlig fruchtlos "Forschung".

#177:  Autor: Marcellinus BeitragVerfasst am: 18.10.2017, 18:54
    —
joh hat folgendes geschrieben:
Es gibt Wissenschaftler, die der Meinung sind,



The multiverse isn't science any more. It's mathematical fantasy. And it's utterly fruitless "research".
Übersetzt aus dem Englisch von Bing

Das Multiversum ist keine Wissenschaft mehr. Es ist mathematische Fantasie. Und es ist völlig fruchtlos "Forschung".


Meine Rede! zwinkern

#178:  Autor: stepWohnort: Germering BeitragVerfasst am: 18.10.2017, 19:22
    —
Marcellinus hat folgendes geschrieben:
joh hat folgendes geschrieben:
... Übersetzt aus dem Englisch von Bing
Meine Rede! zwinkern

Ach Du bist dieser Bing.

Im Ernst, das quantenmechanische Meßproblem ist nun mal nicht wegzudiskutieren, und es ist ein theoretisch extrem interessantes Problem. Man muß es konsistent auflösen oder wenigstens konsistent deuten, und wer - wie viele Physiker und ich - keine magischen Zustandskollapse mag, dem bleibt nur die multiversale Deutung.

Grundlagenforschung ist nicht primär dazu da, mehr Bratkartoffeln zu ernten.

#179:  Autor: smallie BeitragVerfasst am: 18.10.2017, 19:46
    —
step hat folgendes geschrieben:
smallie hat folgendes geschrieben:
Nur sind wir jetzt kein bisschen klüger, warum Beethoven nicht wie Chuck Berry klingt.

Wieso nicht? Wir können sogar viel besser sagen, ob ein Unterschied and den akustischen Parametern liegt oder z.B. an der kulturellen Vorprägung des Rezipienten oder dem dazu gezeigten Bauchtanz.

Wieso nicht?

Gemäß der Analogie sitzen wir in einem Universum, das nur Beethoven kennt. (Falls es nicht auf Modern Talking hinausläuft. Bitte nicht! ) Nie haben wir etwas anderes gehört, und jetzt wollen wir von Beethoven auf andere Musik schließen.

Alles was wir über andere Musik in dieser Situation sagen können ist: sie muß ein zeitlich veränderliches Gebilde aus Frequenzen sein. Sicher: es könnte andere Musiken und andere Universen geben, aber die Behauptung anderer Universen erklärt nichts über die Musik, die in unserem Universum gespielt wird.

Das ist der Trugschluß im SdW-Artikel: er tut so, als würde er etwas erklären. Doch die Erklärung beschränkt sich auf: zufällig ist es bei uns so, wie es halt ist.


step hat folgendes geschrieben:
Bei den letzten 2 Effekten hinkt natürlich die Analogie, weil wesentlich ein weiteres komplexes System ins Spiel kommt (der Rezipient), was meiner Ansicht nach bei der QFT nicht der Fall ist - klar wird die auch von Menschen gemacht, erhält aber durch Experiment und Mathematik eine hohe Operationalisierbarkeit und übersubjektive Unabhängigkeit.

Die Titus-Bode-Regel ist auch übersubjektiv operationalisierbar, wenn man nur unser Sonnensystem kennt.

Die bestätigte QFT reicht bis zum Standardmodell. Danach beginnt das Reich der mehr oder weniger wilden Hypothesen. Wenn ich populäre Darstellungen zum Thema nur oberflächlich lese, entsteht bei mir oft der Eindruck, die Spekulationen seien bereits ausgemachte Wahrheiten - "a g'mahde Wies". Auf der Umschlagseite der SdW-Ausgabe heißt es: Was verrät die Quantenmechanik über kosmische Parallelwelten? Die korrekte Antwort lautet: nichts. Das steht aber bestenfalls im Kleingedruckten oder zwischen den Zeilen. Nichts gegen Spekulation, aber dann bitte in einer eigenen Rubrik.

#180:  Autor: Marcellinus BeitragVerfasst am: 18.10.2017, 19:57
    —
step hat folgendes geschrieben:
Marcellinus hat folgendes geschrieben:
joh hat folgendes geschrieben:
... Übersetzt aus dem Englisch von Bing
Meine Rede! zwinkern

Ach Du bist dieser Bing.

Im Ernst, das quantenmechanische Meßproblem ist nun mal nicht wegzudiskutieren, und es ist ein theoretisch extrem interessantes Problem. Man muß es konsistent auflösen oder wenigstens konsistent deuten, und wer - wie viele Physiker und ich - keine magischen Zustandskollapse mag, dem bleibt nur die multiversale Deutung.

Grundlagenforschung ist nicht primär dazu da, mehr Bratkartoffeln zu ernten.


Ich bin kein Fachmann, aber etwas zu behaupten, weil dann die eigenen Formeln so schön aufgehen, ohne eine Spur von Empirie, hat mit Wissenschaft nicht mehr viel zu tun. Man könnte ja auch ehrlicherweise einmal zugeben, daß man keine Antwort hat. Aber dann gibt's keine Forschungsmittel mehr, und das wäre schön doof, nicht wahr?

#181:  Autor: stepWohnort: Germering BeitragVerfasst am: 18.10.2017, 20:06
    —
smallie hat folgendes geschrieben:
Das ist der Trugschluß im SdW-Artikel: er tut so, als würde er etwas erklären. Doch die Erklärung beschränkt sich auf: zufällig ist es bei uns so, wie es halt ist.

Weiß nicht, wie es in dem Artikel ist, aber in der Methode, die ich oben beschrieben habe, wird man durch den Symmetriebrechungsansatz ja gerade den Zufall los, man formuliert eine Mechanismus, der (auf dieser Ebene) keinen Zufall mehr beinhaltet.

smallie hat folgendes geschrieben:
Die Titus-Bode-Regel ist auch übersubjektiv operationalisierbar, wenn man nur unser Sonnensystem kennt.

Richtig. Ist ja auch keine hinreichende Bedingung, nur eine notwendige zwinkern Häufig wird das Ende einer Theorie dadurch eingeläutet, daß sie irgendwo nicht mehr funktioniert. Bei Titus-Bode kommt noch hinzu, daß es nicht wirklich eine Theorie ist, sondern nur eine Rechenregel, oder?

smallie hat folgendes geschrieben:
Die bestätigte QFT reicht bis zum Standardmodell. Danach beginnt das Reich der mehr oder weniger wilden Hypothesen.

Ja, was aber nicht heißt, daß die alle wertlos sind. Einige sind sogar prinzipiell überprüfbar.

smallie hat folgendes geschrieben:
Wenn ich populäre Darstellungen zum Thema nur oberflächlich lese, entsteht bei mir oft der Eindruck, die Spekulationen seien bereits ausgemachte Wahrheiten - "a g'mahde Wies".

Ja mei. Der Lesch gleitet ja sogar häufig in die Theologie ab. Die meisten Physiker sind da aber realistischer.

smallie hat folgendes geschrieben:
Auf der Umschlagseite der SdW-Ausgabe heißt es: Was verrät die Quantenmechanik über kosmische Parallelwelten? Die korrekte Antwort lautet: nichts.

Einspruch. Sie verrät etwas über verschiedene Typen von Parallelwelten, wie sie entstehen und auf welcher Ebene sie sich von unserer Umgebung unterscheiden. Zumindest unsere beste Theorie darüber.

smallie hat folgendes geschrieben:
Wir und die Aliens, die nur die Lie-Gruppe unseres Universums sehen sind wie Marsianer und Erdenmenschen, die nur die Titus-Bode-Regel unseres Sonnensystems sehen. Gemeinsam können wir uns wundern, ob die Regelmäßigkeiten Zufall sind, oder Ergebnis eines zugrundeliegenden Prinzipes.

Nein, da (fett) kann ich nicht zustimmen. Wir finden (potenziell) ein gemeinsames grundlegendes Prinzip, dieselbe übergeordnete Gruppe sozusagen, und begreifen die Symmetriebrechung.

#182:  Autor: stepWohnort: Germering BeitragVerfasst am: 18.10.2017, 20:17
    —
Marcellinus hat folgendes geschrieben:
Ich bin kein Fachmann, aber etwas zu behaupten, weil dann die eigenen Formeln so schön aufgehen, ohne eine Spur von Empirie, hat mit Wissenschaft nicht mehr viel zu tun.

Strohmann! Ich habe immer gesagt, daß Theorien empirisch überprüft werden müssen. Z.B. kann man die Vereinheitlichungen (also die Umkehrung der Symmetriebrechung sozusagen) an großen Beschleunigern untersuchen.

Marcellinus hat folgendes geschrieben:
Man könnte ja auch ehrlicherweise einmal zugeben, daß man keine Antwort hat. Aber dann gibt's keine Forschungsmittel mehr, und das wäre schön doof, nicht wahr?

Äh ... nee. Forschungsmittel gibt es nicht mehr, WENN man eine Antwort hat. Dann muß man sich eine neue Frage suchen. Übrigens gehören Grundlagenforscher wahrscheinlich zu den Leuten, die am häufigsten schriftlich zugeben, daß sie etwas nicht wissen, und die am meisten Fragen stellen.

In die Grundlagenforschung, Großgeräte usw. gehen übrigens nicht mal 10% der öffentlichen Forschungsgelder in D, das meiste wird für Bratkartoffelforschung ausgegeben.

#183:  Autor: johWohnort: Kevelaer BeitragVerfasst am: 18.10.2017, 20:35
    —
Mathematik ist nur ein perfektes Werkzeug für die Theorie, in der sie verwendet wird.

Aber unterschiedliche Theorien mit unterschiedlichen Definitionen führen zu unterschiedlichen Ergebnissen.

Die Mathematik hat den Vorgaben durch die Definitionen zu folgen.

Wenn eine Theorie bereits auf falsche Annahmen beruht, dann ist die Mathematik immer noch perfekt.

Aus der Mathematik Erkenntnisse ableiten ist zweifelhaft. Man kommt dann eher mit Logik weiter.

#184:  Autor: Marcellinus BeitragVerfasst am: 18.10.2017, 21:26
    —
step hat folgendes geschrieben:
Marcellinus hat folgendes geschrieben:
Ich bin kein Fachmann, aber etwas zu behaupten, weil dann die eigenen Formeln so schön aufgehen, ohne eine Spur von Empirie, hat mit Wissenschaft nicht mehr viel zu tun.

Strohmann! Ich habe immer gesagt, daß Theorien empirisch überprüft werden müssen. Z.B. kann man die Vereinheitlichungen (also die Umkehrung der Symmetriebrechung sozusagen) an großen Beschleunigern untersuchen.


Nein, als Strohmann war das nicht gemeint (ich habe einfach keinen Spaß an der hier üblichen "Konfrontationsdebatte"). Ich schreibe nur gelegentlich, was mir so einfällt, hier zB. daß Theorien nicht nur empirisch überprüft werden müssen. Wenn es richtig ist, daß Theorien eigentlich nichts anderes sind als Modelle von Zusammenhängen zwischen beobachtbaren Tatsachen, dann sollten Theorien eben auch auf beobachtbaren Tatsachen beruhen.

Theoretisch-empirische Wissenschaften beruhen eben auf dem Wechsel zwischen Tatsachenbeobachtungen und Theoriebildung. Mal haben die Tatsachenbeobachtungen "die Nase vorn", und die Theoriebildung läßt auf sich warten, mal hat man eine wunderschöne Theorie, nur keine Empirie dazu.

Problematisch wird die Sache nur, wenn man entweder eine Fülle von Beobachtungen hat, aber zu ihnen keine Modell, wie sie wohl nachprüfbar zusammenhängen könnten, oder einen Berg an Theorien im luftleeren Raum.

#185:  Autor: Marcellinus BeitragVerfasst am: 18.10.2017, 21:28
    —
joh hat folgendes geschrieben:
Mathematik ist nur ein perfektes Werkzeug für die Theorie, in der sie verwendet wird.

Aber unterschiedliche Theorien mit unterschiedlichen Definitionen führen zu unterschiedlichen Ergebnissen.

Die Mathematik hat den Vorgaben durch die Definitionen zu folgen.

Wenn eine Theorie bereits auf falsche Annahmen beruht, dann ist die Mathematik immer noch perfekt.

Aus der Mathematik Erkenntnisse ableiten ist zweifelhaft. Man kommt dann eher mit Logik weiter.


Wieso ist die Logik da besser? Wenn du von falschen Voraussetzungen ausgehst, gehst du bei aller Logik in die Irre. Auf Deutsch: Mit Logik holst du aus einen Satz nichts heraus, was nicht schon drinsteckt. Alles andere ist Metaphysik (du kannst es auch Wunderglauben nennen).

#186:  Autor: stepWohnort: Germering BeitragVerfasst am: 18.10.2017, 21:40
    —
Marcellinus hat folgendes geschrieben:
Theoretisch-empirische Wissenschaften beruhen eben auf dem Wechsel zwischen Tatsachenbeobachtungen und Theoriebildung. Mal haben die Tatsachenbeobachtungen "die Nase vorn", und die Theoriebildung läßt auf sich warten, mal hat man eine wunderschöne Theorie, nur keine Empirie dazu.

Sehe ich ebenso.

Marcellinus hat folgendes geschrieben:
Problematisch wird die Sache nur, wenn man entweder eine Fülle von Beobachtungen hat, aber zu ihnen keine Modell, wie sie wohl nachprüfbar zusammenhängen könnten, oder einen Berg an Theorien im luftleeren Raum.

Ohne Probleme wüßte man ja nicht wohin mit den Forschungsgeldern zwinkern

Im Ernst, ein Beispiel: Man hatte bis vor ca. 1 Monat diverse mögliche Modelle, wie ein Neutronenstern innen aufgebaut ist, die alle mit der ART und den Beobachtungen einigermaßen übereinstimmten. Durch die hier schon erwähnte merger-Messung sind es jetzt ein paar Modelle weniger.

#187:  Autor: Marcellinus BeitragVerfasst am: 18.10.2017, 22:04
    —
step hat folgendes geschrieben:

Marcellinus hat folgendes geschrieben:
Problematisch wird die Sache nur, wenn man entweder eine Fülle von Beobachtungen hat, aber zu ihnen keine Modell, wie sie wohl nachprüfbar zusammenhängen könnten, oder einen Berg an Theorien im luftleeren Raum.

Ohne Probleme wüßte man ja nicht wohin mit den Forschungsgeldern zwinkern

Im Ernst, ein Beispiel: Man hatte bis vor ca. 1 Monat diverse mögliche Modelle, wie ein Neutronenstern innen aufgebaut ist, die alle mit der ART und den Beobachtungen einigermaßen übereinstimmten. Durch die hier schon erwähnte merger-Messung sind es jetzt ein paar Modelle weniger.


Ja, manchmal muß man eben Geduld haben, oder Glück, oder beides. zwinkern

#188:  Autor: johWohnort: Kevelaer BeitragVerfasst am: 18.10.2017, 22:04
    —
Marcellinus hat folgendes geschrieben:
joh hat folgendes geschrieben:
Mathematik ist nur ein perfektes Werkzeug für die Theorie, in der sie verwendet wird.

Aber unterschiedliche Theorien mit unterschiedlichen Definitionen führen zu unterschiedlichen Ergebnissen.

Die Mathematik hat den Vorgaben durch die Definitionen zu folgen.

Wenn eine Theorie bereits auf falsche Annahmen beruht, dann ist die Mathematik immer noch perfekt.

Aus der Mathematik Erkenntnisse ableiten ist zweifelhaft. Man kommt dann eher mit Logik weiter.


Wieso ist die Logik da besser? Wenn du von falschen Voraussetzungen ausgehst, gehst du bei aller Logik in die Irre. Auf Deutsch: Mit Logik holst du aus einen Satz nichts heraus, was nicht schon drinsteckt. Alles andere ist Metaphysik (du kannst es auch Wunderglauben nennen).


Definitionen sind in der Physik ein Mindestkonsens

Wenn man bei der Festlegung von Definitionen bereits logische Fehler begeht, wird das auch mit einer Theorie nichts werden.

Empfehlen würde ich dazu die Beiträge von Rafael Haumann zur Physikkritik

http://viaveto.de/vom-unsinn-des-urknalls.html Teil 1-3 und
http://viaveto.de/definitionen-logik.html


Zuletzt bearbeitet von joh am 18.10.2017, 22:37, insgesamt einmal bearbeitet

#189:  Autor: Marcellinus BeitragVerfasst am: 18.10.2017, 22:13
    —
joh hat folgendes geschrieben:
Marcellinus hat folgendes geschrieben:
joh hat folgendes geschrieben:
Mathematik ist nur ein perfektes Werkzeug für die Theorie, in der sie verwendet wird.

Aber unterschiedliche Theorien mit unterschiedlichen Definitionen führen zu unterschiedlichen Ergebnissen.

Die Mathematik hat den Vorgaben durch die Definitionen zu folgen.

Wenn eine Theorie bereits auf falsche Annahmen beruht, dann ist die Mathematik immer noch perfekt.

Aus der Mathematik Erkenntnisse ableiten ist zweifelhaft. Man kommt dann eher mit Logik weiter.


Wieso ist die Logik da besser? Wenn du von falschen Voraussetzungen ausgehst, gehst du bei aller Logik in die Irre. Auf Deutsch: Mit Logik holst du aus einen Satz nichts heraus, was nicht schon drinsteckt. Alles andere ist Metaphysik (du kannst es auch Wunderglauben nennen).


Definitionen sind in der Physik ein Mindestkonsenz

Wenn man bei der Festlegung von Definitionen bereits logische Fehler begeht, wird das auch mit einer Theorie nichts werden.

Empfehlen würde ich dazu die Beiträge von Rafael Haumann zur Physikkritik

http://viaveto.de/vom-unsinn-des-urknalls.html Teil 1-3 und
http://viaveto.de/definitionen-logik.html


Ich weiß nicht, ob du mich mißverstanden hast, oder ich mich mißverständlich ausgedrückt habe. Mir ging es nicht um Physik-Kritik; dafür habe ich von Physik einfach zu wenig Ahnung. Mir ging es um die Vorstellung, man komme mit Logik zu neuen Erkenntnissen. Ich halte das für einen Irrtum. Mit Logik kann ich vielleicht in einem Modell innere Widersprüche aufdecken. Ich kann vielleicht ein Modell so umformulieren, daß es leichter zu belegen oder zu widerlegen ist. Aber neues Wissen gewinnen ich so nicht. Logik ist ebenso ein menschengemachtes Regelsystem wie die Mathematik. Ich bekomme nur heraus, was ich vorher reingesteckt habe.

#190:  Autor: johWohnort: Kevelaer BeitragVerfasst am: 18.10.2017, 22:21
    —
Hallo, schau dir die Videobeiträge an. Danach hast du Ahnung.

#191:  Autor: Kramer BeitragVerfasst am: 18.10.2017, 22:26
    —
joh hat folgendes geschrieben:
Hallo, schau dir die Videobeiträge an. Danach hast du Ahnung.


Du sollst hier ja auch nicht ganz leer ausgehen. Hier kannst Du noch was lernen.

#192:  Autor: smallie BeitragVerfasst am: 19.10.2017, 23:46
    —
step hat folgendes geschrieben:
smallie hat folgendes geschrieben:
Das ist der Trugschluß im SdW-Artikel: er tut so, als würde er etwas erklären. Doch die Erklärung beschränkt sich auf: zufällig ist es bei uns so, wie es halt ist.

Weiß nicht, wie es in dem Artikel ist, aber in der Methode, die ich oben beschrieben habe, wird man durch den Symmetriebrechungsansatz ja gerade den Zufall los, man formuliert eine Mechanismus, der (auf dieser Ebene) keinen Zufall mehr beinhaltet.

Symmetriebrechung war erfolgreich bei den vergangenen Vereinheitlichungen von elektrischer, magnetischer und der schwachen Kraft. Sie könnte erfolgreich sein bei weiteren Vereinheitlichungen. Du kennst bestimmt die Diagramme, die die Kopplungsenergien dafür vorhersagen. Diese Energieniveaus liegen weit jenseits dessen, was auf absehbare Zeit machbar sein wird.

Der Symmetriebrechungsansatz ist gut. Ich könnte aber nur ästhetische Gründe nennen, warum er dabei helfen sollte, auch die Gravitation unter einen Hut zu bekommen.



step hat folgendes geschrieben:
smallie hat folgendes geschrieben:
Die Titus-Bode-Regel ist auch übersubjektiv operationalisierbar, wenn man nur unser Sonnensystem kennt.

Richtig. Ist ja auch keine hinreichende Bedingung, nur eine notwendige zwinkern Häufig wird das Ende einer Theorie dadurch eingeläutet, daß sie irgendwo nicht mehr funktioniert. Bei Titus-Bode kommt noch hinzu, daß es nicht wirklich eine Theorie ist, sondern nur eine Rechenregel, oder?

Eine Rechenregel, in der Tat.

Titus-Bode ist eine beschreibende Regel, aus ihr geht nicht hervor, warum die Planetenabstände ihre Werte annehmen. Gleiches gilt für die Keplerschen Gesetze, die letztlich auch nur beschreibende Regeln sind, aber über die Ursache der Planetenbewegung nichts aussagen.

Die Newtonsche Mechanik ist eine erklärende Regel, aus ihr lassen sich die Keplerschen Gesetze ableiten. Aber: auf einer tieferen Ebene ist die Newtonsche Mechanik auch nur beschreibend, das 1/r²-Gesetz der Gravitation wird als gegeben oder gefunden vorausgesetzt, ohne ursächlich erklärt zu werden.

Das dürfte sich mit dem decken, was du oben mit Zitat: (auf dieser Ebene) sagen wolltest.


Gibt es eine Erklärung für 1/r²? Ich denke schon. Nur 1/r² führt zu stabilen Planetenbahnen. Das läßt sich mathematisch zeigen, womit aber nicht erklärt wird, warum das gilt. Jetzt kommt die Multiversumsgilde und sagt: in unendlich vielen Universen sind von 1/r bis 1/(r^n) alle möglichen Varianten realisiert, und wir leben zufällig in der einen Universums-Untermenge, in der stabile Planetenbahnen möglich sind, während in allen anderen Universen kakophone Verhältnisse herrschen.

Das ist ein anthropisches Argument. Es versucht nicht einmal, ursächlich zu erklären, wie diese unterschiedlichen Potentiale physikalisch erzeugt werden.



step hat folgendes geschrieben:
smallie hat folgendes geschrieben:
Die bestätigte QFT reicht bis zum Standardmodell. Danach beginnt das Reich der mehr oder weniger wilden Hypothesen.

Ja, was aber nicht heißt, daß die alle wertlos sind. Einige sind sogar prinzipiell überprüfbar.

Bau immer größere Collider. Wenn Schwarze Löcher bereits bei geringeren Energien entstehen, als vorhergesagt, dann ist das ein starker Hinweis auf zusätzliche Dimensionen.

Woran hast du gedacht?

Prinzipiell überprüfbar? *läster* Das hört sich an wie "aber nicht in den nächsten hundert Jahren."



step hat folgendes geschrieben:
smallie hat folgendes geschrieben:
Wenn ich populäre Darstellungen zum Thema nur oberflächlich lese, entsteht bei mir oft der Eindruck, die Spekulationen seien bereits ausgemachte Wahrheiten - "a g'mahde Wies".

Ja mei. Der Lesch gleitet ja sogar häufig in die Theologie ab.

Gutes Stichwort.

Die evolutionäre Anthropologie spricht vom Hyperactive Agent Detection Device - HADD, einer Neigung alles und jedes dem Wirken von (übernatürlichen) Kräften und Agenten zuzuschreiben. Damit soll die religiöse Neigung des Menschen erklärt werden.

Aber, hey, es wäre falsch, HADD nur auf theologische und magische Gedankengänge anzuwenden. Die Wissenschaftsgeschichte ist voll von Beispielen für HADD. Kreisförmige Planetenbahnen, Phlogiston, Äther, Stringtheorie ( Pfeifen ). Was gar nicht abwertend gemeint ist, zur jeweiligen Zeit sahen diese Ideen wie gute Ideen aus.



step hat folgendes geschrieben:
Die meisten Physiker sind da aber realistischer.

In der Tat. Die schaffen es aber nicht auf die Titelseite der Magazine.


(Den Rest wegen Überlänge vorerst unbeantwortet gelassen.)

#193:  Autor: stepWohnort: Germering BeitragVerfasst am: 20.10.2017, 17:58
    —
smallie hat folgendes geschrieben:
Der Symmetriebrechungsansatz ist gut. Ich könnte aber nur ästhetische Gründe nennen, warum er dabei helfen sollte, auch die Gravitation unter einen Hut zu bekommen.

Bin anderer Ansicht, lassen wir das mal so stehen.

smallie hat folgendes geschrieben:
Gibt es eine Erklärung für 1/r²? Ich denke schon. Nur 1/r² führt zu stabilen Planetenbahnen. Das läßt sich mathematisch zeigen, womit aber nicht erklärt wird, warum das gilt.

Das sehe ich ebenfalls anders. Man kann zumindest 1-2 Schritte tiefer gehen in der Erklärung "warum 1/r²". Ich will das hier nicht im Detail ausbreiten, aber die Stichworte lauten konservative Kraft/Energieerhaltung, Laplaceoperator, Symmetriegruppe. Die stabilen Planetenbahnen sind nicht eine Folge dieser Dinge, sondern Folge der Dimensionalität der Raumzeit (3+1).

#194:  Autor: zelig BeitragVerfasst am: 21.10.2017, 09:47
    —
Wäre nicht ein wesentlicher Einwand und berechtigter Einwand das Prinzip, mit Entitäten möglichst sparsam umzugehen? Wenn man beispielsweise wie Tegmark die Mathematik und die Realität letztlich für identisch hält, dann scheint mir diese Annahme so ziemlich genau das Gegenteil des Sparsamkeitsprinzips zu sein. Anstatt aus der Existenz von bekannten Entitäten und deren Verhalten Theorien abzuleiten, wird die Existenz aller mathematisch möglichen Entitäten vorausgesetzt. Haben wir da nicht das Problem, daß diese Art von Theoriebildung immer erfolgreich ist? Bis zu dem Punkt natürlich, an dem man die Existenz einer postulierten Entität ausschließen kann? Da müsste in Anbetracht historischer Vorbilder Vorsicht walten. Was ja auch größtenteils der Fall ist, soweit ich das beurteilen kann.
Ansonsten wären das herrliche Glasperlenspiele. Das ist nicht despektierlich gemeint, soll aber schon zeigen, daß ich solche Überlegungen vorläufig im Spekulativen verorte. Selbst wenn das Doppelspaltexperiment dringlich nach einer Lösung verlangt.
Mal eine Frage, gibt es eigentlich eine Wissenschaft, die sich mit dem Verhältniss von postulierten Anteilen einer Theorie und ihrem Güligkeitsanspruch beschäftigt? Gehört sowas zur Sysemtheorie? Ist verständlich was ich meine? Sorry, schwimme an der Stelle.

#195:  Autor: johWohnort: Kevelaer BeitragVerfasst am: 21.10.2017, 11:33
    —
Auch wenn es wieder nervig klingt, daß ich meinen Senf dazu gebe mit meiner Esoterik.

Aus elektromagnetischer Sicht gibt es zu dem Doppelspaltexperiment keine Unklarheiten mehr.

Wer mehr Wissen will, zum Beispiel die Quelle zur Experimentalphysik mit elektromagnetischer Interpretation, kann ja fragen.

#196:  Autor: AlchemistWohnort: Hamburg BeitragVerfasst am: 21.10.2017, 11:36
    —
joh hat folgendes geschrieben:
Auch wenn es wieder nervig klingt, daß ich meinen Senf dazu gebe mit meiner Esoterik.

Aus elektromagnetischer Sicht gibt es zu dem Doppelspaltexperiment keine Unklarheiten mehr.

Wer mehr Wissen will, zum Beispiel die Quelle zur Experimentalphysik mit elektromagnetischer Interpretation, kann ja fragen.


Ich will mehr wissen. Kannst du das mal näher ausführen?

Btw: wo gibt es denn zum Doppelspaltexperiment Unklarheiten?

#197:  Autor: johWohnort: Kevelaer BeitragVerfasst am: 21.10.2017, 12:19
    —
Alchemist hat folgendes geschrieben:


Ich will mehr wissen. Kannst du das mal näher ausführen?

Btw: wo gibt es denn zum Doppelspaltexperiment Unklarheiten?


Zitat zeilig: "Selbst wenn das Doppelspaltexperiment dringlich nach einer Lösung verlangt."

Das bedeutet, wo eine Lösung in Aussicht ist, da ist auch ein Problem. Denn zu jedem Problem gehört eine Lösung.

Vielleicht kann zeilig das Problem des Doppelspaltexperiments aus quantenphysikalischer Sicht mal beschreiben.

#198:  Autor: stepWohnort: Germering BeitragVerfasst am: 21.10.2017, 12:34
    —
zelig hat folgendes geschrieben:
Wäre nicht ein wesentlicher Einwand und berechtigter Einwand das Prinzip, mit Entitäten möglichst sparsam umzugehen? Wenn man beispielsweise wie Tegmark die Mathematik und die Realität letztlich für identisch hält, dann scheint mir diese Annahme so ziemlich genau das Gegenteil des Sparsamkeitsprinzips zu sein. Anstatt aus der Existenz von bekannten Entitäten und deren Verhalten Theorien abzuleiten, wird die Existenz aller mathematisch möglichen Entitäten vorausgesetzt.

Sehe ich genau andersherum. Es scheint mir insgesamt ontologisch sparsamer, keine magischen Einschränkungen zu benötigen, während viele Menschen intuitiv eher ein magisches, kontingentes Teiluniversum sparsamer finden.

Zum zweiten Teil später mehr.

#199:  Autor: zelig BeitragVerfasst am: 21.10.2017, 12:50
    —
step hat folgendes geschrieben:
magischen Einschränkungen


Damit meinst Du den Zusammenbruch der Wellenfunktion, oder?

#200:  Autor: smallie BeitragVerfasst am: 21.10.2017, 13:27
    —
step hat folgendes geschrieben:
smallie hat folgendes geschrieben:
Gibt es eine Erklärung für 1/r²? Ich denke schon. Nur 1/r² führt zu stabilen Planetenbahnen. Das läßt sich mathematisch zeigen, womit aber nicht erklärt wird, warum das gilt.

Das sehe ich ebenfalls anders. Man kann zumindest 1-2 Schritte tiefer gehen in der Erklärung "warum 1/r²". Ich will das hier nicht im Detail ausbreiten, aber die Stichworte lauten konservative Kraft/Energieerhaltung, Laplaceoperator, Symmetriegruppe. Die stabilen Planetenbahnen sind nicht eine Folge dieser Dinge, sondern Folge der Dimensionalität der Raumzeit (3+1).

Die 1-2 Schritte gehe ich jederzeit mit.

Von der Vorstellung ausgehend, daß sich das Feld einer punktförmigen Quelle auf einer Kugeloberfläche ausbreitet, muß es sogar so sein. Aber was antworte ich, wenn jemand kommt und mich fragt, warum sich die Gravitationskraft nicht verhält wie die Kraft zwischen Quarks, für die das 1/r² nicht gilt?

Es gibt einige Prinzipien, die ich für universell - multiversell? - halte. Energieerhaltung hast du bereits genannt. Lagrange und das Minimalprinzip gehört auch dazu. Du kennst das, ich schreib's trotzdem auf: Es bedeutet, daß ein Stein nach unten fällt, und zwar geradeaus nach unten. Nicht erst hoch, dann runter, dann etwas links und wieder etwas rechts, sondern auf dem Weg der minimalen Wirkung. Wäre es anders, ließe sich allerhand Schabernack treiben.

Andererseits fällt mir kein guter Grund ein, warum es ausgerechnet vier Grundkräfte geben sollte, oder nach Vereinheitlichung nur drei oder nur eine. Oder vielleicht mit der hypothetischen Quintessenz dann doch wieder eine mehr.


Bonusfrage: Wohin gehört das Inflaton im Standardmodell? Passt das da überhaupt hinein, oder gehört es zu einem anderen Teilchenzoo?


Zuletzt bearbeitet von smallie am 21.10.2017, 13:58, insgesamt einmal bearbeitet

#201:  Autor: smallie BeitragVerfasst am: 21.10.2017, 13:57
    —
zelig hat folgendes geschrieben:
Wäre nicht ein wesentlicher Einwand und berechtigter Einwand das Prinzip, mit Entitäten möglichst sparsam umzugehen? Wenn man beispielsweise wie Tegmark die Mathematik und die Realität letztlich für identisch hält, dann scheint mir diese Annahme so ziemlich genau das Gegenteil des Sparsamkeitsprinzips zu sein.

Laß mich von einem schwachen und einem starken "Tegmark-Prinzip" sprechen.

    - schwache Version: die Wirklichkeit ist eine mathematische Struktur. Klar, Mathematik ist das Studium regelmäßiger, (oder gerne auch chaotischer) Strukturen. Die Welt müßte "ungesetzlich" sein, um sich diesem Ansatz zu entziehen. Etwa in dem Sinne, daß wir morgen aufwachen und zwei Monde oder zwei Sonnen am Himmel sehen.

    - starke Version: alle mathematisch konsistenten Gebilde sind realisiert. Das verschiebt die Frage nur dorthin, was mathematisch konsistente Gebilde sind.

Als Vergleich: angenommen, es gäbe nicht nur eine Erde mit einer biologischen Evolutionsgeschichte, sondern unendlich viele. Dann ließe sich sagen: jede mögliche biologische Form wird realisiert werden. Trotzdem wären wir kein bisschen klüger, welche Formen tatsächlich möglich sind.

Natürlich kann man sagen: in keiner dieser Erden wird es Insekten - Tracheenatmer - geben, die eine bestimmte Größe überschreiten, weil das prinzipiell nicht geht. Aber damit stecke ich nur Dinge hinein, die ich eh schon weiß und habe nichts gelernt.



zelig hat folgendes geschrieben:
Anstatt aus der Existenz von bekannten Entitäten und deren Verhalten Theorien abzuleiten, wird die Existenz aller mathematisch möglichen Entitäten vorausgesetzt. Haben wir da nicht das Problem, daß diese Art von Theoriebildung immer erfolgreich ist? Bis zu dem Punkt natürlich, an dem man die Existenz einer postulierten Entität ausschließen kann?

Wenn ich Tegmarks These auf die Mathematik anwende, dann fällt sie zusammen.

    alle mathematisch gültigen Sätze existieren.


Die Mathematiker haben das Programm, alle gültigen Sätze aufzählen zu wollen, vor knapp hundert Jahren aufgegeben. Tegmarks These zuende gedacht bedeutet: es wird Thesen zur mathematisch-physikalischen Wirklichkeit geben, die wir nie entscheiden werden können.

Wenn das so bei Tegmark steht, will ich nichts gesagt haben.



zelig hat folgendes geschrieben:
Selbst wenn das Doppelspaltexperiment dringlich nach einer Lösung verlangt.

Dr. Strangelove or: How I Learned to Stop Worrying and Love the Double-Slit-Experiment.

Pfeifen



zelig hat folgendes geschrieben:
Mal eine Frage, gibt es eigentlich eine Wissenschaft, die sich mit dem Verhältniss von postulierten Anteilen einer Theorie und ihrem Güligkeitsanspruch beschäftigt?

Das wird heute durch die Häufigkeit approximiert, mit der man deine Arbeiten zitiert. zwinkern


Zuletzt bearbeitet von smallie am 21.10.2017, 16:37, insgesamt einmal bearbeitet

#202:  Autor: Marcellinus BeitragVerfasst am: 21.10.2017, 15:49
    —
zelig hat folgendes geschrieben:

Mal eine Frage, gibt es eigentlich eine Wissenschaft, die sich mit dem Verhältniss von postulierten Anteilen einer Theorie und ihrem Güligkeitsanspruch beschäftigt? Gehört sowas zur Sysemtheorie? Ist verständlich was ich meine? Sorry, schwimme an der Stelle.

Dafür wäre eigentlich die Wissenssoziologie zuständig, wenn die selbst nicht noch viel mehr "schwimmen" würde. Sehr glücklich

#203:  Autor: AlchemistWohnort: Hamburg BeitragVerfasst am: 21.10.2017, 15:55
    —
joh hat folgendes geschrieben:
Alchemist hat folgendes geschrieben:


Ich will mehr wissen. Kannst du das mal näher ausführen?

Btw: wo gibt es denn zum Doppelspaltexperiment Unklarheiten?


Zitat zeilig: "Selbst wenn das Doppelspaltexperiment dringlich nach einer Lösung verlangt."

Das bedeutet, wo eine Lösung in Aussicht ist, da ist auch ein Problem. Denn zu jedem Problem gehört eine Lösung.

Vielleicht kann zeilig das Problem des Doppelspaltexperiments aus quantenphysikalischer Sicht mal beschreiben.


Und was ist mit:
Zitat:
Wer mehr Wissen will, zum Beispiel die Quelle zur Experimentalphysik mit elektromagnetischer Interpretation, kann ja fragen.


Ich habe doch danach gefragt.


und Warum verweist du denn überhaupt auf andere user, wenn ich explizit dich frage?

#204:  Autor: johWohnort: Kevelaer BeitragVerfasst am: 21.10.2017, 17:10
    —
Elektromagnetismus löst nicht die Probleme anderer Theorien mit dem Doppelspaltexperiment.

Aber mit Elektromagnetismus erklären sich alle Phänomene. Es gibt keine offenen Fragen mehr.

https://www.youtube.com/watch?v=lpI6ikj1G-s&html5=1

#205:  Autor: zelig BeitragVerfasst am: 21.10.2017, 17:18
    —
Lass mich nur auf das Wichtigste eingehen.

smallie hat folgendes geschrieben:

zelig hat folgendes geschrieben:
Selbst wenn das Doppelspaltexperiment dringlich nach einer Lösung verlangt.

Dr. Strangelove or: How I Learned to Stop Worrying and Love the Double-Slit-Experiment.


Du kennst also nicht nur gute Musik. Du kennst auch gute Filme.


Auf den Arm nehmen

#206:  Autor: AlchemistWohnort: Hamburg BeitragVerfasst am: 21.10.2017, 18:08
    —
joh hat folgendes geschrieben:
Elektromagnetismus löst nicht die Probleme anderer Theorien mit dem Doppelspaltexperiment.

Aber mit Elektromagnetismus erklären sich alle Phänomene. Es gibt keine offenen Fragen mehr.

https://www.youtube.com/watch?v=lpI6ikj1G-s&html5=1


joh hat folgendes geschrieben:
Auch wenn es wieder nervig klingt, daß ich meinen Senf dazu gebe mit meiner Esoterik.

Aus elektromagnetischer Sicht gibt es zu dem Doppelspaltexperiment keine Unklarheiten mehr.

Wer mehr Wissen will, zum Beispiel die Quelle zur Experimentalphysik mit elektromagnetischer Interpretation, kann ja fragen.


Deine Quelle, die du im obigen Beitrag angeboten hadt, ist also ein youtube Video? Mit den Augen rollen

#207:  Autor: johWohnort: Kevelaer BeitragVerfasst am: 21.10.2017, 18:19
    —
Für dich wird doch wohl die youtube-uni reichen (Scherz)

Es geht um den Inhalt. Ob ein Professor im Hörsaal einen Monolog hält ist vom Prinzip das Gleiche.

Aber wenn du googelst, findest du mit Sicherheit die schriftlichen Arbeiten dazu.

Die Aussagen in dem Beitrag wirst du nicht widerlegen können.

#208:  Autor: AlchemistWohnort: Hamburg BeitragVerfasst am: 21.10.2017, 18:29
    —
joh hat folgendes geschrieben:
Für dich wird doch wohl die youtube-uni reichen (Scherz)

Es geht um den Inhalt. Ob ein Professor im Hörsaal einen Monolog hält ist vom Prinzip das Gleiche.

Aber wenn du googelst, findest du mit Sicherheit die schriftlichen Arbeiten dazu.

Die Aussagen in dem Beitrag wirst du nicht widerlegen können.


Ist das nun deine Theorie? Oder hat sich das jemand anderer ausgedacht?
Und warum stellst du dich so quer einfach ne Quelle anzugeben?

#209:  Autor: johWohnort: Kevelaer BeitragVerfasst am: 21.10.2017, 18:35
    —
Die Quelle habe ich vor einer Woche schon mal präsentiert.

Hat sich aber keiner angeschaut.

#210:  Autor: AlchemistWohnort: Hamburg BeitragVerfasst am: 21.10.2017, 19:15
    —
joh hat folgendes geschrieben:
Die Quelle habe ich vor einer Woche schon mal präsentiert.

Hat sich aber keiner angeschaut.


Ach so, und nun bist du beleidigt und verlinkst das kein zweites Mal?

meine Fresse...du hast oben angeboten, eine Quelle zu präsentieren. Von dir aus. Und wenn man dich fragt, druckst du dich in 4 Beiträgen herum und spielst dann die beleidigte Leberwurst.

Was ist denn das für ein Kindergarten?

#211:  Autor: johWohnort: Kevelaer BeitragVerfasst am: 21.10.2017, 20:19
    —
@ Alchemist

Du bringst mich auf eine gute Idee. Um nicht die Threads anderer mit Elektromagnetismus, Weltformel und Metaphysik voll zu müllen, werde ich dazu einen eigenen Thread öffenen.

#212:  Autor: stepWohnort: Germering BeitragVerfasst am: 22.10.2017, 13:21
    —
smallie hat folgendes geschrieben:
Aber was antworte ich, wenn jemand kommt und mich fragt, warum sich die Gravitationskraft nicht verhält wie die Kraft zwischen Quarks, für die das 1/r² nicht gilt?

Die Symmetriegruppe SU(3), die die starke WW hervorbringt, ist nicht-abelsch und hat mehr Freiheitsgrade (z.B. Wechselwirkung der Eichbosonen untereinander, Isospin, Farbladung) als die Gruppe U(1) der QED. Das führt zu komplexeren Lagrangedichten und Potenzialen von Quarks/Gluonen. Tatsächlich geht so ein Potenzial für kleine Abstände mit 1/r wie bei der ED, aber es kommt ein Term ~r dazu ("Gummiband"), der bei größeren Abständen überhand nimmt und das sogenannte Confinement der Quarks bewirkt. Das Nukleon als Ganzes hat dann ein Yukawa-förmiges Potential.

Man muß allerdings zugeben, daß der Mechanismus der Symmetriebrechnug, der die Trennung von SU(3) und der elektroschwachen WW hervorbringt, nicht vollständig verstanden ist, anders als zwischen QED und schwacher WW, wo es mit dem Higgs-Mechanismus bekannt ist.

smallie hat folgendes geschrieben:
Es gibt einige Prinzipien, die ich für universell - multiversell? - halte. Energieerhaltung hast du bereits genannt.

Wobei - schauen wir mal, ob die Invarianz bzgl. Zeitverschiebung (und damit die E-Erhaltung) noch gilt, wenn wir unter die Planckgrößen vordringen.

smallie hat folgendes geschrieben:
Lagrange und das Minimalprinzip gehört auch dazu. Du kennst das, ich schreib's trotzdem auf: Es bedeutet, daß ein Stein nach unten fällt, und zwar geradeaus nach unten. Nicht erst hoch, dann runter, dann etwas links und wieder etwas rechts, sondern auf dem Weg der minimalen Wirkung. Wäre es anders, ließe sich allerhand Schabernack treiben.

So isses. Es ist übrigens meines Wissens bis heute unklar, welche tiefere Bedeutung das Minimalprinzip hat. Planck etwa sah darin ein Indiz für die Zielgerichtetheit der Welt, was natürlich aus heutiger Sicht absurd erscheint. Mithilfe der Feynman-Pfadintegrale könnte man evtl. über die Phasenrotation einen Bezug zum Multiversum herstellen zwinkern

smallie hat folgendes geschrieben:
Andererseits fällt mir kein guter Grund ein, warum es ausgerechnet vier Grundkräfte geben sollte, oder nach Vereinheitlichung nur drei oder nur eine. Oder vielleicht mit der hypothetischen Quintessenz dann doch wieder eine mehr.

Dafür gibt es apriori keinen zwingenden Grund. Dennoch vermuten es (fast) alle Physiker, weil man ca. 2,5 Grundkräfte schon vereinheitlicht hat, weil es einfacher ist (Standardodell hat "nur" noch 18 freie Parameter) und vor allem, weil man die ungebrochenen Symmetriezustände benötigt, um Experimente bei hohen Energien, in der Nähe des Urknalls zu erklären.

smallie hat folgendes geschrieben:
Bonusfrage: Wohin gehört das Inflaton im Standardmodell? Passt das da überhaupt hinein, oder gehört es zu einem anderen Teilchenzoo?

Grob gesagt: Die Inflation (bzw. das zugehörige hypothetische Inflatonfeld) gehört nicht in das Standardmodell der 3+1 Kräfte, wäre aber ebenfalls mit einer QFT beschreibbar. Eine vereinheitlichte Theorie der 4 Grundkräfte sollte die Inflation beinhalten, bei einigen Kandidaten ergibt sie sich sogar direkt aus dem Modell der quantisierten Raumzeit.

#213:  Autor: smallie BeitragVerfasst am: 24.10.2017, 23:00
    —
Anmerkungen zur Symmetrie.


step hat folgendes geschrieben:
smallie hat folgendes geschrieben:
Aber was antworte ich, wenn jemand kommt und mich fragt, warum sich die Gravitationskraft nicht verhält wie die Kraft zwischen Quarks, für die das 1/r² nicht gilt?

Die Symmetriegruppe SU(3), die die starke WW hervorbringt, ist nicht-abelsch und hat mehr Freiheitsgrade (z.B. Wechselwirkung der Eichbosonen untereinander, Isospin, Farbladung) als die Gruppe U(1) der QED. Das führt zu komplexeren Lagrangedichten und Potenzialen von Quarks/Gluonen. Tatsächlich geht so ein Potenzial für kleine Abstände mit 1/r wie bei der ED, aber es kommt ein Term ~r dazu ("Gummiband"), der bei größeren Abständen überhand nimmt und das sogenannte Confinement der Quarks bewirkt. Das Nukleon als Ganzes hat dann ein Yukawa-förmiges Potential.

Eine empirisch gefundene Symmetrie unseres Universums zum multiversellen Wirkprinzip zu erklären, halte ich für gewagt.

Warum ausgerechnet diese Symmetrie und nicht eine andere? Kennzeichen einer Symmetrie ist, daß ich Elemente vertauschen darf, und es bleibt symmetrisch. In einem Kreis darf ich jeden Punkt vertauschen, ihn drehen, vergrößern oder verkleinern, und er bleibt symmetrisch. Ein Rohrschach-Klecks hingegen ist nur spiegelsymmetrisch.

Physiker in einem Rohrschach-Klecks-Universum würden die Spiegelsymmetrie zum multiversellen Prinzip erheben. Was würden Physiker aus einem Weihnachtsstern-Universum dazu sagen, wenn sie davon wüßten?

Fermionen und Bosonen als multiverselles Prinzip, da könnte ich vielleicht noch mitgehen, aber wenn ich den Gedanken des Multiversums ernst nehme, dann sollten die Symmetrien zwischen den Teilchenarten so zahlreich und unterschiedlich sein wie bei Kristallen.



step hat folgendes geschrieben:
Man muß allerdings zugeben, daß der Mechanismus der Symmetriebrechnug, der die Trennung von SU(3) und der elektroschwachen WW hervorbringt, nicht vollständig verstanden ist, anders als zwischen QED und schwacher WW, wo es mit dem Higgs-Mechanismus bekannt ist.

Bei der Gravitation ist das noch viel, ahrm, haariger. zwinkern



step hat folgendes geschrieben:
smallie hat folgendes geschrieben:
Lagrange und das Minimalprinzip gehört auch dazu. Du kennst das, ich schreib's trotzdem auf: Es bedeutet, daß ein Stein nach unten fällt, und zwar geradeaus nach unten. Nicht erst hoch, dann runter, dann etwas links und wieder etwas rechts, sondern auf dem Weg der minimalen Wirkung. Wäre es anders, ließe sich allerhand Schabernack treiben.

So isses. Es ist übrigens meines Wissens bis heute unklar, welche tiefere Bedeutung das Minimalprinzip hat.

Hat das nicht unmittelbar mit Erhaltungsgesetzen zu tun? Die - tada - letztlich Symmetrien sind. Zum Beispiel:

    - Verschiebesymmetrie. Örtlich: ein Stein fällt hier genauso wie fünf Meter weiter rechts oder fünf Meter weiter oben. Zeitlich: ein Stein fällt heute genau so wie gestern. Gäbe es das Minimalprinzip nicht könnte der Stein mal so, mal anders fallen.

    - Zeitliche Symmetrie. Zeit und Impuls lassen sich umkehren. Ich kann nicht unterscheiden, ob ich einen Film sehe, in dem eine Kugel aus zehn Meter Höhe fällt und mit Endgeschwindigkeit aufschlägt, oder ob der Film rückwärts läuft und die Kugel vom Boden aus mit Startgeschwindigkeit abgeschossen wurde. Genau so gut kannst du einen Film der Planetenbewegungen vorwärts- oder rückwärts laufen lassen - ich könnte es nicht unterscheiden. Mit einer riesengroßen Einschränkung: wenn ich genau hinschaue, dann strahlen die Planeten auf ihren Bahnen Gravitationswellen ab, wodurch sich der Bahnradius verringert.


Nicht zuletzt schränkt das Minimalprinzip die Zahl möglicher Lösungen auf eine oder einige wenige ein, zum Beispiel Kegelschnitte als Lösungen von Bahngleichungen. Mir erscheint es logisch zwingend, daß es nur eine Lösung gibt, sonst könnte jederzeit alles Mögliche passieren. In der Quantenmechanik allerdings gibt es nicht nur eine Lösung, sondern eine Überlagerung von mehreren auf einmal, was den Alltagsverstand auf die Probe stellt.



step hat folgendes geschrieben:
smallie hat folgendes geschrieben:
Andererseits fällt mir kein guter Grund ein, warum es ausgerechnet vier Grundkräfte geben sollte, oder nach Vereinheitlichung nur drei oder nur eine. Oder vielleicht mit der hypothetischen Quintessenz dann doch wieder eine mehr.

Dafür gibt es apriori keinen zwingenden Grund. Dennoch vermuten es (fast) alle Physiker, weil man ca. 2,5 Grundkräfte schon vereinheitlicht hat, weil es einfacher ist (Standardodell hat "nur" noch 18 freie Parameter) ...

Also doch ein ästhetisches Argument?



step hat folgendes geschrieben:
... und vor allem, weil man die ungebrochenen Symmetriezustände benötigt, um Experimente bei hohen Energien, in der Nähe des Urknalls zu erklären.

Frage

#214:  Autor: smallie BeitragVerfasst am: 24.10.2017, 23:02
    —
Anmerkungen zur Asymmetrie.


Den Spektrum-Artikel habe ich inzwischen nochmals gelesen. Als Grundsatzkritik: mir fehlt da eine Aussage zur Entropie. Entropie nimmt immer zu. Die beste Möglichkeit, Entropie zu erzeugen, ist den Raum inflationär expandieren zu lassen, weil leerer Raum durch die Nullpunktsenergie Entropie besitzt. Soweit ok. Aber warum sollte die Inflation dann jemals aufhören? Ein Minimalprinzip ist nötig. Einfach zu sagen, das Inflatonfeld hat in unserem Raumbereich ein lokales Minimim erreicht, löst das Problem zwar, ist aber eine Stegreifannahme.

Nichts gegen Stegreifannahmen, die empirisch belegt sind. Mir wäre wohler, wenn die Sache durch "entropische Überlegungen" begründet wäre.

Warum? Ein kurzer Exkurs in die Geschichte der Quantenmechanik. Behauptung: Quantenmechanik entstand aus einer Analogieüberlegung zur Entropie.


step hat folgendes geschrieben:
Alchemist hat folgendes geschrieben:
step hat folgendes geschrieben:
... Einstein. Da ist nunmal E = h*ν
Vielleicht negiert uwe auch diese Formel?

Natürlich, aber hier ging es ja darum, ob Einstein sie negiert. Und der tut es sicher nicht.

Hehe. Siehe Einsteins Aufsatz Über einen die Erzeugung und Verwandlung des Lichtes betreffenden heuristischen Gesichtspunkt.

Einstein ist vor allem als Begründer der speziellen und der allgemeinen Relativitätstheorie bekannt. Weniger bekannt ist, daß auch andere - Lorentz, Poincare, Hilbert - an diesen Themen arbeiteten und innerhalb von Monaten oder wenigen Jahren auch darauf gekommen wären.

Daß er nebenbei auch Begründer der Quantenmechanik ist, geht etwas unter. Das wäre einen ausführlichen Beitrag wert, mit Zitaten und Formeln und den Reaktionen der Zeitgenossen. Kurz angerissen, wie Einstein darauf kam.

Damals konnte man sich das Spektrum der Schwarzkörperstrahlung nicht erklären. Planck hatte eine empirische Formel dafür erraten, von Wilhelm Wien stammte ein andere Formel. Einstein sah die Formel von Wien, erkannte eine Ähnlichkeit zu Boltzmanns Entropie s = k log W aus der Theorie der kinetischen Gase. Kühn behauptet er, daß es Energiequanten geben müsse, analog zu den Teilchen aus der Gastheorie.



Was das Multiversum angeht, wundere ich mich, daß Entropie bei diesen Überlegungen keine Rolle spielt. (Liegt vermutlich an meiner Unkenntnis der Fachliteratur, aber auch Artikel im SdW sind sehr wortkarg in dieser Richtung.) Kann man ernsthaft vom Multiversum sprechen und dabei ein so grundlegendes Prinzip wie Entropie ausblenden?

#215:  Autor: stepWohnort: Germering BeitragVerfasst am: 25.10.2017, 10:54
    —
@smallie: viele interessante Gedanken ... erstmal nur ganz kurz zur multiversalen Entropie: Hier würde ich mal beginnen mit der Frage, wie sich die Entropie in einem einfachen überlagerten Quantenzustand bzw. beim Meßvorgang verhält.

#216:  Autor: smallie BeitragVerfasst am: 26.10.2017, 19:31
    —
Lachen Lachen Lachen Schmeiß mich weg.

Wenn ich richtig verstehe, worauf du hinaus willst, ist das das beste Argument für die Quanten-Vielwelttheorie, das ich bisher gehört habe: Nach dem Kollaps der Wellenfunktion wird aus N Superpositionen nur eine. Welche Verschwendung von wertvoller Entropie!



Aber: kollabiert die Wellenfunktion denn wirklich? Falls ja, dann sollte das Teilchen fürderhin gemäß Ziegelsteinmechanik weiterfliegen. Tatsächlich ist es bei der nächsten Messung schon wieder eine Überlagerung. Der Compton-Effekt bietet sich an, um den Gedanken zu überprüfen, auch wenn ich ohne Nachschlagen nichts genaues sagen kann.

Zum Problem wird der Kollaps erst, wenn man ihn zur Hinrichtung von Katzen verwendet. Pfeifen

Einen Meßvorgang stelle ich mir als Operation vor, bei dem eine Art Kreuzprodukt zwischen der Wellenfunktion des vermessenen und des messenden Teilchens gebildet wird. Mit der Messung ergibt sich für die beide Teilchen jeweils eine neue Wellenfunktion. Das verblüffende daran ist nicht der Kollaps, sondern daß die Wahrscheinlichkeiten der neuen Wellenfunktionen im Sinn der Bellschen Ungleichung nicht unserer klassischen Logik folgen.



Unabhängig davon hast du ein sehr gutes Argument für das Multiversum. Aus entropischen Überlegungen erscheint seine Existenz zwingend. Auch hier ein Aber: wenn ich Jeremy England folge, dann ist Leben eine Methode, um die Entropie im Universum zu erhöhen. Es gibt aber nicht auf jedem Planeten leben.

#217:  Autor: stepWohnort: Germering BeitragVerfasst am: 26.10.2017, 20:29
    —
Naja, ganz so schön ist es nicht. Und es ist leider sehr schwierig, über Entropie und Zeitpfeil kosmologisch zu debattieren.

Aber über den Zusammenhang der Entropie mit der Entartung und so auch mit der fehlenden Information über die Mikrozustände kann man einsehen, daß in einer multiversal interpretierten Aufspaltung sich die Gesamtentropie sehr logisch verändert, während man beim Kollapsmodell komische Zusatzannahmen über magische Informationen usw. machen muß.

Allgemein kann man sagen: Die Entropie des Meßobjekts (Katze) nimmt bei der Messung ab (im Idealfall um die Information über den Mikrozustand), die des Gesamtsystems nimmt jedoch zu.

https://de.wikipedia.org/wiki/Entropie#Quantenmechanik

IdZ hier noch ein schönes Plädoyer für das Everett Multiverse, ohne Formeln. Insbesondere auch zu Deiner Vorstellung des Meßvorgangs. Außerdem wird schön aufgezeigt, welche Einwände "silly" sind und welche nicht:
http://www.preposterousuniverse.com/blog/2014/06/30/why-the-many-worlds-formulation-of-quantum-mechanics-is-probably-correct/

Das alles gilt natürlich nur für das Typ-Everett-Multiversum. Es gibt ja weitere Typen (z.B. expansion bubbles) und es gibt sogar Leute, die meinen, im Gesamtmultiversum sei gar keine Zeitrichtung ausgezeichnet.

#218:  Autor: smallie BeitragVerfasst am: 28.10.2017, 20:15
    —
Teil 1 - was die Lehrbücher wirklich sagen.


step hat folgendes geschrieben:
IdZ hier noch ein schönes Plädoyer für das Everett Multiverse, ohne Formeln. Insbesondere auch zu Deiner Vorstellung des Meßvorgangs. Außerdem wird schön aufgezeigt, welche Einwände "silly" sind und welche nicht:
http://www.preposterousuniverse.com/blog/2014/06/30/why-the-many-worlds-formulation-of-quantum-mechanics-is-probably-correct/

Sean Carroll hatten wir schon mal.

smallie hat folgendes geschrieben:
Der Astrophysiker Sean Carroll meint, Falsifizierbarkeit sei reif fürs Altenteil.

Zitat:
2014 : WHAT SCIENTIFIC IDEA IS READY FOR RETIREMENT?
Sean Carroll - Falsifiability


Science is not merely armchair theorizing; [...] Fortunately, science marches on, largely heedless of amateur philosophizing.

http://edge.org/response-detail/25322

Der letzte Satz ist ziemlich fies. Mr. Green



Zum Link: Carroll stellt die Sache sehr einseitig dar. EDIT: farblich markiert.

Zitat:
Why the Many-Worlds Formulation of Quantum Mechanics Is Probably Correct

In any formulation of quantum mechanics, the apparatus starts in a “ready” state, which is a way of saying “it hasn’t yet looked at the thing it’s going to observe” (i.e., the particle). More specifically, the apparatus is not entangled with the particle; their two states are independent of each other. So the quantum state of the particle+apparatus system starts out like this:

    (“spin is up” + “spin is down” ; apparatus says “ready”) (1)


The particle is in a superposition, but the apparatus is not. According to the textbook view, when the apparatus observes the particle, the quantum state collapses onto one of two possibilities:

    (“spin is up”; apparatus says “up”)

    or

    (“spin is down”; apparatus says “down”).


When and how such collapse actually occurs is a bit vague — a huge problem with the textbook approach — but let’s not dig into that right now.

But there is clearly another possibility. If the particle can be in a superposition of two states, then so can the apparatus. So nothing stops us from writing down a state of the form

    (spin is up ; apparatus says “up”)
    + (spin is down ; apparatus says “down”). (2)


The plus sign here is crucial. This is not a state representing one alternative or the other, as in the textbook view; it’s a superposition of both possibilities. In this kind of state, the spin of the particle is entangled with the readout of the apparatus.

http://www.preposterousuniverse.com/blog/2014/06/30/why-the-many-worlds-formulation-of-quantum-mechanics-is-probably-correct/

Ich weiß ja nicht, welche Lehrbücher Sean Carroll liest. Pfeifen Auch habe ich keinen Überblick, wie die Sache in der Gesamtheit der Lehrbücher dargestellt wird.

Aber ich kann ein Gebenbeispiel vorlegen, das die Sache etwas anders darstellt.

Zitat:
Vorlesungen über Physik, Band III : Quantenmechanik
Richard Feynman - 1965


Nehmen wir an, es gäbe zwei Teilchen, die wir Teilchen Nr. 1 und Teilchen Nr. 2 nennen können. Was sollen wir als Basiszustände benutzen? Ein vollkommen ausreichender Satz kann beschrieben werden, indem man sagt, daß Teilchen 1 bei x1 und Teilchen 2 bei x2 ist [...] Beachten Sie, daß die Beschreibung des Ortes von nur einem Teilchen keinen Baisiszustand definiert. Jeder Basiszustand muß den Zustand des gesamten Systems definieren.

Sie dürfen nicht glauben, daß sich jedes Teilchen unabhäng als dreidimensionale Welle bewegt. [...] Diese verallgemeinerte Amplitude ist daher eine Funktion der zwei Koordinatensätze x1 und x2. Sie sehen daß solch eine Funktion keine Welle im Sinner einer Schwingung ist, die sich in drei Dimensionen ausbreitet. Auch ist sie im allgemeinen nicht einfach ein Produkt von zwei einzelnen, zu je einem Teilchen gehörenden Wellen. Sie ist im allgemeinen eine Arte Welle in den sechs Dimensionen, die durch x1 und x2 definiert sind.

Wenn in der Natur zwei Teilchen vorhanden sind, zwischen denen eine Wechselwirkung besteht, gibt es keine Möglichkeit, zu beschreiben, was mit einem der Teilchen geschieht, indem man versucht eine Wellenfunktion für das Teilchen allein aufzuschreiben.

Die berühmten Widersprüche, die wir in früheren Kapiteln behandelt haben - wo behauptet wurde, daß die Messungen, die an einem Teilchen durchgeführt wurden, auch eine Aussage darüber ermöglichen, was mit einem anderen Teilchen geschehen wird, oder daß die Messungen eine Interferenz zerstören konnten - haben allgemein vielerlei Schwierigkeiten verursacht, weil man versuchte, eher an die Wellenfunktion eines einzelnen Teilchen zu denken als an die richtige Wellenfunktion in den Koordinaten beider Teilchen.

Die vollständige Beschreibung kann nur durch Funktionen der Koordinaten beider Teilchen richtig gegeben werden.

(16-4 Normierung der x-Zustände)

Bei Carroll prägt sich der Quantenzustand des vermessenen Teilchens dem messendem Teilchen auf. Bei Feynman haben beide Teilchen bereits vor der Messung ihre eigenen - und unbekannten - Zustände.

Ich möchte später noch in meinen Worten und Beispielen aufschreiben, wie ich das verstehe. Ein passendes Zitat hinzuklatschen ist ja nur ein halbes Argument - so einfach will ich es mir nicht machen, mich hinter Feynman zu verstecken.

Vermutlich verstehe ich dabei vieles falsch. Aber wenn Carroll schreibt

    In any formulation of quantum mechanics, the apparatus starts in a “ready” state

dann hat auch er etwas falsch verstanden. Oder die falschen Lehrbücher gelesen.

#219:  Autor: AlchemistWohnort: Hamburg BeitragVerfasst am: 28.10.2017, 22:43
    —
joh hat folgendes geschrieben:
@ Alchemist

Du bringst mich auf eine gute Idee. Um nicht die Threads anderer mit Elektromagnetismus, Weltformel und Metaphysik voll zu müllen, werde ich dazu einen eigenen Thread öffenen.


Und? Wo bleibt dieser Thread?

#220:  Autor: stepWohnort: Germering BeitragVerfasst am: 28.10.2017, 22:48
    —
smallie hat folgendes geschrieben:
Carroll hat folgendes geschrieben:
Science is not merely armchair theorizing; [...] Fortunately, science marches on, largely heedless of amateur philosophizing.
Der letzte Satz ist ziemlich fies. Mr. Green

Ja, er lehnt sich manchmal ein bißchen weit raus. Hab gerade keine Zeit, den Gesamtkontext zu lesen, aber es klingt hier tatsächlich ein bißchen so, als ob er selbst das betreibt, worüber er die Wissenschaft hinwegschreiten sieht.

smallie hat folgendes geschrieben:
Ich weiß ja nicht, welche Lehrbücher Sean Carroll liest. Pfeifen Auch habe ich keinen Überblick, wie die Sache in der Gesamtheit der Lehrbücher dargestellt wird.

Forscher: ~70% Copenhagen; Lehrbücher: 95% (?) jedenfalls zu meiner Zeit. Feynman nimmt eine echte Sonderstellung ein, nicht nur zu diesem Thema.

smallie hat folgendes geschrieben:
Vorlesungen über Physik, Band III : Quantenmechanik, Richard Feynman - 1965 hat folgendes geschrieben:
.. Die vollständige Beschreibung kann nur durch Funktionen der Koordinaten beider Teilchen richtig gegeben werden ...
Bei Carroll prägt sich der Quantenzustand des vermessenen Teilchens dem messendem Teilchen auf. Bei Feynman haben beide Teilchen bereits vor der Messung ihre eigenen - und unbekannten - Zustände.

Unbekannt heißt aber nicht verschränkt, insofern sehe ich da keinen Widerspruch.

smallie hat folgendes geschrieben:
... wenn Carroll schreibt
    In any formulation of quantum mechanics, the apparatus starts in a “ready” state

dann hat auch er etwas falsch verstanden. Oder die falschen Lehrbücher gelesen.

Ich verstehe es so, daß das Meßgerät p.d. zu Beginn nicht verschränkt mit dem Objektzustand ist.

#221:  Autor: smallie BeitragVerfasst am: 29.10.2017, 19:49
    —
step hat folgendes geschrieben:
smallie hat folgendes geschrieben:
Ich weiß ja nicht, welche Lehrbücher Sean Carroll liest. Pfeifen Auch habe ich keinen Überblick, wie die Sache in der Gesamtheit der Lehrbücher dargestellt wird.

Forscher: ~70% Copenhagen;

Die Vorstellung vom Kollaps gehört inzwischen zur Folklore, sie wird uns auf absehbare Zeit erhalten bleiben.


step hat folgendes geschrieben:
Lehrbücher: 95% (?) jedenfalls zu meiner Zeit.

Ich werde den Göttern eine Schale Reis opfern. Gelegentlich haben sie doch einen sehr feinen Sinn für Ironie. zwinkern

Zwei weitere Lehrbücher zur Sache habe ich im Regal stehen. Eins dürfte aus deiner Zeit stammen, da habe ich noch nicht reingeschaut. Das andere ist aktuell. Die Autoren fangen mit einem "spin- und katzenlosem" Apparat an, genau wie Carroll das beschreibt. Dann aber heißt es:

Zitat:
Quantum Mechanics - The Theoretical Minimum
Leonard Susskind & Art Friedman - 2014


However, the result of a measurement cannot be properly described without taking the apparatus into account as part of the system.


(Am Ende von 3.5 - A Common Misconception)

Die Autoren haben nur aus Gründen der Darstellung mit einem klassischen Apparat begonnen, um nicht gleich mit der Tür ins Haus zu fallen. Das ist das Gegenteil dessen, was Carroll behauptet.

Auf dem Klappentext des Buches steht:

    If you want to know how physicists really think about the world, this book is the place to start - Sean Carroll

Dann kennt Carroll jetzt doch ein textbook, das die Sache etwas anders darstellt.

#222:  Autor: smallie BeitragVerfasst am: 29.10.2017, 21:41
    —
Teil 2 - Auf Kollaps komm raus.


Aus meiner Sicht findet der Kollaps der Wellenfunktion nicht statt. Beispiel: ich schicke Licht durch einen Spalt auf einen Schirm.

    - ist der Spalt weit, dann werde ich einen klar umrissenen Lichtstreifen sehen.
    - mache ich den Spalt enger, wird der Streifen breiter und verschmiert.

Der enge Spalt mißt die Polarisation des Lichtes. In der Version von Carroll kollabiert dabei die Wellenfunktion. Stimmt ja gar nicht, sie wurde nur transformiert in eine neue Unschärfe darüber, wo das gestreute Teilchen auf dem Schirm auftrifft.

Der "Apparat" hat mit dem Photon eine Wechselwirkung ausgetauscht. Der "Apparat" ist hier ein Atom aus der Wand des Spaltes. Die Polarisation des Photons ist jetzt bekannt, dafür sind der weitere Zustand des Atoms ebenso wie andere Quantenparameter des Photons nun unbekannt.

Insbesondere sind keine Mikrozustände verloren gegangen. Mikrozustände gehen nicht einfach so verloren. Das wäre ja ganz was neues. Oops. Mit dem Argument ist man schon oft auf die Nase gefallen. Hier halte ich es für richtig.



Sean Carroll hat folgendes geschrieben:
Why the Many-Worlds Formulation of Quantum Mechanics Is Probably Correct

When and how such collapse actually occurs is a bit vague — a huge problem with the textbook approach.


Die Antwort auf das Wann hat Carroll selbst gegeben: immer wenn eine Messung stattfindet.

Sean Carroll hat folgendes geschrieben:
To describe measurements, we need to add an observer. It doesn’t need to be a “conscious” observer or anything else that might get Deepak Chopra excited; we just mean a macroscopic measuring apparatus. It could be a living person, but it could just as well be a video camera or even the air in a room. To avoid confusion we’ll just call it the “apparatus.”

Das ist eine gute Antwort, weil sie sich ausdrücklich nicht auf einen bewußten Beobachter stützt. Es reicht eine Wechselwirkung zwischen Teilchen. EDIT: Halt, nein. Eigentlich ist es eine schlechte Antwort, weil "Apparat" etwas makroskopisches meint und genau das ist es nicht.

Praktisches Beispiel: der Doppelspaltversuch klappt mit Photonen wie mit Elektronen. Bei Photonen klappt er in Luft, weil Luft für Photonen einigermaßen durchsichtig ist. Bei Elektronen braucht es ein Vakuum, sonst wechselwirken die Elektronen mit den Luftmolekülen.

Das Wie der Wechselwirkung kann ich leider nicht näher erklären, weil ich von Quantenelektrodynamik nichts verstehe. Feynman war daran beteiligt, siehe das Feynman-Diagramm gerade im anderen Thread. Wenn ich mich auf die Experten verlasse, dann gibt es zur Frage nach dem Wie gute Antworten mit ziemlich guter experimenteller Bestätigung.

Warum Carroll dieses Wie unter den Tisch fallen läßt, ist mir nicht klar. Das Standardmodell funktioniert, wie auch die Schrödingergleichung, ohne daß ich auf Interpretationen wie Kopenhagen, Vielwelttheorie, Bohmsche Mechanik, GWR zurückgreifen muß.



step hat folgendes geschrieben:
Ich verstehe es so, daß das Meßgerät p.d. zu Beginn nicht verschränkt mit dem Objektzustand ist.

Beim bisherigen Wissensstand geht das durch.

Aber: wenn Gravitation quantisiert ist und wenn Masseteilchen mit allen anderen wechselwirken, dann läuft doch dauernd ein Meßapparat, der zum kontinuierlichen Kollaps oder zu kontinuierlicher Verschränkung führt. Süffisant merke ich an, daß die beiden unterschiedlichen Sprechweisen in der Praxis auf die selben Beobachtungen hinauslaufen.



Sean Carroll hat folgendes geschrieben:
There are other silly objections to EQM, of course. The most popular is probably the complaint that it’s not falsifiable. That truly makes no sense. It’s trivial to falsify EQM — just do an experiment that violates the Schrödinger equation or the principle of superposition, which are the only things the theory assumes.

Dann sollte sich die Kopenhagener Deutung und der Kollaps ebenso trivial falsifizieren lassen.



Sean Carroll hat folgendes geschrieben:
There are three popular strategies on the market: anger, denial, and acceptance.

The “anger” strategy says “I hate the idea of multiple worlds with such a white-hot passion that I will change the rules of quantum mechanics in order to avoid them.” And people do this! [...]

The “denial” strategy says “The idea of multiple worlds is so profoundly upsetting to me that I will deny the existence of reality in order to escape having to think about it.”

The final strategy is acceptance. That is the Everettian approach. The formalism of quantum mechanics, in this view, consists of quantum states as described above and nothing more, which evolve according to the usual Schrödinger equation and nothing more.

Auch Vielwelten lassen sich als denial, als Leugnen der Wirklichkeit anschwärzen.

Wenn ich mich auf Vielwelten einlasse, dann darf ich im Gegenzug Lokalität und Realismus behalten. Die Einzellösungen der Superposition verhalten sich dann wie klassische Systeme und der Alltagsverstand wird nicht weiter auf die Probe gestellt. Ich darf meine klassischen Vorstellungen behalten, sofern ich das Quantenmultiversum aktzeptiere.

Behauptung: in allen Vielwelten stellt sich das gleiche Muster am Schirm hinter einem Doppelspalt ein. Wofür brauche ich Vielwelten, wenn sich in allen Vielwelten im statistischen Mittel das gleiche Bild ergibt?

Carroll hat eine vierte Strategie ausgelassen: in der Quantenwelt gelten andere Regeln als in Alltagswelt.

#223:  Autor: stepWohnort: Germering BeitragVerfasst am: 30.10.2017, 11:54
    —
smallie hat folgendes geschrieben:
Insbesondere sind keine Mikrozustände verloren gegangen. Mikrozustände gehen nicht einfach so verloren. Das wäre ja ganz was neues. Oops. Mit dem Argument ist man schon oft auf die Nase gefallen. Hier halte ich es für richtig.

Deswegen schrieb ich ja oben, daß die Entropie des Gesamtsystems nicht geringer wird.

smallie hat folgendes geschrieben:
Halt, nein. Eigentlich ist es eine schlechte Antwort, weil "Apparat" etwas makroskopisches meint und genau das ist es nicht.
Praktisches Beispiel: der Doppelspaltversuch klappt mit Photonen wie mit Elektronen. Bei Photonen klappt er in Luft, weil Luft für Photonen einigermaßen durchsichtig ist. Bei Elektronen braucht es ein Vakuum, sonst wechselwirken die Elektronen mit den Luftmolekülen.

In der Tat, und dieser Meinung ist ganz sicher auch Carroll. Sogar die makroskopische Wirkung der "Umgebung" in der Dekohärenztheorie und Superselektion beruhen auf mikroskopischen Wechselwirkungen. Genaugenommen ist die Dekohärenz daher unvereinbar mit der (ursprünglichen) Kopenhagener Deutung, da letztere a priori einen makroskopisch wirkenden Meßapparat annahm, da nur ein solcher einen Kollaps verursachen könne.

smallie hat folgendes geschrieben:
Warum Carroll dieses Wie unter den Tisch fallen läßt, ist mir nicht klar.

Vermutlich weil er in dem Text keinen Formalismus verwenden will.

smallie hat folgendes geschrieben:
Das Standardmodell funktioniert, wie auch die Schrödingergleichung, ohne daß ich auf Interpretationen wie Kopenhagen, Vielwelttheorie, Bohmsche Mechanik, GWR zurückgreifen muß.

Genau, shut up and calculate. Die Interpretationen kommen v.a. dann ins Spiel, wenn es um Fragen geht wie etwa "ist die Wellenfunktion real?" oder eben, wenn man sich gegen Vorwürfe ontologischer Opulenz wehren muß.

smallie hat folgendes geschrieben:
step hat folgendes geschrieben:
Ich verstehe es so, daß das Meßgerät p.d. zu Beginn nicht verschränkt mit dem Objektzustand ist.
Beim bisherigen Wissensstand geht das durch. Aber: wenn Gravitation quantisiert ist und wenn Masseteilchen mit allen anderen wechselwirken, dann läuft doch dauernd ein Meßapparat, der zum kontinuierlichen Kollaps oder zu kontinuierlicher Verschränkung führt. Süffisant merke ich an, daß die beiden unterschiedlichen Sprechweisen in der Praxis auf die selben Beobachtungen hinauslaufen.

Ja, aber Vorsicht: Eine solche Verschränkung gälte bezüglich eines "anderen Hilbertraums". Sie hätte Auswirkungen, aber nur in ganz anderen Experimenten - etwa einer Art gravitativem Doppelspaltexperiment, das man evtl. mit einem Schwarzen Loch oder einem kleinen Urknall durchführen könnte.

smallie hat folgendes geschrieben:
Sean Carroll hat folgendes geschrieben:
There are other silly objections to EQM, of course. The most popular is probably the complaint that it’s not falsifiable. That truly makes no sense. It’s trivial to falsify EQM — just do an experiment that violates the Schrödinger equation or the principle of superposition, which are the only things the theory assumes.
Dann sollte sich die Kopenhagener Deutung und der Kollaps ebenso trivial falsifizieren lassen.

Auch hier Zustimmung. Die Voraussagen, aufgrund derer die genannte Falsifikation möglich wäre, benötigen nur die Schrödingergleichung und keine Interpretation. Es wäre daher eher eine Falsifikation der SG. Was Carroll aber damit meint - jedenfalls verstehe ich es so - ist, daß ide EQM keine weiteren Zusatzannahmen hineinsteckt, die noch extra falsifizierbar sein müßten oder auch nur könnten. In der Kopenhagener Deutung dagegen könnte man mE fordern, daß die überraschende Zusatzannahme eines magischen Kollapses eine eigene Falsifikation erfordert, oder man könnte sogar sagen, daß sie aus theoriehygienischen Gründen (schwächer als Falsifikation, laut Carroll evtl. sogar stärker?) abzulehnen ist, da hier der Schwanz (intuitive Vorstellung von Realität = was ich sehe) mit dem Hund (Lösung der SG usw.) wackelt.

smallie hat folgendes geschrieben:
Behauptung: in allen Vielwelten stellt sich das gleiche Muster am Schirm hinter einem Doppelspalt ein. Wofür brauche ich Vielwelten, wenn sich in allen Vielwelten im statistischen Mittel das gleiche Bild ergibt?

Hmm ... schwer zu sagen, da es ja nur um eine Interpretation geht. Vielleicht zur konsistenteren Interpretation von Fällen, in denen die alternativen Pfade eine zentralere ("realere") Bedeutung haben, z.B. in einem Quantencomputer?

smallie hat folgendes geschrieben:
Carroll hat eine vierte Strategie ausgelassen: in der Quantenwelt gelten andere Regeln als in Alltagswelt.

Das ist doch bereits trivialerweise durch Hilbertraum und Schrödingergleichung gegeben.

#224:  Autor: smallie BeitragVerfasst am: 31.10.2017, 19:37
    —
step hat folgendes geschrieben:
smallie hat folgendes geschrieben:
Insbesondere sind keine Mikrozustände verloren gegangen. Mikrozustände gehen nicht einfach so verloren. Das wäre ja ganz was neues. Oops. Mit dem Argument ist man schon oft auf die Nase gefallen. Hier halte ich es für richtig.

Deswegen schrieb ich ja oben, daß die Entropie des Gesamtsystems nicht geringer wird.

Das kann man nicht oft genug sagen. zwinkern Erst war die Entropie ein Gedanke aus der Gastheorie. Dann tauchte sie plötzlich bei der Schwarzkörperstrahlung auf und brachte Einstein auf die Quantentheorie.

Bildlich gefragt: warum strahlt ein Atom in einem rotglühenden Körper nicht einfach in einem einzigen Gammablitz ab?
Bildlich geantwortet: weil der Gammablitz eine hohe Ordnung besäße und die Entropie abgenommen hätte.



step hat folgendes geschrieben:
smallie hat folgendes geschrieben:
Halt, nein. Eigentlich ist es eine schlechte Antwort, weil "Apparat" etwas makroskopisches meint und genau das ist es nicht.
Praktisches Beispiel: der Doppelspaltversuch klappt mit Photonen wie mit Elektronen. Bei Photonen klappt er in Luft, weil Luft für Photonen einigermaßen durchsichtig ist. Bei Elektronen braucht es ein Vakuum, sonst wechselwirken die Elektronen mit den Luftmolekülen.

In der Tat, und dieser Meinung ist ganz sicher auch Carroll.

Carroll spricht ausdrücklich von "we just mean a macroscopic measuring apparatus". Warum schreibt er das, wenn er es besser weiß?



step hat folgendes geschrieben:
Sogar die makroskopische Wirkung der "Umgebung" in der Dekohärenztheorie und Superselektion beruhen auf mikroskopischen Wechselwirkungen. Genaugenommen ist die Dekohärenz daher unvereinbar mit der (ursprünglichen) Kopenhagener Deutung, da letztere a priori einen makroskopisch wirkenden Meßapparat annahm, da nur ein solcher einen Kollaps verursachen könne.

Fair enough.

Die Vorstellung vom Kollaps, Stand 1920er oder 30er Jahre, hat sich überholt.



step hat folgendes geschrieben:
smallie hat folgendes geschrieben:
Das Standardmodell funktioniert, wie auch die Schrödingergleichung, ohne daß ich auf Interpretationen wie Kopenhagen, Vielwelttheorie, Bohmsche Mechanik, GWR zurückgreifen muß.

Genau, shut up and calculate.

Solange man eine Abbruchbedingung kennt, passt das. Sabine Hossenfelder meint allerdings, wir seien Lost in Math.



step hat folgendes geschrieben:
smallie hat folgendes geschrieben:
step hat folgendes geschrieben:
Ich verstehe es so, daß das Meßgerät p.d. zu Beginn nicht verschränkt mit dem Objektzustand ist.
Beim bisherigen Wissensstand geht das durch. Aber: wenn Gravitation quantisiert ist und wenn Masseteilchen mit allen anderen wechselwirken, dann läuft doch dauernd ein Meßapparat, der zum kontinuierlichen Kollaps oder zu kontinuierlicher Verschränkung führt. Süffisant merke ich an, daß die beiden unterschiedlichen Sprechweisen in der Praxis auf die selben Beobachtungen hinauslaufen.

Ja, aber Vorsicht: Eine solche Verschränkung gälte bezüglich eines "anderen Hilbertraums". Sie hätte Auswirkungen, aber nur in ganz anderen Experimenten - etwa einer Art gravitativem Doppelspaltexperiment, das man evtl. mit einem Schwarzen Loch oder einem kleinen Urknall durchführen könnte.

Das ist interessant. Ich spiele mal durch, was mir dazu einfällt.

- der erste Gedanke war: kann nicht sein. Nach einer Messung sind alle anderen Quantenzustände unbestimmt, außer dem gemessenen. Dann wären die elektrodynamischen Quantenzustände ja unabhängig von den chromodynamischen und den hypothetischen gravitativen.

- der zweite Gedanke war: kann sehr wohl sein. Einem Elektron oder einem Photon ist die Chromodynamik egal. Andere Teilchenfamilie. Sollten die elektrodynamischen Quantenzustände unabhängig sein von den chromodynamischen und den hypothetischen gravitativen?

Ich tendiere zu 1), bin aber gerade verwirrt. Wofür oder wogegen das jetzt ein Argument ist, muß ich mir auch noch überlegen.



step hat folgendes geschrieben:
smallie hat folgendes geschrieben:
Behauptung: in allen Vielwelten stellt sich das gleiche Muster am Schirm hinter einem Doppelspalt ein. Wofür brauche ich Vielwelten, wenn sich in allen Vielwelten im statistischen Mittel das gleiche Bild ergibt?

Hmm ... schwer zu sagen, da es ja nur um eine Interpretation geht. Vielleicht zur konsistenteren Interpretation von Fällen, in denen die alternativen Pfade eine zentralere ("realere") Bedeutung haben, z.B. in einem Quantencomputer?

Ja, das wird noch spannend.


PS:

Um 49 qubits im Rechner zu simulieren, braucht es einige Terabyte an Speicher. Diese fiesen Feynman-Diagramme. Sehen so einfach aus - und dann das. Motzen

Zitat:
2^n is exponential, but 2^50 is finite

Using the methods presented in this paper, the above simulations required 4.5 and 3.0 TB of memory, respectively, to store calculations, which is well within the limits of existing classical computers.

https://www.scottaaronson.com/blog/?p=3512

#225:  Autor: zelig BeitragVerfasst am: 31.10.2017, 19:42
    —
smallie hat folgendes geschrieben:
Dann tauchte sie plötzlich bei der Schwarzkörperstrahlung auf und brachte Einstein auf die Quantentheorie


Echt Einstein?

#226:  Autor: smallie BeitragVerfasst am: 31.10.2017, 19:47
    —
step hat folgendes geschrieben:
smallie hat folgendes geschrieben:
Carroll hat eine vierte Strategie ausgelassen: in der Quantenwelt gelten andere Regeln als in Alltagswelt.

Das ist doch bereits trivialerweise durch Hilbertraum und Schrödingergleichung gegeben.

Mensch, step, das ist ein Fall von Gegenwarts-Chauvinismus.

In den 1920er Jahren fand eine Physikerkonferenz statt. Einstein brachte jeden Tag einen neuen Einwand gegen die Quantentheorie vor, seine Kollegen konnten ihn regelmäßig dagegenhalten. Daraus entstand das Einstein-Podolski-Rosen-Papier.

Es dauerte eine Weile, bis man das Problen experimentell in den Griff bekam und es auch formal besser verstand. Siehe Bell und seine Ungleichung. Und vieles andere mehr.

Ob Schrödinger diese Folgerungen aus seiner Gleichung geahnt hat? Ich bezweifle es.




Grundsatzfrage:

Sean Carroll hat folgendes geschrieben:
The final strategy is acceptance. That is the Everettian approach. The formalism of quantum mechanics, in this view, consists of quantum states as described above and nothing more, which evolve according to the usual Schrödinger equation and nothing more.

Ich habe den Ansatz von Everett immer so verstanden, daß Einzellösungen der Wellengleichung nach der Messung in kausal voneinander getrennten Quantenuniversen weiter existieren.

Sagt Carroll ja auch:

Sean Carroll hat folgendes geschrieben:
Everett, by contrast, says that the universe splits in two: in one the cat is awake, and in the other the cat is asleep. Once split, the universes go their own ways, never to interact with each other again.

Aber die Schrödingergleichung enthält keinen Term, der erklären könnte, warum die Quantenmultiversen kausal getrennt sein sollten.

#227:  Autor: smallie BeitragVerfasst am: 31.10.2017, 20:01
    —
zelig hat folgendes geschrieben:
smallie hat folgendes geschrieben:
Dann tauchte sie plötzlich bei der Schwarzkörperstrahlung auf und brachte Einstein auf die Quantentheorie


Echt Einstein?

Guten Morgen zelig. Willkommen im Thread. Seit wann liest du mit? Auf den Arm nehmen


Steht ein paar Beiträge weiter oben.

smallie hat folgendes geschrieben:
Quantenmechanik entstand aus einer Analogieüberlegung zur Entropie. ... Siehe Einsteins Aufsatz Über einen die Erzeugung und Verwandlung des Lichtes betreffenden heuristischen Gesichtspunkt.

Damals konnte man sich das Spektrum der Schwarzkörperstrahlung nicht erklären. Planck hatte eine empirische Formel dafür erraten, von Wilhelm Wien stammte ein andere Formel. Einstein sah die Formel von Wien, erkannte eine Ähnlichkeit zu Boltzmanns Entropie s = k log W aus der Theorie der kinetischen Gase. Kühn behauptet er, daß es Energiequanten geben müsse, analog zu den Teilchen aus der Gastheorie.

#228:  Autor: zelig BeitragVerfasst am: 31.10.2017, 20:05
    —
smallie hat folgendes geschrieben:
Guten Morgen zelig. Willkommen im Thread. Seit wann liest du mit? Auf den Arm nehmen


Guten Morgen! Die ganze Zeit eigentlich. : )

#229:  Autor: stepWohnort: Germering BeitragVerfasst am: 01.11.2017, 00:02
    —
smallie hat folgendes geschrieben:
step hat folgendes geschrieben:
smallie hat folgendes geschrieben:
Carroll hat eine vierte Strategie ausgelassen: in der Quantenwelt gelten andere Regeln als in Alltagswelt.
Das ist doch bereits trivialerweise durch Hilbertraum und Schrödingergleichung gegeben.
... Siehe Bell und seine Ungleichung. Und vieles andere mehr. Ob Schrödinger diese Folgerungen aus seiner Gleichung geahnt hat? Ich bezweifle es.

Diese Folgerung wohl nicht. Aber daß in der Quantenwelt andere Regeln gelten als in der Alltagswelt? Z.B. die Rolle der Wahrscheinlichkeit, die Kommutationsrelationen, diskrete Eigenwerte, Spin, Unschärferelation usw?

smallie hat folgendes geschrieben:
Ich habe den Ansatz von Everett immer so verstanden, daß Einzellösungen der Wellengleichung nach der Messung in kausal voneinander getrennten Quantenuniversen weiter existieren.

Genau.

smallie hat folgendes geschrieben:
... Aber die Schrödingergleichung enthält keinen Term, der erklären könnte, warum die Quantenmultiversen kausal getrennt sein sollten.

Doch. Wenn man die SG der Wellenfunktion des Gesamtsystems (!) löst, entstehen u.a. automatisch Dekohärenzterme, und die Wellenfunktion evolviert in eine mit 2 kausal getrennten Anteilen. Dazu wird wirklich nur die SG benötigt. Es ist eigentlich erstaunlich, daß die Dekohärenztheorie erst in den 70-ern gefunden wurde, vermutlich war man vorher einfach zu sehr vernarrt in die Objekt/Meßgerät-Dichotomie und den Kollaps.

smallie hat folgendes geschrieben:
Carroll spricht ausdrücklich von "we just mean a macroscopic measuring apparatus". Warum schreibt er das, wenn er es besser weiß?

Hmm ... keine Ahnung. Vielleicht meint er es obdA. zwinkern

smallie hat folgendes geschrieben:
step hat folgendes geschrieben:
Sogar die makroskopische Wirkung der "Umgebung" in der Dekohärenztheorie und Superselektion beruhen auf mikroskopischen Wechselwirkungen. Genaugenommen ist die Dekohärenz daher unvereinbar mit der (ursprünglichen) Kopenhagener Deutung, da letztere a priori einen makroskopisch wirkenden Meßapparat annahm, da nur ein solcher einen Kollaps verursachen könne.
Fair enough. ... Die Vorstellung vom Kollaps, Stand 1920er oder 30er Jahre, hat sich überholt.

Was würdest Du dann als das Wesentliche an der heutigen Kopenhagener Deutung ansehen, wenn es nicht der Kollaps ist? Die Dekohärenz ist ja nicht spezifisch für Kopenhagen. An welcher Stelle wird von der Wellenfunktion plötzlich nur noch eine Hälfte als real angesehen?

smallie hat folgendes geschrieben:
... dann wären die elektrodynamischen Quantenzustände ja unabhängig von den chromodynamischen und den hypothetischen gravitativen ...

Nach dem Standardmodell ist das auch so, wenn ich es richtig verstanden habe. Die erzeugenden Gruppen sind Produkte. Wenn z.B ein Teilchen elektromagnetisch gemessen wird, kann es danach immer noch in einem Isospin-Mischzustand sein.

smallie hat folgendes geschrieben:
Um 49 qubits im Rechner zu simulieren, braucht es einige Terabyte an Speicher. Diese fiesen Feynman-Diagramme. Sehen so einfach aus - und dann das. ...

Genau. Diese Pfade kann man nicht einfach rauslassen. So scheint es doch ziemlich naheliegend anzunehmen, daß die Pfade alle gleich (oder zumindest anteilig) "real" sind und die Physik sozusagen die einfachste Realisierung ihrer selbst ist, oder?

#230:  Autor: smallie BeitragVerfasst am: 01.11.2017, 19:41
    —
zelig hat folgendes geschrieben:
smallie hat folgendes geschrieben:
Guten Morgen zelig. Willkommen im Thread. Seit wann liest du mit? Auf den Arm nehmen


Guten Morgen! Die ganze Zeit eigentlich. : )

*grummel*

Muß kürzere Beiträge schreiben. Und nicht mehr drei Gedanken in einen stopfen.

#231:  Autor: smallie BeitragVerfasst am: 01.11.2017, 19:43
    —
step hat folgendes geschrieben:
smallie hat folgendes geschrieben:
... Aber die Schrödingergleichung enthält keinen Term, der erklären könnte, warum die Quantenmultiversen kausal getrennt sein sollten.

Doch. Wenn man die SG der Wellenfunktion des Gesamtsystems (!) löst, entstehen u.a. automatisch Dekohärenzterme, und die Wellenfunktion evolviert in eine mit 2 kausal getrennten Anteilen. Dazu wird wirklich nur die SG benötigt.

Wenn ich zuerst Quantenparameter A messe und dann B kommt etwas anderes heraus, als wenn ich zuerst B messe und dann A.

Falls du das meinst, weiß ich was du meinst.



step hat folgendes geschrieben:
Was würdest Du dann als das Wesentliche an der heutigen Kopenhagener Deutung ansehen, wenn es nicht der Kollaps ist? Die Dekohärenz ist ja nicht spezifisch für Kopenhagen.

Zur heutigen Deutung kann ich nichts sagen. Bin schon froh, wenn ich perturbative theory buchstabieren kann. zwinkern


Die herkömmliche formale Erklärung ist das Diracsche delta, das im Moment der Messung eine Amplitude aus der Überlagerung herauspickt. Dieses delta ist eine Funktion, die wie eine extreme Spitze aussieht, infinitesimal schmal und unendlich hoch, dabei aber mit einer 1 als Gesamtfläche.

Ich könnte einen ganzen Absatz schreiben mit Überlegungen und Einwänden dazu. Natürlich kann ich mathematisch einen Punkt aus einer Funktion herausgreifen, ohne daß etwas kollabiert, ich muß den Punkt nur in all seinen Ableitungen kennen, dann steckt da immer noch die ganze Information der Kurve drin. Auf die Physik scheint mir das nicht übertragbar zu sein.

Bevor ich mich mit solchem bla-bla weiter aus dem Fenster lehne, sollte ich nachlesen, wie die Funktion in den Lehrbüchern eingeführt wird. Ist sie begründet oder fällt sie vom Himmel?



step hat folgendes geschrieben:
An welcher Stelle wird von der Wellenfunktion plötzlich nur noch eine Hälfte als real angesehen?

Das muß die selbe Stelle sein, bei der EQM sagt, die eine Hälfte sei mit der anderen kausal nicht mehr verbunden. Pfeifen



step hat folgendes geschrieben:
smallie hat folgendes geschrieben:
... dann wären die elektrodynamischen Quantenzustände ja unabhängig von den chromodynamischen und den hypothetischen gravitativen ...

Nach dem Standardmodell ist das auch so, wenn ich es richtig verstanden habe. Die erzeugenden Gruppen sind Produkte. Wenn z.B ein Teilchen elektromagnetisch gemessen wird, kann es danach immer noch in einem Isospin-Mischzustand sein.

Nur ist der Isospin-Mischzustand jetzt ein anderer. In der Änderung stecken die Mikrozustände, die beim "Kollaps" scheinbar verlorengegangen sind.

Laß mich das Problem in Alltagssprache übersetzen:

Ich schreibe einen Brief, schreibe ihn entweder in Druck- oder in Schreibschrift, schreibe ihn mit schwarzer oder blauer Tinte, schreibe ihn auf glattes oder geprägtes Papier. Damit sind die Briefe in ihren Parametern verschränkt. Aus einer Laune heraus, zerreiße ich die Briefe in der Mitte und schicke die Hälften an meine treuen Leser s und z.

Weil es Quantenbriefe sind, können s und z nur eine Eigenschaft des Briefes messen, bevor er sich selbst zerstört. (Wir von der Einstein-Mafia haben sowas.)

s und z tauschen sich darüber aus, was sie bei den Briefhälften gemessen haben. Sie vermuten, daß ein Brief, wenn er einmal geschrieben wurde, seine Eigenschaften beibehält. Wenn bei s ein Viertel aller Briefe in Druckschrift war, ein anderes viertel in Schreibschrift, zwei weitere Viertel in schwarz und blau, dann muß sich bei z eine ähnliche Verteilung einstellen.

Leider stellt sich diese Verteilung nicht ein. s und z folgern, daß der Brief ein Quantenobjekt ist, für das andere Regeln gelten, als für klassische Briefe.


Was sagt EQM zu diesen nicht-klassischen Vorstellungen? Kann sie etwas zur Erklärung der gemessenen Wahrscheinlichkeiten beitragen, das nicht bereits in der Schrödingergleichung steckt?

#232:  Autor: stepWohnort: Germering BeitragVerfasst am: 01.11.2017, 21:50
    —
smallie hat folgendes geschrieben:
step hat folgendes geschrieben:
Wenn man die SG der Wellenfunktion des Gesamtsystems (!) löst, entstehen u.a. automatisch Dekohärenzterme, und die Wellenfunktion evolviert in eine mit 2 kausal getrennten Anteilen. Dazu wird wirklich nur die SG benötigt.
Wenn ich zuerst Quantenparameter A messe und dann B kommt etwas anderes heraus, als wenn ich zuerst B messe und dann A.

Bist Du da sicher? Ohne daß ich das jetzt nachgeprüft hätte, würde ich behaupten, daß für kommutierende Observablen (über die reden wir ja hier, oder?) die beiden Messungen unabhängig voneinander sind, wenn sie ausschließlich die jeweilige Wechselwirkung bestehen.

smallie hat folgendes geschrieben:
... wie die [Dirac'sche Delta-] Funktion in den Lehrbüchern eingeführt wird. Ist sie begründet oder fällt sie vom Himmel?

Die ist in anderen Kontexten begründet, etwa wenn man ein punktförmiges Teilchen modellieren will. Aber bei der Kopenhagener Deutung fällt sie vom Himmel - falls sie da überhaupt benutzt wird.

smallie hat folgendes geschrieben:
step hat folgendes geschrieben:
An welcher Stelle wird von der Wellenfunktion plötzlich nur noch eine Hälfte als real angesehen?
Das muß die selbe Stelle sein, bei der EQM sagt, die eine Hälfte sei mit der anderen kausal nicht mehr verbunden. Pfeifen

Ähm, nö. Dazu noch ein der multiversalen Ketzerei eher unverdächtiger Autor: "Physik ohne Realität: Tiefsinn oder Wahnsinn?" von H. Dieter Zeh.

smallie hat folgendes geschrieben:
step hat folgendes geschrieben:
Wenn z.B ein Teilchen elektromagnetisch gemessen wird, kann es danach immer noch in einem Isospin-Mischzustand sein.
Nur ist der Isospin-Mischzustand jetzt ein anderer. In der Änderung stecken die Mikrozustände, die beim "Kollaps" scheinbar verlorengegangen sind.

?? Beim "Kollaps" sind keine Mikrozustände scheinbar verlorengegangen, die sich auf den Isospin-Raum beziehen.

smallie hat folgendes geschrieben:
... Weil es Quantenbriefe sind, können s und z nur eine Eigenschaft des Briefes messen, bevor er sich selbst zerstört.

Wenn es geschickt gemacht wird, wird nicht der gesamte Zustand zerstört, sondern nur der Anteil dieser einen Eigenschaft. Eine bestimmte Messung (oder auch ein bestimmter Dekohärenztyp) wechselwirkt gemäß SG nur mit bestimmten Anteilen der Wellenfunktion, die von anderen Teilen abfaktorisiert sind.

smallie hat folgendes geschrieben:
Was sagt EQM zu diesen nicht-klassischen Vorstellungen? Kann sie etwas zur Erklärung der gemessenen Wahrscheinlichkeiten beitragen, das nicht bereits in der Schrödingergleichung steckt?

Ich würde sagen nein. Der einzige mir bekannte Vorteil der EQM liegt wie gesagt darin, daß sie als einzige Interpretation nichts zusätzlich in die SG steckt, sondern diese einfach ernstnimmt.

#233:  Autor: smallie BeitragVerfasst am: 03.11.2017, 23:49
    —
step hat folgendes geschrieben:
smallie hat folgendes geschrieben:
step hat folgendes geschrieben:
Wenn man die SG der Wellenfunktion des Gesamtsystems (!) löst, entstehen u.a. automatisch Dekohärenzterme, und die Wellenfunktion evolviert in eine mit 2 kausal getrennten Anteilen. Dazu wird wirklich nur die SG benötigt.
Wenn ich zuerst Quantenparameter A messe und dann B kommt etwas anderes heraus, als wenn ich zuerst B messe und dann A.

Bist Du da sicher?

Ja, ziemlich sicher. Naja, Stand 1960 sicher.

Die Überlegung ist der Ausgangspunkt für das Briefbeispiel. Etwas abstrakter sieht das so aus:

smallie hat folgendes geschrieben:
F S Z FS FZ SZ

0 0 0 G G G
0 0 1 G U U
0 1 0 U G U
0 1 1 U U G
1 0 0 U U G
1 0 1 U G U
1 1 0 G U U
1 1 1 G G G

Falls das nicht stimmt, muß ich nochmal ganz von vorne anfangen. So mancher Lehrbuchautor auch.



step hat folgendes geschrieben:
Ohne daß ich das jetzt nachgeprüft hätte, würde ich behaupten, daß für kommutierende Observablen (über die reden wir ja hier, oder?) die beiden Messungen unabhängig voneinander sind, wenn sie ausschließlich die jeweilige Wechselwirkung bestehen.

Kommutierende Observable ist mir ein Fremdwort zu viel. Und es ist nicht Observable.


Elektronen lassen sich nicht von Quantenzuständen wie strange oder charm beeinflussen. Insofern könnte ich mir gerade noch vorstellen, daß eine Messung auf strange, charm, up bei einem Elektron nichts kollabieren oder dekoherieren läßt.

Die Sache wird deutlicher, wenn ich ein hypothetisches Teilchen betrachte: das Erebon von Roger Penrose. Das ist ein Teilchen, das nur gravitativ wechselwirkt.

Macht es Sinn zu sagen:

    a) eine Messung der elektromagnetischen Wechselwirkung zwischen Erobon und Elektron hat den gravitativen Teil der Wellenfunktion des Erebons nicht verändert.

    b) es fand gar keine Messung statt, die die Wellenfunktion hätte verändern können, weil keine Wechselwirkung besteht.

Beides sollte in der Praxis auf das gleiche hinauslaufen. Satz b) halte ich aus ästhetischen Überlegungen für richtiger.




GEGENÜBERLEGUNG

Es ist mir nicht Wohl bei dem Gedanken, daß Licht durch Luft fliegt und überhaupt keine Wechselwirkung mit den Elektronen der Atomhüllen erfährt. Wesentlich für das Entstehen einer Interferenz im Doppelspaltversuch ist, daß der Weg des Teilchens nicht bekannt sein darf. Weil da zum Beispiel gerade ein Staubteilchen herumgeflogen ist.

Sichtbares Licht ist kurzwellig genug, um ein Staubteilchen zu detektieren. Bei einem Doppeltspaltversuch mit Radiowellen könnte ich mich in den Strahlgang stellen und würde die Messung nicht wesentlich stören.

Mit anderen Worten:

Große Wellenlängen haben niedrige Energie. Wenn die Wechselwirkung zwischen Photon und Elektron so schwach ist, daß die zugehörige Wellenlänge des Austauschpartikels das Photon nicht mehr genau lokalisieren kann, dann ist eine Wechselwirkung erlaubt. (Da ist eine kleine Schummelei versteckt, die wir noch klären müßten.)


Zurück zur Verschränkung über Gravitonen. Wenn die gravitative Wechselwirkung zwischen zwei Quantenobjekten klein genug ist, dann geht damit kein Kollaps oder keine Dekoheränz einher. So zumindest im Sinn der Gegenüberlegung.



step hat folgendes geschrieben:
smallie hat folgendes geschrieben:
... wie die [Dirac'sche Delta-] Funktion in den Lehrbüchern eingeführt wird. Ist sie begründet oder fällt sie vom Himmel?

Die ist in anderen Kontexten begründet, etwa wenn man ein punktförmiges Teilchen modellieren will.

Auch recht. Läuft auf das selbe hinaus.

Punktförmiges Teilchen führt sofort zu einigen Grundsätzen der QM. Die Wahrscheinlichkeit, das Teilchen an einem bestimmten Ort zu finden, ist dann null. Also muß man von einem Bereich sprechen, in dem das Teilchen mit einer bestimmten Wahrscheinlichkeit anzutreffen ist. Das gilt analog für alle anderen Quantenparameter.

fait accompli



step hat folgendes geschrieben:
smallie hat folgendes geschrieben:
step hat folgendes geschrieben:
An welcher Stelle wird von der Wellenfunktion plötzlich nur noch eine Hälfte als real angesehen?
Das muß die selbe Stelle sein, bei der EQM sagt, die eine Hälfte sei mit der anderen kausal nicht mehr verbunden. Pfeifen

Ähm, nö. Dazu noch ein der multiversalen Ketzerei eher unverdächtiger Autor: "Physik ohne Realität: Tiefsinn oder Wahnsinn?" von H. Dieter Zeh.

Du stimmst dieser Darstellung also nicht zu?

Sean Carroll hat folgendes geschrieben:
Textbook quantum mechanics says that opening the box and observing the cat “collapses the wave function” into one of two possible measurement outcomes, awake or asleep. Everett, by contrast, says that the universe splits in two: in one the cat is awake, and in the other the cat is asleep. Once split, the universes go their own ways, never to interact with each other again.

http://www.preposterousuniverse.com/blog/2014/06/30/why-the-many-worlds-formulation-of-quantum-mechanics-is-probably-correct/


Was Ketzerei betrifft: Eine These, die ohne Ketzerei auskommt, kann nicht die richtige sein. Aber nicht jede Ketzerei ist richtig.



step hat folgendes geschrieben:
smallie hat folgendes geschrieben:
step hat folgendes geschrieben:
Wenn z.B ein Teilchen elektromagnetisch gemessen wird, kann es danach immer noch in einem Isospin-Mischzustand sein.
Nur ist der Isospin-Mischzustand jetzt ein anderer. In der Änderung stecken die Mikrozustände, die beim "Kollaps" scheinbar verlorengegangen sind.

?? Beim "Kollaps" sind keine Mikrozustände scheinbar verlorengegangen, die sich auf den Isospin-Raum beziehen.

Mein Fehler, meine Dummheit. Den Isospin habe ich einfach nachgeplappert, ohne zu wissen, was das ist.



step hat folgendes geschrieben:
smallie hat folgendes geschrieben:
Was sagt EQM zu diesen nicht-klassischen Vorstellungen? Kann sie etwas zur Erklärung der gemessenen Wahrscheinlichkeiten beitragen, das nicht bereits in der Schrödingergleichung steckt?

Ich würde sagen nein. Der einzige mir bekannte Vorteil der EQM liegt wie gesagt darin, daß sie als einzige Interpretation nichts zusätzlich in die SG steckt, sondern diese einfach ernstnimmt.

Dann nochmal zum Quantencomputer.


step hat folgendes geschrieben:
smallie hat folgendes geschrieben:
Um 49 qubits im Rechner zu simulieren, braucht es einige Terabyte an Speicher. Diese fiesen Feynman-Diagramme. Sehen so einfach aus - und dann das. ...

Genau. Diese Pfade kann man nicht einfach rauslassen. So scheint es doch ziemlich naheliegend anzunehmen, daß die Pfade alle gleich (oder zumindest anteilig) "real" sind und die Physik sozusagen die einfachste Realisierung ihrer selbst ist, oder?

Anteilig real. Genau.

zelig wollte eine Lösung für das Doppelspaltexperiment. Ein Baustein dazu ist das obige Beispiel vom breiten und vom schmalen Einzelspalt. Das läßt sich durch die Annahme erklären, ein Photon nimmt jeden möglichen Pfad durch den Spalt. Also wirklich jeden, auch absurd anmutende. Am Ende werden die möglichen Pfade addiert und die Wahrscheinlichen sehen wir dann. Die unwahrscheinlichen Pfade fallen heraus, weil sich dabei die Phasen der Photonen abschwächen oder aufheben. Das müßte ich jetzt graphisch aufzeichnen, damit's verständlich wird. Leider habe ich keinen Quanten-Raytracer zu Hand. Aber es gibt Experimente, die die Realität der Pfade zeigen, also Reflexion am Spiegel versus Reflexion am Gitter.

Langer Rede kurzer Sinn: jeder Pfad wird gebraucht, damit das richtige Ergebnis herauskommt.



Zurück zum Speicherverbrauch. Kann es sein, daß wir beide da etwas übersehen haben?

Für 49 qubits brauche ich 2^49 bits. Siehe die Darstellung mit den Einsen und Nullen. Falls ich mich nicht verrechnet habe, sind 2^49 bits mehr als 5 TB. Eher so 70 TB. Stimmt das?

#234:  Autor: smallie BeitragVerfasst am: 05.11.2017, 23:48
    —
smallie hat folgendes geschrieben:
zelig wollte eine Lösung für das Doppelspaltexperiment. ... Das läßt sich durch die Annahme erklären, ein Photon nimmt jeden möglichen Pfad durch den Spalt. Also wirklich jeden, auch absurd anmutende. ... Das müßte ich jetzt graphisch aufzeichnen, damit's verständlich wird. Leider habe ich keinen Quanten-Raytracer zu Hand.

Tsk. Wofür brauche ich einen Quanten-Raytracer, wenn es das Internet gibt? Bildersuche nach quantum theory reflexion mirror liefert passendes. Stack Exchange: Explain reflection laws at the atomic level


Dargestellt ist eine Lichtquelle S und ein Empfänger P, dazwischen ein Sichtschutz Q.

Die Lichtstrahlen treffen auf einen Spiegel, der in Segmente unterteilt ist. Zu jedem Segment ist ein Lichtstrahl eingezeichnet.

Die Kurve darunter ist die Zeit beziehungsweise die Strecke, die ein Lichtstrahl zurücklegen muß. Der Lichtstrahl mit der kürzesten Zeit ist jener, für den Einfallswinkel gleich Ausfallswinkel ist. Das ist das oben erwähnte Minimalprinzip.

Die Pfeile darunter stellen die Phasenlage der einzelnen Photonen dar. Nee. Falsch. Das Bild gilt auch für ein einzelnes Photon. Also besser die Komponenten einer Superposition. Die Länge der Pfeile stellt ihre Wahrscheinlichkeit dar. (Kann keine Längenunterschiede erkennen. Hmm.) Addiert man sie, erhält man eine Resultierende.



(Leider kann ich die Resultierende gerade nicht anschaulich ins Bild rückübersetzen.)


Wie real sind die Pfade, die nicht Einfallswinkel = Ausfallswinkel gehorchen?

Ich nehme eine Feile oder einen Stanzer, und mach Teile des Spiegels unbrauchbar. A, C, E, G, ... sind dann blind, darunter auch der sweet spot. Gemäß der klassischen Vorstellung sollte ich dann nichts mehr sehen.

Ein Alltagsbeispiel für einen derartig gestanzten Gegenstand:



Hoppla. Sieht man ja doch was.

#235:  Autor: stepWohnort: Germering BeitragVerfasst am: 06.11.2017, 19:33
    —
Ach smallie, alles sehr interessant was Du so schreibst - aber einiges davon kann ich gar nicht beantworten und anderes nicht einfach so aus dem Wissen, sondern muß erstmal selber überlegen, nachlesen usw. zwinkern

Nur Weniges kann ich - denke ich - spontan beantworten, z.B. dieses hier:

smallie hat folgendes geschrieben:
Zurück zum Speicherverbrauch. Kann es sein, daß wir beide da etwas übersehen haben? Für 49 qubits brauche ich 2^49 bits. Siehe die Darstellung mit den Einsen und Nullen. Falls ich mich nicht verrechnet habe, sind 2^49 bits mehr als 5 TB. Eher so 70 TB. Stimmt das?

Du hast richtig gerechnet, aber vermutlich nicht bedacht, daß der Witz bei den neueren Veröffentlichungen gerade darin besteht, die Implementation zu optimieren. Eine der wesentlichen Techniken dabei ist das Aufteilen des (z.B. zufällig gewählten) Gesamtarrays in Tensoren niedrigerer Ordnung, wie z.B. hier beschrieben:
https://arxiv.org/abs/1710.05867

Man nutzt quasi zusätzlich die Information über das spezifische Problem.

Hier noch mit schönem Bildchen der "Borstenpinsel"-Technik:
https://www.ibm.com/blogs/research/2017/10/quantum-computing-barrier/

#236:  Autor: stepWohnort: Germering BeitragVerfasst am: 06.11.2017, 20:16
    —
Weiter geht's mit der Optik:

smallie hat folgendes geschrieben:
[Cornu-Spirale] (Leider kann ich die Resultierende gerade nicht anschaulich ins Bild rückübersetzen.)

Ihre Amplitude entspricht der resultierenden Lichtwelle, ihre Richtung deren Phase. Da nur relative Phasen betrachtet werden, ist die Anfangsrichtung willkürlich gewählt, und damit auch die Richtung der Resultierenden. Man nimmt extra eine schiefe, damit niemand etwas hereininterpretiert. Die Phasen ändern sich zwischen den mittleren Wegen kaum, deswegen tragen auch nur die effektiv zur Summe bei. In diesem Sinne würde ich das Folgende gern anders deuten:

smallie hat folgendes geschrieben:
Der Lichtstrahl mit der kürzesten Zeit ist jener, für den Einfallswinkel gleich Ausfallswinkel ist. Das ist das oben erwähnte Minimalprinzip.

Man könnte sagen, das Minimalprinzip ist eigentlich ein Prinzip der Phasenkonstanz. Diese Idee hat übrigens Feynman, in etwas verallgemeinerter Form, eingeführt, worauf ich weiter oben schonmal kurz hingewiesen habe.

smallie hat folgendes geschrieben:
Wie real sind die Pfade, die nicht Einfallswinkel = Ausfallswinkel gehorchen?

Die sind laut Schrödingergleichung ebenso real. Und Feynman hat versucht plausibel zu machen, daß sie es bei der klassischen Optik eigentlich auch schon sind, jedenfalls wenn man Huygens Modell nimmt. Es ist ja so, daß zumindest die nahe der Mitte ziemlich real sein müssen, da sie ja wesentlich zur Amplitude beitragen. Noch klarer zu sehen ist das bei einer Sammellinse. Die müßte sonst ja gar keine flacheren Seiten haben zwinkern
EDIT: Das mit der Linse ziehe ich zurück, das war zu einfach.

smallie hat folgendes geschrieben:
Ich nehme eine Feile oder einen Stanzer, und mach Teile des Spiegels unbrauchbar. A, C, E, G, ... sind dann blind, darunter auch der sweet spot. Gemäß der klassischen Vorstellung sollte ich dann nichts mehr sehen.

Wieso das? Die Spirale sieht dann zwar gröber aus, aber hat immer noch eine Resultierende, weil die Phasen sich z.B. zwischen F und H immer noch weniger ändern als woanders. Jedenfalls wenn die Segmente keine zu falsche Größe haben.

smallie hat folgendes geschrieben:
Ein Alltagsbeispiel für einen derartig gestanzten Gegenstand: ... https://en.wikipedia.org/wiki/Diffraction_grating ...
Hoppla. Sieht man ja doch was.

? Das ist doch ein ganz anderer Effekt.


Zuletzt bearbeitet von step am 09.11.2017, 19:00, insgesamt einmal bearbeitet

#237:  Autor: zelig BeitragVerfasst am: 09.11.2017, 09:29
    —
Für Leute wie mich ist das eine klasse Seite:
http://scienceblogs.de/hier-wohnen-drachen/

Im hiesigen Kontext das:
http://scienceblogs.de/hier-wohnen-drachen/2012/10/10/quantenmechanik-und-realitat/
Zitat:
Damit eins ganz klar ist: in diesem Text geht es um die philosophischen Implikationen der Quantenmechanik – was können wir daraus über das Wesen der Realität folgern? Hierzu gibt es unterschiedliche Ansichten. Über eins sind sich aber alle, die etwas von Quantenmechanik verstehen, einig (auch – na klar – Florian und ich): die Quantenmechanik taugt nicht dazu, irgendwelche esoterischen Konzepte der Art “Wenn man nur fest genug dran glaubt” zu stützen. Für alle praktischen Zwecke ist es egal, welcher Interpretation der Quantenmechanik ihr anhängt und welches Bild von der Realität ihr euch macht – ihr könnt euch trotzdem keine Reichtümer her- oder Krankheiten wegwünschen. (Gerade in dieser Woche gab es ein Interview im Zeitmagazin, wo irgendeine Schauspielerin, die jetzt einen Film mit Woody Allan gedreht hat, sagt, dass es kein Glück war, sondern dass sie dank Ihrer Lektüre über Talumd und Quantenphysik (huh??) verstanden hat, dass sie ihr Ziel visualisieren musste, um es zu erreichen. Mich regen solche Aussagen auf – nicht nur wegen der damit verbundenen Dummheit (jeder kann ja so dumm sein, wie er will), sondern wegen des impliziten Vorwurfs an alle Menschen, die leiden, dass sie an ihrem Leiden selbst schuld sind. Sagt mal einer Mutter, deren Baby eine tödliche Krankheit hat, dass sie die Krankheit nur hätte rechtzeitig weg-visualisieren müssen…) Auch wenn es – wie ich gleich zeigen werde – eine Wahrscheinlichkeit dafür gibt, dass Objekte plötzlich ganz woanders sind oder sich in etwas anderes umwandeln, so ist diese Wahrscheinlichkeit so klein, dass sie für alle praktischen Zwecke Null ist. (Falls ihr es nicht glaubt, denkt immer dran: es gibt eine endliche Wahrscheinlichkeit, dass sich jedes Exemplar der neusten (4.!) Auflage des Buches “Mechanisches Verhalten der Werkstoffe” nächste Woche in pures Gold verwandelt. Besser, ihr sichert euch ein paar Exemplare, bevor es zu spät ist.)

Nachdem das somit hoffentlich ausgeräumt ist, möchte ich euch jetzt mit ein paar kleinen Gedankenexperimenten zeigen, wo die konzeptionellen und philosophischen Schwierigkeiten stecken, wenn man versucht zu argumentieren, dass Objekte auch dann “da sind”, wenn man sie nicht beobachtet.

#238:  Autor: stepWohnort: Germering BeitragVerfasst am: 09.11.2017, 19:44
    —
zelig hat folgendes geschrieben:
Im hiesigen Kontext das:
http://scienceblogs.de/hier-wohnen-drachen/2012/10/10/quantenmechanik-und-realitat/
Zitat:
...

Ja, das ist leicht zu lesen und gut verständlich.

Hier haben wir jedoch leider wieder den berüchtigten Kollpas, an den ja laut smallie seit Jahrzehnten keiner mehr glaubt:

scienceblogs.de/hier-wohnen-drachen hat folgendes geschrieben:
Wenn wir das Elektron dann wieder im Kasten messen, dann ... kann es logischerweise nicht außerhalb des Kastens sein. Die Wellenfunktion ändert sich also ... Man nennt das den “Kollaps” der Wellenfunktion – eben war sie noch an allen möglichen Orten ungleich Null, jetzt ist sie plötzlich überall Null, nur da nicht, wo wir das Elektron gemessen haben ... Erst wenn wir eine Messung durchführen, dann “realisieren” wir dadurch eine dieser Möglichkeiten ... wenn man sie misst, dann kollabiert die Wellenfunktion ... Dieser Kollaps der Wellenfunktion passiert “sofort” ... Die Wellenfunktion kollabiert also “unendlich schnell”.

Aber gut, es geht ja um etwas anderes.

scienceblogs.de/hier-wohnen-drachen hat folgendes geschrieben:
Die Wellenfunktion eines Teilchens kann auch im Zustand einer Überlagerung sein, in der das Teilchen seine Identität geändert hat. Ein Objekt also einfach mit “seiner” Wellenfunktion zu identifizieren, wird dadurch nicht gerade einfacher, oder?

Haha, die Identitätskrise des armen "Teilchens". Aber gut, er findet selbst den Weg raus.

Jetzt wird es interessant:

scienceblogs.de/hier-wohnen-drachen hat folgendes geschrieben:
Wenn der Wert der Wellenfunktion vom Beobachter abhängt, welches Recht habe ich dann zu behaupten, dass die Wellenfunktion ein reales physikalisches Objekt ist? ... Und wenn wir die Wellenfunktion als reales physikalisches Objekt ansehen, dann bekommen wir Schwierigkeiten mit dem Konzept der Kausalität – Messungen können sich dann für einige Beobachter “rückwärts in der Zeit” auswirken.

Das ist zwar ein berechtigter Einwand, aber eben nur unter der Annahme eines engen klassischen Realitätsbegriffs: so etwas wie "real = gleich für alle Beobachter", und da kommt man natürlich in den Konflikt mit der RT - auch ganz ohne Quanten.

Wenn man jedoch ein Konzept wählt für Wirklichkeit, daß mehr von der Wirkung errührt und von der minimalen Modellierung, dann geht es besser. Z.B. wie das von David Deutsch, was ich hier mal vorgestellt habe:

step, 2004 oder so hat folgendes geschrieben:
Aus meiner Sicht trifft es das folgende ganz gut:

Wenn eine Größe nach der einfachsten Erklärung komplex und autonom ist, dann bezeichnen wir sie als wirklich.

nach der einfachsten Erklärung komplex: Unsere Erklärungen für eine hinreichend komplexe Größe würden verkompliziert, wenn wir diese Größe nicht für wirklich halten. Deswegen halten wir z.B. die Planenten für wirklich, weil wir sonst viel kompliziertere Erklärungen für kosmische Planetarien, Engel, heimliche Gravitationssimulatoren oder sowas finden müßten.

autonom: Die Größe bezieht ihre Komplexität nicht von anderer Seite. Nur komplex reicht nicht, sonmst würde man bspw. ein Spiegelbild für wirklich halten. (Natürlich ist die Illusion selbst wiederum wirklich in diesem Sinne.) ...

Man könnte sogar sagen: gute Theorien beschreiben die Wirklichkeit. Und gute Theorien enthalten eben die (gesamte) Wellenfunktion. Dann kann man die Frage als sinnlos ansehen, ob nun die Wellenfunktion selbst real ist. Das ist dann tatsächlich schon nahe bei "shut up & calculate".

#239:  Autor: smallie BeitragVerfasst am: 09.11.2017, 20:52
    —
zelig hat folgendes geschrieben:
Für Leute wie mich ist das eine klasse Seite:
http://scienceblogs.de/hier-wohnen-drachen/

Ich mag den Titel des Blogs. There be Dragons - stand öfter mal auf alten Weltkarten.

Auf der Karte der Quantenmechanik schwimmen auch noch einige Drachen herum. Pfeifen



zelig hat folgendes geschrieben:
Im hiesigen Kontext das:

<schnipp>

Die sieben Seiten werde ich mir noch anschauen, erst möchte ich meine fast fertige Antwort an step zu Ende bringen.



step hat folgendes geschrieben:
Hier haben wir jedoch leider wieder den berüchtigten Kollpas, an den ja laut smallie seit Jahrzehnten keiner mehr glaubt:

Stand schon mal im kleingedruckten:

smallie hat folgendes geschrieben:
Die Vorstellung vom Kollaps gehört inzwischen zur Folklore, sie wird uns auf absehbare Zeit erhalten bleiben.




step hat folgendes geschrieben:
Und gute Theorien enthalten eben die (gesamte) Wellenfunktion. Dann kann man die Frage als sinnlos ansehen, ob nun die Wellenfunktion selbst real ist.

Sieht so aus, als würde die Frage auf die eine oder andere Weise demnächst, in ein oder zwei Jahrzehnten, beantworten werden, wenn es Quantencomputer gibt (oder nicht.)

#240:  Autor: stepWohnort: Germering BeitragVerfasst am: 09.11.2017, 22:25
    —
smallie hat folgendes geschrieben:
Ich mag den Titel des Blogs. There be Dragons - stand öfter mal auf alten Weltkarten.

Oder "HC SVNT DRACONES" auf noch älteren.

smallie hat folgendes geschrieben:
Auf der Karte der Quantenmechanik schwimmen auch noch einige Drachen herum. Pfeifen

Auf jeden Fall.

smallie hat folgendes geschrieben:
step hat folgendes geschrieben:
Hier haben wir jedoch leider wieder den berüchtigten Kollaps, an den ja laut smallie seit Jahrzehnten keiner mehr glaubt:
Stand schon mal im kleingedruckten:
smallie hat folgendes geschrieben:
Die Vorstellung vom Kollaps gehört inzwischen zur Folklore, sie wird uns auf absehbare Zeit erhalten bleiben.

Bei Folklore weiß man nie, wie ernst sie genommen wird. zwinkern

smallie hat folgendes geschrieben:
step hat folgendes geschrieben:
Und gute Theorien enthalten eben die (gesamte) Wellenfunktion. Dann kann man die Frage als sinnlos ansehen, ob nun die Wellenfunktion selbst real ist.
Sieht so aus, als würde die Frage auf die eine oder andere Weise demnächst, in ein oder zwei Jahrzehnten, beantworten werden, wenn es Quantencomputer gibt (oder nicht.)

Ja, und schön, daß wir das vielleicht noch erleben dürfen.

#241:  Autor: stepWohnort: Germering BeitragVerfasst am: 09.11.2017, 22:31
    —
step hat folgendes geschrieben:
zelig hat folgendes geschrieben:
Im hiesigen Kontext das:
http://scienceblogs.de/hier-wohnen-drachen/2012/10/10/quantenmechanik-und-realitat/
Zitat:
...
Ja, das ist leicht zu lesen und gut verständlich.

Auch die ART/Gravitationsthemen sind ganz gut.

#242:  Autor: smallie BeitragVerfasst am: 09.11.2017, 23:02
    —
step hat folgendes geschrieben:
smallie hat folgendes geschrieben:
[Cornu-Spirale] (Leider kann ich die Resultierende gerade nicht anschaulich ins Bild rückübersetzen.)

Ihre Amplitude entspricht der resultierenden Lichtwelle, ihre Richtung deren Phase. Da nur relative Phasen betrachtet werden, ist die Anfangsrichtung willkürlich gewählt, und damit auch die Richtung der Resultierenden. Man nimmt extra eine schiefe, damit niemand etwas hereininterpretiert. Die Phasen ändern sich zwischen den mittleren Wegen kaum, deswegen tragen auch nur die effektiv zur Summe bei.

Bei "Rückübersetzen" dachte ich eher an derartiges:

Der Punkt G liegt auf der Resultierenden, somit folgen daraus die klassischen Gesetze der Optik.



step hat folgendes geschrieben:
smallie hat folgendes geschrieben:
Der Lichtstrahl mit der kürzesten Zeit ist jener, für den Einfallswinkel gleich Ausfallswinkel ist. Das ist das oben erwähnte Minimalprinzip.

Man könnte sagen, das Minimalprinzip ist eigentlich ein Prinzip der Phasenkonstanz.

Du meinst Phasenkonstanz am Ort des Detektors? Ja, das gefällt mir.



step hat folgendes geschrieben:
Diese Idee hat übrigens Feynman, in etwas verallgemeinerter Form, eingeführt, worauf ich weiter oben schonmal kurz hingewiesen habe.

Wenn es die Phasenbetrachtung erst seit Feynman gibt, wundert es mich nicht, daß die Quantenmechanik vorher so völlig unverständlich erschien.



step hat folgendes geschrieben:
smallie hat folgendes geschrieben:
Wie real sind die Pfade, die nicht Einfallswinkel = Ausfallswinkel gehorchen?

Die sind laut Schrödingergleichung ebenso real. Und Feynman hat versucht plausibel zu machen, daß sie es bei der klassischen Optik eigentlich auch schon sind, jedenfalls wenn man Huygens Modell nimmt. Es ist ja so, daß zumindest die nahe der Mitte ziemlich real sein müssen, da sie ja wesentlich zur Amplitude beitragen. Noch klarer zu sehen ist das bei einer Sammellinse. Die müßte sonst ja gar keine flacheren Seiten haben zwinkern

Ziemlich real ist - ziemlich richtig.

Alle Anteile der Superposition werden gebraucht, um Phasenkonstanz herzustellen. Nimm einen Pfad heraus - dann fokussiert die Linse nicht mehr richtig.



step hat folgendes geschrieben:
smallie hat folgendes geschrieben:
Ich nehme eine Feile oder einen Stanzer, und mach Teile des Spiegels unbrauchbar. A, C, E, G, ... sind dann blind, darunter auch der sweet spot. Gemäß der klassischen Vorstellung sollte ich dann nichts mehr sehen.

Wieso das? Die Spirale sieht dann zwar gröber aus, aber hat immer noch eine Resultierende, weil die Phasen sich z.B. zwischen F und H immer noch weniger ändern als woanders. Jedenfalls wenn die Segmente keine zu falsche Größe haben.

Das ist nicht die klassische Lösung. Bei F und H ist Einfallswinkel gleich Ausfallswinkel verletzt.



step hat folgendes geschrieben:
smallie hat folgendes geschrieben:
Ein Alltagsbeispiel für einen derartig gestanzten Gegenstand: ... https://en.wikipedia.org/wiki/Diffraction_grating ...
Hoppla. Sieht man ja doch was.

? Das ist doch ein ganz anderer Effekt.

Falls ich folgendes richtig verstanden habe, passt die Vorstellung vom ausgestanzten Spiegel schon. Ungefähr ab 45:00, vorher wird obiges Diagramm eingeführt. Siehe insbesondere ab 48:30.

QED: Fits of Reflection and Transmission -- Quantum Behaviour -- Richard Feynman (2/4)

#243:  Autor: smallie BeitragVerfasst am: 09.11.2017, 23:30
    —
step hat folgendes geschrieben:
Ach smallie, alles sehr interessant was Du so schreibst - aber einiges davon kann ich gar nicht beantworten und anderes nicht einfach so aus dem Wissen, sondern muß erstmal selber überlegen, nachlesen usw. zwinkern

Macht ja nix. zwinkern

Um auf zeligs Startbeitrag zurückzukommen.

zelig hat folgendes geschrieben:
In einem Beitrag über das Quantenuniversum in der Spektrum habe ich gelesen, daß "alle möglichen Ausgänge eines Experiments in verschiedenen Zweigen der Realität verwirklicht werden."

Ich hätte mehr Vertrauen in Aussagen über "verschiedene Zweige der Realität", wenn die Vielzahl der Möglichkeiten durch ein Minimalprinzip beschränkt würde. Wenn überhaupt Farbe bekannt werden würde, was möglich ist und was nicht.

Die Idee vom permanenten Kollaps der Wellenfunktion durch gravitative Wechselwirkung wird in solchen Artikeln seltenst erwähnt. So geht das nicht, wenn man einen ergebnisoffenen, spekulativen Artikel schreiben will. nee Ergo: auch die Physik ist bisweilen ein Tendenzbetrieb mit Strömungen und Moden.



step hat folgendes geschrieben:
smallie hat folgendes geschrieben:
Zurück zum Speicherverbrauch. Kann es sein, daß wir beide da etwas übersehen haben? Für 49 qubits brauche ich 2^49 bits. Siehe die Darstellung mit den Einsen und Nullen. Falls ich mich nicht verrechnet habe, sind 2^49 bits mehr als 5 TB. Eher so 70 TB. Stimmt das?

Du hast richtig gerechnet, aber vermutlich nicht bedacht, daß der Witz bei den neueren Veröffentlichungen gerade darin besteht, die Implementation zu optimieren. Eine der wesentlichen Techniken dabei ist das Aufteilen des (z.B. zufällig gewählten) Gesamtarrays in Tensoren niedrigerer Ordnung, wie z.B. hier beschrieben:
https://arxiv.org/abs/1710.05867

"Die Implementation optimieren" - gerne. Aber was wird dann aus:

step hat folgendes geschrieben:
So scheint es doch ziemlich naheliegend anzunehmen, daß die Pfade alle gleich (oder zumindest anteilig) "real" sind und die Physik sozusagen die einfachste Realisierung ihrer selbst ist, oder?

Also gibt es doch eine einfachere Beschreibung der Quantenphysik?

#244:  Autor: zelig BeitragVerfasst am: 11.11.2017, 13:39
    —
step hat folgendes geschrieben:
autonom: Die Größe bezieht ihre Komplexität nicht von anderer Seite. Nur komplex reicht nicht, sonmst würde man bspw. ein Spiegelbild für wirklich halten. (Natürlich ist die Illusion selbst wiederum wirklich in diesem Sinne.) ...


Ich bin nicht so ganz einverstanden. Wenn ich Dich richtig verstehe, ist doch im Streit um die Interpretation der Wellenfunktion die Autonomie fraglich. In deinem Bild: Wir wissen nicht, ob wir in einen Spiegel schauen, oder in etwas, was wie ein Spiegel aussieht, jedoch den gleichen ontologischen Status hat, wie die alltägliche Erfahrungswelt. Es kann natürlich sein, daß ich den Autonomiebegriff hier nicht richtig verstehe.


step hat folgendes geschrieben:
Man könnte sogar sagen: gute Theorien beschreiben die Wirklichkeit. Und gute Theorien enthalten eben die (gesamte) Wellenfunktion. Dann kann man die Frage als sinnlos ansehen, ob nun die Wellenfunktion selbst real ist. Das ist dann tatsächlich schon nahe bei "shut up & calculate".


Kleiner Einwand. Wenn die Riemannsche Vermutung niemals bewiesen wird, aber die Zeta-Funktion würde ausnahmslos immer bestätigt werden. Wäre das nicht unbefriedigend? War es nicht Hilbert, der gesagt haben soll, daß er, wenn er nach 1000 Jahren erweckt werden würde, als erstes fragen würde, ob ein Beweis gelungen ist? Und müsste es einen Physiker (oder auch einen interessierten Philosophen) nicht genauso brennend interessieren, wie man den Kollaps zu deuten hat?

#245:  Autor: smallie BeitragVerfasst am: 11.11.2017, 14:02
    —
zelig hat folgendes geschrieben:

Im hiesigen Kontext das:
http://scienceblogs.de/hier-wohnen-drachen/2012/10/10/quantenmechanik-und-realitat/
Zitat:
Damit eins ganz klar ist: in diesem Text geht es um die philosophischen Implikationen der Quantenmechanik – was können wir daraus über das Wesen der Realität folgern?

Der Wesen der Realität? Die Realität ist zu einem großen Teil eine Quantenrealität.

Quantentheorie (QED) beschreibt die Physik der Atomhülle. Alles was mit Elektronen oder Photonen zu tun hat, ist Quantenrealität. Chemie ist eine Anwendung der Quantentheorie, ohne Quantentheorie kein Wasser, keine organischen Moleküle, kein Leben. Wenn unser Auge bestimmte Frequenzen wahrnimmt, liegt das daran, daß diese Frequenzen von Luft nur wenig gestreut werden: ein Quantenphänomen! Daß der Himmel blau ist - ein Quantenphänomen.

Dazu ein Satz von Philip Ball, Link folgt:

    Man könnte sagen, die klassische Welt ist ganz einfach, wie sich Quantenmechanik darstellt, wenn man einen Meter achtzig groß ist.

    You could say that the classical world is simply what quantum mechanics looks like if you are six feet tall.




Martin Bäker hat folgendes geschrieben:
Nachdem das somit hoffentlich ausgeräumt ist, möchte ich euch jetzt mit ein paar kleinen Gedankenexperimenten zeigen, wo die konzeptionellen und philosophischen Schwierigkeiten stecken, wenn man versucht zu argumentieren, dass Objekte auch dann “da sind”, wenn man sie nicht beobachtet.

Objekte? Hmm.

Das klingt, als möchte Martin Bäker eine klassische Vorstellung vom Objekt auf die Quantenwelt übertragen. Das ist der falsche Ansatz. Besser anders herum: bottom-up die Quantenmechanik zur Basis nehmen.



Martin Bäker hat folgendes geschrieben:
Nach der Standard-Interpretation der Quantenmechanik findet eine Messung durch eine Wechselwirkung mit einem “klassischen Objekt” statt, also einem Objekt, das hinreichend groß ist, um den Regeln der klassischen Physik zu genügen. Nach dieser Interpretation kann sich ein Objekt wie der Mond also sozusagen “selbst messen”, weil er groß genug ist. Das Problem dieser Deutung ist, dass sich der Mond ja aus lauter quantenmechanisch zu beschreibenden Einzelteilchen zusammensetzt, so dass konzeptionell nicht klar ist, wie so ein “klassisches Objekt” entstehen soll.

So vielleicht?

Zitat:
Quantum common sense
Philip Ball

Decoherence is a phenomenally efficient process, probably the most efficient one known to science. For a dust grain 100th of a millimetre across floating in air, it takes about 10-31 seconds: a million times faster than the passage of a photon of light across a single proton! Even in the near-isolation of interstellar space, the ubiquitous photons of the cosmic microwave background – the Big Bang’s afterglow – will decohere such a grain in about one second.

...

Zurek and his colleague Jess Riedel have been able to calculate how fast and extensive this proliferation of quantum copies is for a few simple situations, such as a dust speck in a vacuum flooded by sunlight. They find that, after being illuminated for just one microsecond, a grain of dust a micrometre across will have its location imprinted about 100 million times in the scattered photons.

https://aeon.co/essays/the-quantum-view-of-reality-might-not-be-so-weird-after-all


Übrigens dürfte die Masse eines Staubkorns grob bei einer Planck-Masse liegen. Zufall oder bedeutungsvoll?

#246:  Autor: SkeptikerWohnort: 129 Goosebumpsville BeitragVerfasst am: 11.11.2017, 15:43
    —
smallie hat folgendes geschrieben:
zelig hat folgendes geschrieben:

Im hiesigen Kontext das:
http://scienceblogs.de/hier-wohnen-drachen/2012/10/10/quantenmechanik-und-realitat/
Zitat:
Damit eins ganz klar ist: in diesem Text geht es um die philosophischen Implikationen der Quantenmechanik – was können wir daraus über das Wesen der Realität folgern?

Der Wesen der Realität? Die Realität ist zu einem großen Teil eine Quantenrealität.

Quantentheorie (QED) beschreibt die Physik der Atomhülle. Alles was mit Elektronen oder Photonen zu tun hat, ist Quantenrealität. Chemie ist eine Anwendung der Quantentheorie, ohne Quantentheorie kein Wasser, keine organischen Moleküle, kein Leben. Wenn unser Auge bestimmte Frequenzen wahrnimmt, liegt das daran, daß diese Frequenzen von Luft nur wenig gestreut werden: ein Quantenphänomen! Daß der Himmel blau ist - ein Quantenphänomen.

Dazu ein Satz von Philip Ball, Link folgt:

    Man könnte sagen, die klassische Welt ist ganz einfach, wie sich Quantenmechanik darstellt, wenn man einen Meter achtzig groß ist.

    You could say that the classical world is simply what quantum mechanics looks like if you are six feet tall.


Martin Bäker hat folgendes geschrieben:
Nachdem das somit hoffentlich ausgeräumt ist, möchte ich euch jetzt mit ein paar kleinen Gedankenexperimenten zeigen, wo die konzeptionellen und philosophischen Schwierigkeiten stecken, wenn man versucht zu argumentieren, dass Objekte auch dann “da sind”, wenn man sie nicht beobachtet.

Objekte? Hmm.

Das klingt, als möchte Martin Bäker eine klassische Vorstellung vom Objekt auf die Quantenwelt übertragen. Das ist der falsche Ansatz. Besser anders herum: bottom-up die Quantenmechanik zur Basis nehmen.



Martin Bäker hat folgendes geschrieben:
Nach der Standard-Interpretation der Quantenmechanik findet eine Messung durch eine Wechselwirkung mit einem “klassischen Objekt” statt, also einem Objekt, das hinreichend groß ist, um den Regeln der klassischen Physik zu genügen. Nach dieser Interpretation kann sich ein Objekt wie der Mond also sozusagen “selbst messen”, weil er groß genug ist. Das Problem dieser Deutung ist, dass sich der Mond ja aus lauter quantenmechanisch zu beschreibenden Einzelteilchen zusammensetzt, so dass konzeptionell nicht klar ist, wie so ein “klassisches Objekt” entstehen soll.


Dennoch sind die Bewegungsgesetze der Makrowelt andere als die der Mikrowelt und erstere werden durch letztere eben nicht erklärt.

Die These eines Multiversums ist eigentlich Spekulation und Philosophie. Hier wird quasi gefordert, dass sich die Welt gefälligst den theoretischen Gleichungen anzupassen habe, was ja eigentlich umgekehrt sein müsste.

#247:  Autor: stepWohnort: Germering BeitragVerfasst am: 11.11.2017, 17:02
    —
Skeptiker hat folgendes geschrieben:
Hier wird quasi gefordert, dass sich die Welt gefälligst den theoretischen Gleichungen anzupassen habe, was ja eigentlich umgekehrt sein müsste.

Wäre schlimm, wenn es denn so wäre - ist es aber nicht. Die Gleichungen sind nun mal die, welche die Messungen bisher am besten / einfachsten beschreiben. Leider enthalten sie auch Terme, die man nicht messen kann, die man aber auch nicht weglassen kann, weil sie sonst nicht mehr zu den Messungen passen. Es gibt also grob gesagt nur folgende Möglichkeiten:

1. Man nimmt an, daß das Modell umfassender ist als unsere Messungen (Everett)
2. Man nimmt ein magisches Element zur Hilfe (Kollaps, hidden variables ...)
3. Man sucht nach einem ganz anderen Modell, das die Messungen mindestens genauso gut voraussagt
4. Man verabschiedet sich vom klassischen Realitätsbegriff oder faßt diesen weiter
5. Man verzichtet ganz auf ontologische Aktivitäten (shut up & calculate)

Nachdem es bei 3) ziemlich mau aussieht und ich 2) extrem unbefriedigend finde bzw. dadurch auch Probleme entstehen, ist mein Favorit aus pholosophischer Sicht eine Kombi aus 1) und 4), als Physiker ist mir 5) auch sehr recht.

#248:  Autor: stepWohnort: Germering BeitragVerfasst am: 11.11.2017, 17:45
    —
zelig hat folgendes geschrieben:
step hat folgendes geschrieben:
autonom: Die Größe bezieht ihre Komplexität nicht von anderer Seite. Nur komplex reicht nicht, sonmst würde man bspw. ein Spiegelbild für wirklich halten. (Natürlich ist die Illusion selbst wiederum wirklich in diesem Sinne.) ...
Ich bin nicht so ganz einverstanden. Wenn ich Dich richtig verstehe, ist doch im Streit um die Interpretation der Wellenfunktion die Autonomie fraglich. In deinem Bild: Wir wissen nicht, ob wir in einen Spiegel schauen, oder in etwas, was wie ein Spiegel aussieht, jedoch den gleichen ontologischen Status hat, wie die alltägliche Erfahrungswelt. Es kann natürlich sein, daß ich den Autonomiebegriff hier nicht richtig verstehe.

Bin mir auch nicht sicher, ob ich Dich verstehe.

Ich denke, die nach der SG evolvierende Wellenfunktion, die den Durchgang eines Photons durch den Doppelspalt beschreibt, erfüllt das Autonomiekriteium, grob gesagt deshalb, weil man die Meßergebnisse nicht erklären könnte, wenn man im Modell einen Teil der WF wegnähme. Nun könnte man sagen, daß man zumindest die "abgespaltenen" Teile der Wellenfunktion nach der Messung (also die in den anderen Everett-Universen) wegnehmen könnte, ohne die Meßergebnisse zu beeindrucken. Allerdings bräuchte man dazu eine magische (und nicht-autonome) Wirkung, oder die SG des Gesamtsystems wäre falsch.

zelig hat folgendes geschrieben:
Wenn die Riemannsche Vermutung niemals bewiesen wird, aber die Zeta-Funktion würde ausnahmslos immer bestätigt werden. Wäre das nicht unbefriedigend?

Ja, außer man könnte beweisen, daß und warum man sie niemals beweisen kann.

zelig hat folgendes geschrieben:
... Und müsste es einen Physiker (oder auch einen interessierten Philosophen) nicht genauso brennend interessieren, wie man den Kollaps zu deuten hat?

Was denn fürn Kollaps?

Im Ernst, der Vergleich hinkt insofern, als es bei der Mathematik um Beweise von Sätzen innerhalb logischer Systeme geht. Abgesehen davon wäre der "Kollaps" meines Erachtens ein deutlich brennenderes Problem, wenn man ihn unbedingt bräuchte. Viel interessanter finde ich daher solche offenen Probleme der theoretischen Physik, für die es noch gar keine gute Erklärung gibt, z.B. im Bereich Vereinheitlichung. Auch das von hier schon mal angesprochene Problem des tieferliegenden Wesens von Entropie / Zeit / Information würde mich interessieren.

#249:  Autor: stepWohnort: Germering BeitragVerfasst am: 11.11.2017, 18:35
    —
smallie hat folgendes geschrieben:
step hat folgendes geschrieben:
smallie hat folgendes geschrieben:
Der Lichtstrahl mit der kürzesten Zeit ist jener, für den Einfallswinkel gleich Ausfallswinkel ist. Das ist das oben erwähnte Minimalprinzip.
Man könnte sagen, das Minimalprinzip ist eigentlich ein Prinzip der Phasenkonstanz.
Du meinst Phasenkonstanz am Ort des Detektors? Ja, das gefällt mir.

Ich bezog mich auf die Vektorspirale mit ihren Amplituden und Phasen. Man kann das Minimalprinzip in etwa so formulieren: Dort wo eine kleine Änderung des Pfades zu keiner Änderung der Phase führt, liegt die Resultierende. Kann man sehr schön auf alle Pfadintegrale jeder Art übertragen und auch auf den Lagrangeformalismus. "Kürzeste Zeit" geht hier natürlich auch, aber die Phase ist "schöner" wegen der in den QFT allgegenwärtigen exp(iwt) Phasenterme.

smallie hat folgendes geschrieben:
step hat folgendes geschrieben:
Diese Idee hat übrigens Feynman, in etwas verallgemeinerter Form, eingeführt, worauf ich weiter oben schonmal kurz hingewiesen habe.
Wenn es die Phasenbetrachtung erst seit Feynman gibt, wundert es mich nicht, daß die Quantenmechanik vorher so völlig unverständlich erschien.

Es gibt Physiker (v.a. Feynman-Jünger), die sagen, daß Huygens mehr davon verstand als Bohr. Ich bin kein Historkier, aber ich kenne diesen Gedanken erstmalig von Feynman. Ist auch irgendwie kein Wunder, denn er hat sich ja quasi hauptberuflich mit Problemen beschäftigt, bei denen es um die Beiträge vieler "Pfade" zum Ergebnis ging. Und auch das Minimalprinzip war so ein Steckenpferd von ihm, da hat er halt versucht, einen tieferen Zusammenhang zu sehen.

smallie hat folgendes geschrieben:
step hat folgendes geschrieben:
smallie hat folgendes geschrieben:
Ich nehme eine Feile oder einen Stanzer, und mach Teile des Spiegels unbrauchbar. A, C, E, G, ... sind dann blind, darunter auch der sweet spot. Gemäß der klassischen Vorstellung sollte ich dann nichts mehr sehen.
Wieso das? Die Spirale sieht dann zwar gröber aus, aber hat immer noch eine Resultierende, weil die Phasen sich z.B. zwischen F und H immer noch weniger ändern als woanders. Jedenfalls wenn die Segmente keine zu falsche Größe haben.
Das ist nicht die klassische Lösung. Bei F und H ist Einfallswinkel gleich Ausfallswinkel verletzt.

Egal. Wenn Du es ausrechnest bzw. die Vektorspirale aufmalst, tragen immer noch die mittigeren der verbleibenden Segmente am meisten bei (phasenkonstant mit Loch sozusagen). Die Spirale sieht dann nicht ganz so gerade in der Mitte aus, aber es reicht immer noch für eine ähnliche Resultierende. In Summe hast Du sogar Einfalls- = Ausfallswinkel.

smallie hat folgendes geschrieben:
Falls ich folgendes richtig verstanden habe, passt die Vorstellung vom ausgestanzten Spiegel schon. Ungefähr ab 45:00, vorher wird obiges Diagramm eingeführt. Siehe insbesondere ab 48:30. QED: Fits of Reflection and Transmission -- Quantum Behaviour -- Richard Feynman (2/4)

Ah, jetzt verstehe ich, was Du meinst. Klar, durch Ausstanzen an der richtigen Stelle kann man die Reflektion bestimmter Frequenzen verstärken. Mich hatte irritiert, daß Du die CD als Beispiel für das Ausstanzen jedes zweiten Elements und insbesondere des sweet spots genannt hattest.

smallie hat folgendes geschrieben:
Ich hätte mehr Vertrauen in Aussagen über "verschiedene Zweige der Realität", wenn die Vielzahl der Möglichkeiten durch ein Minimalprinzip beschränkt würde.

Schön formuliert, allerdings glaube ich nicht, daß das Minimalprinzip gültig auf die klassische ontologische Mächtigkeit anwendbar ist. Ich frage mich (oder bessergesagt Dich) zudem, warum es nicht auch ein Minimalprinzip für das Meßergebnis hinter dem Doppelspalt gibt, anstatt so eine doofe Wahrscheinlichkeitsverteilung. Und beim Lotto hätte ich auch gern eins.

Unausgegoren: Ich vermute, man kann die Multiversumszweige tatsächlich durch ein Minimalprinzip beschreiben, allerdings minimiert dieses nicht die Anzahl der Zweige. Ähnlich wie beim Lotto zwinkern

smallie hat folgendes geschrieben:
Die Idee vom permanenten Kollaps der Wellenfunktion durch gravitative Wechselwirkung wird in solchen Artikeln seltenst erwähnt.

Das stimmt. Der Grund ist nach meinem Verständnis, daß man - in Ermangelung einer vereinheitlichten Theorie - davon ausgeht, daß die gravitative Wechselwirkung bei gemäßigtem Klima von der elektromagnetischen WW entkoppelt ist und daher nur zu einer "gravitativen Dekohärenz" führen könnte. Bewiesen ist das aber nicht.

smallie hat folgendes geschrieben:
auch die Physik ist bisweilen ein Tendenzbetrieb mit Strömungen und Moden.

Ja, aber es imponiert trotzdem, wie - außer an den ausgefransten Frontlinien - ein Gebäude wächst, das immer größer, genauer und allgemein akzeptierter wird.

smallie hat folgendes geschrieben:
step hat folgendes geschrieben:
... Eine der wesentlichen Techniken dabei ist das Aufteilen des (z.B. zufällig gewählten) Gesamtarrays in Tensoren niedrigerer Ordnung, wie z.B. hier beschrieben: https://arxiv.org/abs/1710.05867
"Die Implementation optimieren" - gerne. Aber was wird dann aus:

step hat folgendes geschrieben:
So scheint es doch ziemlich naheliegend anzunehmen, daß die Pfade alle gleich (oder zumindest anteilig) "real" sind und die Physik sozusagen die einfachste Realisierung ihrer selbst ist, oder?
Also gibt es doch eine einfachere Beschreibung der Quantenphysik?

Den letzten Schluß verstehe ich nicht. Die Vereinfachung im obigen Beispiel ist nur deshalb möglich, weil das zu modellierende Gesamtsystem (49 Qbits + Problemstellung) weniger als 49 bit Information beinhaltet. Anders ausgedrückt, der dort beschriebene "getweakte" Quantencomputer ist nicht geeignet, jedes mit 49 bit beschreibbare Problem zu lösen, sondern nur eben dieses. Man könnte sagen, die Jungs haben gemogelt ...

#250:  Autor: stepWohnort: Germering BeitragVerfasst am: 11.11.2017, 18:53
    —
smallie hat folgendes geschrieben:
Übrigens dürfte die Masse eines Staubkorns grob bei einer Planck-Masse liegen. Zufall oder bedeutungsvoll?

Es gibt ein weites Spektrum von Staubkörnern, aber nur eine Planck-Masse.

#251:  Autor: zelig BeitragVerfasst am: 11.11.2017, 19:38
    —
step hat folgendes geschrieben:
Im Ernst, der Vergleich hinkt[...]


Ja, er hinkt. Ich finde aber er verdeutlicht, daß "shut up and calculate" nicht unbedingt einen adäquaten Schlusspunkt für diese Art von Diskussion setzen kann. Womit ich nicht sagen will, daß er kein adäquater Schlusspunkt für eine andere Art von Tätigkeit sein kann, eben das "Berechnen".

#252:  Autor: stepWohnort: Germering BeitragVerfasst am: 11.11.2017, 20:29
    —
zelig hat folgendes geschrieben:
... daß "shut up and calculate" nicht unbedingt einen adäquaten Schlusspunkt für diese Art von Diskussion setzen kann. Womit ich nicht sagen will, daß er kein adäquater Schlusspunkt für eine andere Art von Tätigkeit sein kann, eben das "Berechnen".

OK, aber wie willst Du dann zur "richtigen" Interpretation kommen?

Und beim Realitätsbegriff, da kann man lange etwas zurechtphilosophieren, aber der klassische Realitätsbegriff ist nicht zu retten, auch wenn es noch so oft versucht wird (hier z.B. von Myron).

#253:  Autor: stepWohnort: Germering BeitragVerfasst am: 11.11.2017, 20:30
    —
smallie hat folgendes geschrieben:
Besser anders herum: bottom-up die Quantenmechanik zur Basis nehmen.

Sehe ich ebenso.

#254:  Autor: smallie BeitragVerfasst am: 11.11.2017, 20:33
    —
step hat folgendes geschrieben:
zelig hat folgendes geschrieben:
... Und müsste es einen Physiker (oder auch einen interessierten Philosophen) nicht genauso brennend interessieren, wie man den Kollaps zu deuten hat?

Was denn fürn Kollaps?

Laß mich kurz diesen Kollaps deuten.

Fett von mir:
Martin Bäker hat folgendes geschrieben:
5. Wann kollabiert die Wellenfunktion?

Betrachten wir noch einmal unser erstes Experiment mit dem Photon. Ich wandle das Experiment ein bisschen ab, indem ich den halbdurchlässigen Spiegel senkrecht stelle. Dann wird das Photon entweder durchgelassen oder zum Anfang zurückreflektiert:



Die dicke Linie zeigt das ankommende Photon, das dann nach A und/oder B läuft, wo es gemessen wird.

Bei der Messung kollabiert seine Wellenfunktion. Nehmen wir (ohne Beschränkung der Allgemeinheit, wie die MathematInnen sagen) an, wir messen das Photon bei A. Wir bauen unsere Apparate so, dass die Messung an beiden Orten genau gleichzeitig stattfindet (im Ruhesystem der Messapparate, die relativ zu einander stillstehen). Weil die Messungen gleichzeitig waren, können wir nicht sagen, ob es die Messung bei A oder die bei B ist, die die Wellenfunktion kollabieren lässt, beides passiert gleichzeitig.

http://scienceblogs.de/hier-wohnen-drachen/2012/10/10/quantenmechanik-und-realitat/?all=1

Das erinnert mich fatal an Buridans Esel. Der stand genau in der Mitte zwischen zwei Heuhaufen, konnte sich nicht entscheiden, von welchem er fressen sollte - und ist verhungert. Mit Quantenphysik hätte der Esel überlebt. Die läßt nicht zu, daß die Heuhaufen genau gleich weit vom Esel entfernt sind. Sie läßt auch den Bau des vorgeschlagenen Apparates nicht zu.

Wenn man von einer klassischen Vorstellung ausgehend zu einem Widerspruch kommt, liegt das an der Quantenmechanik oder an der klassischen Vorstellung?

#255:  Autor: Zumsel BeitragVerfasst am: 11.11.2017, 20:36
    —
step hat folgendes geschrieben:
zelig hat folgendes geschrieben:
... daß "shut up and calculate" nicht unbedingt einen adäquaten Schlusspunkt für diese Art von Diskussion setzen kann. Womit ich nicht sagen will, daß er kein adäquater Schlusspunkt für eine andere Art von Tätigkeit sein kann, eben das "Berechnen".

OK, aber wie willst Du dann zur "richtigen" Interpretation kommen?

Und beim Realitätsbegriff, da kann man lange etwas zurechtphilosophieren, aber der klassische Realitätsbegriff ist nicht zu retten, auch wenn es noch so oft versucht wird (hier z.B. von Myron).


Könntest du für unbedarfte Physik-Laien erklären, welchen Mehrwert Berechnungen haben sollen, die zu nichts außer anderen Berechnungen einen erkennbaren Bezug haben?

#256:  Autor: stepWohnort: Germering BeitragVerfasst am: 11.11.2017, 20:40
    —
Zumsel hat folgendes geschrieben:
Könntest du für unbedarfte Physik-Laien erklären, welchen Mehrwert Berechnungen haben sollen, die zu nichts außer anderen Berechnungen einen erkennbaren Bezug haben?

Welche meinst Du denn jetzt? Falls Du die Schrödingergleichung und die Evolution der Wellenfunktion (als deren Lösung) meinst: Die haben doch auch einen sehr großen Wert für Voraussage und Erklärung meßbarer Phänomene, für die Technik usw.

#257:  Autor: stepWohnort: Germering BeitragVerfasst am: 11.11.2017, 20:44
    —
smallie hat folgendes geschrieben:
Laß mich kurz diesen Kollaps deuten.

Fett von mir:
Martin Bäker hat folgendes geschrieben:
5. Wann kollabiert die Wellenfunktion?

Betrachten wir noch einmal unser erstes Experiment mit dem Photon. Ich wandle das Experiment ein bisschen ab, indem ich den halbdurchlässigen Spiegel senkrecht stelle. Dann wird das Photon entweder durchgelassen oder zum Anfang zurückreflektiert:



Die dicke Linie zeigt das ankommende Photon, das dann nach A und/oder B läuft, wo es gemessen wird.

Bei der Messung kollabiert seine Wellenfunktion. Nehmen wir (ohne Beschränkung der Allgemeinheit, wie die MathematInnen sagen) an, wir messen das Photon bei A. Wir bauen unsere Apparate so, dass die Messung an beiden Orten genau gleichzeitig stattfindet (im Ruhesystem der Messapparate, die relativ zu einander stillstehen). Weil die Messungen gleichzeitig waren, können wir nicht sagen, ob es die Messung bei A oder die bei B ist, die die Wellenfunktion kollabieren lässt, beides passiert gleichzeitig.

http://scienceblogs.de/hier-wohnen-drachen/2012/10/10/quantenmechanik-und-realitat/?all=1

Das erinnert mich fatal an Buridans Esel. Der stand genau in der Mitte zwischen zwei Heuhaufen, konnte sich nicht entscheiden, von welchem er fressen sollte - und ist verhungert. Mit Quantenphysik hätte der Esel überlebt. Die läßt nicht zu, daß die Heuhaufen genau gleich weit vom Esel entfernt sind. Sie läßt auch den Bau des vorgeschlagenen Apparates nicht zu.

Wenn man von einer klassischen Vorstellung ausgehend zu einem Widerspruch kommt, liegt das an der Quantenmechanik oder an der klassischen Vorstellung?

Ganz genau. Und wenn ich mich recht entsinne, erwähnt Bäker selbst ja dann auch, daß hier die relativistische Informationsübertragung zwischen A und B eine Rolle spielt. Oder hab ich mir das von woanders dazugedichtet?

#258:  Autor: SkeptikerWohnort: 129 Goosebumpsville BeitragVerfasst am: 11.11.2017, 23:16
    —
step hat folgendes geschrieben:
Skeptiker hat folgendes geschrieben:
Hier wird quasi gefordert, dass sich die Welt gefälligst den theoretischen Gleichungen anzupassen habe, was ja eigentlich umgekehrt sein müsste.

Wäre schlimm, wenn es denn so wäre - ist es aber nicht. Die Gleichungen sind nun mal die, welche die Messungen bisher am besten / einfachsten beschreiben. Leider enthalten sie auch Terme, die man nicht messen kann, die man aber auch nicht weglassen kann, weil sie sonst nicht mehr zu den Messungen passen.


Ok, die SG beschreibt ein ideales Teilchen, etwa ein Elektron im Atomorbital oder sonstige Teilchen-Wellen.

In der Chemie gilt für ideale Gase, dass diese bei gleichem Druck und Temperatur jeweils aus genau gleich vielen Teilchen bestehen. Bei idealen Gasen kann man die Wechselwirkungen seiner Teilchen vernachlässigen. (Helium kommt einem idealen Gas sehr nahe.)

Ein Welle-Teilchen bzw. eine Teilchen-Welle lässt sich auch nur dann mit der SG beschreiben, wenn nicht makroskopische Strukturen damit eine Wechselwirkung auslösen. Wie alles in der realen Welt ist auch eine Teilchen-Welle nur theoretisch ein geschlossenes System, so dass auch in größeren Molekülen schnell Dekohärenz-Effekte auftreten.

Das hatten wir schon damals diskutiert, als es um das "Durchschlagen" des Quantenzufalls in mikroskopischen Strukturen auf das Gehirn sowie um die Zufall-/Determinismus-Diskussion ging. Damals hattest du bestritten, dass Quantenzufallseffekte irgend einen Effekt auf Hirnprozesse haben könnten aufgrund der Größe der Synapsen.

step hat folgendes geschrieben:
Es gibt also grob gesagt nur folgende Möglichkeiten:

1. Man nimmt an, daß das Modell umfassender ist als unsere Messungen (Everett)
2. Man nimmt ein magisches Element zur Hilfe (Kollaps, hidden variables ...)
3. Man sucht nach einem ganz anderen Modell, das die Messungen mindestens genauso gut voraussagt
4. Man verabschiedet sich vom klassischen Realitätsbegriff oder faßt diesen weiter
5. Man verzichtet ganz auf ontologische Aktivitäten (shut up & calculate)

Nachdem es bei 3) ziemlich mau aussieht und ich 2) extrem unbefriedigend finde bzw. dadurch auch Probleme entstehen, ist mein Favorit aus pholosophischer Sicht eine Kombi aus 1) und 4), als Physiker ist mir 5) auch sehr recht.


Wenn die SG aufgrund der Dekohärenz zu einer Fallunterscheidungs-Funktion degeneriert, dann kann man dies doch auch einfach als natürliche Folge davon ansehen, dass die Mikroteilchen nun mal nicht für sich selbst existieren können, sondern die Welt aus ständig miteinander wechselwirkenden Systemen und Bestandteilen besteht.

Wieso man da so eine weit hergeholte Annahme wie das Multiversum an den Haaren herbei zerren will, erscheint mir nicht besonders plausibel.

Die Wechselwirkungen sind ja auch kein "magisches Element", sondern z.B. der Messvorgang selbst, welcher die mathematisch ästhetische SG beendet, und zwar als Ausdruck eines real geänderten Zustandes des Gesamtsystems.

Physiker wie Ellis und Laughlin äußern ja eine ähnliche Kritik, ohne 'hidden variables' einführen zu wollen.

#259:  Autor: stepWohnort: Germering BeitragVerfasst am: 11.11.2017, 23:35
    —
Skeptiker hat folgendes geschrieben:
Ok, die SG beschreibt ein ideales Teilchen, etwa ein Elektron im Atomorbital oder sonstige Teilchen-Wellen.

Was ist den ein ideales Teilchen? Gibt es auch unideale Teilchen, die nicht von der SG beschrieben werden?

Skeptiker hat folgendes geschrieben:
Ein Welle-Teilchen bzw. eine Teilchen-Welle lässt sich auch nur dann mit der SG beschreiben, wenn nicht makroskopische Strukturen damit eine Wechselwirkung auslösen. Wie alles in der realen Welt ist auch eine Teilchen-Welle nur theoretisch ein geschlossenes System, so dass auch in größeren Molekülen schnell Dekohärenz-Effekte auftreten.

Die SG beschreibt auch das Gesamtsystem.

Skeptiker hat folgendes geschrieben:
Das hatten wir schon damals diskutiert, als es um das "Durchschlagen" des Quantenzufalls in mikroskopischen Strukturen auf das Gehirn sowie um die Zufall-/Determinismus-Diskussion ging. Damals hattest du bestritten, dass Quantenzufallseffekte irgend einen Effekt auf Hirnprozesse haben könnten aufgrund der Größe der Synapsen.

Das ist richtig, die Dekohärenz im Gehirn ist einfach zu schnell. Die SG gilt natürlich trotzdem bzw. sie erzwingt sogar die Dekohärenz.

Skeptiker hat folgendes geschrieben:
Wenn die SG aufgrund der Dekohärenz zu einer Fallunterscheidungs-Funktion degeneriert, dann kann man dies doch auch einfach als natürliche Folge davon ansehen, dass die Mikroteilchen nun mal nicht für sich selbst existieren können, sondern die Welt aus ständig miteinander wechselwirkenden Systemen und Bestandteilen besteht.

Ich verstehe zwar nicht genau, was Du hier sagen willst, aber diese Bestandteile und Systeme werden auch durch die SG beschrieben.

Skeptiker hat folgendes geschrieben:
Wieso man da so eine weit hergeholte Annahme wie das Multiversum an den Haaren herbei zerren will, erscheint mir nicht besonders plausibel.

Die ist nicht weit hergeholt oder irgendwie gezerrt, sondern ergibt sich, wenn man die SG ersnt nimmt. Diese Interpretation erscheint uns nur erstmal merkwürdig, weil wir makroskopisch dekohärente Effekte gewohnt sind.

Skeptiker hat folgendes geschrieben:
Die Wechselwirkungen sind ja auch kein "magisches Element", sondern z.B. der Messvorgang selbst, welcher die mathematisch ästhetische SG beendet, und zwar als Ausdruck einer real geänderten Zustandes des Gesamtsystems.

Wie bitte? Die Dekohärenz beendet die SG? Wo hast Du denn studiert?

Skeptiker hat folgendes geschrieben:
Physiker wie Ellis und Laughlin äußern ja eine ähnliche Kritik, ohne 'hidden variables' einführen zu wollen.

Was genau? Ich kann mir nicht vorstellen, daß ein seriöser Physiker meint, das Gesamtsystem würde nicht nach der SG evolvieren.

#260:  Autor: zelig BeitragVerfasst am: 12.11.2017, 12:44
    —
step hat folgendes geschrieben:
Und beim Realitätsbegriff, da kann man lange etwas zurechtphilosophieren, aber der klassische Realitätsbegriff ist nicht zu retten, auch wenn es noch so oft versucht wird (hier z.B. von Myron).


Ja, das ist mir klar. Und ehrlich, genau das fasziniert.

#261:  Autor: smallie BeitragVerfasst am: 13.11.2017, 21:36
    —
step hat folgendes geschrieben:
smallie hat folgendes geschrieben:
Ich hätte mehr Vertrauen in Aussagen über "verschiedene Zweige der Realität", wenn die Vielzahl der Möglichkeiten durch ein Minimalprinzip beschränkt würde.

Schön formuliert, allerdings glaube ich nicht, daß das Minimalprinzip gültig auf die klassische ontologische Mächtigkeit anwendbar ist. Ich frage mich (oder bessergesagt Dich) zudem, warum es nicht auch ein Minimalprinzip für das Meßergebnis hinter dem Doppelspalt gibt, anstatt so eine doofe Wahrscheinlichkeitsverteilung. Und beim Lotto hätte ich auch gern eins.

Unausgegoren: Ich vermute, man kann die Multiversumszweige tatsächlich durch ein Minimalprinzip beschreiben, allerdings minimiert dieses nicht die Anzahl der Zweige. Ähnlich wie beim Lotto zwinkern

Ganz zitiert, um den Kontext nicht zu zerreißen. Einzeln beantwortet:


step hat folgendes geschrieben:
Schön formuliert, allerdings glaube ich nicht, daß das Minimalprinzip gültig auf die klassische ontologische Mächtigkeit anwendbar ist.

Frag Physiker nach eine Rangfolge der wichtigsten Gesetze - Thermodynamik wird ganz weit oben stehen. Da liegt es auf der Hand, auch an anderen Stellen nach Thermodynamik zu suchen. Entropie - tauchte bei Boltzmann auf im Zusammenhang mit Gasteilchen, tauchte bei Einstein auf im Zusammenhang mit Lichtteilchen. Entropie scheint ein Strukturgesetz zu sein, das nicht auf einen speziellen Anwendungsfall beschränkt ist.

Behauptung: die folgenden Punkte sind verbandelt:

    - Entropie nimmt im geschlossenen System nie ab.
    - der mittlere Zustand gleicht sich an.
    - Systeme "versuchen" den Zustand niedrigster Energie einzunehmen. Hier versteckt sich das gesuchte Minimalprinzip.
    - niedrigste Energie bedeutet maximale Entropie


Beispiel: ich habe ein Gas aus zwei Teilen Wasserstoff und einem Teil Sauerstoff, dem aufgrund der Freiheitsgrade der Bewegung eine bestimmte Entropie zukommt. Was passiert, wenn ich einen Funken im Gas erzeuge? Klar, es gibt eine Knallgasexplosion. Warum? Obwohl ein Wassermolekül weniger Freiheitsgrade hat, als drei Gasatome, bringt die thermische Energie mehr Freiheitsgrade ins System, so daß die Reaktion abläuft.


Einwand 1: Was ist hier das grundlegende Prinzip? Ergibt sich thermodynamische Bilanz aus den technischen Details der Chemie? Oder erzwingt die Thermodynamik eine bestimmte Form der Chemie? Mein Bauchgefühl sagt, beide Darstellungen seien äquivalent.

Einwand 2: Hilft Entropie überhaupt weiter, sobald es um kosmologische Fragestellungen geht? Eher nicht. Was bedeutet "geschlossenes System" im Zusammenhang mit Multiversum, Inflation oder Dunkler Energie? Es scheint, als würde Entropie gerade dort nicht weiterhelfen, wie sie am Dringendsten gebraucht wird.


Thermodynamik erlaubt die Vorhersage, daß sich mittlere Teilchenzustände angleichen. Systeme bewegen sich auf einen Gleichgewichtszustand zu, meinetwegen so, daß 68% aller Teilchen innerhalb einer Standardabweichung vom Mittelwert liegen.

Solche Überlegungen vermisse ich bei den Multiversumstheorien. Ich könnte mich auch mit 10^500 verschiedenen Multiversen anfreunden. Mir wäre aber wohler, wenn es hieße, daß 68% davon innerhalb einer Standardabweichung liegen.




RE: klassische ontologische Mächtigkeit

Wörtlich gelesen hast du Recht. Die klassische Ontologie hat eine Obsession mit Dingen, mit Objekten. Eine moderne Ontologie sollte auf den Beziehungen von Dingen basieren. Ein Atom wäre dann kein unteilbares Teilchen (Demokrit, Dalton), auch kein Planetensystem mit einem Kern und Elektronen drumherum (Bohr), sondern einfach die Summe dessen, was wir darüber wissen. Dann gibt's auch keine langen Gesichter, wenn sich die Vorstellung vom Atom mal ändert. Die Formulierung "Summe dessen, was wir darüber wissen" beinhaltet bereits, daß wir auch noch mehr darüber wissen könnten, ohne daß das zuvor gewußte falsch wird.



step hat folgendes geschrieben:
Ich frage mich (oder bessergesagt Dich) zudem, warum es nicht auch ein Minimalprinzip für das Meßergebnis hinter dem Doppelspalt gibt, anstatt so eine doofe Wahrscheinlichkeitsverteilung.

Doofe Wahrscheinlichkeitsverteilung?

Wenn beim Würfel eine bestimmte Zahl mit Wahrscheinlichkeit 1/6 herauskommt, wundert sich niemand, weil der Würfel sechs gleichmäßige Seiten hat. Was entspricht den sechs Seiten in der Quantenmechanik?

    - Nils Bohr nahm für sein Atommodell an, daß das Elektron in einem Wasserstoffatom auf einer Kreisbahn läuft und sein Drehimpuls ein ganzzahliges Vielfaches der "Drehimpulseinheit" ist, die selbst wiederum ein ganzahliges Vielfaches der Planckschen Konstanten ist. Damit konnte Bohr die Energienieveaus des ionisierten Helium-Atoms auf mehrere Nachkommastellen genau vorhersagen.

    - Schrödinger nahm die deBroglie-Formel für Materiewellen und setzte sie in eine Wellengleichung ein, unter Annahme einer stehenden Welle.


Beim Würfel ist die regelmäßige 6-Seitigkeit offensichtlich ist. Wie offensichtlich, wie real sind die Annahmen von Bohr und Schrödinger?



Zum Doppelspalt.

Eine Quelle Q sendet Licht zu einem Empfänger E, durch einen Spalt in einer Blende #. Der Spalt soll groß sein, im Vergleich zu Wellenlänge des Lichtes. Das Licht im Bild sieht übrigens nach koheräntem Laserlicht aus, was nur an der Darstellung liegt und so nicht gemeint ist.

Code:


                          #
                          #
                          #
   - - - - - - - - - - - - - - - - - - - - - - -
Q  - - - - - - - - - - - - - - - - - - - - - - -  E
   - - - - - - - - - - - - - - - - - - - - - - -
                          #
                          #
                          #


Im Sinne der Quantenmechik darf mir andere mögliche Pfade dazu denken. Hier machen zwei Photonen einen Schlenker und biegen nach unten ab. Sie interferieren am Beobachtungspunkt zu Amplitude null.

(Ich gebe zu, daß ich das jetzt nicht vorrechnen könnte.)

Code:


                          #
                          #
                          #
   - - - - - - - - - - - - * - - - - - - - - - -
Q  - - - - - - - - - - - - - * - - - - - - - - -  E
   - - - - - - - - - - - - * - * - - - - - - - -
                          #  *   *
                          #     *  *
                          #       *  *
                          #          * *
                          #            **
                          #               * 
                          #                       
                          #                   keine Messung



Die Situation an einem Spalt in der Größenordnung einer Wellenlänge. Der abgebogene Lichtstrahl ist jetzt ein "Einzelgänger". Es gibt keine Interferenz, der abgebogene Lichtstrahl wird sichtbar.

Code:


                          #
                          #
                          #
Q  - - - - - - - - - - - * - - - - - - - - - - -  E
                          #   *   
                          #     *   
                          #       *   
                          #         *   
                          #           * 
                          #             * 
                          #               *     
                          #                    Messung     




Der Doppelspalt wird dann leicht einsehbar: an manchen Stellen heben sie die Phasen von * und X weg, an anderen verstärken sie sich.


Code:


                          #
  - - - - - - - - - - - - X - - - - - - - - - - -
                          #   X
Q                         #       X                E
                          #           X
  - - - - - - - - - - - - * - - - - - - - X - - -
                          #   *               X   
                          #     *                  Messung
                          #       *   
                          #         *   
                          #           * 
                          #             * 
                          #               *       
                          # 
                          #                         Messung     




Mist. Jetzt habe ich es nicht geschafft, das Wort Minimalprinzip unterzubringen. Ich verweise auf deinen Satz: "Man könnte sagen, das Minimalprinzip ist eigentlich ein Prinzip der Phasenkonstanz."


@zelig: du sagtest, das Doppelspaltproblem verlange dringend nach einer Lösung. Angenommen, meine Darstellung wäre halbwegs zutreffend, hältst du sie für befriedigend?



step hat folgendes geschrieben:
Und beim Lotto hätte ich auch gern eins.

Unausgegoren: Ich vermute, man kann die Multiversumszweige tatsächlich durch ein Minimalprinzip beschreiben, allerdings minimiert dieses nicht die Anzahl der Zweige. Ähnlich wie beim Lotto zwinkern

Das Minimalprinzip beim Lotto? Es wird nicht fünfmal hintereinander 1, 2, 3, 4, 5, 6, 7 gezogen.

#262:  Autor: stepWohnort: Germering BeitragVerfasst am: 13.11.2017, 22:58
    —
Nur ganz kurz dazu:

smallie hat folgendes geschrieben:
Entropie - tauchte bei Boltzmann auf im Zusammenhang mit Gasteilchen, tauchte bei Einstein auf im Zusammenhang mit Lichtteilchen. Entropie scheint ein Strukturgesetz zu sein, das nicht auf einen speziellen Anwendungsfall beschränkt ist.

Genau, und der Grund dafür ist, daß man für die Entropie gar keine Physik braucht. Quasi "nur" ein Gesetz der großen Zahl mit Zusammenhang zur Informationstheorie.

#263:  Autor: stepWohnort: Germering BeitragVerfasst am: 13.11.2017, 23:00
    —
Und dazu:

smallie hat folgendes geschrieben:
Das Minimalprinzip beim Lotto? Es wird nicht fünfmal hintereinander 1, 2, 3, 4, 5, 6, 7 gezogen.

Hmm - das minimiert aber auch nicht die Anzahl der Möglichkeiten.

#264:  Autor: stepWohnort: Germering BeitragVerfasst am: 15.11.2017, 22:14
    —
smallie hat folgendes geschrieben:
Systeme bewegen sich auf einen Gleichgewichtszustand zu, meinetwegen so, daß 68% aller Teilchen innerhalb einer Standardabweichung vom Mittelwert liegen. Solche Überlegungen vermisse ich bei den Multiversumstheorien. Ich könnte mich auch mit 10^500 verschiedenen Multiversen anfreunden. Mir wäre aber wohler, wenn es hieße, daß 68% davon innerhalb einer Standardabweichung liegen.

Bei den Teilchen im Raum stellt sich die Verteilung ein, indem sie wechselwirken (z.B. über Stöße in einem idealen Gas). Ich denke nicht, daß man das einfach so auf einen Wellenfunktionsanteile in Hilberträumen übertragen kann. Es könnte aber durchaus Eigenschaften geben, die multiversal besonders häufig auftreten, ähnlich wie auch de Eigenwerte bei Einzelmessungen eine Wahrscheinlichkeitsverteilung haben.

smallie hat folgendes geschrieben:
... Dann gibt's auch keine langen Gesichter, wenn sich die Vorstellung vom Atom mal ändert. Die Formulierung "Summe dessen, was wir darüber wissen" beinhaltet bereits, daß wir auch noch mehr darüber wissen könnten, ohne daß das zuvor gewußte falsch wird.

Ja, das gefällt mir.

smallie hat folgendes geschrieben:
Wenn beim Würfel eine bestimmte Zahl mit Wahrscheinlichkeit 1/6 herauskommt, wundert sich niemand, weil der Würfel sechs gleichmäßige Seiten hat.

Doch, ich wundere mich da. Und bei der Begründung fehlt auch etwas Entscheidendes: daß die Anfangs- und Randbedingungen jedesmal anders sind und ihre Wirkung auf den Würfel fern aller 6 Fixpunkte im Phasenraum stattfindet. Und dann wundere ich mich nicht mehr. Bei deterministischem Chaos habe ich keine Verschränkung von 6 Würfelzuständen und daher auch keine multiversale Aufspaltung. Die "Wahrscheinlichkeit" ist hier einfach eine zeitliche Folge hinreichend chaotischer Anfangsbedingungen. Man könnte auch sagen, der "Kollaps" besteht hier in der Annäherung an einen Fixpunkt.

smallie hat folgendes geschrieben:
Was entspricht den sechs Seiten in der Quantenmechanik?

Hmm - nichts. Jedenfalls kein Quantenwürfel. Und auch nicht 2 Universen. Das Minimalprinzip sagt eben nichts über die Anzahl der Möglichkeiten aus, sondern wenn dann über eine Resultierende (so eine Art Gesamtlösung, deren Phase robust gegen minimale Änderungen ist). Und eine in 2 Universen gespaltene Wellenfunktion hat auch nicht mehr Energie als die vorherige Überlagerung in einem Universum, wegen des Entropiegewinns sogar eher im Gegenteil, würde ich sagen.

smallie hat folgendes geschrieben:
Beim Würfel ist die regelmäßige 6-Seitigkeit offensichtlich ist. Wie offensichtlich, wie real sind die Annahmen von Bohr und Schrödinger?

Well, da halte ich es mit Deiner eigenen Aussage: "Summe dessen, was wir darüber wissen".

smallie hat folgendes geschrieben:
Der Doppelspalt wird dann leicht einsehbar: an manchen Stellen heben sie die Phasen von * und X weg, an anderen verstärken sie sich.

Das ist ja genau der ursprüngliche Wellenansatz z.B. von Young. Er erklärt die Wellennatur, Interferenz und Intensität, aber nicht den quantenmechanischen Meßprozess. Wenn man versucht, alles mit der klassischen Wellentheorie zu beschreiben, scheitert man.

#265:  Autor: smallie BeitragVerfasst am: 16.11.2017, 00:29
    —
step hat folgendes geschrieben:
smallie hat folgendes geschrieben:
Das Minimalprinzip beim Lotto? Es wird nicht fünfmal hintereinander 1, 2, 3, 4, 5, 6, 7 gezogen.

Hmm - das minimiert aber auch nicht die Anzahl der Möglichkeiten.

Wie bedeutend ist eine einzelne Messung? Im Allgemeinen haben wir es mit Ensembles zu tun, die aus Dutzenden von Teilchen bestehen, gerne auch bis hin zur Avogadro Zahl oder mehr.



Folgendes habe ich als Antwort an zelig aufgeschrieben, aber nicht abgeschickt, weil die Analogie zum Kollaps doch zu sehr hinkt. Für den Aspekt "Gesetze der großen Zahl" könnte sie taugen:


zelig hat folgendes geschrieben:
Und müsste es einen Physiker (oder auch einen interessierten Philosophen) nicht genauso brennend interessieren, wie man den Kollaps zu deuten hat?

Ich probier's mal mit einer Analogie und hoffe, daß sie nicht allzusehr hinkt.


Luft, Stille und Musik

In Gasen stoßen die Luftmoleküle fortwährend aufeinander. Das geht kreuz und quer und mit unterschiedlichen Geschwindigkeiten. Wir nehmen das als Druck und Temperatur wahr.

Jetzt nehme ich einen Lautsprecher und einen Synthesizer und produziere einen 40-Hz-Ton. Und behaupte mit Augenzwinkern: du kannst den Ton nicht hören. Beweis: nimm ein Zeitlupen-Mikroskop, verfolge die Bahn eines Luftmoleküls. Es fliegt kreuz und quer und mit unterschiedlichen Geschwindigkeiten herum. Weit und breit kein Ton erkennbar.

In Wahrheit hörst du natürlich einen Ton, weil nicht ein Molekül beteiligt ist, sondern sehr, sehr viele. Ich rate: 10^19 Moleküle, die gerade auf dein Trommelfell eintrommeln. Klang ist ein bestimmter mittlerer Zustand von Luftmolekülen, eine periodische Druckänderung.

Wenn ich Klang aber auf der Ebene eines einzelnen Gasteilchens suche, werde ich ihn nicht finden und ich komme zum falschen Schluß, der Klang und seine Wellenfunktion sei kollabiert.





Die Analogie hinkt, genau aus dem Grund, den du nanntest:

step hat folgendes geschrieben:
Bei den Teilchen im Raum stellt sich die Verteilung ein, indem sie wechselwirken (z.B. über Stöße in einem idealen Gas). Ich denke nicht, daß man das einfach so auf einen Wellenfunktionsanteile in Hilberträumen übertragen kann.



step hat folgendes geschrieben:
Es könnte aber durchaus Eigenschaften geben, die multiversal besonders häufig auftreten....

Richtig. Ich kann mich aber nicht erinnern, das jemals in einem Artikel ausdrücklich so gelesen zu haben.

Ebensowenig habe ich gelesen, daß die herkömmliche Vorstellung vom Mulitversum das kosmologische Prinzip verletzt, welches behauptet, wir befänden uns in einer durchschnittlichen Beobachterposition. Sicher: das ist nur ein Prinzip und kein Gesetz. Nur sollte das Prinzip nicht zu Grabe getragen werden, ohne sein Ableben zu erwähnen.



step hat folgendes geschrieben:
smallie hat folgendes geschrieben:
Der Doppelspalt wird dann leicht einsehbar: an manchen Stellen heben sie die Phasen von * und X weg, an anderen verstärken sie sich.

Das ist ja genau der ursprüngliche Wellenansatz z.B. von Young. Er erklärt die Wellennatur, Interferenz und Intensität, aber nicht den quantenmechanischen Meßprozess. Wenn man versucht, alles mit der klassischen Wellentheorie zu beschreiben, scheitert man.

Yep.

Der Doppelspalt läßt sich semi-klassisch erklären. Genau so wie Martin Bäkers erstes Beispiel vom halbdurchlässigen Spiegel. Solange ich nur einen Quantenparameter betrachte, ist die Sache relativ harmlos. Die klassische Vorstellung scheitert, sobald mehrere Parameter im Spiel sind. Das ist das Ergebnis von EPR und Bellscher Ungleichung.

#266:  Autor: stepWohnort: Germering BeitragVerfasst am: 18.11.2017, 15:56
    —
smallie hat folgendes geschrieben:
Ebensowenig habe ich gelesen, daß die herkömmliche Vorstellung vom Mulitversum das kosmologische Prinzip verletzt, welches behauptet, wir befänden uns in einer durchschnittlichen Beobachterposition. Sicher: das ist nur ein Prinzip und kein Gesetz. Nur sollte das Prinzip nicht zu Grabe getragen werden, ohne sein Ableben zu erwähnen.

Wird die durchschnittliche Beobachterposition nicht eher in der klassischen Deutung zu Grabe getragen? Dort hast Du eine unerklärliche Feinabstimmung. In der multiversalen Deutung dagegen erscheint es trivial, daß sich ein unwahrscheinlicher Beobachter in einem untypischen Zweig wiederfindet.

#267:  Autor: smallie BeitragVerfasst am: 18.11.2017, 23:17
    —
Trivial? Beim Artikel von Bäker dachte ich mir, der sieht andere Probleme als ich. Wie schaut's bei uns zwei aus - sind wir uns bei den Problemen einig?


1) Solange nur ein Teilchen beteiligt ist, das mit sich selbst wechselwirkt, läßt sich dafür eine semi-klassische Beschreibung finden.


2) Sind es zwei verschränkte Teilchen, dann funktioniert die semi-klassische Beschreibung nicht mehr. Verschränktes Teilchen ist dabei zum Beispiel ein Photonenpaar, das bei einer Elektron-Positron-Zerstrahlung entstanden ist. Die entstehenden Photonen haben dann eine entgegengesetzte (orthogonale) Polarisation, egal welche Achse ich auswähle.

Alternativ kann ich den Spin des Teilchenpaares betrachten

Zitat:
Discussion of Experimental Proof for the Paradox of Einstein, Rosen, and Podolskj
D. BOHM AND Y. AHARONOV - 1957


We consider a molecule of total spin zero consisting of two atoms, each of spin one-half.

The two atoms are then separated by a method that does not influence the total spin. After they have separated enough so that they cease to interact, any desired component of the spin of the first particle (A) is measured. Then, because the total spin is still zero, it can immediately be concluded that the same component of the spin of the other particle (B) is opposite to that of A.

Ich messe den Spin von A und weiß damit den Spin von B, ohne daß bei B ein Apparat etwas gemessen hätte. Genauer: ich messe bei A die Spin-Komponente entweder in x, y, z-Achse und stelle auf der gleichen Achse bei B den gegenteiligen Wert fest.

Klassisch betrachtet sollten nach einer Messung in x auch y und z noch korrelieren. Tun sie aber nicht. Woher weiß das Teilchen B, daß bei A auf x gemessen wurde, und es jetzt bei einer Messung auf y oder z nicht mehr korreliert sein muß? Der Apparat bei B hat ja gar nichts gemacht. Beim "Standardkollaps" mit nur einem Teilchen kann ich mich darauf herausreden, das Meßteilchen hätte den Zustand der Wellenfunktion verändert, so daß er jetzt unbekannt ist. Bei zwei verschränkten Teilchen hilft mir das nichts. Das Merkwürdige am Kollaps ist nicht der Kollaps, sondern die Nichtlokalität.

#268:  Autor: stepWohnort: Germering BeitragVerfasst am: 18.11.2017, 23:53
    —
smallie hat folgendes geschrieben:
1) Solange nur ein Teilchen beteiligt ist, das mit sich selbst wechselwirkt, läßt sich dafür eine semi-klassische Beschreibung finden.

Darüber habe ich noch nicht nachgedacht, spontan würde ich sagen, es ist falsch.

smallie hat folgendes geschrieben:
2) Sind es zwei verschränkte Teilchen, dann funktioniert die semi-klassische Beschreibung nicht mehr. Verschränktes Teilchen ist dabei zum Beispiel ein Photonenpaar, das bei einer Elektron-Positron-Zerstrahlung entstanden ist. Die entstehenden Photonen haben dann eine entgegengesetzte (orthogonale) Polarisation, egal welche Achse ich auswähle. ... Woher weiß das Teilchen B, daß bei A auf x gemessen wurde, und es jetzt bei einer Messung auf y oder z nicht mehr korreliert sein muß? Der Apparat bei B hat ja gar nichts gemacht. ... Das Merkwürdige am Kollaps ist nicht der Kollaps, sondern die Nichtlokalität.

Das mit dem Kollaps sehe ich anders. Aber ich gebe Dir insofern recht, als die Nichtlokalität merkwürdig (= nichtintuitiv nach klassicher Kausalität) ist, und das natürlich auch in der multiversalen Interpretation. Also denke ich schon, daß wir uns einig sind in bezug auf die Natur des Meßproblems bzw. des Problems der anscheinenden Unmöglichkeit eines lokalen Realismus.

Willst Du auf sowas hinaus wie hier beschrieben?
http://www.spektrum.de/news/wie-real-ist-die-quantenverschraenkung/1445463

#269:  Autor: smallie BeitragVerfasst am: 20.11.2017, 21:42
    —
step hat folgendes geschrieben:
Willst Du auf sowas hinaus wie hier beschrieben?

Ich wollte vor allem vermeiden, daß wir aneinander vorbeireden, weil wir zum selben Begriff andere Vorstellungen und Beispiele haben.

Bevor ich Quantenmechanik oder EQM auf den Prüfstand stellen kann, muß ich erst meine Annahmen darüber auf den Prüfstand stellen.



step hat folgendes geschrieben:
smallie hat folgendes geschrieben:
1) Solange nur ein Teilchen beteiligt ist, das mit sich selbst wechselwirkt, läßt sich dafür eine semi-klassische Beschreibung finden.

Darüber habe ich noch nicht nachgedacht, spontan würde ich sagen, es ist falsch.

Auch dann, wenn semi-klassisch bedeutet, ich darf zwischen Wellen- und Teilchendarstellung wechseln, wie es mir gerade in den Kram passt, ohne das näher begründen zu müssen?

Nebenaspekt: Eine Begründung scheint mir spätestens dann nötig, wenn ein Photon auf eine der allgegenwärtigen Quantenfluktuationen aus der Vakuumsenergie trifft. Weiter oben sagte ich Wechselwirkungen seien erlaubt, wenn die Wellenlänge der beteiligten Partikel keine Lokalisation erlaubt, weil die Wellenlänge zu groß ist.



Ich gehe nochmal zurück zu Martin Bäker. Seine Darstellung zum halbdurchlässigen Spiegel:

Martin Bäker hat folgendes geschrieben:
1. Wo ist das Photon?

Fangen wir ganz einfach an: Wir schicken ein Photon (also ein Lichtquant) auf einen halbdurchlässigen Spiegel.



Das Photon kommt von links und trifft auf den Spiegel. Es hat eine 50%-Wahrscheinlichkeit dafür, nach A zu laufen, und eine 50%-Wahrscheinlichkeit für den Weg nach B. (Das ist die Definition von “halbdurchlässig”.)

Die Regeln der Quantenmechanik sagen uns, dass das Photon tatsächlich beide Wege geht: Solange wir das Photon nicht beobachten, ist es in einem Mischzustand (auch Überlagerungszustand genannt) – es hat eine Wahrscheinlichkeit, auf dem Weg nach A zu sein, und eine, auf dem Weg nach B zu sein. Das gilt auch dann, wenn A und B extrem weit voneinander entfernt sind. Sobald einer der beiden Detektoren anspricht, wissen wir, dass das Photon jetzt bei diesem Detektor ist, (und wir werden es dann nicht auch noch beim anderen Detektor messen).

Die Energieerhaltung lässt keinen Zweifel daran zu, dass das Photon die ganze Zeit existiert, aber wo es sich aufhält, ist bis zur Messung nicht eindeutig – man kann sagen, es ist an zwei Orten gleichzeitig, solange, bis wir es gemessen haben.

Da versteckt sich meiner Ansicht nach ein subtiler Fehler.

Es ist nicht so, daß das Photon beide Wege geht - als ob es sich in Miniatur-Kopien seiner selbst aufgespalten hätte. Wäre das Photon an zwei Orten gleichzeitig, hätten wir plötzlich die doppelte Energie im System. Alternativ könnten die Kopien nur jeweils halbe Energie haben - was ganz böse ist, weil sich dann die Wellenlänge verändert.

Ich würde lieber von zwei Wahrscheinlichkeitsamplituden sprechen, die zusammen einem Teilchen entsprechen.


Martin Bäker hat folgendes geschrieben:
(Man darf hier nicht den Fehler machen zu glauben, dass das Problem nur darin steckt, dass wir einfach nur nicht wissen, wo das Photon hingelaufen ist – der Überlagerungszustand ist tatsächlich real und mit geeigneten Experimenten lässt sich das auch nachweisen – hier ginge das dadurch, dass man die beiden Wege des Photons durch weitere Spiegel wieder zusammenführt und die Interferenz ausnutzt. Das will ich aber in diesem Text nicht erklären, hier soll es nur darum gehen, was wir wissen, nicht, wie wir es herausgefunden haben.)

Schade, daß Bäker hier nicht wenigstens einen Namen nennt, um seinen Lesern eine Internetsuche zu ersparen. Sein Diagramm vom halbdurchlässigen Spiegel ist wenig hilfreich, es zeigt das Problem nicht. Ich kann es so lesen, daß das Photon abgelenkt worden oder durchgegangen ist, je nachdem ob es gerade ein Elektron in den Atomen des Spiegels getroffen hat oder nicht.



Apropos semi-klassisch.

step hat folgendes geschrieben:
die Phase ist "schöner" wegen der in den QFT allgegenwärtigen exp(iwt) Phasenterme.


Oft lese ich, Wellen in der Quantenmechanik seien Wellen im komplexen Zahlenraum. Sind sie ja auch. Aber die Aussagen kommen so bedeutungsschwanger und geheimnisvoll daher:


Zitat:
The state space is a complex vector space: these linear combinations can and do crucially involve complex numbers, in an inescapable way. In the classical case only real numbers appear, with complex numbers used only as an inessential calculational tool.

http://www.math.columbia.edu/~woit/QM/qmbook.pdf

Fehlt nur der Zusatz: darum ist die Quantenphysik für den Menschen so unverständlich. Komplexe Zahlen, Geschockt wer kann sich das schon vorstellen? Stimmt aber nicht. Das exp(iwt) taucht auch in der Fouriertransformation auf. Die braucht man für mp3 - dort hält niemand die komplexen Anteile für geheimnisvoll.

Was will der Autor sagen? Sollte ich etwas ganz wesentliches übersehen haben?


step hat folgendes geschrieben:
smallie hat folgendes geschrieben:
... Das Merkwürdige am Kollaps ist nicht der Kollaps, sondern die Nichtlokalität.

Das mit dem Kollaps sehe ich anders.

Nach meinem Verständnis gilt:

1) von einem Quantensystem mit mehren Parametern, meinetwegen Spin, Polarisierung, ... läßt sich immer nur einer durch Messung bestimmen.
2) ohne Messung bleiben die Quantenzustände erhalten.
3) die Messung hat Auswirkungen auf die nicht gemessenen Parameter. Wenn vor der Messung die Polarisation bekannt ist und ich einen anderen Parameter messe, ist die Polarisation hinterher unbekannt.

Naheliegende Einwände lasse ich vorerst weg. Weiter oben erwähntest du Kommutativität, womit ich nichts anfangen konnte. Ich vermute, das spielt hier mit hinein.


Die Kopenhagener Deutung in meinen Worten:

Beim Doppelspalt ergibt sich am Schirm ein Interferenzmuster, auch dann, wenn ich nur ein Photon durchschicke. Das geht nur, wenn alle Pfade real sind, aber am Ende wird nur einer gemessen - alle übrigen sind verschwunden. Ergo Kollaps.



step hat folgendes geschrieben:
Willst Du auf sowas hinaus wie hier beschrieben?
http://www.spektrum.de/news/wie-real-ist-die-quantenverschraenkung/1445463

Daran kann ich mich erinnern, hab's damals aber nicht verstanden.

Artikel hat folgendes geschrieben:
[es] könnte es unbekannte Einflüsse, Beschränkungen, Randbedingungen, Erhaltungssätze geben, die unsere Freiheit der Wahl subtil einschränken.

Einige Kandidaten:

    - Wie wär's mit dem holographischen Prinzip? Der 3D-Raum kann möglicherweise auf einen gekrümmten 2D-Raum projiziert werden. Hatten wir ganz am Anfang des Threads schon.

    - wenn Raum keine kontinuierliche Größe ist, dann sollten die Freiheitsgrade auch auf irgend eine Weise beschränkt sein.

    - der Renormierungsfehler - oder wie das richtig heißt. Quantenmechanische Berechnung der Nullpunktsenergie liefert höhere Werte als die kosmologischen Beobachtung. Das heißt: die QM hat Freiheitsgrade, die in der Natur nicht auftreten.

#270:  Autor: zelig BeitragVerfasst am: 25.11.2017, 14:20
    —
step hat folgendes geschrieben:
Willst Du auf sowas hinaus wie hier beschrieben?
http://www.spektrum.de/news/wie-real-ist-die-quantenverschraenkung/1445463


Danke für den Link. Den Artikel habe ich damals gelesen, aber ich hatte die Quelle vergessen.
Ich finde den Gedanken der "subtilen Einschränkung" der Wahlfreiheit faszinierend und einleuchtend zugleich. Kommt man mit solchen Überlegungen nicht an eine Grenze, die die Begrifflichkeit, mit der wir operieren, unbrauchbar macht? Was soll denn "Superdeterminismus" bedeuten, wenn das Prinzip von Ursache und Wirkung einzig im Zeit-Raum-Gebilde geltung hat. Und was bedeutet es, wenn man höherdimensionale Konfigurationsräume theoretisch mathematisch beschreiben kann? Man müsste am Ende konstatieren, daß da was vorliegt, wovon unsere Realität nur ein winziger Teil ist. Der riesige Rest wäre aber auf gewisse Weise irrelevant.

#271:  Autor: stepWohnort: Germering BeitragVerfasst am: 26.11.2017, 18:25
    —
zelig hat folgendes geschrieben:
Was soll denn "Superdeterminismus" bedeuten, wenn das Prinzip von Ursache und Wirkung einzig im Zeit-Raum-Gebilde geltung hat.

Das sagt dann etwas Allgemeineres aus, z.B. über die unitären mathematischen Zusammenhänge zwischen Zuständen eines Zustandsraums, prinzipielle Berechenbarkeit oder so. Eignet sich dann natürlich noch weniger zum fälschlichen Ableiten eines Fatalismus zwinkern

zelig hat folgendes geschrieben:
Und was bedeutet es, wenn man höherdimensionale Konfigurationsräume theoretisch mathematisch beschreiben kann? Man müsste am Ende konstatieren, daß da was vorliegt, wovon unsere Realität nur ein winziger Teil ist. Der riesige Rest wäre aber auf gewisse Weise irrelevant.

Jain, würde ich sagen:
- irrelevant, insofern es keine direkten Auswirkungen auf unser täglich Leben und Streben hat
- relevant für Leute, die ein konsistentes Modell wünschen, z.B. weil sie sonst auf magischen Annahmen über unser Universum sitzenbleiben würden, mit denen man allerlei metaphysischen Unsinn treiben könnte.

Übrigens gibt es auch ohne multiversales Modell bereits eine Physik, von der unsere Realität nur ein winziger Teil ist. Hätte man etwa schwarze Löcher oder Gravitationswellen nicht mit großem Aufwand aufgespürt, gäbe es sie faktisch nicht. Die Grenze ist fließender, als man gemeinhin so denkt.

#272:  Autor: Marcellinus BeitragVerfasst am: 27.11.2017, 12:39
    —
step hat folgendes geschrieben:

zelig hat folgendes geschrieben:
Und was bedeutet es, wenn man höherdimensionale Konfigurationsräume theoretisch mathematisch beschreiben kann? Man müsste am Ende konstatieren, daß da was vorliegt, wovon unsere Realität nur ein winziger Teil ist. Der riesige Rest wäre aber auf gewisse Weise irrelevant.

Jain, würde ich sagen:
- irrelevant, insofern es keine direkten Auswirkungen auf unser täglich Leben und Streben hat
- relevant für Leute, die ein konsistentes Modell wünschen, z.B. weil sie sonst auf magischen Annahmen über unser Universum sitzenbleiben würden, mit denen man allerlei metaphysischen Unsinn treiben könnte.


Was ist an einen Modell "konsistent", das über weite Strecken aus Spekulationen besteht, wenn auch sicherlich aus mathematisch hochinteressanten Spekulationen, wenn man sie denn versteht, was ich eindeutig nicht tue?

Bitte mich nicht mißzuverstehen, ich finde faszinierend, was ihr beide hier tut, gerade, weil ich nicht die Spur einer Ahnung habe, worum es sich dabei dreht. Ok, das ist jetzt vielleicht etwas übertrieben. "Universen", "Raum-Zeit-Gefüge", das sind schon Begriffe, mit denen ich eine laienhafte Vorstellung verbinde, aber mehr sicherlich nicht.

Aber ein wie auch immer "konsistentes" Modell unseres Universum, so wie ihr es versteht, habe ich nicht, ohne daß ich den Eindruck hätte, ich würde auf "magischen Annahmen über unser Universum sitzenbleiben, mit denen man allerlei metaphysischen Unsinn treiben könnte". Aber vielleicht habt ihr ja ein paar Hinweise auf mögliche Defekte meines Weltmodells, die auch ein physikalischer Blindfisch wie ich versteht. zwinkern

#273:  Autor: stepWohnort: Germering BeitragVerfasst am: 27.11.2017, 17:19
    —
Marcellinus hat folgendes geschrieben:
Aber ein wie auch immer "konsistentes" Modell unseres Universum, so wie ihr es versteht, habe ich nicht, ohne daß ich den Eindruck hätte, ich würde auf "magischen Annahmen über unser Universum sitzenbleiben, mit denen man allerlei metaphysischen Unsinn treiben könnte". Aber vielleicht habt ihr ja ein paar Hinweise auf mögliche Defekte meines Weltmodells, die auch ein physikalischer Blindfisch wie ich versteht. zwinkern

Ich denke, Dein Weltbild beinhaltet bereits viele heute selbstverständliche Teile, die früher eingeführt wurden, um Inkonsistenzen zu beseitigen. Z.B. in der Physik das Sternmodell mit Kernfusion - zuvor mußte man magische Energiequellen annehmen, um zu erklären, warum die Sonne so heiß und lange strahlt. Oder in der Biologie die Evolution - zuvor mußte man magische Eingriffe annehmen, um die Entstehung, Vielfalt und Anpassung der Arten zu erklären.

Bei der hier diskutierten Konsistenzfrage geht es um den quantenmechanischen Meßprozeß - also klassisch ausgedrückt um die Frage, wie die Entscheidung fällt, welchen der zwei möglichen, zuvor überlagerten Zustände wir messen. In der "Kopenhagener Deutung" nimmt man an der Stelle eine "magische" zufällige Auswahl eines Zustandes an, so daß es letztlich offen bleibt, wieso gerade dieser Zustand ausgewählt wird. Das ist irgendwie unbefriedigend. In noch schlimmerer Form sehen wir dieses Problem etwa bei der Symmetriebrechung, die kurz nach dem Urknall stattfand: Hier entstand offensichtlich ein Zustand mit extrem spezifischen Parametern ("Feinabstimmung"), aber warum genau die? Ein Physik-Gott?

Viele Physiker empfinden es in dieser Situation als konsistenter, auf die magische Auswahl zu verzichten und stattdessen eine der folgenden alternativen Deutungen anzunehmen:

- "verborgene Variablen" - es fehlt eine unbekannte Kraft in der Theorie, die den gemessenen Wert erzwingt. Dieser Ansatz führt aber zu anderen fundamentalen Problemen etwa bei der Berechnung von Wahrscheinlichkeiten, außerdem müßte man die fehlende Kraft erstmal finden

- "Vielwelten" - hier rechnet man einfach mit der Gesamtwellenfunktion auch während und nach der Messung weiter, und es ergibt sich dann eine Welt, von der der gemessene Spezialwert nur eine Teilwelt repräsentiert, damit ist er nix Besonderes mehr. Ich persönlich finde dies die bisher konsistenteste Deutung.

- es gibt noch andere Varianten, z.B. die Bohm'sche Interpretation, dann noch eine mit Führungswellen usw.

#274:  Autor: fwoWohnort: im Speckgürtel BeitragVerfasst am: 27.11.2017, 18:05
    —
step hat folgendes geschrieben:
....
Bei der hier diskutierten Konsistenzfrage geht es um den quantenmechanischen Meßprozeß - also klassisch ausgedrückt um die Frage, wie die Entscheidung fällt, welchen der zwei möglichen, zuvor überlagerten Zustände wir messen. In der "Kopenhagener Deutung" nimmt man an der Stelle eine "magische" zufällige Auswahl eines Zustandes an, so daß es letztlich offen bleibt, wieso gerade dieser Zustand ausgewählt wird. Das ist irgendwie unbefriedigend. In noch schlimmerer Form sehen wir dieses Problem etwa bei der Symmetriebrechung, die kurz nach dem Urknall stattfand: Hier entstand offensichtlich ein Zustand mit extrem spezifischen Parametern ("Feinabstimmung"), aber warum genau die? Ein Physik-Gott?
....

Aber das ist eine Frage vor der jedes hypothetische Leben steht, dessen Steuerung so hypertrophiert ist, dass es beginnen kann seine Welt im Sinne einer Wissenschaft zu untersuchen. Warum zuckt man da nicht einfach mit den Schultern und fragt Na und? Schulterzucken

Oder gäbe es einen möglichen Nutzen für die Beantwortung dieser Frage?

#275:  Autor: Marcellinus BeitragVerfasst am: 27.11.2017, 18:37
    —
step hat folgendes geschrieben:

Ich denke, Dein Weltbild beinhaltet bereits viele heute selbstverständliche Teile, die früher eingeführt wurden, um Inkonsistenzen zu beseitigen. Z.B. in der Physik das Sternmodell mit Kernfusion - zuvor mußte man magische Energiequellen annehmen, um zu erklären, warum die Sonne so heiß und lange strahlt. Oder in der Biologie die Evolution - zuvor mußte man magische Eingriffe annehmen, um die Entstehung, Vielfalt und Anpassung der Arten zu erklären.

Bei der hier diskutierten Konsistenzfrage geht es um den quantenmechanischen Meßprozeß - also klassisch ausgedrückt um die Frage, wie die Entscheidung fällt, welchen der zwei möglichen, zuvor überlagerten Zustände wir messen. In der "Kopenhagener Deutung" nimmt man an der Stelle eine "magische" zufällige Auswahl eines Zustandes an, so daß es letztlich offen bleibt, wieso gerade dieser Zustand ausgewählt wird. Das ist irgendwie unbefriedigend. In noch schlimmerer Form sehen wir dieses Problem etwa bei der Symmetriebrechung, die kurz nach dem Urknall stattfand: Hier entstand offensichtlich ein Zustand mit extrem spezifischen Parametern ("Feinabstimmung"), aber warum genau die? Ein Physik-Gott?


Siehst du, und das verstehe ich nicht. Wissenschaften, so wie ich sie verstehe, dienen dazu, nachprüfbare Modelle für beobachtbare Tatsachen zu finden. Die Suche nach absoluten Anfängen oder dogmatischen, endgültigen Antworten ist nicht ihre Aufgabe. Eine offene Frage ist damit eben das und nur das: eine offene Frage. Götter haben da nichts zu suchen, es sei denn, man versteht es als die Aufgabe der eigenen Wissenschaft, eine Art Religionsersatz zu sein.

Wissenschaften sind aber der Ausdruck der Erfahrung der Menschen, daß man erst dann zu empirisch nachprüfbaren Theorien kommt, wenn man das, was wir in dieser Welt beobachten können, nicht auf die Handlungen und Absichten übernatürlicher Verursacher zurückführt. Insofern stellt sich die Frage mit dem „Physik-Gott“ nicht. Auch die Beispiele, die du anführst, passen meiner Ansicht nach nicht. Die Kernfusion ist ein empirisch nachprüfbares Modell, die biologische Evolution ebenfalls.

Die „Feinabstimmung“ mag ein ungelöstes Problem sein. Aber dem kommt man nicht dadurch bei, daß man zu Scheinerklärungen greift, die noch fantastischer sind als das eigentliche Problem. Was ist so schlimm daran, auf eine Frage (vorübergehend) keine Antwort zu haben? Schlimm finde ich dagegen, wenn man sich daran gewöhnt, bloße Spekulationen schon für Wissenschaft zu halten. Wissenschaften leben vom wechselseitigen Prozeß von Tatsachenbeobachtung und Modellbildung, von Empirie und Theorie. Das eine funktioniert nicht ohne das andere.

Könnte es sein, daß ihr in diesem Fall die physikalischen Wissenschaften nur benutzt, um philosophische oder religiöse Fragen zu diskutieren?

#276:  Autor: stepWohnort: Germering BeitragVerfasst am: 27.11.2017, 19:27
    —
Marcellinus hat folgendes geschrieben:
Wissenschaften, so wie ich sie verstehe, dienen dazu, nachprüfbare Modelle für beobachtbare Tatsachen zu finden.

Ja.

Marcellinus hat folgendes geschrieben:
Die Suche nach absoluten Anfängen oder dogmatischen, endgültigen Antworten ist nicht ihre Aufgabe.

Strohmann? Übrigens wurde dies früher schon gegen allerlei heute akzeptierte Forschungsgebiete eingewandt.

Marcellinus hat folgendes geschrieben:
Eine offene Frage ist damit eben das und nur das: eine offene Frage.

Das ist trivial richtig.

Marcellinus hat folgendes geschrieben:
Götter haben da nichts zu suchen, es sei denn, man versteht es als die Aufgabe der eigenen Wissenschaft, eine Art Religionsersatz zu sein.

Strohmann?

Marcellinus hat folgendes geschrieben:
... wenn man das, was wir in dieser Welt beobachten können, nicht auf die Handlungen und Absichten übernatürlicher Verursacher zurückführt.

Ja, und zwar indem man sie stattdessen auf natürliche Wirkungen zurückführt. In unserem Fall haben wir plötzlich eine "übernatürliche" Zustandsauswahl, und die ist unbefriedigend. Wir wollen sie nict auf etwas Übernatürliches zurückführen, also müssen wir versuchen, sie als natürliches Ergebnis unseres Modells zu beschreiben.

Marcellinus hat folgendes geschrieben:
Insofern stellt sich die Frage mit dem „Physik-Gott“ nicht.

Wenn man sich mit der Feinabstimmung einfach so zufriedengibt, glaubt man an etwas Übernatürliches.

Marcellinus hat folgendes geschrieben:
Auch die Beispiele, die du anführst, passen meiner Ansicht nach nicht. Die Kernfusion ist ein empirisch nachprüfbares Modell, die biologische Evolution ebenfalls.

Natürlich ist es nicht 1:1 übertragbar, aber da bewegst Du Dich auf ganz dünnem Eis! Versuch mal, die "Nachprüfbarkeit" zu operationalisieren ... da kriegst Du sehr schnell ein Grenzziehungsproblem. Was ist z.B. mit so grundlegenden Schlüssen wie der Annahme, daß bestimmte gute Theorien auch da gelten, wo wir sie nicht überprüft haben oder nicht überprüfen können?

Z.B. selbst bei den unstrittigeren Teilen der Quantenphysik; nehmen wir den Überlagerungszustand. Wir können ihn niemals direkt nachprüfen, sondern nur indirekt schließen, daß dies die beste Theorie ist. Natürlich darf man daraud keine ontologischen Fehlschlüsse ziehen, eine Theorie ist immer nur eine möglichst konsitente und einfache Rechenvorschrift für eine Simulation des Beobachteten.

Marcellinus hat folgendes geschrieben:
Die „Feinabstimmung“ mag ein ungelöstes Problem sein. Aber dem kommt man nicht dadurch bei, daß man zu Scheinerklärungen greift, die noch fantastischer sind als das eigentliche Problem.

Strohmann? Ich finde die konsequente Ernstnahme der Schrödingergleichung ist die bisher am wenigsten fantastische Deutung. Eben weil sie keinerlei fantastische Zusatzannahmen macht. Trotzdem kann man natürlich argumentieren, daß man eigentlich gar keine Deutung benötigt ("shut up & calculate"). Ich habe damit i.a. kein Problem, solange nicht jemand mit der Kopenhagener Deutung ankommt.

Marcellinus hat folgendes geschrieben:
Was ist so schlimm daran, auf eine Frage (vorübergehend) keine Antwort zu haben?

Gar nix. Aber wieso "vorübergehend"? Hast Du nicht oben gefordert, man solle von dieser Frage die Finger lassen, weil sie "endgültig" sei?

Marcellinus hat folgendes geschrieben:
Schlimm finde ich dagegen, wenn man sich daran gewöhnt, bloße Spekulationen schon für Wissenschaft zu halten. Wissenschaften leben vom wechselseitigen Prozeß von Tatsachenbeobachtung und Modellbildung, von Empirie und Theorie. Das eine funktioniert nicht ohne das andere.

Das wurde ja oft genug betont, auch von mir.

Marcellinus hat folgendes geschrieben:
Könnte es sein, daß ihr in diesem Fall die physikalischen Wissenschaften nur benutzt, um philosophische oder religiöse Fragen zu diskutieren?

Nein, könnte nicht sein. Obwohl, Dein letzter Beitrag und meine Antwort (also diese) sind natürlich philosophisch, insofern sie die Methoden der Wissenschaft von einer Metaperspektive aus betrachten.

#277:  Autor: stepWohnort: Germering BeitragVerfasst am: 27.11.2017, 19:39
    —
fwo hat folgendes geschrieben:
step hat folgendes geschrieben:
... In noch schlimmerer Form sehen wir dieses Problem etwa bei der Symmetriebrechung, die kurz nach dem Urknall stattfand: Hier entstand offensichtlich ein Zustand mit extrem spezifischen Parametern ("Feinabstimmung"), aber warum genau die? Ein Physik-Gott?
....
Aber das ist eine Frage vor der jedes hypothetische Leben steht, dessen Steuerung so hypertrophiert ist, dass es beginnen kann seine Welt im Sinne einer Wissenschaft zu untersuchen. Warum zuckt man da nicht einfach mit den Schultern und fragt Na und? Schulterzucken Oder gäbe es einen möglichen Nutzen für die Beantwortung dieser Frage?

Nutzen ... hmm ... da kann ich nur ein paar "geringere" Nutzen aufzählen, nicht so was Tolles wie eine Atombombe oder eine Genschere.

- es befriedigt meine Neugier (das kann man jetzt natürlich als Kontrollzwang herabwürdigen usw.)
- es hat einen hohen ästhetischen Wert (ähnlich wie Musik oder Kunst)
- es bietet eine Alternative zu übernatürlichen Modellen. Selbst wenn wir am Ende eine Lie-Gruppe hätten und die nicht weiter begründen könnten, so wäre deren metaphysisches Suchtpotential doch wohl sehr gering.

Übrigens kann ich die Frage auch umdrehen: wozu das ganze Gedöns um die Gravitationswellen? Die sind auch wahrscheinlich niemals zu etwas nutze. Ebenso wie große Teile der Paläoanthropologie usw.

#278:  Autor: zelig BeitragVerfasst am: 27.11.2017, 19:42
    —
Was mich betrifft, ich finde es ist pure Lust über Konzepte der Wirklichkeit nachzudenken.

#279:  Autor: fwoWohnort: im Speckgürtel BeitragVerfasst am: 27.11.2017, 20:01
    —
step hat folgendes geschrieben:
fwo hat folgendes geschrieben:
step hat folgendes geschrieben:
... In noch schlimmerer Form sehen wir dieses Problem etwa bei der Symmetriebrechung, die kurz nach dem Urknall stattfand: Hier entstand offensichtlich ein Zustand mit extrem spezifischen Parametern ("Feinabstimmung"), aber warum genau die? Ein Physik-Gott?
....
Aber das ist eine Frage vor der jedes hypothetische Leben steht, dessen Steuerung so hypertrophiert ist, dass es beginnen kann seine Welt im Sinne einer Wissenschaft zu untersuchen. Warum zuckt man da nicht einfach mit den Schultern und fragt Na und? Schulterzucken Oder gäbe es einen möglichen Nutzen für die Beantwortung dieser Frage?

Nutzen ... hmm ... da kann ich nur ein paar "geringere" Nutzen aufzählen, nicht so was Tolles wie eine Atombombe oder eine Genschere.

- es befriedigt meine Neugier (das kann man jetzt natürlich als Kontrollzwang herabwürdigen usw.)
- es hat einen hohen ästhetischen Wert (ähnlich wie Musik oder Kunst)
- es bietet eine Alternative zu übernatürlichen Modellen. Selbst wenn wir am Ende eine Lie-Gruppe hätten und die nicht weiter begründen könnten, so wäre deren metaphysisches Suchtpotential doch wohl sehr gering.

Übrigens kann ich die Frage auch umdrehen: wozu das ganze Gedöns um die Gravitationswellen? Die sind auch wahrscheinlich niemals zu etwas nutze. Ebenso wie große Teile der Paläoanthropologie usw.

Die Frage nach dem Nutzen war natürlich blöde und Deine Antwort mit der Befriedigung der Neugier war ein passende Backpfeife.

Auf solchen Blödsinn kommt man natürlich immer wieder auch aus fachlichem Neid, wenn man z.B. sieht, dass die Kosten für die Erforschung des Teilchenzoos in die Milliarden gehen, wogegen es in den Biowissenschaften tatsächlich auch noch Amateurarbeiten an der Grenze gibt.

step hat folgendes geschrieben:
...
Marcellinus hat folgendes geschrieben:
Insofern stellt sich die Frage mit dem „Physik-Gott“ nicht.

Wenn man sich mit der Feinabstimmung einfach so zufriedengibt, glaubt man an etwas Übernatürliches.

Nö. Ein Zufall, den Du zwangsläufig bemerkst, weil Du an seinem Ende stehst, bleibt immer noch ein Zufall.
step hat folgendes geschrieben:
....
Marcellinus hat folgendes geschrieben:
Was ist so schlimm daran, auf eine Frage (vorübergehend) keine Antwort zu haben?

Gar nix. Aber wieso "vorübergehend"? Hast Du nicht oben gefordert, man solle von dieser Frage die Finger lassen, weil sie "endgültig" sei?

Es ist natürlich richtig, dass eine nicht willkürlich begrenzte Forschung zwangsläufig regelmäßige Grenzüberschreitungen zu Religion usw. beinhaltet und diese Grenzen auch verschiebt. Das ist im Endeffekt auch weniger eine Frage an die Forschung, die immer soviel spielt, wie die Gesellschaft ihr finanziert. Das ist eigentlich immer eher eine Frage an die Gesellschaft wieviel ihr dieses Gebiet wert ist.

#280:  Autor: stepWohnort: Germering BeitragVerfasst am: 27.11.2017, 21:26
    —
fwo hat folgendes geschrieben:
... auch aus fachlichem Neid, wenn man z.B. sieht, dass die Kosten für die Erforschung des Teilchenzoos in die Milliarden gehen, wogegen es in den Biowissenschaften tatsächlich auch noch Amateurarbeiten an der Grenze gibt.

Das stimmt - wobei die Grundlagenphysiker genauso schlecht bezahlt werden wie die Biologen. Ich denke, ein Grund für die teureren Investitionen in der Physik ist einfach, daß man an ihre tieferen Grundlagen schwerer herankommt. Und gäbe es denn in der Biologie überhaupt ein grundlegendes Experiment oder Vorhaben, das sich nur mangels Großinvestition nicht realisieren läßt? Welche Grundfrage würde es beantworten?

Im Bereich der emergenten Wissenschaften fällt mir da die Hirnforschung ein, da könnte man mit Investitionen sicher auch was Grundlegendes schneller herausfinden.

fwo hat folgendes geschrieben:
step hat folgendes geschrieben:
Wenn man sich mit der Feinabstimmung einfach so zufriedengibt, glaubt man an etwas Übernatürliches.
Nö. Ein Zufall, den Du zwangsläufig bemerkst, weil Du an seinem Ende stehst, bleibt immer noch ein Zufall.

Aber was ist ein Zufall? Es gibt ja in unserer Welt keinen Zufall, sondern nur auf Mechanismen beruhenden Pseudozufall. Daher will ich wissen, welcher Mechanismus hinter dem (scheinbaren) Zufall der QM steht, in diesem Sinne wäre ein "echter" Zufall metaphysisch.

fwo hat folgendes geschrieben:
Das ist eigentlich immer eher eine Frage an die Gesellschaft wieviel ihr dieses Gebiet wert ist.

Ja, sicher.

#281:  Autor: Marcellinus BeitragVerfasst am: 27.11.2017, 21:27
    —
step hat folgendes geschrieben:

Marcellinus hat folgendes geschrieben:
Die Suche nach absoluten Anfängen oder dogmatischen, endgültigen Antworten ist nicht ihre Aufgabe.

Strohmann? Übrigens wurde dies früher schon gegen allerlei heute akzeptierte Forschungsgebiete eingewandt.


Daß Wissenschaften keine endgültigen Antworten liefern können (Religionen und Philosophien natürlich auch nicht), daß wir uns gewissermaßen „emporirren“, dürfte sich auch bis zu dir rumgesprochen haben.

step hat folgendes geschrieben:

Marcellinus hat folgendes geschrieben:
... wenn man das, was wir in dieser Welt beobachten können, nicht auf die Handlungen und Absichten übernatürlicher Verursacher zurückführt.

Ja, und zwar indem man sie stattdessen auf natürliche Wirkungen zurückführt. In unserem Fall haben wir plötzlich eine "übernatürliche" Zustandsauswahl, und die ist unbefriedigend. Wir wollen sie nict auf etwas Übernatürliches zurückführen, also müssen wir versuchen, sie als natürliches Ergebnis unseres Modells zu beschreiben.


Erst einmal ist es nur eine Beobachtung, für die man keine Erklärung hat. Daran ist nichts Übernatürliches. Wie schrieb hier jemand mal sinngemäß: Jenseits unseres Wissens beginnt Nichtwissen, nicht das „Übernatürliche“.

step hat folgendes geschrieben:

Marcellinus hat folgendes geschrieben:
Insofern stellt sich die Frage mit dem „Physik-Gott“ nicht.

Wenn man sich mit der Feinabstimmung einfach so zufriedengibt, glaubt man an etwas Übernatürliches.


Es geht nicht um „zufiedengeben“, sondern darum, dafür im Moment keine Erklärung zu haben, die sich durch Tatsachenbeobachtungen belegen läßt. Also nö!

step hat folgendes geschrieben:

Marcellinus hat folgendes geschrieben:
Was ist so schlimm daran, auf eine Frage (vorübergehend) keine Antwort zu haben?

Gar nix. Aber wieso "vorübergehend"? Hast Du nicht oben gefordert, man solle von dieser Frage die Finger lassen, weil sie "endgültig" sei?


Nein, habe ich nicht. Das kommt davon, wenn man den Post eines anderen in Einzelzeilen zerlegt. zwinkern

step hat folgendes geschrieben:

Marcellinus hat folgendes geschrieben:
Könnte es sein, daß ihr in diesem Fall die physikalischen Wissenschaften nur benutzt, um philosophische oder religiöse Fragen zu diskutieren?

Nein, könnte nicht sein. Obwohl, Dein letzter Beitrag und meine Antwort (also diese) sind natürlich philosophisch, insofern sie die Methoden der Wissenschaft von einer Metaperspektive aus betrachten.


Ich wüßte nicht, was die Philosophie berechtigte, sich für eine „Metawissenschaft“ zu halten.

#282:  Autor: stepWohnort: Germering BeitragVerfasst am: 27.11.2017, 21:47
    —
Marcellinus hat folgendes geschrieben:
step hat folgendes geschrieben:
Marcellinus hat folgendes geschrieben:
Die Suche nach absoluten Anfängen oder dogmatischen, endgültigen Antworten ist nicht ihre Aufgabe.
Strohmann? Übrigens wurde dies früher schon gegen allerlei heute akzeptierte Forschungsgebiete eingewandt.
Daß Wissenschaften keine endgültigen Antworten liefern können (Religionen und Philosophien natürlich auch nicht), daß wir uns gewissermaßen „emporirren“, dürfte sich auch bis zu dir rumgesprochen haben.

Eben darum solltest Du nicht sugggerieren, ich würde nach "Absolutem", "Dogmatischen" oder "Endgültigem" suchen.

Marcellinus hat folgendes geschrieben:
step hat folgendes geschrieben:
Dein letzter Beitrag und meine Antwort (also diese) sind natürlich philosophisch, insofern sie die Methoden der Wissenschaft von einer Metaperspektive aus betrachten.
Ich wüßte nicht, was die Philosophie berechtigte, sich für eine „Metawissenschaft“ zu halten.

Erstens habe ich "Metaperspektive" und nicht "Metawissenschaft" geschrieben. Zweitens gilt das mE durchaus, für den Teil der Philosophie, auf den ich oben bezug nahm, nämlich die Wissenschaftstheorie. Da geht es genau um solche Fragen wie etwa "was ist Falsifikation" oder "was ist eine gute Theorie".

#283:  Autor: Marcellinus BeitragVerfasst am: 27.11.2017, 22:53
    —
step hat folgendes geschrieben:

Marcellinus hat folgendes geschrieben:
step hat folgendes geschrieben:
Dein letzter Beitrag und meine Antwort (also diese) sind natürlich philosophisch, insofern sie die Methoden der Wissenschaft von einer Metaperspektive aus betrachten.
Ich wüßte nicht, was die Philosophie berechtigte, sich für eine „Metawissenschaft“ zu halten.

Erstens habe ich "Metaperspektive" und nicht "Metawissenschaft" geschrieben. Zweitens gilt das mE durchaus, für den Teil der Philosophie, auf den ich oben bezug nahm, nämlich die Wissenschaftstheorie. Da geht es genau um solche Fragen wie etwa "was ist Falsifikation" oder "was ist eine gute Theorie".


Ja, davon träumt die Philosophie spätestens seit Popper. Vergeblich, würde ich sagen, aber das ist nun wirklich ein anderes Thema.

#284:  Autor: fwoWohnort: im Speckgürtel BeitragVerfasst am: 27.11.2017, 23:32
    —
step hat folgendes geschrieben:
fwo hat folgendes geschrieben:
... auch aus fachlichem Neid, wenn man z.B. sieht, dass die Kosten für die Erforschung des Teilchenzoos in die Milliarden gehen, wogegen es in den Biowissenschaften tatsächlich auch noch Amateurarbeiten an der Grenze gibt.

Das stimmt - wobei die Grundlagenphysiker genauso schlecht bezahlt werden wie die Biologen. Ich denke, ein Grund für die teureren Investitionen in der Physik ist einfach, daß man an ihre tieferen Grundlagen schwerer herankommt. Und gäbe es denn in der Biologie überhaupt ein grundlegendes Experiment oder Vorhaben, das sich nur mangels Großinvestition nicht realisieren läßt? Welche Grundfrage würde es beantworten?

Im Bereich der emergenten Wissenschaften fällt mir da die Hirnforschung ein, da könnte man mit Investitionen sicher auch was Grundlegendes schneller herausfinden.
...

Zum Beispiel. Aber die Großprojekte der Physik haben es irgendwie auch an sich, dass sie politisch einen höheren Image-Gewinn versprechen mit magischen Titeln wie "Die Such nach der Weltformel". Das klingt dann nach Weisheit, während z.B (ich nehme mal nicht die Biologie) künstliche Intelligenz
1. nicht nur schillert, sondern auch Gefahren verheißt, und
2. jeder Philosoph oder was immer sich so nennt, behauptet, mitreden zu können.

In der Biologie bräuchten wir keine großen vorzeigefähigen Maschinen, sondern ganz viele Leute, die die ökologischen Daten sammeln, die wir für Vorhersagen brauchen, denn im Prinzip haben wir viel zu wenig Ahnung von den Ansprüchen und Fähigkeiten der Arten so wie den Abhängigkeiten dazwischen auf diesem Globus, um für irgendein Gebiet wirklich Vorhersagen machen zu können. Das grundsätzliche Problem besteht zusätzlich noch darin, dass all das Wissen, das da gesammelt wird, sich als potentiell wirtschaftsfeindlich herausstellen kann. Das ist in der Physik und den angrenzenden Gebieten fast genau andersherum, weshalb da auch fast jede Arbeitsgruppe ihre Drittmittel hat.

#285:  Autor: stepWohnort: Germering BeitragVerfasst am: 28.11.2017, 13:50
    —
fwo hat folgendes geschrieben:
die Großprojekte der Physik haben es irgendwie auch an sich, dass sie politisch einen höheren Image-Gewinn versprechen mit magischen Titeln wie "Die Such nach der Weltformel". Das klingt dann nach Weisheit, ...

Stimmt, und ein Grund dafür ist auch, daß die Fragen nun mal grundlegender sind. Dazu kommt, daß Physik sehr mathematisch ist und von vielen Leuten mit Unverständnis und Ehrfurcht besetzt ist. Ausgeschlachtet wird das allerdings vor allem von den Medien, weniger von den Physikern selbst.

fwo hat folgendes geschrieben:
... während z.B (ich nehme mal nicht die Biologie) künstliche Intelligenz
1. nicht nur schillert, sondern auch Gefahren verheißt, und

Das gibt es aber bei der Pghysik auch - z.B. erinnerst Du Dich noch, als plötzlich alle Zeitungen schrieben, am CERN könnten aus Versehen gefährliche schwarze Löcher erzeugt werden, die dann alles fressen? Es gab zimlich viele Leute, die damals konkret Angst davor hatten.

fwo hat folgendes geschrieben:
2. jeder Philosoph oder was immer sich so nennt, behauptet, mitreden zu können.

Stimmt. Es wird als "weniger schwierig" empfunden, wahrscheinlich weil es weniger mathematisch komplex und besser in Alltagssprache zu beschrieben ist.

fwo hat folgendes geschrieben:
... Das grundsätzliche Problem besteht zusätzlich noch darin, dass all das Wissen, das da gesammelt wird, sich als potentiell wirtschaftsfeindlich herausstellen kann. Das ist in der Physik und den angrenzenden Gebieten fast genau andersherum, weshalb da auch fast jede Arbeitsgruppe ihre Drittmittel hat.

Das stimmt allerdings nicht wirklich für die Grundlagenforschung in der Physik. Ich glaube nicht, daß es etwa für Stringtheoretiker einfach ist, Drittmittel zu bekommen.

#286:  Autor: fwoWohnort: im Speckgürtel BeitragVerfasst am: 28.11.2017, 15:17
    —
step hat folgendes geschrieben:
....
fwo hat folgendes geschrieben:
2. jeder Philosoph oder was immer sich so nennt, behauptet, mitreden zu können.

Stimmt. Es wird als "weniger schwierig" empfunden, wahrscheinlich weil es weniger mathematisch komplex und besser in Alltagssprache zu beschrieben ist.

Und gleichzeitig kann man es so schön philosophisch verquasen oder verquälen? - ich erinnere in diesem Zusammenhang an die berühmte Qualiadiskussion oder auch an das "chinesische Zimmer".

Bei der Biologie ist Mathe höchtens intern ein Alptraum (Die Leute, die Methoden benutzen, die sie nicht verstanden haben, und im Ausgleich dafür anschließend an die Zahlen glauben, die diese Methoden produzieren, sind mir ein ewiger Quell des Ärgernisses), die Einzelarbeiten sind für Normalbürger einfach nur Pillepalle, genauso wie die größeren Themen, denen sie zuarbeiten.
step hat folgendes geschrieben:
fwo hat folgendes geschrieben:
... Das grundsätzliche Problem besteht zusätzlich noch darin, dass all das Wissen, das da gesammelt wird, sich als potentiell wirtschaftsfeindlich herausstellen kann. Das ist in der Physik und den angrenzenden Gebieten fast genau andersherum, weshalb da auch fast jede Arbeitsgruppe ihre Drittmittel hat.

Das stimmt allerdings nicht wirklich für die Grundlagenforschung in der Physik. Ich glaube nicht, daß es etwa für Stringtheoretiker einfach ist, Drittmittel zu bekommen.

Da ist mein Horizont sehr begrenzt und meine erste Frage ist sofort, inwiefern auch ohne die Berücksichtigung finanzieller Probleme überhaupt zur Zeit Experimente in Sicht wären, mit denen man in dieser Richtung forschen könnte. Das, was ich bisher davon mitbekommen habe, klang für mich alles nur spekulativ theoretisch und ohne eine Aussicht, es überhaupt überprüfen zu können.

#287:  Autor: stepWohnort: Germering BeitragVerfasst am: 28.11.2017, 17:59
    —
fwo hat folgendes geschrieben:
step hat folgendes geschrieben:
Ich glaube nicht, daß es etwa für Stringtheoretiker einfach ist, Drittmittel zu bekommen.
Da ist mein Horizont sehr begrenzt und meine erste Frage ist sofort, inwiefern auch ohne die Berücksichtigung finanzieller Probleme überhaupt zur Zeit Experimente in Sicht wären, mit denen man in dieser Richtung forschen könnte. Das, was ich bisher davon mitbekommen habe, klang für mich alles nur spekulativ theoretisch und ohne eine Aussicht, es überhaupt überprüfen zu können.

Viele Aspekte spezieller Stringtheorien, Supersymmetrie usw. sind mit heutigen Mitteln experimentell falsifizierbar. Aber eben nur mit extrem hohen Energien.

#288:  Autor: smallie BeitragVerfasst am: 30.11.2017, 22:41
    —
zelig hat folgendes geschrieben:
step hat folgendes geschrieben:
Willst Du auf sowas hinaus wie hier beschrieben?
http://www.spektrum.de/news/wie-real-ist-die-quantenverschraenkung/1445463


Danke für den Link. Den Artikel habe ich damals gelesen, aber ich hatte die Quelle vergessen.
Ich finde den Gedanken der "subtilen Einschränkung" der Wahlfreiheit faszinierend und einleuchtend zugleich.

Ebenso.


zelig hat folgendes geschrieben:
Kommt man mit solchen Überlegungen nicht an eine Grenze, die die Begrifflichkeit, mit der wir operieren, unbrauchbar macht?

Ich tu' mich schwer, im Artikel eine zusammenhängende Überlegung zu finden, mit der ich am Ende bei Zeilingers Versuch lande. Über die Entropie von Schwarzen Löchern geht es zur Annahme, Gravitation könne die gesuchte Einschränkung sein. Ohne weitere Erklärungen fällt dann der Versuchsaufbau vom Himmel.

Die "wahre" Erklärung hab' ich noch nicht ganz verstanden. Grob: die Detektoren stehen in einem gemeinsamen Lichtkegel, und könnten deshalb bereits verschränkt sein, oder mit dem Gerät verschränkt sein, das die verschränkten Teilchen erzeugt. Deshalb nimmt man Licht von Sternen, die weit auseinander stehen, so daß der Teilchengenerator nicht mit ihnen verschränkt ist. So ungefähr.

Ein Fehlschlag des Versuchs wäre eine Sensation gewesen. Licht von weit entfernten Sternen soll sich anders verhalten als Licht von der Sonne, das 8 Minuten nach dem Bau eines Detektors mit ihm verschränkt ist? Hat jemand erwartet, daß der Versuch fehlschlägt und die bekannte Quantenmechanik widerlegt? Ich glaube kaum.

Ein bisschen Steam-Punk-Physik als Gegenargument. Das Argument ist alt, damals stritt man noch darüber, ob es überhaupt Atome gebe, aber der Grundgedanke ist zeitlos.

Zitat:
James Clerk Maxwell, Moleküle - 1873

Jedoch finden wir durch ihr Licht, und nur durch ihr Licht, Sterne, die so weit voneinander entfernt sind, daß kein materielles Ding je von einem zum anderen gelangen haben könnte. Und doch erzählt uns dieses Licht, welches der einzige Beleg des Daseins dieser fernen Welten ist, daß jede von ihnen aus den gleichen Molekülen besteht, die wir auch auf der Erde finden. Ein Wasserstoffmolekül zum Beispiel, führt seinen Schwingungen sowohl im Sirius wie im Arcturus in genau gleicher Zeit aus.

But in the heavens we discover by their light, and by their light alone, stars so distant from each other that no material thing can ever have passed from one to another, and yet this light, which is to us the sole evidence of the existence of these distant worlds, tells us also that each of them is built up of molecules of the same kinds as those which we find on earth. A molecule of hydrogen, for example, whether in Sirius or in Arcturus, executes its vibrations in precisely the same time.

http://www.victorianweb.org/science/maxwell/molecules.html


Modern und stärker formuliert heißt das, die Kopplungskonstanten wie die Feinstrukturkonstante sind im beobachtbaren Universum gleich.




zelig hat folgendes geschrieben:
Was soll denn "Superdeterminismus" bedeuten, wenn das Prinzip von Ursache und Wirkung einzig im Zeit-Raum-Gebilde geltung hat.

Mir scheint, Superdeterminismus sei nur ein Wort für den Wunsch, die Regeln der Quantenmechanik aus grundlegenderen Prinzipien herleiten zu können.

Zitat aus dem Artikel: Es könnte unbekannte Einflüsse, Beschränkungen, Randbedingungen, Erhaltungssätze geben. Heißt auf Deutsch, jenseits von vagen Vermutungen gibt es nichts Konkretes zu sagen. Aber, hey, immerhin haben wir ein Wort für das Unbekannte.

Dabei hat vor einiger Zeit schon mal jemand Superdeterminismus vertreten, was man ihm als Starrsinn angekreidet hat. Vergleiche:

Artikel hat folgendes geschrieben:
"Wenn tatsächlich alle Korrelationen bereits beim Urknall festgelegt wurden, dann ist alles vorherbestimmt", sagt Larsson. "Ich empfinde ein solches Weltbild als öde. Ich kann einfach nicht glauben, dass es so ist."

mit dem Einstein-Wort:

    Der Alte würfelt nicht.

Damit steht jetzt Aussage gegen Aussage.

Mir ist die Einsteinsche Sicht sympathisch - aber ich komm' allmählich auch ins passende Alter für Starrsinn. zwinkern

#289:  Autor: zelig BeitragVerfasst am: 16.10.2020, 20:01
    —
Eine Zeptosekunde ist ein Billionstel einer Milliardstel, schreibt die FAZ. Nur wenige Billionstel einer Milliardstel Sekunde dauert es, bis Lichtteilchen ein Molekül mit zwei Atomen durchquert haben. Die kürzeste je gemessene Zeitspanne. : )

https://www.faz.net/aktuell/rhein-main/frankfurt/physiker-messen-im-labor-kuerzeste-zeitspanne-der-welt-17005812.html

#290:  Autor: jdfWohnort: Nekropole E|B BeitragVerfasst am: 16.10.2020, 22:44
    —
zelig hat folgendes geschrieben:
Eine Zeptosekunde ist ein Billionstel einer Milliardstel, schreibt die FAZ. Nur wenige Billionstel einer Milliardstel Sekunde dauert es, bis Lichtteilchen ein Molekül mit zwei Atomen durchquert haben. Die kürzeste je gemessene Zeitspanne. : )

https://www.faz.net/aktuell/rhein-main/frankfurt/physiker-messen-im-labor-kuerzeste-zeitspanne-der-welt-17005812.html

Ich liebe es...

#291:  Autor: zelig BeitragVerfasst am: 25.11.2020, 20:36
    —
Zitat:
Sollte sich Quintessenz tatsächlich als Erklärung für die Dunkle Energie herausschälen, hätte dies eine beträchtliche Folgewirkung, etwa für das Alter des Universums. Derzeit schätzen es Experten anhand der Hintergrundstrahlung auf 13,8 Milliarden Jahre, die Quintessenz würde es etwas jünger machen. Mit ihr ließe sich auch erklären, warum die Hintergrundstrahlung eine Verlangsamung der kosmischen Expansion vorhersagt.

https://www.spektrum.de/news/verdrehtes-licht-liefert-hinweise-auf-dunkle-energie/1799180

#292:  Autor: zelig BeitragVerfasst am: 10.02.2021, 16:54
    —
Eine Frage, die mich aufgrund der Lektüre eines Reclam-Heftchens beschäftigt:
Ist das Multiversum mächtiger als der Minkowski-Raum? Und wie verhalten sich beide zur Stringtheorie?

#293:  Autor: smallie BeitragVerfasst am: 13.02.2021, 14:39
    —
Alles folgende cum grano salis. zwinkern


Was meinst du mit "mächtiger"?

zelig hat folgendes geschrieben:
Ist das Multiversum mächtiger als der Minkowski-Raum?

"Mächtig" so wie die Mächtigkeit in der Mathematik? Als Zahl der Elemente einer Menge?

Für das Mulitiversum wurden fünf verschiedene Typen behauptet. Die höheren Untertypen können selbst eine Vielzahl von Universen aufweisen. Muß man die mitzählen?

Wie viele Minkowski-Räume gibt es? Ist ein MR denkbar, in dem die Lichtgeschwindigkeit nicht 299 999 km/s beträgt, sondern 299 998 km/s? Wären das dann zwei? Ich würde sagen: nur einer, weil Lichtgeschwindigkeit ein Parameter ist. Was ist mit einem Minkowski-Raum, der mehr als drei Raumdimensionen hat? Usw.

Analog gilt: eine anständige Weltformel wäre auch nur eine, in Zahlen 1, Formel mit einem Satz von Parametern. So gesehen hat das Multiversum eine Mächtigkeit von 1 und ist genau so mächtig wie der Minkowski-Raum. Geschockt

Errm. Das gefällt mir jetzt auch nicht. Die Idee vom Minkowski-Raum, wenn ich sie salopp aber berechtigt mit Relativität gleichsetze, enthält nicht die ganze Physik. Es fehlt die Quantenmechanik. Minkowski-Raum ist also nicht so "mächtig" wie die ganze Physik. So gesehen hat das Universum (egal ob Single- oder Multiversum) eine höhere Mächtigkeit.

Das war jetzt wie bei den Rechentricks, wo am Ende 1 = 2 herauskommt, weil man versteckt durch 0 geteilt hat oder sowas. Pfeifen


-----------------------------------------------------------------------------------------------------------------------

Oder meinst du "mächtiger" so wie "grundlegender"? Darauf würde ich analog zu oben antworten. Es ist schwer, Dinge zu zählen, von denen man nicht weiß, ob es sie gibt. Es ist schwer zu sagen, was grundlegender ist, wenn man die Grundlagen nicht oder nur zum Teil kennt.


zelig hat folgendes geschrieben:
Und wie verhalten sich beide zur Stringtheorie?

Multiversums-Vorstellungen und den Minkowski-Raum gab es lange vor der Stringtheorie.

Die Stringtheorie kann auch ein Multiversum hervorbringen. Gehen die zehn oder elf Dimensionen der Stringtheorie als verallgemeinerter Minkowski-Raum durch? Ich habe nicht die leiseste Ahnung.

#294:  Autor: zelig BeitragVerfasst am: 13.02.2021, 15:15
    —
smallie hat folgendes geschrieben:
Alles folgende cum grano salis. ;)
Was meinst du mit "mächtiger"?
"Mächtig" so wie die Mächtigkeit in der Mathematik? Als Zahl der Elemente einer Menge?


Ja, so ungefähr. In meinem laienhaften Verständnis unterscheiden sich die Theorien auch hinsichtlich der Gesamtenergie/Masse, die sie zum Objekt haben. Beispielsweise würde ich erwarten, daß die Stringtheorie aufgrund der spekulativen vielen differenten Welten mächtiger ist, als der Minkowski-Raum, der "nur" die Realität des uns bekannten Universums umfasst. Uff.



Freigeisterhaus -> Wissenschaft und Technik


output generated using printer-friendly topic mod. Alle Zeiten sind GMT + 1 Stunde

Seite 1 von 1

Powered by phpBB © 2001, 2005 phpBB Group